Vous êtes sur la page 1sur 230

2: General and Computer Adaptive Test 3.

Pulse 112, blood pressure 100/60, respirations 20


Strategies 4. Pain at the biopsy site
The following is an example of a knowledge-based Can you select an answer based on recall or
question you might have seen in nursing recognition? No. Let’s analyze the question and
school. answer choices.
Which of the following is a complication that occurs The question is: What is a complication of a liver
during the first 24 hours after a biopsy? In order to begin to analyze this
percutaneous liver biopsy? question, you must know that hemorrhage is the
(1) Nausea and vomiting major complication. However, it’s not listed
(2) Constipation as an answer. Can you find hemorrhage in one of the
(3) Hemorrhage answer choices?
(4) Pain at the biopsy site ANSWERS:
The question restated is, “What is a common (1) “Anorexia, nausea, and vomiting.” Does this
complication of a liver biopsy?” You may or indicate that the client is hemorrhaging?
may not remember the answer. So, as you look at the No, these are not symptoms of hemorrhage.
answer choices, you hope to see an item (2) “Abdominal distention and discomfort.” Does this
that looks familiar. You do see something that looks indicate that the client is hemorrhaging?
familiar: “Hemorrhage.” Perhaps. Abdominal distention could indicate internal
Recall/Recognition bleeding.
Understanding (3) “Pulse 112, blood pressure 100/60, respirations
Application 20.” Does this indicate that the client is
Analysis hemorrhaging? Yes. An increased pulse, a decreased
Figure 1: Levels of Questions in Nursing Tests blood pressure, and increased respirations indicate
In nursing school, you are also given test questions shock. Shock is a result of hemorrhage.
written at the comprehension level. These (4) “Pain at the biopsy site.” Does this indicate the
questions require you to understand the meaning of client is hemorrhaging? No. Pain at the
the material. Let’s look at this same question biopsy site is expected due to the procedure.
written at the comprehension level. Ask yourself, “Which is the best indicator of
The nurse understands that hemorrhage is a hemorrhage?” Abdominal distention or a change
complication of a liver biopsy due to in vital signs? Abdominal distention can be caused by
which of the following reasons? liver disease. The correct answer is (3).
(1) There are several large blood vessels near the This question tests you at the application level. You
liver. were not able to answer the question
(2) The liver cells are bathed with a mixture of venous by recalling or recognizing the word hemorrhage. You
and arterial blood. had to take information you learned
(3) The test is performed on clients with elevated (hemorrhage is the major complication of a liver
enzymes. biopsy) and select the answer that best indicates
(4) The procedure requires a large piece of tissue to hemorrhage. Application involves taking the facts that
be removed. you know, and using them to
The question restated is, “Why does hemorrhage make a nursing judgment. You must be able to answer
occur after a liver biopsy?” In order to answer this questions at the application level in
question, the nurse must understand that the liver is order to prove your competence on the NCLEX-RN®
a highly vascular organ. The portal vein and the exam.
hepatic artery join in the liver to form the sinusoids Let’s look at a question that is written at the analysis
that bathe the liver in a mixture of venous and arterial level.
blood. The nurse is caring for a 56-year-old man receiving
The NCLEX-RN® exam asks few minimum- haloperidol (Haldol) 2 mg PO
competency questions at the comprehension bid. The nurse assists the client to choose which of the
level. It assumes you know and understand the facts following menus?
you learned in nursing school. 1. 3 oz. roast beef, baked potato, salad with dressing,
Minimum-competency NCLEX-RN® exam questions dill pickle, baked apple
are written at the application and/or pie, and milk
analysis level. Remember, the NCLEX-RN® exam 2. 3 oz. baked chicken, green beans, steamed rice, 1
tests your ability to make safe judgments slice of bread, banana, and
about client care. Your ability to solve problems is not milk
tested with questions at the recall/recognition 3. Cheeseburger on a bun, french fries with catsup,
or comprehension level. chocolate chip cookie,
Let’s look at this same question written at the apple, and milk
application level. 4. 3 oz. baked fish, slice of bread, broccoli, ice cream,
Which of the following symptoms observed by the and pineapple drink taken
nurse during the first 24 30–60 minutes after the meal
hours after a percutaneous liver biopsy would indicate Many students panic when they read this question
a complication from the because they can’t immediately recall any
procedure? diet restriction required by a client taking Haldol.
1. Anorexia, nausea, and vomiting Because students can’t recall the information,
2. Abdominal distention and discomfort
they assume that they didn’t learn enough 27
information. Analysis questions are often 2: General and Computer Adaptive Test
written so that a familiar piece of information is put in Strategies
an unfamiliar setting. Let’s think Strategies That Don’t Work on the NCLEX-RN ®
about this question. Exam
26 Whether you realize it or not, you developed a set of
NCLEX-RN® Exam Overview and Test Taking strategies in nursing school to answer
Strategies teacher-generated test questions that are written at
What type of diet do you choose for a client receiving the knowledge/comprehension level.
Haldol? In order to begin analyzing These strategies include the following:
this question, you must first recall that Haldol is an • “Cramming” in hundreds of facts about disease
antipsychotic medication used to treat processes and nursing care
psychotic disorders. There are no diet restrictions for • Recognizing and recalling facts rather than
clients taking Haldol. Because there are understanding the pathophysiology and the
no diet restrictions, you must problem-solve to needs of a client with an illness
determine what this question is really asking. • Knowing who wrote the question and what is
Based on the answer choices, it is obviously a diet important to that instructor
question. What kind of diet should you • Predicting answers based on what you remember or
choose for this client? Because you have been given who wrote the test question
no other information, there is only one • Selecting the response that is a different length
type of diet that can be considered: a regular balanced compared to the other choices
diet. This is an example of taking the • Selecting the answer choice that is grammatically
familiar (a regular balanced diet) and putting it into correct
the unfamiliar (a client receiving Haldol). • When in doubt, choosing answer choice (C)
In this question, the critical thinking is deciding what These strategies will not work on the NCLEX-RN®
this question is really asking. exam. Remember, the NCLEX-RN®
QUESTION: “What is the most balanced regular diet?” exam is testing your ability to make safe, competent
ANSWERS: decisions.
(1) “3 oz. roast beef, baked potato, salad with Becoming a Better Test Taker
dressing, dill pickle, baked apple pie, and The first step to becoming a better test taker is to
milk.” Is this a balanced diet? Yes, it certainly has assess and identify the following:
possibilities. • The kind of test taker you are
(2) “3 oz. baked chicken, green beans, steamed rice, • The kind of learner you are
1 slice of bread, banana, and milk.” Is Successful NCLEX-RN® Exam Test Takers
this a balanced diet? Yes, this is also a good answer • Have a good understanding of nursing content.
because it contains foods from each • Have the ability to tackle each test question with a
of the food groups. lot of confidence because they
(3) “Cheeseburger on a bun, french fries with catsup, assume that they can figure out the right answer.
chocolate chip cookie, apple, and • Don’t give up if they are unsure of the answer. They
milk.” Is this a balanced diet? No. This diet is high in are not afraid to think about the
fat and does not contain all of the question, and the possible choices, in order to select
food groups. Eliminate this answer. the correct answer.
(4) “3 oz. baked fish, slice of bread, broccoli, ice • Possess the know-how to correctly identify the
cream, and pineapple drink taken 30–60 question.
minutes after the meal.” Does this sound like a • Stay focused on the question.
balanced diet? The choice of foods isn’t Unsuccessful NCLEX-RN® Exam Test Takers
bad, but why would the intake of fluids be delayed? • Assume that they either know or don’t know the
This sounds like a menu to prevent answer to the question.
dumping syndrome. Eliminate this answer. • Memorize facts to answer questions by recall or
Which is the better answer choice: (1) or (2)? Dill recognition.
pickles are high in sodium, so the correct 28
answer is (2). NCLEX-RN® Exam Overview and Test Taking
Choosing the menu that best represents a balanced Strategies
diet is not a difficult question to answer. • Read the question, read the answers, read the
The challenge lies in determining that a balanced diet question again, and pick an answer.
is the topic of the question. Note that • Choose answer choices based on a hunch or a feeling
answer choices (1) and (2) are very similar. Because instead of thinking carefully.
the NCLEX-RN® exam is testing your • Answer questions based on personal experience
discretion, you will be making a decision between rather than nursing theory.
answer choices that are very close in meaning. • Give up too soon, because they aren’t willing to think
Don’t expect obvious answer choices. hard about questions and answers.
These questions highlight the difference between the • Don’t stay focused on the question.
knowledge/comprehension-based questions If you are a successful test taker, congratulations!
that you may have seen in nursing school, and the This book will reinforce your test-taking
application/analysis-based questions skills. If you have many of the characteristics of an
that you will see on the NCLEX-RN® exam. unsuccessful test taker, don’t despair! You
can change. If you follow the strategies in this book, 1. Two-point gait
you will become a successful test taker. 2. Three-point gait
What Kind of Learner Are You? 3. Four-point gait
It is important for you to identify whether you think 4. Swing-through gait
predominantly in images or words. Why? Don’t panic if you can’t remember crutch-walking
This will assist you in developing a study plan that is gaits. Instead, visualize!
specific for your learning style. Read Step 1. “See” a person (or yourself) walking normally.
the following statement: First the right leg and left arm are
A nurse walks into a room and finds the client lying on extended, and then the left leg and right arm are
the floor. extended.
As you read those words, did you hear yourself Step 2. Put crutches in your hands. Now walk. Each
reading the words? Or did you see a nurse foot and each crutch is a point.
walking into a room, and see the client lying on the Step 3. “See” a person (or yourself) with a full cast on
floor? If you heard yourself reading the the left leg, with the foot never touching
sentence, you think in words. If you formed a mental the ground.
image (saw a picture), you think in Step 4. Visualize the answers.
images. (1) Two-point gait. One leg and one crutch would be
Students who think in images sometimes have a touching the ground at the same time.
difficult time answering nursing test questions. Sounds like normal walking. Eliminate this choice
These students say things like: because the client is non-weightbearing.
“I have to study harder than the other students.” (2) Three-point gait. Both crutches and one foot are
“I have to look up the same information over and over on the ground. This would be appropriate
again.” for a non-weight-bearing client.
“Once I see the procedure (or client), I don’t have any (3) Four-point gait. This would require both legs and
difficulty understanding or remembering crutches to touch the ground. However,
the content.” in this question the client is non-weight-bearing.
“I have trouble understanding procedures from Eliminate this option.
reading the book. I have to see the procedure (4) Swing-through gait. This gait means advancing
to understand it.” both crutches, then both legs, and
“I have trouble answering test questions about clients requires weight-bearing. The gait is not as stable as
or procedures I’ve never seen.” the other gaits. Eliminate this option:
Why is that? For some people, imagery is necessary the client in this question is non-weight-bearing.
to understand ideas and concepts. If this The correct answer is (2). Even if you are unsure of
is true for you, you need to visualize information that crutch-walking gaits, imaging and thinking
you are learning. As you prepare for through the answer choices will enable you to select
the NCLEX-RN® exam, try to form mental images of the correct answer.
terminology, procedures, and diseases. The nurse cares for a client diagnosed with a right-
For example, if you’re reviewing information about sided cerebrovascular
traction but you have never seen traction, accident (CVA) with dysphagia. Which of the following
it would be ideal for you to see a client in traction. If actions by the nurse
that isn’t possible, find a picture of reflects appropriate care for the client? Select all
traction and rig up a traction setup with whatever that apply.
material you have available. As you read £ 1. The nurse assesses the client’s ability to swallow.
about traction, use the photo or model to visualize £ 2. The nurse positions the client at a 45-degree
care of the client. If you can visualize the angle.
theory that you are trying to learn, it will make recall £ 3. The nurse offers the client scrambled eggs.
and understanding of concepts much £ 4. The nurse instructs the client to place food on the
easier for you. left side of the mouth.
29 £ 5. The nurse turns off the television.
2: General and Computer Adaptive Test You will know that the question is a “Select all that
Strategies apply” alternate format question because
It is also important that you visualize test questions. after the question stem and before the answer choices
As you read the question and possible you will be instructed to “Select all that
answer choices, picture yourself going through each apply.” You will note that there are more than four
suggested action. This will increase your possible answer choices; usually five or six
chances of selecting correct answer choices. are provided. Also, there is a box in front of each
Let’s look at a test question that requires imagery. answer choice rather than the radio button
An adolescent is seen in the emergency room for a you see with multiple-choice, four-option, text-based
fracture of the left femur questions.
sustained in a sledding accident. The fracture is To answer this type of question, determine which of
reduced and a cast is applied. the answer choices provided are correct.
The client is taught how to use crutches for It is important to remember that in order for the
ambulating without bearing weight question to be scored as correct, you must
on the left leg. The nurse would expect the client to select all of the answer choices that apply, not just the
learn which of the following best response. You will not receive any
crutch-walking gaits?
partial credit if you do not. Left-click on the box in When you have selected all the responses you believe
front of each answer choice that you think to be correct, click on the NEXT (N)
is correct. A small check mark appears in the box button in the bottom left of the screen or press the
indicating that you selected that answer. If Enter key on the keyboard to lock in your
you change your mind about a particular answer answer and go on to the next question. Remember,
choice, just click on the box again: the check once you click on the NEXT (N) button
mark disappears and the answer choice is no longer or press the Enter key, you have entered your answer
selected. to the question and you cannot return
How should you approach this type of question? What to the question.
doesn’t work is to compare and contrast The nurse cares for a client diagnosed with a right-
the individual answer choices. For a “Select all that sided cerebrovascular
apply” question, any number of accident (CVA) with dysphagia. Which of the following
answer choices may be correct. Instead, consider actions by the nurse
each answer choice a True/False question. reflects appropriate care for the client? Select all
Reword this question to ask, “What is appropriate care that apply.
for a client with a right-sided CVA 1. The nurse assesses the client’s ability to swallow.
who has dysphagia?” Dysphagia means the client is £ 2. The nurse positions the client at a 45-degree
having difficulty swallowing; if the CVA angle.
is in the right hemisphere, the client’s left side is 3. The nurse offers the client scrambled eggs.
affected. £ 4. The nurse instructs the client to place food on the
left side of the mouth.
NCLEX-RN® Exam Overview and Test Taking 5. The nurse turns off the television.
Strategies 33
Let’s look at the answers. The strategy is to change Strategies
each answer choice into a statement, and Here’s the answer to this hot spot question.
then determine if the statement is true or false. The nurse performs a physical assessment on an adult
(1) “I should assess the client’s ability to swallow.” Is male. Identify the area
this true for a client with dysphagia? where the nurse should place the stethoscope to
Yes. This is a correct response because the nurse auscultate heart sounds heard
needs to make sure that the client in the tricuspid area.
can swallow food before giving him anything to eat. _
The results of the evaluation will It is important for you to know where to listen to
also determine whether the nurse should offer the specific heart sounds. In addition to the
client clear liquids or thickened liquids. tricuspid area, you should be able to locate other
Some clients will require thickened liquids while others anatomical landmarks to evaluate heart
will not. Select this answer sounds:
choice. • Angle of Louis—manubrial sternal junction at the
(2) “I should position the client at a 45-degree angle.” second rib
Is this the correct position for a client • Aortic area—second intercostal space to the right of
with dysphagia? No. The client should be sitting the sternum
upright in a chair or the bed. Eliminate • Pulmonic area—second intercostal space to the left
this answer choice. of the sternum
(3) “I should offer the client scrambled eggs.” Is this • Erb’s point—third intercostal space to the left of the
an appropriate food for a client with sternum
dysphagia? Yes. Soft or semi-soft foods are more • Mitral area—fifth intercostal space at the left
easily tolerated than a regular diet. midclavicular line
Select this answer choice. In the mitral area of an adult is the apical impulse,
(4) “I should instruct the client to place food on the also known the point of maximal impulse
left side of his mouth.” Is this what (PMI), where the impulse of the left ventricle is felt
should be done? If the client has a right-sided CVA, most strongly; on an infant, the apical
that means the left side of the client’s impulse is lateral to the left nipple.
body is affected. The food should be placed on the Fill-in-the-Blank—Enter the answer
unaffected side—the right side of the This type of alternate format question asks you to fill
mouth for this client. Eliminate this answer. in the blank with a number based on
(5) “I should turn off the television.” What are they a calculation.
getting at with this statement? Many 35
clients are easily distracted after a CVA. If the client 2: General and Computer Adaptive Test
has dysphagia, you don’t want him Strategies
to aspirate because he is distracted by the television. The following is an example of a fill-in-the-blank
It is best to turn off the TV during question.
meals. Select this answer choice. The nurse cares for a client receiving hourly peritoneal
So, which answers should be checked as correct? For dialysis exchanges.
this question, choices (1), (3), and (5) During a one-hour exchange, the nurse infuses 2,000
are correct. Left-click on the boxes in front of each of mL of dialysate and 1,900
these answer choices to select them. mL of outflow is returned. During the exchange, the
client drinks 8 oz. of apple
juice, 2 cups of water, and voids 150 mL of urine. solution.
Calculate and record the client’s Inflate the balloon of the
intake in milliliters. catheter to check for leaks.
mL Place the client supine
To answer this question, calculate the client’s intake with knees flexed.
from the information provided. Note: Lubricate the tip of the
Pay close attention to the unit of measure you catheter.
need for your final answer. In this situation, you Put on the sterile gloves.
are asked for the client’s intake in milliliters, not cups Unordered Options Ordered Response
or ounces. The strategy to use in answering this kind of question
You can use the drop-down calculator provided on the is to picture yourself performing the
computer to do the math. The button procedure. First, prepare the client. Next, prepare the
that displays the calculator is on the bottom of the equipment in the correct order, using
right side of the computer screen. Use sterile technique. Open the sterile pack between the
your mouse to click on the numbers or functions you client’s legs. Next, put on the sterile
want. Remember, the slash (/) is used 2: General and Computer Adaptive Test
for division. Strategies
To answer this question you need to know that intake gloves. Inflate the balloon of the catheter to check for
includes what the client drinks along leaks. Lubricate the tip of the catheter.
with the amount of dialysate that is retained after the Once the equipment is ready, prepare the client for
one-hour exchange of solution. the insertion of the catheter. The last step
First, convert cups into ounces. One cup of fluid = 8 from those provided is to wipe the urinary meatus with
oz. Then convert ounces into milliliters. a cotton ball saturated with cleansing
One ounce = 30 milliliters. solution.
The client’s intake is: To place the options in the correct order, click on an
8 oz. apple juice = 240 mL option and drag it to the box on the
2 cups = 16 oz. water = 480 mL right. You can also move an answer from the left
100 mL = retained dialysate column to the right column by highlighting
Use the computer mouse to move the cursor inside the option and clicking the arrow key that points to
the text box. Left-click on the cursor. the column on the right. You may also
Type in the correct intake using the number keys on rearrange
the keyboard. The correct answer is 820. the order of the options in the right column using the
Do not put mL or any unit of measure after the arrow keys pointing up and
number. Only the number goes into the box. down.
Rules for rounding are typically provided with the Here’s the answer to this question.
question. The nurse prepares to insert an indwelling Foley
36 catheter in an elderly female
NCLEX-RN® Exam Overview and Test Taking client. Arrange the following steps in the order the
Strategies nurse should perform them.
The nurse cares for a client receiving hourly peritoneal All options must be used.
dialysis exchanges. Unordered Options Ordered Response
During a one-hour exchange, the nurse infuses 2,000 Wipe the urinary meatus
mL of dialysate and 1,900 with a cotton ball
mL of outflow is returned. During the exchange, the saturated with cleansing
client drinks 8 oz. of apple solution.
juice, 2 cups of water, and voids 150 mL of urine. Lubricate the tip of the
Calculate and record the client’s catheter.
intake in milliliters. Inflate the balloon of the
820 mL catheter to check for leaks.
Drag and Drop/Ordered Response—Arrange the Put on the sterile gloves.
answers in the correct Open the sterile pack
order between the client’s legs.
This is one of the newer alternate format question Place the client supine
types introduced by NCSBN. These questions with knees flexed.
ask you to place answers in a specific order. Multiple-Choice Test Questions
Take a look at the following question. Multiple-choice questions with four answer options
The nurse prepares to insert an indwelling Foley may take the form of a traditional textbased
catheter in an elderly female question, or may be in the form of an alternate
client. Arrange the following steps in the order the question that includes an exhibit/chart,
nurse should perform them. is based on an audio clip, or contains graphics in place
All options must be used. of some of the text. No matter the
Open the sterile pack
between the client’s legs.
Wipe the urinary meatus 38
with a cotton ball NCLEX-RN® Exam Overview and Test Taking
saturated with cleansing Strategies
form, to effectively apply the strategies discussed in of play therapy is to give children the opportunity to
this book, you need to understand the communicate using their own
components of an NCLEX-RN® exam multiple-choice “language.” This is the correct answer.
question. They are as follows: (3) Assess her developmental level. The nurse might
• The stem of the question. The stem includes the be able to assess whether a child is
situation that describes the client, his functioning at an age-appropriate level, but this is not
or her problems or health care needs, and other the primary purpose of play
relevant information. It also includes a therapy. This is a distracter.
question or an incomplete statement. This is the (4) Find out what type of abuse she has experienced.
question that you must answer. The child might communicate
• Three incorrect answers, referred to here as the type of abuse she has experienced if that is what
distracters. she chooses to communicate.
• The correct answer. The nurse should focus on the purpose of play
The three distracters will probably sound logical to therapy, not the type of abuse. This
you. They may even be based on information is a distracter.
provided in the stem, but they don’t really answer the Let’s try another question.
question. Other incorrect answers A client is being treated for heart failure with diuretic
may be actions that are common nursing practice but therapy. Which of the
not ideal nursing practice. following assessments best indicates to the nurse that
The correct answer is the only choice that is the client’s condition is
recognized as correct by the NCLEX-RN® exam, improving?
so you need to learn to select it. Remember that most 1. The client’s weight has remained stable since
answer choices are written at the application admission.
level: you will not be able to select answers based on 2. The client’s systolic blood pressure has decreased.
recognition or recall. You must 3. There are fewer crackles heard when auscultating
understand the whys of nursing care in order to select the client’s lungs.
the correct response. 4. The client’s urinary output is 1,500 mL per day.
Read the following exam-style question. In addition to The Components
selecting an answer, identify the components • The stem:
of this question. ‚ . Heart failure
The nurse plans care for a 4-year-old girl who has ‚ . Treatment is diuretic therapy
been sexually abused by her ‚ . How do you know the client’s condition is
father. Play therapy is scheduled. The nurse knows improving?
that the primary goal of play • The answer choices:
therapy for a 4-year-old is which of the following? (1) Weight has remained stable. The client’s weight
1. Provide her with the opportunity to express anger should decrease because he is taking
and hostility by playing a diuretic. Weight addresses issues involved with
with dolls. diuretic therapy. However, it
2. Promote communication because she may lack the is not the best indication of improvement in a client
emotional and intellectual with heart failure. This is a
capacity to express her perceptions verbally. distracter.
3. Assess whether she is functioning at an age- (2) The systolic blood pressure has decreased.
appropriate developmental level. Decreased blood pressure may be
4. Reveal through direct observation of her at play the result of diuretic therapy, but the reduction could
what type of abuse has been also be due to other causes
experienced. (change of position, calm rather than in an excited
The Components state, etc.). This is not the best
• The stem: indication of an improvement in a client with heart
‚ . 4-year-old girl failure. This is a distracter.
‚ . Sexually abused by her father 40
‚ . Play therapy is scheduled NCLEX-RN® Exam Overview and Test Taking
‚ . What is the primary goal of play therapy for a 4- Strategies
year-old? (3) There are fewer crackles. A client with heart failure
• The answer choices: has crackles due to pulmonary
39 edema. Diuretics are given to promote excretion of
2: General and Computer Adaptive Test sodium and water through the
Strategies kidneys. Decreased crackles would indicate that the
(1) Provide opportunity to express anger and hostility. pulmonary edema is improving.
Play therapy will allow children This is the correct answer.
to express anger and hostility if that’s what they want (4) Urinary output of 1,500 mL in 24 hours. This is
to communicate. Some within normal limits. Although a
students select this answer because they focus on the normal output addresses diuretic therapy, it is not the
treatment of sexual abuse best indication of improvement
mentioned in the situation. This is a distracter. of heart failure. This is a distracter.
(2) Promote communication. Play is the universal Critical Thinking Strategies
language of children. The purpose
• The NCLEX-RN® exam is not a test about or a no, or with a specific bit of information. Begin
recognizing facts. your questions with what, when, or why.
• You must be able to correctly identify what the We will refer to this reworded version as THE
question is asking. REWORDED QUESTION in the examples
• Do not focus on background information that is not that follow.
needed to answer the question. Step 4. If you can’t complete step 3, read the answer
• The NCLEX-RN® exam focuses on thinking through choices for clues.
a problem or situation. Let’s practice rewording a question.
Now that you are more knowledgeable about the A preschooler with a fractured femur is brought to the
components of a multiple-choice test question, emergency room by her
let’s talk about specific strategies that you can use to parents. When asked how the injury occurred, the
problem-solve your way to correct child’s parents state that she
answers on the NCLEX-RN® exam. fell off the sofa. On examination, the nurse finds old
Remember, the NCLEX-RN® exam is testing your and new lesions on the
ability to think critically. Critical thinking child’s buttocks. Which of the following statements
for the nurse involves the following: most appropriately reflects
• Observation how the nurse should document these findings?
• Deciding what is important 1.
• Looking for patterns and relationships 2.
• Identifying the problem 3.
• Transferring knowledge from one situation to 4.
another We omitted the answer choices to make you focus on
• Applying knowledge the question stem this time. The answer
• Discriminating between possible choices and/or choices will be provided and discussed later in this
courses of action chapter.
• Evaluating according to criteria established Step 1. Read the question stem carefully.
Are you feeling overwhelmed as you read these Step 2. Pay attention to the adjectives. Most
words? Don’t be! We are going teach you a appropriately tells you that you need to select
step-by-step method to choose the appropriate path. the best answer.
The Kaplan Nursing team has developed Step 3. Reword the question stem in your own words.
a decision tree that shows you how to approach every In this case, it is, “What is the best
NCLEX-RN® exam question. In charting for this situation?”
this book, these strategies appear as 10 critical 42
thinking paths. NCLEX-RN® Exam Overview and Test Taking
There are some strategies that you must follow on Strategies
every NCLEX-RN® exam test question. Step 4. Because you were able to reword the question,
You must always figure out what the question is the fourth step is unnecessary. You
asking, and you must always eliminate answer didn’t need to read the answer choices for clues.
choices. We have all missed questions on a test because we
Choosing the right answer often involves choosing the didn’t read accurately. The following question
best of several answers that have correct illustrates this point.
information. This may entail your correct analysis and A construction worker is admitted to the hospital for
interpretation of what the question treatment of active
is really asking. So let’s talk about how to figure out tuberculosis (TB). The nurse teaches the client about
what the question is asking. TB. Which of the following
41 statements by the client indicates to the nurse that
2: General and Computer Adaptive Test further teaching is necessary?
Strategies 1.
Reword the Question 2.
The first step to correctly answering NCLEX-RN® 3.
exam questions is to find out what each 4.
question is really asking. Again, just the question stem is given to encourage
Step 1. Read each question carefully from the first you to focus on rewording the question.
word to the last word. Do not skim over We will discuss the answer choices for this question
the words or read them too quickly. later in this chapter.
Step 2. Look for hints in the wording of the question Step 1. Read the question stem carefully.
stem. The adjectives most, first, best, Step 2. Look for hints. Pay particular attention to the
primary, and initial indicate that you must establish statement “further teaching is necessary.”
priorities. The phrase further teaching You are looking for negative information.
is necessary indicates that the answer will contain Step 3. Reword the question stem in your own words.
incorrect information. The phrase client In this case, it is, “What is incorrect
understands the teaching indicates that the answer information about TB?”
will be correct information. Step 4. Because you were able to reword the question,
Step 3. Reword the question stem in your own words the fourth step is unnecessary. You
so that it can be answered with a yes didn’t need to read the answer choices for clues to
determine what the question is asking.
Try rewording this test question. A preschooler with a fractured femur is brought to the
A woman admitted to the hospital in premature labor emergency room by her
has been treated parents. When asked how the injury occurred, the
successfully. The client is to be sent home on an oral child’s parents state that she
regimen of terbutaline. fell off the sofa. On examination, the nurse finds old
Which of the following statements by the client and new lesions on the
indicates to the nurse that the child’s buttocks. Which of the following statements
client understands the discharge teaching about the most appropriately reflects
medication? how the nurse should document these findings?
1. 1. “Six lesions noted on buttocks at various stages of
2. healing.”
3. 2. “Multiple lesions on buttocks due to child abuse.”
4. 3. “Lesions on buttocks due to unknown causes.”
43 4. “Several lesions on buttocks caused by cigarettes.”
2: General and Computer Adaptive Test THE REWORDED QUESTION: “What is good charting?”
Strategies Step 1. Do not look at any of the answer choices
Again, just the question stem is given to encourage except for answer choice (1). Thoughtfully
you to focus on rewording the question. consider each answer choice individually.
We will discuss the answer choices for this question Step 2. Read answer choice (1). Does it answer the
later in this chapter. question, “What is good charting for this
Step 1. Read the question stem carefully. situation?”
Step 2. Look for hints. Pay attention to the words (1) “Six lesions noted on buttocks at various stages of
client understands. You are looking for healing.” Is this good charting? Maybe.
true information. Leave it in for consideration.
Step 3. Reword the question stem. This question is Step 3. Repeat the process with each remaining
asking, “What is true about terbutaline answer choice.
(Brethine)?” (2) “Multiple lesions on buttocks due to child abuse.”
Step 4. Because you were able to reword this Is this good charting? No, because the
question, the fourth step is unnecessary. You nurse is making a judgment about the cause of the
didn’t need to obtain clues about what the question is lesions.
asking from the answer choices. (3) “Lesions on buttocks due to unknown causes.” Is
Eliminate Incorrect Answer Choices this good charting? Maybe. Leave it in
Now that you’ve mastered rewording the question, for consideration.
let’s examine how to select the correct (4) “Several lesions on buttocks caused by
answer. cigarettes.” Is this good charting? No. The question
Remember the characteristics of unsuccessful test does not include information about how the lesions
takers? One of their major problems is that occurred.
they do not thoughtfully consider each answer choice. Step 4. Answer choices (1) and (3) remain.
They react to questions using feelings Step 5. Reread the question to make sure you have
and hunches. Unsuccessful test takers look for a correctly identified THE REWORDED
specific answer choice. The following strategy QUESTION. This question asks you to identify good
will enable you to consider each answer choice in a charting.
thoughtful way. Step 6. Which is better charting? “Six lesions noted on
Step 1. Do not look at any of the answer choices buttocks at various stages of healing,”
except answer choice (1). or “Lesions due to unknown causes”? Good charting is
Step 2. Read answer choice (1). Then repeat THE accurate, objective, concise, and
REWORDED QUESTION after reading complete. It must reflect the client’s current status.
the answer choice. Ask yourself, “Does this answer The correct answer is (1).
THE REWORDED QUESTION?” If you Some students will select answer (3), thinking, “How
know the answer choice is wrong, eliminate it. If you can I be sure about the stages of
aren’t sure, leave the answer choice in healing?” But the purpose of this question is to test
for consideration. your ability to select good charting.
Step 3. Repeat the above process with each remaining 45
answer choice. 2: General and Computer Adaptive Test
Step 4. Note which answer choices remain. Strategies
Step 5. Reread the question to make sure you have Select the answer choice that shows you are a safe
correctly identified THE REWORDED and effective nurse. Remember, questions
QUESTION. on the NCLEX-RN® exam are not designed to trick
Step 6. Ask yourself, “Which answer choice best you. Stay focused on the question.
answers the question?” That is your answer. Let’s select the correct answer for the second
44 question.
NCLEX-RN® Exam Overview and Test Taking A construction worker is admitted to the hospital for
Strategies treatment of active
Let’s practice the elimination strategy using the same tuberculosis (TB). The nurse teaches the client about
questions. TB. Which of the following
statements by the client indicates to the nurse that Which of the following statements by the client
further teaching is necessary? indicates to the nurse that the
1. “I will have to take medication for 6 months.” client understands the discharge teaching about the
2. “I should cover my nose and mouth when coughing medication?
or sneezing.” 1. “As long as I take my medication, I can be sure I
3. “I will remain in isolation for at least 6 weeks.” will not deliver
4. “I will always have a positive skin test for TB.” prematurely.”
THE REWORDED QUESTION: What is incorrect 2. “It is important that I count the fetal movements
information about TB? for one hour, twice a day.”
Step 1. Do not look at any of the answer choices 3. “I may feel a rapid heartbeat and some muscle
except answer choice (1). tremors while on this
Step 2. Read answer choice (1). Does it answer THE medication.”
REWORDED QUESTION, “What is 4. “Bed rest is necessary in order for the medication
incorrect (or wrong) information about TB?” to work properly.”
(1) “I will have to take medication for 6 months.” Is THE REWORDED QUESTION: What is true about
this wrong information? No, it is a terbutaline (Brethine)?
true statement. The client will need to take a Step 1. Do not look at any of the answer choices
medication, such as isonicotinyl hydrazine except answer choice (1).
(INH), for 6 months or longer. Eliminate this choice. Step 2. Read answer choice (1). Does it answer the
Step 3. Repeat the process with each remaining question, “What is true about terbutaline?”
answer choice. (1) “As long as I take my medication, I won’t deliver
(2) “I should cover my nose and mouth when prematurely.” Is this true about terbutaline?
coughing or sneezing.” Is this wrong information No. Terbutaline will inhibit uterine contractions, but
about TB? No, this is a true statement. TB is there is no guarantee that
transmitted by droplet contamination. there won’t be a premature delivery. Eliminate it.
Eliminate it. Step 3. Repeat the process with each remaining
(3) “I will remain in isolation for at least 6 weeks.” Is answer choice.
this wrong information about TB? 47
Maybe. Leave it in for consideration. 2: General and Computer Adaptive Test
(4) “I will always have a positive skin test for TB.” Is Strategies
this a wrong statement about TB? No, (2) “It is important that I count the fetal movements
this is true. A positive skin test indicates that the client for one hour, twice a day.” Is this true
has developed antibodies to the about terbutaline? Maybe. Clients are told to be aware
tuberculosis bacillus. Eliminate this choice. of fetal movement. Keep it as a
Step 4. Only answer choice (3) remains. possibility.
Step 5. Reread the question to make sure you have (3) “I may feel a rapid heartbeat and some muscle
correctly identified THE REWORDED tremors while on this medication.” Is this
QUESTION. The question is, “What is incorrect true of terbutaline? Yes. Terbutaline is a smooth-
information about TB?” muscle relaxant. Side effects include
46 increased maternal heart rate, palpitations, and
NCLEX-RN® Exam Overview and Test Taking muscle tremors. Leave this choice in for
Strategies consideration.
Step 6. The correct answer is (3). You “know” this is (4) “Bed rest is necessary in order for the medication
the correct answer because you’ve eliminated to work properly.” Is this true about
the other three answer choices. The client does not terbutaline? No. Terbutaline will work whether the
need to be isolated for 6 weeks. The client is on bed rest or not. Eliminate
client’s activities will be restricted for about 2–3 weeks it.
after medication therapy is initiated. Step 4. Note that only answer choices (2) and (3)
A few things to remember when using this strategy: remain.
• Eliminate only what you know is wrong. However, Step 5. Reread the question to make sure you have
once you eliminate an answer choice, correctly identified THE REWORDED
do not retrieve it for consideration. You may be QUESTION. The reworded question is, “What is true
tempted to do this if you do not feel about Brethine?”
comfortable with the one answer choice that is left. Step 6. Which choice best answers the question, (2)
Resist the impulse! or (3)? If you are focused on the question,
• Stay focused on THE REWORDED QUESTION. How you will select (3). Some students focus on the
many times have you missed background information (pregnancy). This
a question that asked for negative information question has nothing to do with pregnancy. If you
because you selected the answer choice chose (2), you fell for a distracter.
that contained correct information? Remember: Focus on the question, and not the
Here’s another question. background information. If you can answer
A woman admitted to the hospital in premature labor the question—“What is true about Brethine?”—without
has been treated considering the background information
successfully. The client is to be sent home on an oral (pregnancy), do it. Many students answer a question
regimen of terbutaline. incorrectly because they don’t
focus on THE REWORDED QUESTION. Don’t fall for the for glucose and ketones. It is not relevant to a clean-
distracters. catch urine specimen. Eliminate.
At this point you’re probably thinking, “Will I have (3) This is true of a clean-catch urine specimen for
enough time to finish the test using culture and sensitivity. The urinary
these strategies?” or “How will I ever remember how meatus is cleansed, a sterile container is used, and
to answer questions using these steps?” the penis must not touch the container.
Yes, you will have time to finish the test. Unsuccessful Leave it in for consideration.
test takers spend time agonizing (4) This does describe a clean-catch urine specimen.
over test questions. By using these strategies, you will The client does void a few drops of
be using your time productively. You urine, stops, and then continues voiding into the
will remember the steps because you are going to container. There is only one problem.
practice, practice, practice with test questions. For a culture and sensitivity, the container must be
You will not be able to absorb this strategy by sterile. Eliminate.
osmosis; the process must be practiced The correct answer is (3). Many students will select
repeatedly. answer choice (4) because they see the
Don’t Predict Answers expected words: “Void a few drops, then stop;
On the NCLEX-RN® exam, you are asked to select the continue voiding.” Be careful. This question is
best answer from the four choices that a good example of why scanning for expected words
you are given. Many times, the “ideal” answer choice could get you into trouble. You may see
is not there. Don’t sit and moan because expected words in an answer choice that is not
the answer that you think should be there isn’t correct.
provided. Remember: 49
• Identify THE REWORDED QUESTION. 2: General and Computer Adaptive Test
• Select the best answer from the choices given. Strategies
48 Okay. You’ve practiced how to identify the topic of the
NCLEX-RN® Exam Overview and Test Taking question and how to eliminate answer
Strategies choices. You know that predicting answers does not
Look at this question. work on the NCLEX-RN® exam. You are
The nurse describes the procedure to a male client for well on your way to correctly answering NCLEX-RN®
collecting a clean-catch exam test questions. Unfortunately,
urine specimen for culture and sensitivity testing. this is just the starting point. Let’s talk about specific
Which of the following paths and how you can correctly decide
explanations by the nurse would be most accurate? which paths to use on the NCLEX-RN® exam.
1. “The urinary meatus is cleansed with an iodine Remember, the correct answer is at the end
solution and then a urinary of the path!
drainage catheter is inserted to obtain urine.” Recognize Expected Outcomes
2. “You will be asked to empty your bladder one-half Correct
hour before the test; you Expected Answer
will then be asked to void into a container.” Outcomes
3. “Before voiding, the urinary meatus is cleansed 12
with an iodine solution and What is the
urine is voided into a sterile container; the container expected
must not touch the outcome?
penis.” What is the
4. “You must void a few drops of urine, then stop; best action for
then void the remaining urine the expected
into a clean container, which should be immediately outcome?
covered.” You spent much of your time in nursing school
Step 1. Read the question stem. learning about what might go wrong with
Step 2. Focus on the adjectives. “Most accurate” tells clients and their care. This makes sense; after all,
you that more than one answer may nurses need to deal with problems and
seem correct. illnesses. Many test questions that your nursing school
Step 3. Reword the question stem. What is true about faculty wrote focused on what was
a clean-catch urine specimen for culture wrong with clients and their care. In order to prove
and sensitivity? minimum competence, the beginning
Step 4. Read each answer choice and ask yourself, “Is practitioner must demonstrate the ability to make
this true about a clean-catch urine appropriate nursing judgments. Competent
specimen for culture and sensitivity?” nursing judgments include recognizing both expected
(1) This choice describes how to obtain a catheterized and unexpected behaviors, so it
urine specimen. Urine isn’t usually is important for you to recognize expected outcomes
collected by catheterization due to the increased risk on the NCLEX-RN® exam. Expected
of infection. This answer does not outcomes are the behaviors and changes you think
answer the question about a clean-catch urine are going to occur as a result of nursing
specimen. Eliminate. care. These outcomes allow the nurse to evaluate
(2) This describes a double-voided specimen. This whether goals have been met.
action is usually done when testing urine Look at the following question.
The physician orders an arterial blood gas (ABG) for a 2. Place the client in semi-Fowler’s position, and
client receiving oxygen at administer O2 at 4 L.
6 L/min. Results show pH 7.37, HCO3 26 mm Hg, 3. Administer a second dose of nitroglycerin.
pCO2 42 mm Hg, pO2 90 mm Hg. 4. Document the results, and continue to monitor the
Which of the following should the nurse do first? client.
1. Increase the rate of oxygen flow the client is 51
receiving. 2: General and Computer Adaptive Test
2. Elevate the head of the bed. Strategies
3. Document the results in the chart. THE REWORDED QUESTION: What should you do for
4. Instruct the client to cough and deep-breathe. this client? To answer this question
If this question were included on one of your you need to know what these vital signs indicate.
medical/surgical tests, you would assume that Step 1. Recognize normal. Nitroglycerin is a potent
a problem was being described. So you would choose vasodilator with anti-anginal, antiischemic,
an answer choice that involves “fixing” and antihypertensive actions. It increases blood flow
the problem. Let’s look at this question. through the coronary arteries.
50 Side effects include orthostatic hypotension,
NCLEX-RN® Exam Overview and Test Taking tachycardia, dizziness, and palpitations. A
Strategies decreased blood pressure, increased pulse, and stable
THE REWORDED QUESTION: What should you do with respirations after administration of a
a client with these ABGs? potent vasodilator are normal and expected.
Step 1. Recognize normal. Interpret the ABGs. All are Step 2. Decide how you should use this information.
within normal limits. The question should be reworded as,
Step 2. Decide how you should use this information. “What should you do for a client who has responded
Because they are all normal, let’s reword as expected to a dose of nitroglycerin?”
the question again using this information. ANSWERS:
Now THE REWORDED QUESTION is: What should you (1) “Notify the physician that the client has become
do for a client with normal ABGs? hypotensive and obtain an order to
ANSWERS: administer IV fluids.” The blood pressure has
(1) “Increase the rate of oxygen flow the client is decreased due to vasodilatation. Decreased
receiving.” This is unnecessary because his blood pressure is expected. Eliminate.
O2 is within normal limits. Eliminate. (2) “Place the client in semi-Fowler’s position and
(2) “Elevate the head of the bed.” This is unnecessary administer O2 at 4 L.” Respirations are
because the ABGs are within normal stable and there is no indication of respiratory
limits. Eliminate. distress. Eliminate.
(3) “Document the results in the chart.” This action (3) “Administer a second dose of nitroglycerin.” The
should be done because the ABGs are nurse should assess the client for chest
normal. pain first, and administer a second dose of the
(4) “Instruct the client to cough and deep-breathe.” medication only if the client continues to
This is usually recommended in a situation complain of chest pain. Eliminate.
in which there is some limitation of respiratory (4) “Document the results and continue to monitor the
function, due to immobility or postoperative client.” This is the correct choice
conditions, for example. The only information you are because you recognized the client’s response as
given in this question is normal, thus eliminating the other three
the client’s ABGs, which are within normal limits. answer choices.
Although this could be done, you are The correct answer is (4). You would expect a client’s
given no indication that it is necessary. Eliminate. blood pressure to decrease after administration
The correct answer is (3). The ABGs are within normal of nitroglycerin. The key to this question is
limits. Some students select answer understanding how the medication
choice (2) because they think there’s something they works, and correctly identifying the expected
missed, or it must be a trick question. outcome.
The “trick” is deciding whether the information that Read Answer Choices to Obtain Clues
you are given is normal or abnormal, Because the NCLEX-RN® exam is testing your critical
and then answering the question accordingly. thinking, the topic of the questions
Try this question. may be unstated. You may see a question that
A client is brought to the emergency room concerns a disease process or procedure with
complaining of pressure in her chest. which you are unfamiliar. Most test takers who are
Her blood pressure is 150/90, pulse 88, respirations “clueless” about a question will read the
20. The nurse administers question and answer choices over and over again.
nitroglycerin 0.4 mg sublingually as ordered. After five They do this because they hope that:
minutes her blood • They will remember seeing the topic in their notes
pressure is 100/60, pulse 96, respirations 20. Which or on a textbook page.
of the following should the • The light will dawn and they will remember
nurse do next? something about the topic.
1. Notify the physician that the client has become • They believe there is some clue in the question that
hypotensive, and obtain an will point them toward the correct
order to administer IV fluids. answer.
52 (1) “Hold your regular dose of insulin.” This is an
NCLEX-RN® Exam Overview and Test Taking implementation that would increase the
Strategies blood glucose level. The nurse should assess first.
What usually happens? Absolutely nothing! The Eliminate.
student then randomly selects an answer (2) “Check your blood glucose level every 3–4 hours.”
choice. When you randomly select an answer, you This is an assessment. Before you can
have 1 chance in 4 of getting it right. You advise the client, you must identify whether the client
can better those odds, and here’s how: When you is hypoglycemic or hyperglycemic.
encounter a question that deals with unfamiliar Keep this answer for consideration.
nursing content, look for clues in the answer choices (3) “Increase your consumption of foods containing
instead of in the question stem. simple sugars.” This is an implementation
If you find yourself “clueless” after you carefully read and would increase the client’s blood glucose level.
a question, follow these steps: The nurse should assess first. Eliminate.
Step 1. Resist the impulse to read and reread the (4) “Increase your activity level.” This is an
question. Read the question only once. implementation that would decrease the client’s
Identify the topic of the question. It is often unstated. blood glucose level. The nurse should assess first.
Step 2. Read the answer choices, not to select the Eliminate.
correct answer but to figure out, “What is The nurse should always assess before implementing
the topic of the question?” or “What should I be nursing care. The correct answer is (2).
thinking?” You are looking for clues from No matter how much you prepare for the NCLEX-RN®
the answer choices. exam, there may be topics you see on
Step 3. After reading the answer choices, reword the your test with which you are unfamiliar. Reading the
question using the clues that you have answer choices for clues will increase
obtained. your chances of selecting a correct answer.
Step 4. Then use the strategies previously discussed Remember, you do have a body of knowledge. You
to answer the question you have formulated. just have to be calm and access this knowledge.
Question? Read this question.
Correct A client is being treated for Addison’s disease. The
3 Answer physician orders cortisone 25
2 mg PO daily. The nurse should explain to the client
1 that adjustment of the dosage
Read the may be required in which of the following situations?
stem one 1. Dosage is increased when the blood glucose level
time. increases.
Read answer 2. Dosage is decreased when dietary intake is
choices for increased.
clues to topic. 3. Dosage is decreased when infection stimulates
Reword question endogenous steroid secretion.
using clues from 4. Dosage is increased relative to an increase in the
answer choices. level of stress.
Let’s try this strategy with a question. Not sure what Addison’s disease is? Not sure how to
A client contacts his home care nurse with complaints adjust the dose of cortisone?
of nausea and abdominal Step 1. Read the question once. Resist the impulse to
pain. He has type 1 diabetes. The nurse should advise reread the question.
the client to do which of Step 2. Read the answer choices. What should you be
the following? thinking? The question concerns cortisone.
1. “Hold your regular dose of insulin.” If the client is receiving cortisone, Addison’s disease
2. “Check your blood glucose level every 3–4 hours.” must be something that requires
3. “Increase your consumption of foods containing cortisone, a hormone from the adrenal glands. You
simple sugars.” notice that dosages are both increased
4. “Increase your activity level.” and decreased.
Step 1. Read the stem of the question. Can you Step 3. Use these clues to reword the question: “What
identify the topic of the question? No, you is true about adjusting cortisone
can’t. The nurse is telling the client to do something, dosage?”
but about what topic? The topic is 54
unstated in the question. NCLEX-RN® Exam Overview and Test Taking
Step 2. Read the answer choices to obtain clues about Strategies
the topic of the question. Each answer Step 4. Consider each answer choice. Does it answer
choice deals with ways to maintain a normal blood THE REWORDED QUESTION?
sugar. (1) Dosage is increased when the blood glucose level
53 increases. Is this true about cortisone?
2: General and Computer Adaptive Test No. This sounds like insulin. Eliminate.
Strategies (2) Dosage is decreased when dietary intake is
Step 3. Reword the question: “What does the nurse increased. Is this true about cortisone? No.
tell the client about ‘sick day rules’?” Cortisone requirements are not related to diet.
ANSWERS: Eliminate.
(3) Dosage is decreased when infection stimulates ANSWERS:
endogenous steroid secretion. Endogenous (1) “Do your ankles swell at the end of the day?” Why
means “within the client.” If the client is receiving would you ask a client this question?
cortisone for Addison’s disease, he must Because edema is a symptom of right-sided heart
have adrenal insufficiency. Therefore, infection can’t failure. Is right-sided failure your priority?
stimulate steroid secretion. Eliminate. No, left-sided failure takes priority because it affects
The correct answer is (4) because it is the only choice the lungs. Eliminate this answer.
remaining. Even if you are not confident (2) “Where do you sleep at night?” Why would you ask
that cortisone is increased during periods of stress, a client this question? If he is sleeping
you can conclude that this is the correct in his bed, his breathing is not compromised. If he has
answer because the other choices have been to sleep in his recliner, he is having
eliminated. orthopnea. Orthopnea is a symptom of left-sided
If you’re not sure about the topic of the question, read failure, and this would be a priority.
the answer choices for clues. Keep this answer for consideration.
Let’s look at another path. (3) “How do you feel after you eat dinner?” Why would
Correct you ask a client this question? Bloating
Answer after meals is a symptom of right-sided failure. This is
Read the not as important as breathing
stem. problems. Eliminate this answer.
Identify (4) “Do you have chest pain when you inhale?” Why
the topic. Read the would you ask a client this question? It
answer does indicate a breathing problem. The student who
choices. reacts rather than thinks may select
Identify the this answer. Pain on inspiration may indicate irritation
nursing concept of the parietal pleura of the lung,
contained in which is not associated with heart failure. Eliminate
answer choices. this answer.
4 The correct answer is (2). In order to select this
3 answer, you must recognize that “Where do
2 you sleep at night?” represents orthopnea. The
Answers? 1 NCLEX-RN® exam can take important concepts
In some questions, the NCLEX-RN® exam asks you to such as this, and “hide” the concept in some fairly
figure out the topic of the question. In simple behaviors.
other questions you are required to use critical Let’s try another question where you have to figure
thinking skills to figure out what the answer out what the answer choices really mean.
choices really mean. The NCLEX-RN® exam can take The nurse is caring for a client immediately after a
a concept with which you are very paracentesis. It is most
familiar and make it difficult to recognize. The important for the nurse to ask which of the following
following question illustrates this point. questions?
A client with a history of heart failure visits the clinic. 1. “Do your clothes still feel tight?”
He states, “I have not been 2. “Do you need to void?”
feeling like my old self for about 2 weeks.” It would 3. “Are you feeling dizzy?”
be most important for the 4. “Do you have any pain?”
nurse to ask which of the following questions? NCLEX-RN® Exam Overview and Test Taking
1. “Do your ankles swell at the end of the day?” Strategies
2. “Where do you sleep at night?” 56
3. “How do you feel after you eat dinner?”
4. “Do you have chest pain when you inhale?” Step 1. Read the stem of the question.
It is not difficult to identify the topic of this question, Step 2. Reword the question in your own words.
“What is a priority for a client with Step 3. Read the answer choices.
heart failure?” Many students get tripped up on this Step 4. Think: “What nursing concept should I identify
question by not thinking through the in the answer choices?”
answers as carefully as they should. In some THE REWORDED QUESTION: What is the highest
questions, you have to figure out the topic of the priority for a client after a paracentesis?
question. In this question, you have to figure out what ANSWERS:
the answer choices mean. (1) “Do your clothes still feel tight?” Why would you
55 ask a client this question? Clothes
2: General and Computer Adaptive Test should fit looser because the abdominal girth has
Strategies decreased after fluid has been removed
Step 1. Read the stem of the question. with a paracentesis. This is an expected outcome.
Step 2. Reword the question in your own words. Eliminate.
Step 3. Read the answer choices. (2) “Do you need to void?” Why would you ask a client
Step 4. Think: “What nursing concept should I identify this question? It is imperative to
in the answer choices?” empty the bladder prior to the procedure, not after
THE REWORDED QUESTION: What is a priority for a the procedure. There is no compelling
client with heart failure? reason to ask the client this question. Eliminate.
(3) “Are you feeling dizzy?” What makes a client NCLEX-RN® Exam Overview and Test Taking
dizzy? One of the causes is a decrease in Strategies
cerebral perfusion due to a fall in blood pressure. (3) “Sounds reasonable. I have seen this done in some
Could this client have a decreased circumstances.”
blood pressure? Yes. Hypotension and hypovolemic (4) “A picture? What picture? I’ve never seen a picture
shock are complications of a paracentesis of a client in a chart!”
due to removal of a large volume of fluid. Keep this Possible conclusions drawn by this person would
answer for consideration. include: “OK, I’ve seen one nurse ask another
(4) “Do you have any pain?” You ask this question to for information so (3) must be the answer,” or “Well,
assess pain level. This client may maybe the client isn’t all that confused, so
have discomfort where the paracentesis was I’ll select (2).”
performed, but this is an expected outcome. According to nursing textbooks, asking another health
Eliminate. care professional is not the correct
The correct answer is (3). way to identify a client. Many acute-care settings now
These questions illustrate why knowing nursing include a photo of the client in the
content is not enough to answer application/ chart for just this type of situation. The correct answer
analysis-level questions. You must be able to to this question is (4). Many students
effectively use the information you learned reject this answer because there are rarely pictures of
in nursing school to answer NCLEX-RN® exam-style clients in the charts. Real-world experience
test questions. Here is a brief review of doesn’t count, though; in this case, the client does
some of the lessons you have learned in this chapter: have a picture in his chart.
• Reword the question. The NCLEX-RN® exam is a standardized exam
• Eliminate answer choices you know to be incorrect. administered by NCSBN. Because the
• Don’t predict answers. NCLEX-RN® exam is a national exam, students should
• Recognize expected outcomes. be aware that in some parts of the
• Read answer choices to obtain clues. country, nursing is practiced slightly differently.
57 However, to ensure that the test is reflective
Now that you understand what kind of questions the of national trends, questions and answers are all
NCLEX-RN® exam is going to ask, you need carefully documented. The test makers
to learn more specific strategies for success on the ensure that the correct answers are documented in at
NCLEX-RN® exam. least two standard nursing textbooks,
The NCLEX-RN® Exam Versus Real-World or in one textbook and one nursing journal.
Nursing Real World
Some of you are LPNs or LVNs completing your RN Correct
studies, while others are EMTs. Some Answer
of you worked during school as student techs. All of 1
you, however, spent time in a clinical Don’t use real-world
setting during your nursing education. All of this adds experience to
up to a significant amount of experience. answer NCLEX-RN®
Experience will help you get a job, but answering questions.
questions based on your experience 2
can be dangerous on the NCLEX-RN® exam. You have the
Look at the following question. time, the staff,
On admission to the hospital, an elderly client appears and the
disheveled and is equipment.
restless and confused. During the client’s second day 3
on the unit, a nurse Take care
approaches the client to administer medication. The of the
nurse is unable to identify client _rst.
the client because his armband is missing. Which of 4
the following actions by The NCLEX-RN®
the nurse is the best? exam tests the
1. Have the client’s roommate identify him. nurse’s judgment.
2. Ask the client to state his full name. When you are unsure of an answer choice, don’t ask
3. Ask another nurse to identify the client. yourself, “What do they do on my floor?”
4. Look in the chart at the picture of the client. but “What does the medical/surgical textbook writer
Let’s see how someone using his or her real-world Brunner say?” or “What do Potter and Perry
experience would approach this question: say to do?” This test does not necessarily reflect what
(1) “The roommate is never involved in identification happens in the real world, but is based on
of a client.” textbook nursing.
(2) “A confused client cannot be relied on for an Remember the following when taking the NCLEX-RN®
accurate identification.” exam:
NCLEX-RN® Exam Strategies • You have all of the time and resources you need to
chapter 3 provide appropriate care to your client.
58 (Checking for bowel sounds for five minutes in all four
quadrants, no problem!)
• You have all of the equipment you need. (Remember mother decides to bottle-feed her infant. Which of the
the bath thermometer you learned following statements by the
to use in the nursing lab? For the NCLEX-RN® exam, mother after a teaching session indicates to the nurse
you will have one available to test the that the client needs further
temperature of bath water.) instruction?
59 1. “I’ll pump my breasts and use warm packs to
3: NCLEX-RN® Exam Strategies relieve breast pain.”
• There are no staffing problems on the NCLEX-RN® 2. “I’ll use a tight bra and ice packs to relieve
exam. You are caring only for the engorgement discomfort.”
client described in the question, and that person is 3. “I’ll take the medication prescribed by the doctor
your only concern. for pain.”
• All care given to clients is “by the book.” No 4. “I’ll take the pills ordered by my doctor to help stop
shortcuts are used. (You would not turn off an IV the production of milk.”
solution, flush the line, give another IV solution, flush Let’s look at these answers more closely.
the line, and then restart the original (1) Pumping the breasts will stimulate milk
IV solution that was ordered to be run continuously.) production. This is clearly wrong.
Answer the following question. (2) Wearing a tight bra and using ice packs are
A client is treated in the emergency room for acute appropriate interventions for a nonbreastfeeding
alcohol intoxication. He has mother.
a five-year history of alcohol abuse. He is agitated and (3) Taking a medication (mild analgesic) is an
verbally abusive. His appropriate intervention for a nonbreastfeeding
admission orders include chlordiazepoxide 50 mg IM mother.
or PO every 4–6 hours (4) Medication to prevent lactation is not frequently
for agitation. The nurse should take which of the prescribed because of potentially dangerous
following precautions after side effects. However, a medication may be
chlordiazepoxide is administered? prescribed to prevent lactation. This would
1. Place the client in restraints. be considered an appropriate intervention.
2. Leave the client in a room by himself until the The correct answer is (1).
tranquilizer takes effect. First Take Care of the Client, Then the Equipment
3. Assign a practical nurse to stay with the client and The NCLEX-RN® exam tests your ability to use critical
assess his condition. thinking skills to make nursing
4. Ask the security guard to stay with the client. judgments. It is very important that you remember
Let’s look at this using real-world logic. to:
(1) “Place the client in restraints.” Yes, that is done in • Take care of the client first.
the real world. • Take care of the equipment second.
(2) “Leave the client in a room by himself until the Look at the following question.
tranquilizer takes effect.” Yes, that is done in A client sustains a fractured left femur in a car
the real world, but most students recognize that it is accident. She is placed in
not the best answer. balanced suspension skeletal traction using a Thomas
(3) “Assign a practical nurse to stay with the client splint and a Pearson
and assess his condition.” Sounds good, but attachment. The client tells the nurse that she has
what if you don’t have enough staffing to assign an “terrible” pain in her left
LPN/LVN to sit with this client? thigh. Which of the following should the nurse do
(4) “Ask the security guard to stay with the client.” FIRST?
Yes, in the real world, security is called when 1. Determine that all the weights and ropes from the
clients are agitated. traction apparatus are in
According to real-world logic, the correct answer must line and hanging free.
be (1) or (4). However, textbook theoretical 2. Ask the client for more information about the
nursing practice states that this client should not be location and characteristics of her
left alone while in an agitated state. A pain.
professional should remain with the client. Therefore, 3. Check the Thomas splint and Pearson attachment
the correct answer is (3). to make sure they are
Use your real-world experience to help you visualize appropriately positioned.
the client described in the test question, but 4. Explain to the client that the pain she is
select your answers based on what is found in nursing experiencing in the affected leg is a
textbooks. common occurrence.
Your nursing faculty has probably been conscientious Let’s review the answers:
about instructing you in the most upto- (1) All weights should be hanging free in balanced
date nursing practice. According to the National suspension skeletal traction. This answer
Council, the primary source for documenting choice has you checking the equipment, not the client.
correct answers is in nursing textbooks, and the most Your first concern should be the client,
up-to-date practice might not always not the traction.
agree with the textbooks. When in doubt, always (2) The nurse should focus on assessing the client and
select the textbook answer! her problem before assessing the function
A woman is admitted to the hospital and delivers a of the equipment. All complaints of pain should be
healthy 7 lb., 2 oz. girl. The thoroughly investigated by the
nurse. • How the specific gravity and hematocrit levels are
(3) This answer choice has you checking the affected by a fluid volume deficit
equipment, not the client. Your first concern should Fluid volume deficit occurs when water and
be the client, not the traction. electrolytes are lost in the same proportion as
(4) Any complaints of pain are considered abnormal, they exist in the body. When a client is dehydrated,
and you should investigate them both the specific gravity of urine and the
thoroughly. hematocrit become elevated. The correct answer is
The correct answer is (2). (2).
Laboratory Values Answer the following question.
Answering questions about lab values is another A client is hospitalized with a diagnosis of atrial
example of how the real world does not work fibrillation. Heparin 5,000 units
on the NCLEX-RN® exam. In nursing school, you is ordered every 12 hours to be given subcutaneously.
learned lab values for a specific test and you may The physician orders daily
not have remembered them after the test. While you partial thromboplastin times (PTT). The result of the
were in the clinical setting, the emphasis client’s most recent PTT is 55.
was on interpretation of lab values. Because most lab Which of the following actions should be taken by the
slips contained a listing of normal values, nurse?
you were able to compare the client’s results to the 1. Document the results and administer the heparin.
normal levels. Questions on the NCLEX-RN® 2. Withhold the heparin.
exam will not provide you with a listing of normal lab 3. Notify the physician.
values. 4. Have the test repeated.
To answer questions on the NCLEX-RN® exam, you In order to answer this question you need to know:
must: • The normal range for a PTT is 20–45 seconds.
• Know normal lab test results. • The therapeutic range for a client receiving heparin,
• Correctly interpret normal or abnormal lab test an anticoagulant, is 1.5–2 times the
results. control or normal level.
Compare the following two questions. • To calculate the therapeutic range, take the lower
A client is admitted to the hospital with flu-like number for the normal range for
symptoms. When taking the a PTT (20) and multiply it by 1.5. The result is 30.
history, the nurse learns that the client has been Multiply the higher number (45) by 2.
taking digoxin 0.125 mg PO The result is 90. Thus the therapeutic range goes from
daily and furosemide 40 mg PO daily for 3 years. Last 30 to 90. If a PTT reading is between
month her physician those endpoints, no medication is needed.
changed the prescription for digoxin to 0.25 mg qd. Evaluate the answer choices:
The nurse would expect the (1) “Document the results and administer the
physician to order which of the following laboratory heparin.” The client’s most recent PTT is 55. This
tests? is within the therapeutic range of 30 to 90, so no
1. Serum electrolytes and digoxin level medication should be given.
2. White blood cell count, hemoglobin, and hematocrit (2) “Withhold the heparin.” The PTT level is within
3. Cardiac enzymes and an arterial blood gas what is considered an effective therapeutic
4. Blood cultures and urinalysis level. This client does not need the anticoagulant.
You are probably familiar with the concepts presented (3) “Notify the physician.” There is no reason to notify
in this question. The physician has the physician. The client has reached the
increased the client’s dose of digoxin. Furosemide is a therapeutic level of heparin.
potassium-wasting diuretic. The client (4) “Have the test repeated.” There is no reason to
will likely develop digitalis toxicity if she has a low have the test repeated. The client has achieved
potassium level. Serum electrolytes and the therapeutic level.
digoxin level (1) is the correct answer. The correct answer is (2).
Now look at this question. Medication Administration
The nurse plans care for a teenager admitted with An important function in providing safe and effective
complaints of fever, vomiting, care to clients is the administration of medications.
and diarrhea. The nurse writes the following nursing Because this is one of the responsibilities of a
diagnosis on the client’s care beginning practitioner, questions about
plan: “fluid volume deficit.” Which of the following medications are often an important part of the NCLEX-
changes in laboratory values RN® exam. The nurse who is minimally
would demonstrate an improvement in the client’s competent is knowledgeable about medications and
condition? uses the “six rights” when administering
1. Urine specific gravity, 1.015; hematocrit, 37% medication.
2. Urine specific gravity, 1.020; hematocrit, 45% In nursing school, most questions about medication
3. Urine specific gravity, 1.032; hematocrit, 52% followed the same pattern. You were told
4. Urine specific gravity, 1.025; hematocrit, 35% the client’s diagnosis and the name of the medication,
In order to correctly answer this question, you must and then were asked a question. Even if
know: you didn’t know the information about the medication,
• The specific gravity of urine (1.010–1.030) and the sometimes you were able to select the
normal levels of hematocrit (male correct answer by knowing the diagnosis.
42–50%, female 40–48%)
The NCLEX-RN® exam does not give you any clues for the NCLEX-RN
from the context of the question. The questions ® exam.
on this exam include the name of the medication, Notify the Physician
almost always identifying it by both trade Another behavior that commonly occurs in the real
and generic names. Most of the time, you will not be world is calling the physician. In nursing
given the reason the client is receiving the school you were encouraged to notify your instructor
medication. of changes in your client’s condition.
Let’s look at some medication questions. Be very careful how you handle this on the NCLEX-
The physician orders furosemide and spironolactone RN® exam. More often than not,
for a client. Prior to the answer choice that states “call the physician,”
administering the medication, the nurse determines “contact the social worker,” or “refer to
that the client’s potassium the chaplain” is the WRONG answer. Usually there is
is 3.2 mEq/L. In addition to notifying the physician, something you need to do first before
the nurse should anticipate you make that call. The NCLEX-RN® exam does not
taking which of the following actions? want to know what the physician is
1. Do not administer the furosemide or going to do. The NCLEX-RN® exam wants to know
spironolactone. what you, the registered professional
2. Administer the spironolactone only. nurse, will do in a given situation.
3. Administer the furosemide only.
4. Administer the furosemide and spironolactone.
This is a typical exam-style medication question. The A client is receiving packed red blood cells. Several
question concerns the side effects and minutes after the infusion
nursing implications of furosemide and is started, the client complains of itching and develops
spironolactone. hives on his chest and
(1) The potassium level is below normal (3.5–5.0 abdomen. Which of the following actions should the
mEq/L). Furosemide is a potassium-wasting nurse take FIRST?
diuretic. spironolactone is a potassium-sparing 1. Slow the rate of the transfusion.
diuretic. There is no reason to hold the 2. Call the physician for an order for an antihistamine.
spironolactone because the client has a low potassium 3. Mix IV fluid with the blood to dilute it.
level. Eliminate this answer. 4. Stop the transfusion.
(2) The spironolactone should be administered. THE REWORDED QUESTION: What should you do first
(3) Do not administer the furosemide because it is a for this client?
potassium-wasting diuretic. The client’s It sounds like the client is having an allergic reaction
potassium level is already low. Eliminate. to the transfusion. If this is what’s going
(4) Do not administer the furosemide. Eliminate. on, what should you do?
The correct answer is (2). (1) If the client is having a transfusion reaction,
Let’s try this next question. slowing the rate of the transfusion is not the
A client returns to the clinic 2 weeks after being right action.
started on allopurinol 200 mg (2) Antihistamines are given for allergic reactions. The
PO daily. The nurse reviews information about this doctor needs to be notified. This
medication with the client. answer might be a possibility, but is there something
Which of the following statements by the client you should do first?
indicates that the teaching was (3) Mixing IV fluids with blood is done to decrease the
effective? viscosity of RBCs. This doesn’t have anything
1. “I should take my medication on an empty to do with an allergic transfusion reaction. Eliminate.
stomach.” (4) If the client was having a transfusion reaction, the
2. “I should take my medication with orange juice.” best action is to stop the transfusion. This
3. “I should increase my intake of protein.” is the correct action to take first, before the physician
4. “I should drink at least 8 glasses of water every is called.
day.” The correct answer is (4). After the transfusion is
To answer this question you need to know information stopped, you will contact the physician and
about allopurinol, an antigout agent antihistamines will probably be ordered.
that reduces uric acid. Before you want to choose the answer choice that
(1) Allopurinol is best tolerated with or immediately involves “call the physician,” look at the other
after meals to reduce gastrointestinal answer choices very carefully. Make sure that there
(GI) irritation. Eliminate. isn’t an answer that contains an assessment
(2) Orange juice makes the urine acidic. Allopurinol is or action you should do before making the phone call.
more soluble in alkaline urine. The test makers want to know what you
Eliminate. would do in a situation, not what the doctor would do!
(3) It is not necessary to increase the intake of protein 67
when taking allopurinol. Eliminate. 3:Here is one more real-world question.
(4) Allopurinol can cause renal calculi. The client Upon returning from lunch, the nurse is approached in
should drink 3,000 mL/day to reduce the the elevator by a hospital
risk of kidney stone formation. employee from another unit. The employee states that
The correct answer is (4). You must know the side her close friend is a client
effects and nursing implications of medications
on the nurse’s unit. The employee asks how her friend what you observe. You must complete an assessment
is doing and if all of her tests before you analyze, plan, and implement
were normal. The nurse should do which of the nursing care. The correct answer is (3).
following? The following situation might sound familiar: You are
1. Answer the employee’s questions softly so other called to a client’s room by a family
people on the elevator will not member and find the client lying on the floor. He is
hear. bleeding from a wound on the forehead,
2. Refuse to discuss her friend’s medical condition. and his indwelling catheter is dislodged and hanging
Suggest that she visit her from the side of the bed. Where do you
friend. begin? Do you call for help? Do you return him to bed?
3. Give the employee the name of the client’s Do you apply pressure to the cut? Do
physician to call for this information. you reinsert the catheter? Do you call the doctor?
4. Tell the employee about the results of the client’s What do you do first? This is why establishing
tests because they were priorities is so important.
within normal limits. Your nursing faculty recognized the importance of
THE REWORDED QUESTION: What should a nurse do teaching you how to establish priorities.
when asked about a client by a hospital They required you to establish priorities both in clinical
employee? situations and when answering test
(1) Discussing client information in a public place is a questions. These are the type of questions that
breach of confidentiality. Eliminate. nursing students find most controversial.
(2) Refusing to discuss a client’s medical condition Here is an example of a nursing school test question:
does not violate the client’s right to privacy Which of the following would most concern the nurse
and confidentiality. Keep in consideration. during a client’s recovery from
(3) Providing any information about a client to surgery?
someone not directly involved in the client’s care (1) Safety
is a breach of privacy. Eliminate. (2) Hemorrhage
(4) It is a breach in the client’s right to privacy to (3) Infection
share information with others without the client’s (4) Pain control
permission. Eliminate. A conversation in class with your instructor may then
The correct answer is (2). go something like this:
Expect to see real-world situations on your NCLEX- Instructor: “The correct answer is (2).”
RN® exam, but make sure that you do not Student: “Why isn’t infection the correct answer? It
choose real-world answers! These strategies should says right here [pointing to textbook]
help you use your previous nursing experience that infection is a major complication after surgery.”
without encountering any pitfalls. Instructor: “Yes, infection is an important concern
Strategies for Priority Questions after surgery. But if the client has a lifethreatening
You will recognize priority questions on the NCLEX- hemorrhage, then the fact that the wound is infected
RN® exam because they will ask you is immaterial.”
what is the “best,” “most important,” “first,” or “initial Student: “But you can’t count this answer wrong!”
response” by the nurse. In some situations, the faculty member will give you
Take a look at this sample question. partial credit for your answer, or will
An hour after admission to the nursery, the nurse “throw the question out” because there is more than
observes a newborn baby one right answer. But you won’t get the
having spontaneous jerky movements of the limbs. opportunity to argue about questions on the NCLEX-
The infant’s mother had RN® exam. You either select the answer
gestational diabetes mellitus (GDM) during the test makers are looking for, or you get the
pregnancy. Which of the following question wrong. In the question above, all of
actions should the nurse take FIRST? the answers listed are important when caring for a
1. Give dextrose water. postoperative client, but only one answer
2. Call the physician immediately. is the best.
3. Determine the blood glucose level. The critical thinking required for priority questions is
4. Observe closely for other symptoms. for you to recognize patterns in the
As you read this question you are probably thinking, answer choices. By recognizing these patterns, you
“All of these look right!” or “How can will know which path you need to choose
I decide what I will do first?” The panic sets in as you to correctly answer the question. There are three
try to decide what the best answer is strategies to help you establish priorities on
when they all seem “correct.” the NCLEX-RN® exam:
As a registered professional nurse, you will be caring • Maslow strategy
for clients who have multiple problems • Nursing process strategy
and needs. You must be able to establish priorities by • Safety strategy
deciding which needs take precedence We will outline each strategy, describe how and when
over the other needs. You probably recognized the it should be used, and show you how to
baby’s jerky movements as an indication apply these strategies to exam-style questions. By
of hypoglycemia. Don’t forget that an important part using these strategies, you will be able to
of the assessment process is validating eliminate the second-best answer and correctly
identify the highest priority.
Strategy One: Maslow don’t eliminate it yet. Remember, Maslow states that
Maslow’s hierarchy of needs (Figure 1) is crucial to physiological needs must be met first.
establishing priorities on the NCLEX-RN® Although pain certainly has a physiological
exam. Maslow identifies five levels of human needs: component, reactions to pain are considered
physiological, safety or security, love and “psychosocial” on this exam and will become a lower
belonging, esteem, and self-actualization. priority.
Physiological Needs Step 3. Look at each of the answer choices that you
Safety and Security have not yet eliminated and ask yourself
Love and Belonging if the answer choice makes sense with regard to the
Self-esteem disease or situation described in the
Selfactualization question. If it makes sense as an answer choice, keep
Figure 1: Maslow’s Hierarchy of Needs it for consideration and go on to the
Because physiological needs are necessary for next choice.
survival, they have the highest priority and Step 4. Can you apply the ABCs?
must be met first. Physiological needs include oxygen, Look at the remaining answer choices. Can you apply
fluid, nutrition, temperature, elimination, the ABCs? The ABCs stand for airway,
shelter, rest, and sex. If you don’t have oxygen to breathing, and circulation. If there is an answer that
breathe or food to eat, you really don’t involves maintaining a patent airway, it
care if you have stable psychosocial relationships! will be correct. If not, is there a choice that involves
Safety and security needs can be both physical and breathing problems? It will be correct. If
psychosocial. Physical safety includes not, go on with the ABCs. Is there an answer
decreasing what is threatening to the client. The pertaining to the cardiovascular system? It will
threat may be an illness (myocardial infarction), be correct. What if the ABCs don’t apply? Compare the
accidents (a parent transporting a newborn in a car remaining answer choices and ask
without using a car seat), or environmental yourself, “What is the highest priority?” This is your
threats (the client with COPD who insists on walking answer.
outside in 10° F [−12° C] 4
temperatures). 3
To attain psychological safety, the client must have 2
the knowledge and understanding about Maslow 1
what to expect from others in his environment. For Correct
example, it is important to teach the client Answer
and his family what to expect after a cerebrovascular Eliminate
accident (CVA). It is also important psychosocial
that you allow a woman preparing for a mastectomy answers.
to verbalize her concerns about changes “Does this
that might occur in her relationship with her partner. make sense?”
To achieve love and belonging, the client needs to feel Apply
loved by family and accepted by others. ABCs.
When a client feels self-confident and useful, he will Recognize that
achieve the need of self-esteem as answers are
described by Maslow. both physical
The highest level of Maslow’s hierarchy of needs is and psychosocial.
self-actualization. To achieve this level, 72
the client must experience fulfillment and recognize NCLEX-RN® Exam Overview and Test Taking
his or her potential. In order for selfactualization Strategies
to occur, all of the lower-level needs must be met. Let’s apply this technique to a few sample exam-style
Because of the stresses of test questions.
life, lower-level needs are not always met, and many A woman is admitted to the hospital with a ruptured
people never achieve this high level of ectopic pregnancy. A
functioning. laparotomy is scheduled. Preoperatively, which of the
71 following goals is most
3: NCLEX-RN® Exam Strategies important for the nurse to include on the client’s plan
The Maslow Four-Step Process of care?
The first strategy to use in establishing priorities is a 1. Fluid replacement
four-step process, beginning with 2. Pain relief
Maslow’s hierarchy. To use the Maslow strategy, you 3. Emotional support
must first recognize the pattern in the 4. Respiratory therapy
answer choices. Look at the stem of the question. The words most
Step 1. Look at your answer choices. important mean:
Determine if the answer choices are both physiological • This is a priority question.
and psychosocial. If they are, apply • There probably will be more than one answer choice
the Maslow strategy detailed in Step 2. that is a correct nursing action, but
Step 2. Eliminate all psychosocial answer choices. If only one will be the most important or highest priority
an answer choice is physiological, action.
Step 1. Look at the answer choices.
You see that both physical and psychosocial (1) “Altered nutrition: more than body requirements
interventions are included. Apply Maslow. related to high-fat intake” does make
Step 2. Eliminate all psychosocial answer choices. sense. This diet is high in fat.
Answer choice (2), which is pain relief, should be (3) “Altered nutrition: less than body requirements
discarded. Remember, pain is considered related to increased nutritional demands
a psychosocial problem on the NCLEX-RN® exam. of pregnancy” also makes sense. This diet has an
Answer choice (3), emotional support, adequate number of calories, but it is
is also a psychosocial concern. Eliminate this answer. deficient in the needed vitamins and minerals.
You have now eliminated two of the (4) “Risk for injury: fetal malnutrition related to poor
possible choices. maternal diet” does not make sense. There
Step 3. Now look at the remaining answer choices and is an adequate number of calories to support fetal
ask yourself if they make sense. growth. Eliminate this choice.
Answer choice (1), fluid replacement, makes sense You have now eliminated two of the choices. Let’s go
because this client has a ruptured ectopic on.
pregnancy. An ectopic pregnancy is implantation of Step 4. Answer choices (1) and (3) remain. Can you
the fertilized ovum in a site other apply the ABCs to these choices? No.
than the endometrial lining, usually the fallopian tube. So compare the answer choices. Which is higher
Initially, the pregnancy is normal; priority: the fact that this pregnant 16-yearold’s
but as the embryo outgrows the fallopian tube, the diet contains too much fat, or that the diet does not
tube ruptures, causing extensive bleeding have enough nutrients? Insufficient
into the abdominal cavity. Answer choice (4), nutrients is a higher priority, so the correct answer is
respiratory therapy, does not make sense (3).
with a ruptured ectopic pregnancy. The obstetrical Many students, when they first read this question,
client is not likely to need respiratory choose (2), knowledge deficit. According
care prior to surgery. Eliminate this answer choice. to Maslow, physiological needs always take priority
You are left with the correct answer, (1). After reading over psychosocial needs. Using this strategy
this question, many students select answer on the NCLEX-RN® exam will enable you to choose
choices (2) or (3) as the correct answer. They justify the correct answer.
this by emphasizing the importance of managing Now, let’s try another question.
this woman’s pain, or addressing her grief about The nurse plans care for a 14-year-old girl admitted
losing the pregnancy. Neither answer with an eating disorder.
choice takes priority over the physiological demand of On admission, the girl weighs 82 lb. and is 5'4" tall.
fluid replacement prior to surgery. Lab tests indicate severe
hypokalemia, anemia, and dehydration. The nurse
Ready for another question? Try this one. should give which of the
The nurse obtains a diet history from a pregnant 16- following nursing diagnoses the highest priority?
year-old girl. The girl tells 1. Body image disturbance related to weight loss
the nurse that her typical daily diet includes cereal and 2. Self-esteem disturbance related to feelings of
milk for breakfast, pizza inadequacy
and soda for lunch, and a cheeseburger, milk shake, 3. Altered nutrition: less than body requirements
fries, and salad for dinner. related to decreased intake
Which of the following is the MOST accurate nursing 4. Decreased cardiac output related to the potential
diagnosis based on this for dysrhythmias
data? The first thing you should notice in this question stem
1. Altered nutrition: more than body requirements is the phrase “highest priority.” This
related to high-fat intake alerts you that there may be more than one answer
2. Knowledge deficit: nutrition in pregnancy that could be considered correct.
3. Altered nutrition: less than body requirements Step 1. Look at the answer choices.
related to increased nutritional Both physical and psychosocial interventions
demands of pregnancy are included. Apply the Maslow strategy.
4. Risk for injury: fetal malnutrition related to poor Step 2. Eliminate all phychosocial answer choices.
maternal diet It is easy to see that answer choice (1), body image
The first thing you should notice about this question disturbance, is a psychosocial concern.
stem is the phrase “most accurate.” This The same is true of answer choice (2), self-esteem
alerts you that there may be more than one answer disturbance. Answer choices (3) and (4) are
choice that could be considered correct. physiological. You have now eliminated all but two
Step 1. Look at the answer choices. answer choices.
You will see that both physical and psychosocial Step 3. Ask yourself whether the remaining answer
interventions are included. Apply the choices make sense.
Maslow strategy. Answer choice (3), “Altered nutrition: less than body
Step 2. Eliminate all psychosocial answer choices. In requirements related to decreased
this case, that means answer choice (2). intake,” does make sense. Remember, the client has
Knowledge deficit is a psychosocial need. anorexia, is 5'4" tall, and weighs 82 lb.
Step 3. Ask yourself whether the remaining answer Answer choice (4), “Decreased cardiac output related
choices make sense. to the potential for dysrhythmias,” also
makes sense. Dysrhythmias are a concern for a client don’t access the airway before performing mouth-to-
with severe hypokalemia, which often mouth resuscitation, your actions may be
occurs with anorexia. harmful!
You still have work to do. Implementation is the care you provide to your
Step 4. Can you apply the ABCs? Yes. clients. Implementation includes: assisting in
Decreased cardiac output is a higher priority than the performance of activities of daily living (ADLs),
altered nutrition. One answer choice counseling and educating the client and
remains: (4). the client’s family, giving care to clients, and
75 supervising and evaluating the work of other
3: NCLEX-RN® Exam Strategies members of the health team. Nursing interventions
When you first read this question, you probably may be independent, dependent, or interdependent.
identified each of the answer choices as Independent interventions are within the scope of
appropriate for a client with anorexia. Only one nursing practice and do not
nursing diagnosis can be the highest priority. require supervision by others. Instructing the client to
By using strategies involving Maslow and the ABCs, turn, cough, and breathe deeply after
you will choose the correct answer on surgery is an example of an independent nursing
your NCLEX-RN® exam. intervention. Dependent interventions are
Strategy Two: Nursing Process (Assessment versus 76
Implementation) NCLEX-RN® Exam Overview and Test Taking
A second strategy that will assist you in establishing Strategies
priorities involves the assessment and based on the written orders of a physician. On the
implementation NCLEX-RN® exam, you should assume
steps of the nursing process. As a nursing student, that you have an order for all dependent interventions
you have been drilled so that you that are included in the answer choices.
can recite the steps of the nursing process in your This may be a different way of thinking from the way
sleep—assessment, analysis, planning, you were taught in nursing school. Many
implementation, students select an answer on a nursing school test
and evaluation. In nursing school, you did have some (that is later counted wrong) because the
test questions about the nursing intervention requires a physician’s order. Everyone
process, but you probably did not use the nursing walks away from the test review muttering,
process to assist you in selecting a correct “Trick question.” It is important for you to remember
answer on an exam. On the NCLEX-RN® exam, you that there are no trick questions on the
will be given a clinical situation and asked to NCLEX-RN® exam. You should base your answer on
establish priorities. The possible answer choices will an understanding that you have a physician’s
include both the correct assessment and order for any nursing intervention described.
implementation for this clinical situation. How do you Interdependent interventions are shared with other
choose the correct answer when both the members of the health team. For instance,
correct assessment and implementation are given? nutrition education may be shared with the dietitian.
Think about these two steps of the nursing Chest physiotherapy may be shared with
process. a respiratory therapist.
Assessment is the process of establishing a data The following strategy, utilizing the assessment and
profile about the client and his or her health implementation phases of the nursing process,
problems. The nurse obtains subjective and objective will assist you in selecting correct answers to
data in a number of ways: talking to questions that ask you to identify priorities.
clients, observing clients and/or significant others, Step 1. Read the answer choices to establish a
taking a health history, performing a pattern.
physical examination, evaluating lab results, and If the answer choices are a mix of
collaborating with other members of the assessment/validation and implementation, use the
health care team. Nursing
Once you collect the data, you compare it to the Process (Assessment vs. Implementation) strategy.
client’s baseline or normal values. On the Step 2. Refer to the question to determine whether
NCLEX-RN® exam, the client’s baseline may not be you should be assessing or implementing.
given, but as a nursing student you have Step 3. Eliminate answer choices, and then choose the
acquired a body of knowledge. On this exam, you are best answer.
expected to compare the client information If after Step 2 you find that, for example, it is an
you are given to the “normal” values learned from assessment question, eliminate any answers
your nursing textbooks. that clearly focus on implementation. Then choose the
Assessment is the first step of the nursing process and best assessment answer.
takes priority over all other steps. It is Correct
essential that you complete the assessment phase of 3 Answer
the nursing process before you implement 2
nursing activities. This is a common mistake made by Nursing 1
NCLEX-RN® exam takers: don’t implement Process
before you assess. For example, when performing Recognize both
cardiopulmonary resuscitation (CPR), if you assess and
implement
answers. 1. Immobilize the affected limb with a splint and ask
Read stem to him not to move.
decide whether 2. Make a thorough assessment of the circumstances
to assess or surrounding the accident.
implement. 3. Put him in semi-Fowler’s position for comfort.
Select best 4. Check the pedal pulse and blanching sign in both
assessment or legs.
implementation. The words “ first action” tell you that this is a priority
77 question.
3: NCLEX-RN® Exam Strategies THE REWORDED QUESTION: What is the highest
Try this strategy on the next question. priority for a fractured femur?
The mother of a boy with type 1 diabetes calls the Step 1. Read the answer choices to establish a
physician’s office to discuss the pattern.
child’s self-monitoring blood glucose (SMBG) home The answer choices are a mix of
reading. He is being tightly assessment/validation and implementation. Use the
regulated with a combination of NPH and regular Nursing
insulin before breakfast and Process (Assessment vs. Implementation) strategy.
supper. The past two mornings his blood sugar Step 2. Refer to the question to determine whether
readings were 220 mg/dL and 210 you should be assessing or implementing.
mg/dL. Which of the following should the nurse tell According to the question, the nurse has determined
the boy’s mother? that the boy has a possible fracture.
1. “Continue with his medication regimen.” This implies that the nurse has completed the
2. “Check his blood sugar during the night.” assessment step. It is now time to implement.
3. “Give his NPH insulin later in the evening.” Step 3. Eliminate answer choices, and then choose the
4. “Serve his bedtime snack earlier in the evening.” best answer.
THE REWORDED QUESTION: What advice should the Eliminate answers (2) and (4) because they are
nurse give the mother about her assessments. This leaves you with choices (1)
diabetic child who is hyperglycemic in the morning? and (3). Which takes priority: immobilizing the
Step 1. Read the answer choices to establish a affected limb, or placing the boy in a semi-
pattern. Fowler’s position to facilitate breathing? The question
There is one assessment answer, (2), and three does not indicate any respiratory distress.
implementation answers, (1), (3), and (4). You The correct answer is (1), immobilize the affected
can use the Nursing Process (Assessment vs. limb.
Implementation) strategy. Some students will choose an answer involving the
Step 2. Refer to the question to determine whether ABCs without thinking
you should be assessing or implementing. it through. Students,
The child’s mother tells you that blood sugars have beware. Use the ABCs to establish priorities, but make
been elevated the last two mornings. This sure that the answer is appropriate
indicates that there is a problem. According to the to the situation. In this question, breathing was
nursing process, you should assess first. mentioned in one of the answer choices.
Step 3. Eliminate answer choices, and then choose the If you thought of the ABCs immediately without
best answer. looking at the context of the question, you
Eliminate answers (1), (3), and (4), which are would have answered this question incorrectly.
implementation answers. You are left with only 79
one answer choice, (2). This question is about the 3: NCLEX-RN® Exam Strategies
Somogyi effect, which is rebound hyperglycemia Look at this question in another form.
that occurs in response to a rapid decrease in blood A boy was riding his bike to school when he hit the
glucose during the night. Treatment curb. The boy tells the school
includes adjusting the evening diet, changing the nurse, “I think my leg is broken.” Which of the
insulin dose, and altering the amount following actions is the fir st
of exercise to prevent nocturnal hypoglycemia. Even action the nurse should take?
if you’ve never heard of the Somogyi 1. Immobilize the affected limb with a splint and ask
effect, you are still able to correctly answer this the client not to move.
question using the Nursing Process (Assessment 2. Ask the client to explain what happened.
vs. Implementation) strategy. 3. Put the client in semi-Fowler’s position to facilitate
78 breathing.
NCLEX-RN® Exam Overview and Test Taking 4. Check the appearance of the client’s leg.
Strategies In this question, the client has stated, “My leg is
Let’s look at another question. broken.” This statement is not the nurse’s
A boy was riding his bike to school when he hit the assessment.
curb. He fell and hurt his leg. This alerts the nurse that there is a problem, and the
The school nurse was called and found him alert and nurse should begin the steps of the
conscious, but in severe nursing process. The first step is assessment, so
pain with a possible fracture of the right femur. Which eliminate answers (1) and (3); these are
of the following is the implementations.
fir st action that the nurse should take?
So what takes priority? Assessment of the leg takes Answer choice (2), postural drainage, may cause
priority over an assessment of bleeding. Eliminate. Answer choice (3),
what happened to cause the accident. The correct coughing and deep-breathing, may cause bleeding.
answer is (4). Eliminate. Answer choice (4), giving ice
Strategy Three: Safety cream, may cause the child to clear his throat, causing
Nurses have the primary responsibility of ensuring the bleeding. Eliminate. The correct answer
safety of clients. This includes clients in is (1). The nurse must prevent postoperative
health care facilities, in the home, at work, and in the hemorrhage, a complication seen after this type
community. Safety includes: meeting basic of surgery. Crying would irritate the child’s throat and
needs (oxygen, food, fluids, etc.), reducing hazards increase the chance of hemorrhage.
that cause injury to clients (accidents, obstacles Let’s try another question.
in the home), and decreasing the transmission of A client is receiving intravenous cimetidine. After 20
pathogens (immunizations, sanitation). minutes of the infusion, the
Remember that the NCLEX-RN® exam is a test of client complains of a headache and dizziness. Which
minimum competency to determine that of the following actions
you are able to practice safe and effective nursing should the nurse take FIRST?
care. Always think safety when selecting 1. Stop the infusion.
correct answers on the exam. When answering 2. Take the client’s vital signs.
questions about procedures, this strategy will 3. Reposition the client.
help you to establish priorities. 4. Call the pharmacist.
123 THE REWORDED QUESTION: What should you do if a
Correct client is having side effects to a
Safety Answer medication being administered?
All answers Step 1. Are all answers implementations? Yes.
must be 81
implementations. 3: NCLEX-RN® Exam Strategies
Try to answer Step 2. Can you answer this question based on your
based on knowledge? If not, proceed to Step 3.
knowledge; Step 3. Ask yourself, “What will cause the client the
if you can’t... least amount of harm?”
What will cause (1) Stopping the infusion would not harm the client. If
the client the least the symptoms described are due to a
amount of harm? side effect of the medication, this action would help
Step 1. Are all the answer choices implementations? the client. Retain this choice.
If so, use the Safety strategy illustrated (2) Taking vital signs would not harm the client.
above. Retain it for consideration.
Step 2. Can you answer the question based on your (3) Repositioning the client would not harm the client,
knowledge? If not, continue to Step 3. but would not help the client.
Step 3. Ask yourself, “What will cause the client the Eliminate.
least amount of harm?” and choose the (4) Calling the pharmacist would not harm the client,
best answer. but would not help him. Eliminate.
80 Choices (1) and (2) are left to consider. The infusion
NCLEX-RN® Exam Overview and Test Taking may be the cause of the client’s reported
Strategies symptoms. The client’s vital signs can be taken after
Apply this strategy to the following question. the infusion is stopped. Choice (1) is the
A child undergoes a tonsillectomy for treatment of correct answer.
chronic tonsillitis unresponsive Let’s look at one more question.
to antibiotic therapy. After surgery, the child is A client is admitted with a diagnosis of dementia. He
brought to the recovery room. attempts several times to
Which of the following actions should the nurse pull out his nasogastric tube. An order for cloth wrist
include in the child’s plan of care? restraints is received by the
1. Institute measures to minimize crying. nurse. Which of the following actions by the nurse is
2. Perform postural drainage every 2 hours. most appropriate?
3. Cough and deep-breathe every hour. 1. Attach the ties of the restraints to the bed frame.
4. Give ice cream as tolerated. 2. Perform range of motion to the restrained
THE REWORDED QUESTION: What should you do extremities once a shift.
after a tonsillectomy? 3. Remove the restraints when the client is up in a
Step 1. Are all the answer choices implementations? wheelchair.
Yes. 4. Explain the need for restraints only to the family
Step 2. Can you answer the question based on your because the client is confused.
knowledge of a tonsillectomy? If not, THE REWORDED QUESTION: What is the safest way
continue to Step 3. to apply restraints?
Step 3. Ask yourself, “What will cause the client the Step 1. Are all answers implementations? Yes.
least amount of harm?” Step 2. Can you answer based on your knowledge? If
Answer choice (1), minimizing crying, will help not, proceed to Step 3.
prevent bleeding. Keep in consideration.
Step 3. Ask yourself, “What will cause the client the 1. Teaching the client about the importance of taking
least amount of harm?” lithium as prescribed
(1) Attaching the restraint ties to the bed frame will 2. Providing the client with a safe environment with
not harm the client. Retain this answer. few distractions
(2) Performing range of motion once a shift will not 3. Arranging for food and rest for the client
harm the client. However, it should be 4. Setting limits on the client’s behavior
performed more frequently. Retain this answer. 83
(3) Removing the restraints when the client is up in a 3: NCLEX-RN® Exam Strategies
wheelchair will be harmful to the client. Question 3
Restraints should not be removed when the client is The physician orders a nasogastric (NG) tube inserted
unattended. Eliminate. and connected to low
(4) Explaining the need for restraints only to the intermittent suction for a client with an intestinal
family can cause harm to the client. Restraints obstruction. Two hours after
can increase the confusion or combativeness of the insertion of the NG tube, the client vomits 200 mL.
client. Even though the client is confused, While irrigating the NG tube,
he needs to receive an explanation. Eliminate. the nurse notes resistance. Which of the following
82 actions should the nurse take
NCLEX-RN® Exam Overview and Test Taking FIRST?
Strategies 1. Replace the NG tube with a larger one.
You are now considering answer choices (1) and (2). 2. Turn the client on his left side.
What will cause the least amount of harm to 3. Change the suction from intermittent to continuous.
the client—attaching the ties of the restraint to the 4. Continue the irrigation.
bed frame, or performing range of motion to Let’s see if you were able to correctly identify which
the extremities once a shift? Range of motion should strategy you should use to determine
be performed every 2–4 hours to prevent priorities.
loss of joint mobility. Eliminate (2). The correct Question 1
answer is (1). Attaching the ties of the restraint The answer choices include both assessments and
to the bed frame will allow the nurse to raise and lower implementations. Use the Nursing Process
the side rail without injury to the client. strategy to select the correct answer.
Priority questions are an important component of the Step 1. Read the answer choices to establish a
NCLEX-RN® exam. To help you select correct pattern.
answers, think: Choices (1) and (3) are assessments; choices (2) and
• Maslow (4) are implementations.
• The Nursing Process Step 2. Refer to the question to determine whether
• Safety you should be assessing or implementing.
Answer the following three questions using the According to the situation, the client has begun to
appropriate priority strategy. The explanations choke. This alerts the nurse that there is a
follow the questions. problem. The first step of the Nursing Process is to
Question 1 assess.
The nurse cares for a client with a diagnosis of Step 3. Eliminate answer choices, and then choose the
cerebrovascular accident (CVA). best answer.
The nurse is feeding the client in a chair when he Eliminate answer choices (2) and (4) because they are
suddenly begins to choke. implementations. Now choose the best
Which of the following actions should the nurse take answer from the remaining answer choices, (1) and
FIRST? (3).
1. Check for breathlessness by placing an ear over the What takes priority—assessing for breathlessness by
client’s mouth and placing an ear over the client’s mouth,
observing the chest. or assessing the client by asking, “Are you choking?”
2. Leave the client in the chair and apply vigorous Inability to speak or cough indicates
abdominal or chest thrusts from the airway is obstructed. Breathlessness should be
behind the client. checked only in an unconscious client. The
3. Ask the client, “Are you choking?” correct answer is (3).
4. Return the client to the bed and apply vigorous Question 2
abdominal or chest thrusts Look at the answer choices. They include both
while straddling the client’s thighs. physiological and psychosocial interventions.
Question 2 Apply the Maslow strategy.
A client with a history of bipolar disorder is admitted 84
to the psychiatric hospital. NCLEX-RN® Exam Overview and Test Taking
She was found by the police attempting to climb onto Strategies
the wing of a plane at the Step 1. Look at the answer choices and identify which
airport. Her husband reports that she has not eaten are physiological—choices (2) and
or slept in 2 days, and he (3)—and which are psychosocial—choices (1) and (4).
suspects she has stopped taking lithium. On Step 2. Eliminate all psychosocial answer choices—(1)
admission, the nurse should place and (4).
the highest priority on which of the following client Step 3. Ask yourself if the remaining answer choices
care needs? make sense. Choice (2), providing the
client with a safe environment, does make sense. quality client care. Appropriate supervision of
Retain this answer. Choice (3), arranging for LPN/LVN and/or NAPs by the registered
food and rest, also makes sense. Retain this answer. professional nurse is essential for safe and effective
Step 4. Can you apply the ABCs to the remaining client care.
answer choices? No; neither choice refers To reflect these changes, the NCLEX-RN® exam
to airway, breathing, or circulation. Since the ABCs contains questions about delegation and assignment
don’t apply, ask yourself “What is the of client care. There are several reasons why you may
highest priority—providing for a safe environment, or find these questions difficult to
providing for food and rest?” According answer correctly on the NCLEX-RN® exam:
to Maslow, food and rest take highest priority. The • Many nursing schools test the content presented in
correct answer is (3). the management course with essay
Question 3 questions rather than multiple-choice questions.
This question is about a procedure: What should the • You may have received lectures regarding
nurse do when resistance is met while management of care, but your clinical rotation
irrigating an NG tube? If you are unsure about a in management may have been less than ideal.
procedure, think safety. Regardless, do not choose answers based
Step 1. Are all the answer choices implementations? on decisions you may have observed during your
Yes. clinical experience in the hospital or clinic
Step 2. Can you answer the question based on your setting. Remember, theNCLEX-RN® examisivory-
knowledge? If not, continue to Step 3. towernursing. Alwaysaskyourself,“Isthis
Step 3. Ask yourself, “What will cause the client the textbook nursing care?”
least amount of harm?” • Your experience may have been restricted to caring
(1) Replacing the nasogastric tube with a larger one for one or two clients without any
could harm the client by damaging the opportunity to supervise others, or you may have
mucosa. Eliminate. spent time on a hospital unit providing
(2) Turning the client to his left side would not hurt client care under the supervision of a preceptor.
the client. Retain this answer. Even if you have no direct experience in these areas,
(3) Changing the suction from intermittent to the Rules of Management will get you
continuous is never done because it will erode through the test. They will help you choose more right
the mucosa. Eliminate. answers when answering management
(4) Continuing the irrigation when there is resistance questions on the NCLEX-RN® exam.
might be harmful. Never force an irrigation. The Rules of Management
Eliminate. Correct
The correct answer is (2). The tip of the tube may be Answer
against the stomach wall. Repositioning the Do not
client might allow the tip to lay unobstructed in the delegate
stomach. assessment,
Using these critical thinking strategies will help you teaching,
unlock the secrets of correctly answering evaluation,
priority questions. Now let’s look at some strategies or nursing
for answering another type of question, judgement.
Management of Care. Delegate tasks
85 that involve
3: NCLEX-RN® Exam Strategies standard,
Strategies for Management of Care Questions unchanging
Every three years, the National Council conducts a job procedures.
analysis study to determine the activities Remember
required of a newly licensed registered nurse. Based priorities:
on this study, the National Council Maslow, ABCs,
adjusts the content of the test to accurately reflect and stable vs.
what is happening in the workplace. This unstable.
ensures that the NCLEX-RN® exam tests what is Delegate care
needed to be a safe and effective nurse. for stable
The role of the nurse has expanded in today’s health patients with
care environment. In addition to providing expected
quality client care, the nurse is also responsible for outcomes.
coordination and supervision of care 4
provided by other health care workers. Many health 3
care settings are staffed by registered 2
nurses, licensed practical nurses/licensed vocational Delegation 1
nurses (LPN/LVN), and nursing assistive 86
personnel (NAPs) such as nursing assistants and NCLEX-RN® Exam Overview and Test Taking
support staff. It is the responsibility of Strategies
the registered nurse to coordinate the efforts of these Rule #1: Do not delegate the functions of assessment,
health care workers to provide affordable evaluation, and nursing judgment.
During your nursing education, you learned that (3) There is no assessment, evaluation, or nursing
assessment, evaluation, and nursing judgment judgment involved in this option, so leave it
are the responsibility of the registered professional in for consideration.
nurse. You cannot give this responsibility (4) The nurse is with the child and his parents while
to someone else. the NAP obtains needed equipment. There
Rule #2: Delegate activities for stable clients with is no assessment, evaluation, or nursing judgment
predictable outcomes. when gathering equipment, so leave this
If the client is unstable, or the outcome of an activity choice in for consideration.
not assured, it should not be delegated. Step 3. Select an answer from the remaining choices.
Rule #3: Delegate activities that involve standard, You are left with answer choices (3) and (4). You are
unchanging procedures. halfway to the correct answer!
Activities that frequently reoccur in daily client care Answer (3) indicates that the nurse is on the phone
can be delegated. Bathing, feeding, and the LPN/LVN is with the client. Have
dressing, and transferring clients are examples. you seen this done in the real world? Probably. Is this
Activities that are complex or complicated what nursing textbooks and journals
should not be delegated. say should be done in this situation? Probably not.
Rule #4: Remember priorities! Eliminate this answer. Remember, on the
Remember Maslow, the ABCs, and “stable versus NCLEX-RN® exam, emphasis is placed on providing
unstable” when determining which client care to clients according to how nursing
the RN should attend to first. Keep in the mind that care is defined in textbooks and journals.
you can see only one client or perform The correct answer is (4). The nurse is caring for the
one activity when answering questions that require child and his parents while delegating tasks
you to establish priorities. to nursing assistive personnel.
Let’s use the Rules of Management to eliminate Let’s look at another Management of Care question.
answer choices in exam-like Management Which of the following tasks is appropriate for the
of Care questions. nurse to delegate to an
A child with a compound fracture of the left femur is experienced NAP?
being admitted to a pediatric 1. Obtain a 24-hour diet recall from a client recently
unit. Which of the following actions is best for the admitted with anorexia
nurse to take? nervosa.
1. Ask the NAP to obtain the child’s vital signs while 2. Obtain a clean-catch urine specimen from a client
the nurse obtains a history suspected of having a urinary
from the parents. tract infection.
2. Ask the LPN/LVN to assess the peripheral pulses of 3. Observe the amount and characteristics of the
the child’s left leg while the returns from a continuous
nurse completes the admission forms. bladder irrigation for a client after a transurethral
3. Ask the LPN/LVN to stay with the child and his resection.
parents while the nurse obtains 4. Observe a client newly diagnosed with diabetes
phone orders from the physician. mellitus practice injection
4. Ask the NAP to obtain equipment for the child’s care techniques using an orange.
while the nurse talks with Step 1. Reword the question.
the child and his parents. “What task will you assign to an NAP?” The fact that
Step 1. Reword the question in your own words. the NAP is “experienced” is a distracter.
The question asks what the nurse should do when a Step 2. Eliminate answer choices based on the Rules
child with a fractured femur is first of Management.
admitted. That question is very broad. To establish 88
exactly what is being asked, you must NCLEX-RN® Exam Overview and Test Taking
read the answer choices. In each answer, the RN is Strategies
delegating tasks to the LPN/LVN or NAP. (1) Obtain 24-hour diet recall from a client with
The real question is, “What is appropriate delegation?” anorexia nervosa. Some students may
Step 2. Eliminate answer choices based on the Rules consider this answer choice because eating is certainly
of Management. a recurring daily activity, but
87 this answer isn’t about feeding a client. Eating has
3: NCLEX-RN® Exam Strategies special significance for a client with
(1) Obtaining vital signs is an important part of anorexia nervosa. An important assessment that the
assessment. According to Rule #1, the registered nurse must make is the quantity of
nurse cannot delegate assessment. Eliminate this food consumed by this client. The nurse cannot
answer choice. delegate assessment. Eliminate.
(2) Checking the peripheral pulses is an important (2) Obtain a clean-catch urine specimen from a client
assessment for this client because of the with suspected UTI. Rule #4 states, “Delegate
diagnosis of a fractured left femur. The nurse needs activities that involve standard, unchanging
to assess the client before delegating procedures.” There is no indication that
activities to someone else. Assessment of the client is the client has a catheter, so this is a routine
much more important than completing procedure. Keep for consideration.
paperwork. Eliminate. (3) Observe bladder irrigation returns after a
transurethral resection. The color of the fluid needs
to be assessed to determine if hemorrhage is Many graduate nurses are not comfortable answering
occurring. This is an assessment. Eliminate. these questions because:
(4) Observe a newly diagnosed DM client practicing • They don’t understand the “whys” of positioning.
injection techniques. This answer choice • They don’t know the terminology.
involves the evaluation of client teaching. According • They have difficulty imagining the various positions.
to Rule #1, the nurse cannot delegate If you have difficulty answering positioning questions,
evaluation of client care. Eliminate. the following strategy will assist you in
Step 3. Select an answer from the remaining choices. selecting the correct answer.
That leaves only answer choice (2), the correct Correct
answer. Answer
Let’s look at one more question. Are you trying
Which of the following clients should the nurse on a to prevent or
pediatric unit assign to an LPN/ promote?
LVN? What are you
1. A 3-year-old girl admitted yesterday with trying to prevent
laryngotracheobronchitis who has a or promote?
tracheostomy Think A&P.
2. A 5-year-old girl admitted after gastric lavage for 3
Tylenol ingestion 2
3. A 6-year-old boy admitted for a fracture of the Positioning 1
femur, in balanced suspension Step 1. Decide if the position for the client is designed
traction to prevent something or promote
4. A 10-year-old boy admitted for observation after an something.
acute asthmatic attack Step 2. Identify what it is you are trying to prevent or
Step 1. Reword the question. promote.
The question is asking for the appropriate assignment Step 3. Think about anatomy, physiology, and
for an LPN/LVN. pathophysiology (“A&P”).
Step 2. Eliminate answer choices based on the Rules 90
of Management. NCLEX-RN® Exam Overview and Test Taking
After reading the answer choices, you may have Strategies
already seen that Rule #3 (Delegate activities Step 4. Which position best accomplishes what you
for stable clients with predictable outcomes) will be are trying to prevent or promote?
particularly helpful. Does this sound a little confusing? Hang in there. Let’s
(1) Ask yourself, is this a stable client with a walk through a question using this
predictable outcome? A 3 year-old with a new strategy.
tracheostomy is not stable or predictable. Eliminate Immediately after a percutaneous liver biopsy, the
this answer choice. nurse should place the client in
(2) This child may be unstable and the outcome of a which of the following positions?
poisoning is unpredictable. Eliminate 1. Supine
this answer choice. 2. Right side-lying
89 3. Left side-lying
3: NCLEX-RN® Exam Strategies 4. Semi-Fowler’s
(3) This child has a problem that has a predictable Before you read the answers, let’s go through the four
outcome. No information is provided in the steps outlined above.
choice to lead you to believe that this child is unstable Step 1. By positioning the client after a liver biopsy,
at this time. Keep this answer choice are you trying to prevent something or
in consideration. promote something? Think about what you know
(4) Because of the narrow airway of a child, this child about a liver biopsy. You position a client
may be unstable and the outcome is after this procedure to prevent something.
unpredictable. Eliminate this answer choice. Step 2. What are you trying to prevent? The most
Step 3. Select an answer from the remaining choices. serious and important complication after
Answer choice (3) is the correct answer. a percutaneous liver biopsy is hemorrhage.
Strategies for Positioning Questions Step 3. Think about the principles of anatomy,
Because many illnesses affect body alignment and physiology, and pathophysiology. What do
mobility, you must be able to safely care for you do to prevent hemorrhage? You apply pressure.
these clients in order to be an effective nurse. These Where would you apply pressure? On the
topics are also important on the NCLEXRN liver. Where is the liver? On the right side of the
® exam. The successful test taker must correctly abdomen under the ribs.
answer questions about impaired mobility Step 4. How should the client be positioned to prevent
and positioning. hemorrhage from the liver, which is
Immobility occurs when a client is unable to move on the right side of the body? Look at your answer
about freely and independently. To answer choices.
questions on positioning, you need to know the (1) Supine. If you lay the client flat on his back, no
hazards of immobility, normal anatomy and pressure will be applied to the right side.
physiology, and the terminology for positioning. Eliminate.
(2) Right side-lying. If you lay the client in a right degrees in this position. The leg is lower than the
side-lying position, will pressure be applied heart. If the right leg is bent at the knee,
to the right side? Yes. Keep it in for consideration. this could constrict
(3) Left side-lying. No pressure is applied to the right arterial blood flow. Eliminate.
side. Eliminate. 92
(4) Semi-Fowler’s. If you lay the client on his back NCLEX-RN® Exam Overview and Test Taking
with head partially elevated, no pressure is Strategies
applied to the right side. Eliminate. (2) “Side-lying with a pillow between the knees.” Use
The correct answer is (2). Some students select (3) of a pillow in this position could create
because they don’t know normal anatomy and pressure points in the right leg. You don’t want the
physiology. Some students select (4) because semi- knees bent. Eliminate.
Fowler’s position is used for a lot of reasons. (3) “Supine with the right leg extended.” In this
91 position, the leg is at the level of the heart. Circulation
3: NCLEX-RN® Exam Strategies will not be constricted because the leg is straight.
Things to Remember Keep for consideration.
• Even if you didn’t memorize what position to use (4) “High Fowler’s with her right leg elevated.” The
before, during, and after a procedure, head of the bed is elevated 60–90 degrees
think about the question for a moment. You can figure in this position. Elevating the leg promotes venous
out what position is needed. return. Eliminate.
• You cannot figure out the correct position if you do The correct answer is (3). The client is on bed rest for
not know what the terms (such as 8–12 hours in a supine position after an
supine or Fowler’s) mean. angiogram.
• You cannot figure out a correct position if you do not If you didn’t know the specific positioning needed after
know anatomy and physiology. If you an angiogram, you can apply your
think the liver is on the left side of the body, you are knowledge to select the correct answer by just
in trouble! thinking about it.
• You cannot figure out a correct position if you do not Let’s look at another question.
know what you are trying to accomplish. The nurse cares for a client after a lumbar
If you couldn’t remember that a complication after a laminectomy. Which of the following
liver biopsy is hemorrhage, you statements BEST describes the method of turning a
will simply be taking a random guess at the correct client following a lumbar
answer. laminectomy?
• If you think in images, you should form a mental 1. The head of the bed is elevated 30 degrees; the
image of each position. Picture yourself client locks her knees when
placing the client in each position, and then see if the turning.
position makes sense. 2. A pillow is placed between the client’s legs; her
Let’s try another question using the strategies for body is turned as a unit.
positioning. 3. The client straightens her back and grasps the side
An angiogram is scheduled for a client with decreased rail on the opposite side of
circulation in her right leg. the bed.
After the angiogram, the nurse should place the client 4. The head of the bed is flat; the client bends her
in which of the following knees and rolls to the side.
positions? This question isn’t about positioning after a procedure.
1. Semi-Fowler’s with right leg bent at the knee It asks how to turn the client after surgery.
2. Side-lying with a pillow between the knees Step 1. When turning the client after a laminectomy,
3. Supine with right leg extended are you trying to prevent or promote
4. High Fowler’s with right leg elevated something? Promote.
Let’s go through the steps. Step 2. What are you trying to promote? A straight
Step 1. By positioning the client after an angiogram, back. The client can’t bend or twist the torso.
are you trying to prevent something or Step 3. Think about the principles of anatomy,
promote something? You are trying to promote physiology, and pathophysiology.
something. A laminectomy
Step 2. What are you trying to promote? Adequate is removal of one or more vertebral laminae. After a
circulation of the right leg. laminectomy, the back should
Step 3. Think about the principles of anatomy, be kept straight.
physiology, and pathophysiology. What promotes Step 4. How should the client be turned in order to
adequate circulation in the right leg? Keeping the leg keep the back straight?
at or below the level of the heart (1) If the head of the bed is elevated 30 degrees, the
so blood flow is not constricted. back will not be straight. Eliminate.
Step 4. How will the client be positioned after an (2) If a pillow is placed between the legs and the body
angiography to prevent constriction of is rolled as a unit, the client’s back will
vessels and keep the right leg at or below the level of be kept straight. Keep in for consideration.
the heart? Look at the answer choices. 93
(1) “Semi-Fowler’s with the right leg bent at the 3: NCLEX-RN® Exam Strategies
knee.” The head of the bed is elevated 30–45 (3) If the client grabs the opposite side rail, the client’s
torso will twist. The back will not be
straight even though the client straightened her back • High Fowler’s: 80–90 degrees
before turning and twisting. Eliminate. • Fowler’s: 45–60 degrees
(4) If the head of the bed is flat, the client’s back will • Semi-Fowler’s: 30–45 degrees
be straight. If the client bends her knees • Low Fowler’s: 15–30 degrees
and rolls to her side, her back will not be kept straight. Increases venous return; allows maximal lung
Eliminate. expansion
The correct answer is (2). That is a textbook Feet and leg elevated Increases blood return to heart
description of log-rolling. But if you didn’t recall Feet elevated and head lowered
logrolling, (Trendelenburg)
you were able to select the correct answers by Used to insert central venous pressure (CVP) line, or
thoughtfully considering each answer for
choice. treatment of umbilical cord compression
Sometimes a positioning question will be difficult to Feet elevated 20 degrees, knees
identify, such as in the following example. straight, trunk flat, and head slightly
The nurse cares for a client after an appendectomy. elevated (modified Trendelenburg)
The client continues to Increases venous return; used for shock; may be used
complain of discomfort to the nurse shortly after to
receiving an analgesic. Which of prevent shock
the following measures by the nurse would be MOST Elevation of extremity Increases venous return;
appropriate? decreases blood volume to
1. Notify the physician. extremity
2. Place the client in Fowler’s position. Flat on back, thighs flexed, legs
3. Massage his abdomen. abducted (lithotomy)
4. Provide him with reading material. Increases vaginal opening for examination
As you can see, not all of the answer choices involve Prone Promotes extension of hip joint; not well
positioning! How should you approach this tolerated by persons
question? with respiratory or cardiovascular difficulties
First, reword the question so you know what to focus Knee-chest Provides maximal visualization of rectal
on in the answer choices. The question area
really being asked is, “What should the nurse do to Strategies for Communication Questions
help this client with pain relief?” Let’s look at Communication is emphasized on the NCLEX-RN®
the answer choices. exam because it is critical to your success
(1) Calling the doctor, as you know, is almost never as a beginning practitioner. Therapeutic
the right answer. See if another answer communication means listening to and understanding
choice is more appropriate. the client while promoting clarification and insight. It
(2) Fowler’s position. Why change this client’s enables the nurse to form a working
position? To promote pain relief. Will Fowler’s relationship with both the client and the health care
position decrease the client’s pain? Yes, by relieving team, using both verbal and nonverbal
pressure on the client’s abdomen. This communication. Remember that nonverbal
answer is a possibility. communication is the most accurate reflection
(3) Massaging his abdomen will increase the client’s of attitude. Therapeutic responses include the
pain. Eliminate. following:
(4) Providing him with reading materials might 95
distract him from his discomfort, but this is not 3: NCLEX-RN® Exam Strategies
an appropriate intervention for a client in pain. Response Goal/Purpose
Eliminate. Using silence Allows the client time to think and
The correct answer is (2). reflect; conveys acceptance.
Positioning is an important part of the NCLEX-RN® Allows the client to take the lead in conversation.
exam. You must be able to answer these questions Using general leads or broad
correctly in order to prove your competence. If you opening
use the strategies just discussed, you Encourages the client to talk. Indicates your interest
will be thinking about nursing principles and you will in the
select correct answers! client. Allows the client to choose the subject.
94 Clarification Encourages recall and details of a
NCLEX-RN® Exam Overview and Test Taking particular experience.
Strategies Encourages description of feelings. Seeks
Essential Positions to Know for the NCLEX-RN® Exam explanation; pinpoints
Position Therapeutic Function specifics.
Flat (supine) Avoids hip flexion, which can compress Reflecting Paraphrases what client says. Reflects on
arterial flow what client says,
Dorsal recumbent Supine with knees flexed; more especially the feelings conveyed.
comfortable There are many questions on the NCLEX-RN® exam
Side lateral Allows drainage of oral secretions that require you to select the correct
Side with leg bent (Sims’) Allows drainage of oral therapeutic communication response. As with other
secretions; used for rectal exam NCLEX-RN® exam questions, one of the
Head elevated (Fowler’s)
biggest errors that test takers commit when trying to become defensive. A “why” question can come in
answer this type of question is to look many forms, and need not always
for the correct answer. Remember, you are selecting begin with “why.” Any response that puts the client on
the best answer from the four possible the defensive is nontherapeutic
answers that you are given. and therefore incorrect. Examples include:
To select the best answer, you must eliminate answer ‚ . “What makes you think that?”
choices. Let’s look at some different ‚ . “Why do you feel this way?”
answer choices you can eliminate: • Authoritarian answers: Eliminate answer choices in
Correct which the nurse is telling the client
Answer what to do without regard for the client’s desires or
Eliminate feelings. Examples include:
“Don’t ‚ . Insisting that the client follow unit rules
worry.” ‚ . Insisting that the client do what you command
Eliminate immediately
“explore” • Nurse-focused answers: Eliminate answer choices in
answers. which the focus of the comment
Don’t ask is on the nurse. Be careful, because these answer
“Why?” choices may sound very empathetic.
Eliminate The focus of your communication should always be on
“authoritarian” the client. Examples include:
answers. ‚ . “That happened to me once.”
Eliminate ‚ . “I know from experience this is hard for you.”
“focus on • Closed-ended questions: Eliminate answer choices
the nurse” that include closed-ended questions
answers. that can be answered with the words yes, no, or
5 another monosyllabic response. Closedended
34 questions discourage the client from sharing thoughts
2 and feelings. Examples
Eliminate include:
closedended ‚ . “Are you feeling guilty about what happened?”
questions. ‚ . “How many children do you have?”
6 Eliminating these types of nontherapeutic responses
1 that appear as answer choices is an effective
Therapeutic strategy when answering therapeutic communication
Communication questions. Don’t simply look for the
• “Don’t worry” answers: Eliminate answer choices specific words that you see here; you may need to
that offer false reassurance. These “translate” the answer choices into the above
type of responses discourage communication between errors of therapeutic communication.
the nurse and the client by not So how do you select the correct response? By
allowing the client to explore his or her own ideas and choosing from the answer choices that are left!
feelings. False reassurance also The correct response will usually contain one or both
discounts what the client is feeling. Examples include: of the following elements:
‚ . “It’s going to be OK.” • Gives correct information: Offering information
‚ . “Don’t worry. Your doctors will do everything encourages further communication
necessary for your care.” from the client. Examples of giving correct information
• “Let’s explore” answers: Another incorrect answer include:
choice that many graduate nurses ‚ . “You are experiencing acute alcohol withdrawal;
select is the choice that includes the word “explore.” you may see and feel things that
On the NCLEX-RN® exam, avoid aren’t real.”
being a junior psychiatrist. It isn’t the nurse’s role to ‚ . “There are many reasons for memory loss; tell me
delve into the reasons why the client more about what you have
is feeling a particular way. The client must be allowed noticed.”
to verbalize the fact that he or she 97
is sad, angry, fearful, or overwhelmed. 3: NCLEX-RN® Exam Strategies
Examples include: • Is empathetic and reflects the client’s feelings:
‚ . “Let’s talk about why you didn’t take your Empathy is the ability to perceive what
medication.” another person experiences using that person’s frame
‚ . “Tell me why you really injured yourself.” of reference. Reflection communicates
96 to the client that the nurse has heard and understands
NCLEX-RN® Exam Overview and Test Taking what the client is trying
Strategies to communicate. When reflecting feelings, the nurse
• “Why” questions: Eliminate answer choices that focuses on the feelings and not the
include “why” questions: ones that content of what is said. The following are examples of
seek reasons or justification. “Why” questions imply empathetic, reflective statements
disapproval of the client, who may include:
‚ . “I can see that you are frightened about being You have eliminated three of the four answer choices.
here.” The correct answer is the only answer
‚ . “You seem very upset. Tell me how you’re feeling.” choice remaining, (3).
Let’s practice therapeutic communication with a few Let’s look at one more question.
exam-style questions. A client in the psychiatric unit asks the nurse, “Am I
A client is admitted to the emergency room with a in a special radioactive
diagnosis of acute myocardial shelter? When was it last checked for radioactivity?”
infarction. The client tells the nurse, “I’m scared. I Which of the following
think I’m going to die.” Which responses by the nurse would be MOST appropriate?
of the following responses by the nurse would be 1. “This is a hospital, and we do not have a Nuclear
MOST appropriate? Medicine Department here.”
1. “Everything is going to be fine. We’ll take good care 2. “Don’t worry, you’re safe. There’s no radioactivity
of you.” here.”
2. “I know what you mean. I thought I was having a 3. “I’m sure your safety is of concern to you, but this
heart attack once.” is a hospital.”
3. “I’ll call your doctor so you can discuss it with him.” 4. “Please share with me what makes you think there
4. “It’s normal to feel frightened. We’re doing is radioactivity here.”
everything we can for you.” Step 1. Eliminate answer choices.
Step 1. Eliminate incorrect answer choices. (1) This response provides information. Leave it in for
(1) This is a “don’t worry” response. There is no consideration.
acknowledgment of the client’s fears. (2) This response offers false reassurances. Eliminate
Eliminate. it.
(2) The focus of this response is on the nurse, not the (3) This response reflects the client’s concern about
client. Eliminate. safety and provides information. Keep
(3) It is within the scope of nursing practice for the it in for consideration.
nurse to respond to the client’s feelings. (4) This response allows the client to verbalize, but
Don’t pass the responsibility to the physician. you don’t want to encourage a client with
Eliminate. psychological problems to talk about hallucinations or
(4) This answer choice responds to feelings and delusions. Rather, you want your
provides information. Keep it in consideration. discussion to focus on the feelings that accompany
Step 2. Select an answer from the remaining choices. them. Eliminate this choice.
One answer was not eliminated: (4). This is the Step 2. Select an answer from the remaining choices.
correct answer. The nurse is empathetic, You have more than one possible answer choice: (1)
acknowledging that the client feels frightened, and and (3). Look for the answer choice that
provides information. reflects feelings and gives information. The correct
Let’s look at another question. answer is (3).
A mother is to undergo a breast biopsy. She tells the ork for Kaplan, the oldest test prep company in the
nurse, “If lose my breast, nation. We have
I know my husband will no longer find me attractive.” been preparing graduate nurses and international
Which of the following nurses for the NCLEX-RN® exam for more than
responses by the nurse would be MOST appropriate? 25 years. We know what works to prepare for the
1. “You don’t know if you are going to lose your exam and what doesn’t work.
breast. They are just doing the correctly. Tear out the Chart of Critical Thinking Paths
biopsy now.” in Appendix A and consult
2. “You should focus on your children. They are young it while you are answering practice test questions.
and they need you.” This will help you become
3. “You seem to be concerned that your relationship more comfortable with putting the strategies into
with your husband might practice. As you answer more
change.” and more questions, put the diagram aside and rely
4. “Why don’t you wait and see what your husband’s on your memory to identify
reaction is before you get and implement a critical thinking strategy.
upset.” Question: “Am I going to have enough time when I
Step 1. Eliminate answer choices. take the NCLEX-RN® exam to figure
(1) This response gives false reassurance and out which strategy to use?”
discounts the client’s feelings. Eliminate it. Answer: Timing is a concern on the NCLEX-RN®
(2) This response is authoritarian: the nurse tells the exam. You need to maximize your efforts
client what to do. Eliminate it. on each test question. Practice answering test
(3) This response reflects the fears of the client. The questions using the various strategies
response is open-ended and allows the we’ve outlined. As you get more proficient, you will
client to express what she is feeling. Keep it in for discover that it takes you less
consideration. time to identify the strategy or path that will lead you
(4) This response dismisses the feelings that the client to the correct answer.
is experiencing and gives advice. Eliminate Question: “I don’t have to use these strategies on
it. every question, do I? I think I’ll use them
Step 2. Select an answer from the remaining choices. only when I can’t figure out the correct answer on my
own.”
Answer: Wrong! You should use critical thinking to that situation. Notice how much easier your test
answer every question on the seems in that situation.
NCLEX-RN® exam to make sure that you pass. Go Here’s another variation. Close your eyes and think
through the steps that we about a situation in which you did well on
have outlined for every practice question that you a test. If you can’t come up with one, pick a situation
answer as you prepare for the in which you did some good academic
exam. If you practice these steps, you will not need work that you were really proud of, or some other kind
to randomly guess the correct of genuine accomplishment. Not a
answer on the NCLEX-RN® exam. fiction, mind you: it has to be from real life. Make it
Question: “So all I have to do is memorize the as detailed as possible. Think about the
strategies, right?” sights, the sounds, the smells, and even the tastes
Answer: Just memorizing the various strategies will that you associate with this experience of
not ensure your success on the academic success. Now think about your test in line
NCLEX-RN® exam. Remember, the exam does not with that experience. Don’t make comparisons
test your ability to memorize between them. Just imagine taking your test with that
either critical thinking strategies or nursing content. same feeling of relaxed control.
The NCLEX-RN® exam * Some of these methods were originally
tests your ability to think critically and use the nursing conceptualized by Dr. Émile Coué, who in the 1920s
knowledge that you told everyone
have. It’s relatively easy to just memorize nursing that the key to a happy life was to constantly repeat
content. The hard part is to the phrase, “Every day in every way I am getting
figure out how to use this knowledge to make nursing better
judgments. It’s relatively and better.” As advice to test takers, that isn’t bad at
easy to memorize the critical thinking strategies. The all!
hard part is to figure out 2. Exercise
which strategy to use on each and every question. Whether it be jogging, walking, yoga, push-ups, or a
That takes practice. pickup basketball game, physical exercise is
Question: “What if I use the strategies but still can’t a great way to stimulate the mind and body and
figure out the correct answer?” improve one’s ability to think and concentrate. A
Answer: It’s not unusual that students will read a surprising number of those who prepare for
question, read the answers, and think standardized tests don’t exercise regularly because
“Huh? Something is missing!” If you feel like they spend so much time preparing. Sedentary
something is missing, reread the people—this is a medical fact—get less oxygen
question to determine if you have correctly identified in the blood, and therefore to the brain, than active
what the question is asking. people.
If you have identified the question correctly, then read Do the Following on Exam Day
the answer choices to make • Keep moving forward. By test day, do enough
sure you haven’t missed the nursing concept preparation with a review course or practice
contained in the answer choices. questions so that it becomes an instinct to keep
Question: “Will these strategies work on every moving forward instead of getting bogged
practice question that I answer?” down in a difficult question. You don’t need to get
Answer: The critical thinking strategies discussed in everything right to pass, so don’t linger
this book will enable you to answer on a question that is going nowhere. The best test
all kinds of multiple-choice test questions. The critical takers don’t get bothered by difficult
thinking strategies apply questions because they accept that everyone
to test questions written at the application/analysis encounters them on the NCLEX-RN® exam.
level and do not work with • Don’t listen to negative words or behavior. Don’t be
knowledge-based test questions. If you feel that the distracted by the ignorant babble or the
strategies don’t work with behavior of other, less-prepared, less-skilled
102 candidates around you. Negative thoughts
Mental Preparation* lead to negative feelings and may interfere with
1. Visualize performing your best on Test Day.
You have probably learned how to do this with clients; • Don’t be anxious if other test takers seem to be
now it’s your turn. Sit back and let working harder or answering questions more
your shoulders and arms relax. Close your eyes and quickly. Continue to spend your time patiently but
imagine yourself in a relaxing situation— persistently thinking through your
it can be fictional, but a real-life memory is best. Make answers; it’s going to lead to higher-quality test
it as detailed as possible. Think taking and better results. Set your own
about the sights, the sounds, the smells, even the pace and stick to it.
tastes that you associate with the relaxing • Keep breathing! Weak standardized test takers tend
situation. Keep your eyes shut; keep sinking back into to share one major trait: forgetting
your chair. Now that you’re in that to breathe steadily as the test proceeds. They do not
situation, start bringing your test in—think about the to know the value of proper breathing.
experience of taking the test while in They start holding their breath without realizing it, or
that relaxing situation. Imagine how much easier it begin breathing erratically
would be if you could take your test in
or arrhythmically. This can hurt confidence and • Information technology
accuracy. Do what you can to instill an • Informed consent
awareness of proper breathing before and during each Safe and Effec tive Care
study or testing session. Environmen t: Managemen t of Care
• Do some quick isometrics during the test. This is chapter 4
helpful especially if your concentration 108
is wandering or energy is waning. For example, put NCLEX-RN® Exam Content Review and Practice
your palms together and press • Legal rights and responsibilities
intensely for a few seconds. • Performance improvement (quality improvement)
Here is a brief review of the various strategies that • Referrals
you have learned in this chapter: • Supervision
• The NCLEX-RN® exam isn’t the real world, so don’t Now let’s review the most important concepts covered
rely on your real-world experience by the Management of Care subcategory
to answer NCLEX-RN® exam questions. on the NCLEX-RN® exam.
• To answer priority questions correctly, think Maslow, Advance Directives
the nursing process, and safety. An advance directive is a legal document, such as a
• The Rules of Management will help you answer living will, a health care proxy, or a
questions about delegation and assignment Durable Power of Attorney for Health Care (DPAHC).
of client care. Advance directives provide guidance
• Use the Positioning strategy when you encounter to caregivers about the client’s wishes and are
questions about positioning and mobility. followed if a client’s decision-making powers
• The Therapeutic Communication strategy will help become impaired. The 1990 Patient Self-
you eliminate incorrect answer choices Determination Act requires that upon admission
in communication questions. to hospitals, long-term care facilities, and home
• Identify your strength and weaknesses, and choose health agencies, patients be informed that
an effective method of study that they have the right to accept or refuse medical care,
works for you. as well as to specify in advance (through
• Use mental preparation techniques to alleviate advance directives) what their wishes are.
stress and manage your test day experience. Your role as a nurse is to integrate advance directives
NCLEX-RN® Exam into the client care plan. To accomplish
Content Review this, evaluate client status regarding advance
and Practice directives, and help to determine whether
Part 2 family members and/or significant others should be
involved in conversations and decisionmaking.
107 If the client, a family member, significant other, or
One of the most important parts of your job in client staff member is not familiar with
care is keeping the care environment the details of advance directives, provide the
safe for all involved. In addition, it’s also important to information as needed.
provide care effectively. Providing a You must also ensure that copies of advance directives
safe and effective care environment involves both are placed in the client’s medical
proper management of care, and safety and record. This includes information on organ or tissue
infection control. donation for clients over 18 years of age.
Management of care refers specifically to the way The Uniform Anatomical Gift Act, for example,
nursing care is provided and directed so governs organ donations for transplantation
that the client receives proper treatment, and so that and how to donate one’s cadaver as an anatomical
health care personnel remain safe. It gift.
also covers management, delegation, and other skills Advocacy
you are expected to have, as well as your Client advocacy—promoting your clients’ rights and
ethical and legal obligations regarding client care. interests—is an important part of nursing.
On the NCLEX-RN® exam, you can expect Discuss treatment options with clients, including what
approximately 20 percent of the questions to the options are, how they work,
relate to Management of Care. Exam content related and what the side effects may be, so the client
to this subcategory includes, but is not understands all available choices. You must
limited to, the following areas: respect client decisions even if you do not agree with
• Advance directives them. You may need to provide information
• Advocacy regarding these discussions to other staff members so
• Case management you can advocate for your client.
• Client rights When necessary, use an interpreter or translator for
• Collaboration with interdisciplinary teams non-English-speaking clients. Know
• Concepts of management when it is appropriate to engage others higher in the
• Confidentiality/information security chain of command or with different
• Consultation areas of expertise, such as a social worker, on your
• Continuity of care client’s behalf.
• Delegation 109
• Establishing priorities 4: Safe and Effective Care Environment:
• Ethical practice Management of Care
Case Management options and take part in decisions about care. Parents,
It is important to assist your clients in achieving guardians, family, and significant
and/or maintaining their independence by others can represent the client if the client cannot
identifying and utilizing the resources available to make his or her own decisions.
them. The individualized care plan you • Confidentiality of health information: The client has
develop for each client should be aimed at providing the right to talk privately with health
safe, cost-effective care for the client. care providers and have health care information
The plan is based on your assessment of client needs protected; it also includes the right to
as well as goals, such as providing selfcare. read and copy one’s own medical records.
You should also incorporate evidence-based research • Complaints and appeals: The client has the right to
from medical literature and other a fair, fast, and objective review of any
resources, where applicable, into the care plan. In complaint against a health plan, a physician, other
addition to initiating the care plan for each health care personnel, or a hospital.
client, you are expected to evaluate and revise that • Consumer responsibilities: This includes, among
plan, as needed. other things, a client’s responsibility to
When a client leaves the hospital, provide the client provide information about medications and past
with information on discharge procedures illnesses.
to home, hospice, or community living, whichever Evaluate the client’s understanding of his or her rights
may be relevant to the client’s situation. and responsibilities, including the
This includes information about medications the client right to informed consent and the difference between
should be taking, follow-up privileged communication and the duty
visits, future lab tests, and so on. to disclose, as well as staff understanding of client
Client Rights rights.
Part of your job as a health care provider is to discuss Collaboration with Interdisciplinary Teams
treatment options and decisions with The term interdisciplinary or multidisciplinary refers to
your clients, and educate them about client rights and situations in which various disciplines
responsibilities. As noted previously, are involved in reaching a common goal, with each
the Patient Self-Determination Act requires that upon contributing his or her specific expertise.
admission to hospitals, long-term care The interdisciplinary interaction between different
facilities, and home health agencies, patients be health care professions such as nursing,
informed that they have the right to accept medicine, and social work is known as collaboration.
or refuse medical care. At times, you may need to Such collaboration in the management
recognize the client’s right to refuse treatment. of a particular disorder enables caregivers to provide
The Health Insurance Portability and Accountability a more comprehensive and individualized
Act (HIPAA) protects personally approach. Collaboration with an interdisciplinary team
identifying information, such as the client’s name, requires cooperation, integration,
social security number, date of birth, and and teamwork.
information about diagnosis and treatment. HIPAA Because nurses are often the caregivers that clients
provides that such information should see most often, be prepared to identify the
only be shared with individuals directly involved in the need for interdisciplinary conferences regarding a
client’s care, the payment of care, and/ client, and know how to initiate such conferences.
or the management of the client’s care. This includes identification of significant information
The Patients’ Bill of Rights, adopted by the President’s to report to other disciplines,
Advisory Commission on Consumer including health care providers, pharmacists, social
Protection and Quality in the Health Care Industry, is workers, and respiratory therapists.
a statement about the rights to which You should be ready to act as the point person to
individuals are entitled as recipients of health care, review the care plan and ensure continuity
and their responsibilities. It covers the across disciplines, and to collaborate with health care
following areas: members in other disciplines to provide
• Information disclosure: The client has a right to efficient and effective client care.
accurate and easily understood information Concepts of Management
about health plans, health care professionals, and It’s important to identify the roles and responsibilities
health care facilities. of all members of the health care team.
• Choice of providers and plans: The client has the You’ll often need to act as the liaison between those
right to choose health care providers who team members and the client to coordinate
can provide high-quality health care when needed. and manage care.
• Access to emergency services: The client has the As issues arise regarding client treatment, apply the
right to be screened and stabilized using principles of conflict resolution, as
emergency services whenever and wherever the client needed, when working with health care staff. You
needs them, without having to should also be able to plan overall strategies
wait for authorization, and without any financial 111
penalty. 4: Safe and Effective Care Environment:
110 Management of Care
NCLEX-RN® Exam Content Review and Practice to address client problems. Know how to supervise
• Participation in treatment decisions: The client has care provided by others (see the “Delegation”
the right to know about treatment
and “Supervision” sections later in this chapter), and the authority to do the job. Good delegators provide
know which staff members can support and monitoring, provide sufficient
perform particular procedures related to client care. time to complete the task, retain responsibility for
Confidentiality/Information Security knowing the outcome, and praise
Like all health care providers, you should maintain and acknowledge a job well done.
client confidentiality and take the necessary Do not delegate the following to nonprofessional staff:
steps to ensure that client information security is not • Nursing assessments
breached. An individual not • Diagnosis, care goals, or progress plans
involved in the care of the client does not have a • Interventions that require professional knowledge
legitimate need to access the client’s medical and skill
record. Know the provisions of HIPAA (summarized in Remember the five “rights” of delegation:
the Client Rights section of this • Right task: Can the task be safely delegated?
chapter) and protect the client’s right to privacy. • Right circumstance: Is the client stable, and is the
Ensure that only authorized individuals outcome predictable?
access medical records, that no medical records are • Right person: Does the person to whom the task will
viewable by the general public, and that be delegated have the necessary
no conversations about client information can be knowledge and appropriate skills?
overheard by unauthorized persons. • Right direction/communication: Has the nurse
You may need to intervene when confidentiality is communicated appropriate instructions for
breached by other staff members. You’ll accomplishing the task?
also be expected to assess staff members’ and your • Right supervision: Will the delegating nurse remain
clients’ understanding of confidentiality responsible for the task and
requirements, such as those governed by HIPAA. outcomes?
Consultation You can delegate activities for stable clients with
Consultation involves communication with another predictable outcomes, and activities that
nurse or health care professional, such as involve standard, unchanging procedures, such as
a dietician or pharmacist, about an aspect of client bathing, feeding, dressing, and transferring
care. Determine when a consultation with clients. Do not delegate an activity if the client is
other health care providers is appropriate, and then unstable, if the outcome of the activity is not
initiate such consultations as needed. assured, or if the activity is complex or complicated.
You should also be able to identify the expected Establishing Priorities
outcomes of consultations, and revise the There are several other frameworks for establishing
care plan if the client’s needs change. the priority of client care. They include:
Continuity of Care • ABCs (airway, breathing, circulation/cardiovascular
Continuity of care is the process by which a client and system)
health care providers are cooperatively • Maslow’s hierarchy of needs (physiological needs,
involved in the ongoing health care management of safety and security, love and belonging,
the client, with the goal of providing highquality self-esteem, and self-actualization)
and cost-effective health care. Ideally, all people • Agency policies and procedures
involved in a client’s health care, • Time
including the client, communicate with one another to • Client and family preferences
coordinate care, as well as agree and 113
understand the goals of health care for the client. 4: Safe and Effective Care Environment:
To help ensure continuity of care, know the proper Management of Care
procedures to admit, transfer, and discharge • Care related to client activity
a client. This includes maintaining continuity of care • Priorities in medication therapy
between/among health care Assess/triage (French for sort) clients to prioritize
agencies when clients are transferred or handed off order of care delivery, and focus on the
from one department to another, or from least stable clients first. Use your knowledge of
one agency to another. It also includes using pathophysiology when establishing priorities
documents and proper forms to enter client for interventions with multiple clients. Once you have
information into medical records or on provided care to multiple clients, evaluate
transfer/referral forms. You may also need to follow and adjust your care plans as needed.
up on unresolved issues regarding client care (e.g., The following general problems indicate priority
laboratory results and client requests) and needs:
provide reports on assigned clients. • Postoperative clients just out of surgery
112 • Clients whose status has deteriorated from their
NCLEX-RN® Exam Content Review and Practice normal baseline
Delegation • Clients exhibiting signs of shock
Delegation is a crucial skill. You must be able to • Clients with allergic reactions
identify an appropriate person to carry out • Clients with chest pain
a specific task or set of tasks, explain the tasks • Postdiagnostic-procedure clients who require
clearly, and make sure you are understood. temporary monitoring
It is also your responsibility to make sure the person • Clients who tell you they have unusual symptoms
to whom you are delegating a task has • Clients with malfunctioning equipment or tubing
Ethical Practice
Ethical principles help you determine whether an Typically, the health care provider who is performing
action is right or wrong. In addition to the procedure or providing the treatment
understanding basic ethics and morals, you should be (usually the physician) is responsible for obtaining the
familiar with the American Nurses client’s informed consent. One of your
Association (ANA) Code of Ethics for Nurses. These roles in the process is to advocate for the client by
guidelines delineate values and standards ensuring he or she has been provided the
for professional practice. necessary information to make an informed decision.
Make sure you understand the following ethical In cases where the client does not speak
principles: English, provide written materials in the client’s native
• Autonomy: The right of individuals to make language, when possible. Another one of
decisions for themselves your roles is to ensure that a client has actually given
• Beneficence: A nurse’s duty to do what is in the best informed consent for treatment before that
interests of the client treatment occurs. One way to do so is to act as a
• Justice: A fair, equitable, and appropriate treatment witness to the informed consent. As a witness,
• Nonmaleficence: A nurse’s duty to do no harm you confirm that the client gave his or her informed
• Fidelity: Keeping faithful to ethical principles and the consent voluntarily, the client’s signature
ANA Code of Ethics for Nurses is authentic, and the client is competent to give
• Virtues: Compassion, trustworthiness, integrity, and consent. You may be called upon to evaluate
veracity (truthfulness) clients to determine whether they are capable of
• Confidentiality: Maintaining the client’s privacy by providing informed consent, and identify an
not disclosing personal information appropriate person to do so, such as a parent or legal
about the client guardian, if the client is a minor.
• Accountability: Responsibility for one’s actions If the client waives consent, ensure it is documented
You should be able to identify ethical issues affecting in the medical record. If the client is
staff or clients, provide information deemed incompetent to give informed consent, a
on ethics, and intervene appropriately to promote court-appointed guardian may do so on the
ethical practice. You’ll also be expected to client’s behalf.
review outcomes of interventions to promote ethical 115
practice. 4: Safe and Effective Care Environment:
114 Management of Care
NCLEX-RN® Exam Content Review and Practice The requirement to obtain the client’s informed
Information Technology consent can be waived in an emergency situation
Electronic medication administration records have the in which the client is incapacitated and the situation
potential to reduce medication administration requires immediate treatment.
errors and improve access to client information at the Legal Rights and Responsibilities
point of care. You must know Know the confines of applicable laws and understand
how to use information technology and information the parameters of your nursing license.
systems to enter computer documentation Legal limits and the scope of practice for nursing are
in a client’s medical record and other databases in a dictated by federal and state laws,
timely and accurate manner. You may such as the Nurse Practice Acts (NPAs) and related
also need to access data for clients and staff through guidelines, and are regulated by each
online databases and journals. state’s Board of Nursing. Nurses are accountable and
Whenever you access a client record, apply your responsible for incorrect or inappropriate
knowledge of the facility’s specific regulations. actions or inactions. These may include negligence,
You’ll also be expected to receive and/or transcribe malpractice, or other legal charges.
primary health care provider Negligence involves the unintentional failure to act as
orders. a reasonable person would in similar
You should also know how to use information circumstances that results in an injury to the client.
technology (e.g., a computer or video) to Elements include a breach of a duty
enhance the care provided to a client. Telehealth, for of care, with a resultant injury that has been
example, uses transmissions via telecommunications proximately caused (i.e., there is reasonably
technology to transmit health information remotely. close connection between the nurse’s actions and the
Informed Consent resulting injury), and actual damages
Informed consent is the right of clients to be to the injured party. Malpractice involves the failure
adequately informed of the risks and benefits of a by a medical professional to carry out
proposed procedure or treatment before determining or perform his or her duties that result in injury to the
whether or not to consent to that procedure client. The specific requirements for
or treatment. The components of informed consent malpractice are typically defined by the statutes and
include an explanation of the following: rules/regulations of each state.
• Details of the procedure or treatment There are specific areas with which you should be
• Risks and benefits of the procedure or treatment, familiar. They include:
including the potential for serious • Identifying legal issues affecting clients (e.g.,
injury or death refusing treatment) and knowing how to
• Alternative procedures or treatments respond appropriately
• Potential consequences of refusing the procedure or • Recognizing tasks and assignments you are not
treatment prepared to perform and seeking
assistance Some of these may require specific approvals,
• Identifying and managing clients’ valuables although in some cases you can refer a client
according to facility or agency policy directly to a dietary or wound care specialist.
• Educating clients and staff on legal and ethical issues Assess the need to refer clients for assistance with
• Complying with state and/or federal regulations for actual or potential problems (physical
reporting client conditions (e.g., therapy, speech therapy), and match community
abuse/neglect, communicable disease, gunshot resources to the client’s needs (respite care,
wound, or dog bite) social services, shelters). In all referral situations, you
• Reporting unsafe practices of health care personnel need to know which documents to
to internal or external entities include when referring a client, such as a medical
• Intervening appropriately when you observe unsafe record or referral form.
practices by staff members Supervision
Performance Improvement (Quality Leadership will be critical to your success. You should
Improvement) be able to create a common vision for
Each institution may define it differently, but a staff, and promote a sense of urgency. This helps
standard definition of quality involves meeting connect daily activities to a larger strategic
or exceeding the expectations of customers and plan, and keeps nursing activities in line with the
standards, and achieving planned outcomes. overall goals of the institution.
Quality management principles include total quality 117
management (TQM), continuous 4: Safe and Effective Care Environment:
quality improvement (CQI), and evidence-based Management of Care
decision making, among others. Quality A supervisor is someone who has authority to manage
improvement includes activities such as identifying other employees. Supervision includes
opportunities and developing policies guidance and direction, evaluation, and follow-up to
for improving the quality of nursing practice. Methods ensure tasks are accomplished. A good
include establishing a comprehensive supervisor provides the following:
116 • Clear direction and communication
NCLEX-RN® Exam Content Review and Practice • Timely follow-up
quality management plan, establishing benchmarks, • Active listening
completing performance appraisals, • Complete technical knowledge of supervised work
performing intradisciplinary and interdisciplinary • Feedback and resolution of problems and conflicts
assessments, performing nursing audits, As a supervisor, you are expected to select and
conducting peer reviews and utilization reviews, and implement strategies for interventions with
managing outcomes. Mock codes can staff members as necessary, to report staff member
improve performance by encouraging teamwork, performance, and to evaluate the skills
improving communication and skill building, and abilities of staff members, particularly as they
and enhancing confidence of caregivers. relate to time management.
You must report identified client care issues or Types of staff members you might be called upon to
problems to appropriate personnel (e.g., the supervise include other RNs, licensed
nurse manager or risk manager). A nurse is also practical nurses (LPNs), licensed vocational nurses
expected to participate in the performance (LVNs), and nursing assistive personnel
improvement and quality assurance process, which (NAPs).
may include data collection or participation NCLEX-RN® Exam Content Review and Practice
on a team. 118
You may be asked to utilize research and other 1. A 58-year-old man with head and neck
references when determining how best to cancer is admitted to the hospital and
improve performance, and you will be expected to tells the nurse he does not want parenteral
evaluate the impact of performance nutritional therapy as his cancer progresses.
improvement measures on client care and resource The nurse explains he can specify his wishes
utilization using a variety of specific by creating an advance directive. The nurse
indicators. knows that the requirement to provide clients
Nurse-sensitive indicators are measurements of client with this type of information can be found in
care that are impacted by nursing interventions. which of the following?
Examples include maintenance of skin integrity, 1. The Patient Self-Determination Act
pressure ulcer prevalence and incidence, 2. Nursing Scope and Standards of Practice
fall injury rate, medication incident rate, restraint 3. The Patient Protection and Affordable
utilization rate, client satisfaction Care Act
with pain management, client satisfaction with overall 4. The Patients’ Bill of Rights
nursing care, and nurse satisfaction. 2. A 14-year-old girl newly diagnosed with
Referrals diabetes is preparing for discharge. Which of
Nurses often have a role to play in assisting and the following activities BEST describes the
coordinating client care that requires referrals. nurse’s role as a client advocate?
There are different types of referrals: authorization for 1. Arranging for a visit with a home health
care or a service, recommendation nurse
of a specific provider, referral to specialists, and 2. Providing written medication instructions
referral to a different facility for care. to the client’s parents
3. Instructing the client to follow up with interaction is BEST referred to as which of
her provider in 4 weeks the following?
4. Teaching the client how to administer 1. Case management
insulin injections 2. Collaboration
3. A client is seen for an outpatient 3. Cooperation
appointment and asks the nurse if he can 4. Collegiality
obtain a copy of his medical record. The 8. A pregnant woman at 15 weeks’ gestation
nurse knows the client has the right to read is scheduled for an amniocentesis. As the
and copy his medical records, and that client is being prepped for the procedure,
this is guaranteed by virtue of which of the it becomes clear to the nurse that the client
following? doesn’t fully understand the risks and
1. The Code of Ethics for Nurses benefits associated with the procedure.
2. The Health Insurance Portability and Which of the following describe the nurse’s
Accountability Act (HIPAA) role in obtaining informed consent? Select all
3. The Patient Self-Determination Act that apply.
4. The Americans with Disabilities Act 1. Explain the risks and benefits associated
4. After receiving report at the start of the with the procedure.
evening shift, which of the following clients 2. Describe alternatives to the procedure.
should the nurse attend to FIRST? 3. Witness the client’s signature on the
1. A 34-year-old man undergoing treatment consent form.
for non-Hodgkin lymphoma with a 4. Advocate for the client by ensuring she is
potassium level of 7.5 mEq/L making an informed decision.
2. A 21-year-old woman with sickle-cell 9. The nurse noticed an increase in the prevalence
anemia with pain of 6 on a scale of 1–10 of pressure ulcers among clients in an intensive
3. A 55-year-old woman with ovarian cancer care unit. She documented her findings and
waiting to be discharged worked with her manager to develop and
4. A 72-year-old man with chronic implement a new policy using a pressure ulcer
obstructive pulmonary disease (COPD) risk assessment scale. Which of the following
and a pulse oximetry of 96% on room air BEST describes the nurse’s actions?
5. A 34-year-old woman who developed 1. Quality improvement
Stevens-Johnson syndrome while undergoing 2. Collaboration
treatment with carbamazepine (Tegretol) is 3. Advocacy
being transferred in stable condition from 4. Case management
the intensive care unit to the medical unit. 10. The nurse is working on a surgical unit.
There are 4 beds available. The nurse knows Which of the following tasks would be
the BEST choice of roommates for this client appropriate for the nurse to delegate to
is which of the following? nursing assistive personnel (NAP)?
1. A 40-year-old man with methicillinresistant 1. Assist a new postoperative client to the
Staphylococcus aureus (MRSA) bathroom.
2. A 28-year-old woman diagnosed with 2. Set up the clients’ lunch trays.
diarrhea 3. Change a central line dressing.
3. A 72-year-old man with fever of unknown 4. Teach a client how to administer
origin discharge medications.
4. A 68-year-old woman with atrial fibrillation 11. The nurse has been asked to administer a
6. A 72-year-old man who had a stroke is drug by IV push. She is uncertain whether
being transferred from a medical unit to a or not this task falls within her scope of
rehabilitation center. The nurse case manager practice. The nurse knows that which of
is assisting in the process. The nurse knows the following are the BEST sources to refer
that the goals of case management include to for information related to her scope of
which of the following? Select all that apply. practice in this situation? Select all that
1. Improving the coordination of care apply.
2. Increasing referrals to local organizations 1. Hospital and unit policies and
3. Reducing the fragmentation of care procedures
4. Discharging clients quickly 2. Nurse Practice Act
Chapter Quiz 3. Ordering physician
4: Safe and Effective Care Environment: 4. Hospital pharmacist
Management of Care 12. A 20-year-old client with leukemia has
119 consented to a blood transfusion against the
7. An 18-year-old client with acute lymphocytic wishes of his family, who are all Jehovah’s
leukemia is admitted to the bone marrow Witnesses. The nurse knows that which
transplantation unit. His family is having of the following ethical principles BEST
trouble dealing with the emotional and supports this decision?
financial pressures of his disease. The 1. Autonomy
nurse, case manager, physician, and social 2. Beneficence
worker meet to discuss the plan of care. The 3. Nonmaleficence
nurse knows this type of interdisciplinary 4. Justice
13. The nurse wants to delegate the task of man being admitted for Crohn’s disease.
showering an elderly client in a wheelchair The nurse knows that according to the
to the nursing assistive personnel (NAP). Patients’ Bill of Rights, this client is
Before delegating a task to the NAP, the responsible for which of the following?
nurse should FIRST ensure which of the Select all that apply.
following is accomplished? 1. Consenting to treatment
NCLEX-RN® Exam Content Review and Practice 2. Providing information about
120 medications
1. The UAP is supervised at all times. 3. Providing proof of insurance
2. The UAP demonstrated competency for 4. Providing information about past
the task during orientation. illnesses
3. The UAP has performed the task before. 19. The nurse is caring for a 41-year-old man
4. The UAP has received the assignment with a new colostomy. As part of the care
during report. planning for this client, the nurse knows a
14. A well-known actor has been admitted to referral to which of the following will be the
an ambulatory surgical unit. The nurse priority?
notices a staff member who is not involved 1. A certified wound, ostomy, and
in the client’s care reading his medical continence nurse (CWOCN)
record. The nurse knows she should FIRST 2. Social services
do which of the following? 4: Safe and Effective Care Environment:
1. Nothing. The staff member has a Management of Care
hospital ID badge and is authorized to 121
read the medical record. 3. Physical therapy
2. Inform the staff member that without 4. Occupational therapy
a legitimate need for the information, 20. An RN is in charge of a team on a medical/
staff should not be reading the medical surgical unit that includes an LPN. The RN
record. understands that which of the following
3. Tell the client his medical records is an activity that falls within the scope of
have been read by an unauthorized practice of an LPN?
individual. 1. Administer oral medications to a client.
4. Page the physician and ask if it’s 2. Collaborate with social services to
acceptable for the staff member to develop a discharge plan.
access the medical records. 3. Formulate a nursing diagnosis.
15. The nurse is learning how to use the 4. Develop a policy.
hospital’s new electronic medication 21. The nurse in a maternity unit is caring for
administration record. The nurse knows a client who has just delivered twins. The
this tool has the potential to do which of the client voices concern about her ability to
following? Select all that apply. manage when she gets home. Which of
1. Reduce medication administration the following statements BEST illustrates
errors. quality care delivery by the nurse? Select all
2. Improve access to information at the that apply.
point of care. 1. “Just focus on how lucky you are to have
3. Eliminate the need for the nurse to two healthy babies.”
document medication administration. 2. “We can arrange for follow-up visits
4. Eliminate the need for the nurse to with a home health nurse.”
verify dose calculations. 3. “Here is some information on support
16. The nurse uses the Internet to receive groups for parents of multiples.”
electrocardiogram results from a client 4. “You will find it easier to formula-feed
living in a nursing home. The nurse knows your babies at home.”
this type of information technology is 22. After responding to a code, several
BEST described as which of the following? staff nurses express concerns over their
1. Encryption confidence levels and performance to
2. Telecommunications the nurse in charge of the hospital’s
3. Telehealth performance improvement program. The
4. Nursing informatics nurse in charge knows the BEST way to
17. The nurse is preparing to transfer a client evaluate and improve performance is to
to the operating room. She knows that implement which of the following?
adhering to the hospital policy for client 1. A program that collects and analyzes
handoffs BEST ensures which of the performance data
following? 2. Mock codes
1. Case management 3. Inservice training
2. Continuity of care 4. Written competency exams
3. Confidentiality protection 23. A client is being treated for uncontrolled
4. Collaboration hypertension. The nurse knows that
18. The nurse is preparing to perform an the involvement of nursing, pharmacy,
admission assessment on a 28-year-old cardiology, and nutritional services is
an example of which of the following (1) Arranging for a visit with a home health nurse
approaches? may be important in the overall management of
1. Managed care this client’s care, but does not directly assist in
2. Multidisciplinary teaching the client the necessary skills to manage
3. Case management her diabetes.
4. Performance improvement (2) Providing written medication instructions to the
24. The nurse is caring for a client newly client’s parents may be important in the overall
diagnosed with diabetes, and performs the management of this client’s care, but does not
following tasks. Place the tasks the nurse directly assist in teaching the client the necessary
would perform in the appropriate order. All skills to manage her diabetes.
options must be used. (3) Instructing the client to follow up with her provider
1. The nurse establishes a goal with the in 4 weeks may be important in the overall
client to be able to self-administer management of this client’s care, but does not
insulin injections. directly assist in teaching the client the necessary
2. The nurse assesses the client’s level of skills to manage her diabetes.
knowledge about how to administer (4) CORRECT: Teaching the client how to administer
insulin injections. her own medication is the best example of
3. The nurse evaluates the client while the nurse’s role as a client advocate, because this
selfadministering action directly helps the client develop self-advocacy
insulin injections. skills.
4. The nurse establishes the diagnosis of 3. The Answer is 2
knowledge deficit. A client is seen for an outpatient appointment and
25. The nurse administers the first dose of asks the nurse if he can obtain a copy of his medical
chemotherapy to a client on an oncology record. The nurse knows the client has the right to
unit. The nurse knows that which of read and copy his medical records, and that this is
the following activities is appropriate to guaranteed by virtue of which of the following?
delegate to the LPN? Category: Client rights
1. Obtain the client’s blood pressure. (1) The Code of Ethics for Nurses does not address
2. Provide teaching about the side effects this issue.
of chemotherapy. (2) CORRECT: HIPAA protects the patient’s right
3. Administer the second dose of to review, copy, and request amendments to his
chemotherapy. medical records.
4. Flush the client’s central line with (3) The Patient Self-Determination Act does not
heparin. address whether a patient may read and copy his
NCLEX-RN® Exam Content Review and Practice medical records.
122 (4) The Americans with Disabilities Act does not
1. The Answer is 1 address whether a patient may read and copy his
A 58-year-old man with head and neck cancer is medical records.
admitted to the hospital and tells the nurse he does 4. The Answer is 1
not want parenteral nutritional therapy as his cancer After receiving report at the start of the evening
progresses. The nurse explains he can specify his shift, which of the following clients should the nurse
wishes by creating an advance directive. The nurse attend to FIRST?
knows that the requirement to provide clients with Category: Establishing priorities
this type of information can be found in which of (1) CORRECT: Hyperkalemia is a potentially
the following? serious condition that, in a client undergoing
Category: Advance directives treatment for non-Hodgkin lymphoma, could
(1) CORRECT: The 1990 Patient Self-Determination indicate tumor lysis syndrome.
Act was passed by Congress to ensure that (2) A 21-year-old woman with sickle-cell anemia
upon admission to hospitals, long-term care with pain of 6 on a scale of 1–10 should be
facilities, and home health agencies, patients Chapter Quiz Answers and Explanations
are informed that they have the right to accept 4: Safe and Effective Care Environment:
or refuse medical care, as well as to specify in Management of Care
advance (through advance directives) what their 123
wishes are. attended to, but her condition is not as urgent
(2) Nursing Scope and Standards of Practice do not as the client’s condition described in answer
address advance directives. choice (1).
(3) The Patient Protection and Affordable Care Act (3) A 55-year-old woman with ovarian cancer waiting
does not address advance directives. to be discharged should be attended to but
(4) The Patients’ Bill of Rights does not address does not require immediate attention.
advance directives. (4) A 72-year-old man with COPD and a pulse
2. The Answer is 4 oximetry
A 14-year-old girl newly diagnosed with diabetes of 96% on room air does not require immediate
is preparing for discharge. Which of the following attention.
activities BEST describes the nurse’s role as a client 5. The Answer is 4
advocate? A 34-year-old woman who developed Stevens-
Category: Advocacy Johnson
syndrome while undergoing treatment with A pregnant woman at 15 weeks’ gestation is
carbamazepine scheduled
(Tegretol) is being transferred in stable for an amniocentesis. As the client is being
condition from the intensive care unit to the medical prepped for the procedure, it becomes clear to the
unit. There are 4 beds available. The nurse knows the nurse that the client doesn’t fully understand the
BEST choice of roommates for this client is which of risks and benefits associated with the procedure.
the following? Which of the following describe the nurse’s role in
Category: Concepts of management obtaining informed consent? Select all that apply.
(1) A client with MRSA may be an infection risk for Category: Informed consent
an individual with altered skin integrity. NCLEX-RN® Exam Content Review and Practice
(2) A client diagnosed with diarrhea may be an 124
infection risk for an individual with altered skin (1) It is the physician’s duty to provide information
integrity. to the client-related to risks and benefits.
(3) A client with fever of unknown origin may be an (2) It is the physician’s duty to provide information
infection risk for an individual with altered skin to the client related to alternatives.
integrity. (3) CORRECT: One of the nurse’s roles in the
(4) CORRECT: A client with Stevens-Johnson informed consent process is to witness the
syndrome is likely to have severe skin integrity signature on the consent form.
issues, including blistering and skin shedding, (4) CORRECT: One of the nurse’s roles in the
which can place the client at high risk for infection. informed consent process is to advocate for the
Atrial fibrillation is not an infectious process. client by ensuring she has been provided the
6. The Answer is 1 and 3 necessary
A 72-year-old man who had a stroke is being information to make an informed decision.
transferred 9. The Answer is 1
from a medical unit to a rehabilitation center. The nurse noticed an increase in the prevalence of
The nurse case manager is assisting in the process. pressure ulcers among clients in an intensive care
The nurse knows that the goals of case management unit. She documented her findings and worked with
include which of the following? Select all that apply. her manager to develop and implement a new policy
Category: Case management using a pressure ulcer risk assessment scale. Which
(1) CORRECT: One of the primary goals of case of the following BEST describes the nurse’s actions?
management is to improve the coordination of Category: Performance improvement (quality
care. improvement)
(2) Although case managers do make referrals to (1) CORRECT: Quality improvement includes
local organizations, this is not a goal of case activities such as identifying opportunities and
management. developing policies for improving the quality of
(3) CORRECT: One of the primary goals of case nursing practice. Identifying an increase in pressure
management is to reduce fragmentation of care. ulcers and implementing a policy aimed at
(4) Although case managers help to make discharges improving the assessment and prevention of
more efficient, this is not a goal of case management. pressure ulcers best fits the definition of quality
7. The Answer is 2 improvement.
An 18-year-old client with acute lymphocytic leukemia (2) The nurse may have collaborated (or worked
is admitted to the bone marrow transplantation together) with colleagues, but this is not the best
unit. His family is having trouble dealing with the choice.
emotional and financial pressures of his disease. The (3) Advocacy refers to the nurse’s duty to act on
nurse, case manager, physician, and social worker behalf of the client. Although reducing pressure
meet to discuss the plan of care. The nurse knows ulcers may indirectly advocate for the client, it is
this type of interdisciplinary interaction is BEST not the best answer choice.
referred to as which of the following? (4) Case management refers to the coordination of
Category: Collaboration with interdisciplinary team care to reduce fragmentation and costs, as well
(1) Case management refers to the coordination of as to improve quality and outcomes.
care to reduce fragmentation and improve quality 10. The Answer is 2
and outcomes, as well as to reduce costs. The nurse is working on a surgical unit. Which of the
(2) CORRECT: The interdisciplinary interaction following tasks would be appropriate for the nurse to
between different health care professions, such delegate to nursing assistive personnel (NAP)?
as nursing, medicine, and social work, is known Category: Delegation
as collaboration. (1) Assisting a new postoperative client to the
(3) Although the health care team may have been bathroom
cooperating, or operating as a team, the term is a task the registered nurse or another
“cooperation” does not specifically refer to the licensed individual, such as an LVN/LPN, should
concept of interdisciplinary action. perform.
(4) Although the health care team may have been (2) CORRECT: Setting up the client’s lunch trays is
operating in a collegial (cooperative and professional) an appropriate task to delegate to the UAP.
manner, this term does not specifically (3) Changing a central line dressing is a task the
refer to the concept of interdisciplinary action. registered
8. The Answer is 3 and 4 nurse or another licensed individual, such
as an LVN/LPN, should perform.
(4) Teaching a client how to administer discharge NAP’s competency has been verified.
medications is a task the registered nurse or 14. The Answer is 2
another licensed individual, such as a pharmacist, A well-known actor has been admitted to an
should perform. ambulatory
11. The Answer is 1 and 2 surgical unit. The nurse notices a staff member
The nurse has been asked to administer a drug by IV who is not involved in the client’s care reading his
push. She is uncertain whether or not this task falls medical record. The nurse knows she should FIRST
within her scope of practice. The nurse knows that do which of the following?
which of the following are the BEST sources to refer Category: Confidentiality and information security
to for information related to her scope of practice in (1) A staff member who is not involved in the client’s
this situation? Select all that apply. care is not authorized to access private
Category: Legal rights and responsibilities information.
(1) CORRECT: Hospital and unit policies and (2) CORRECT: An individual not involved in the
procedures care of the client does not have a legitimate need
may outline specific information about to access the medical record. The nurse should
who can administer which drugs by what route. protect the client’s right to privacy by ensuring
(2) CORRECT: Nurse Practice Acts (NPAs) are only authorized individuals access medical
laws in each state that define the scope of practice records.
for nursing. (3) The nurse should do more than simply inform
(3) Although the ordering physician may be able the client of the breach.
to provide helpful information related to the (4) The nurse should do more than simply ask a
drug itself, the ordering physician is not the best physician
source of information related to nurse licensing if it’s acceptable for the staff member to
and scope-of-practice issues. access the client’s medical records.
(4) Although the hospital pharmacist may be able to 15. The Answer is 1 and 2
provide helpful information related to the drug The nurse is learning how to use the hospital’s new
itself, the pharmacist is not the best source of electronic medication administration record. The
information related to nurse licensing and scopeof- nurse knows this tool has the potential to do which
practice issues. of the following? Select all that apply.
12. The Answer is 1 Category: Information technology
A 20-year-old client with leukemia has consented to (1) CORRECT: Electronic medication administration
a blood transfusion against the wishes of his family, records have the potential to reduce medication
who are all Jehovah’s Witnesses. The nurse knows administration errors.
that which of the following ethical principles BEST (2) CORRECT: Electronic medication administration
supports this decision? records have the potential to improve access
4: Safe and Effective Care Environment: to client information at the point of care.
Management of Care (3) It is always the nurses’ responsibility to document
125 medication administration.
Category: Ethical practice (4) It is always the nurses’ responsibility to verify the
(1) CORRECT: Autonomy refers to the right of doses of drugs being administered.
individuals to make decisions for themselves. 16. The Answer is 3
(2) Beneficence refers to the nurse’s duty to do what The nurse uses the Internet to receive
is good for the client. electrocardiogram
(3) Nonmaleficence refers to the nurse’s duty to do results from a client living in a nursing home.
no harm. The nurse knows this type of information technology
(4) Justice refers to the concept of fair and equitable is BEST described as which of the following?
treatment. Category: Information technology
13. The Answer is 2 (1) Encryption refers to the conversion of information
The nurse wants to delegate the task of showering an to code during transmission to keep the
elderly client in a wheelchair to nursing assistive information secure.
personnel (2) Telecommunications refers to the electronic
(NAP). Before delegating a task to the NAP, transmission of data over phone-based lines.
the nurse should FIRST ensure which of the following (3) CORRECT: Telehealth uses transmissions via
is accomplished? telecommunications technology to transmit
Category: Delegation health information remotely.
(1) Supervising the NAP does not ensure that the (4) Nursing informatics refers to a specialty of nursing
NAP’s competency has been verified. that integrates nursing and computer science.
(2) CORRECT: Prior to delegating a task appropriate NCLEX-RN® Exam Content Review and Practice
for the NAP, the nurse should first ensure 126
that competency has been verified during the 17. The Answer is 2
NAP’s orientation. The nurse is preparing to transfer a client to the
(3) The fact that the NAP has performed the task operating room. She knows that adhering to the
before does not ensure that the NAP’s competency hospital
has been verified. policy for client handoffs BEST ensures which
(4) The fact that the NAP has received the assignment of the following?
during report does not ensure that the Category: Continuity of care
(1) Case management does not address the issue of practice.
handoffs between caregivers. (4) Developing policies are activities that fall within
(2) CORRECT: Improving handoff communication registered nurses’ scope of practice.
allows each caregiver to communicate completely, 21. The Answer is 2 and 3
effectively, and consistently as the client The nurse in a maternity unit is caring for a client
transitions to different departments in the hospital. who has just delivered twins. The client voices
This process improves the continuity of care. concern about her ability to manage when she gets
(3) Confidentiality protection does not address the home. Which of the following statements BEST
issue of handoffs between caregivers. 4: Safe and Effective Care Environment:
(4) Collaboration does not address the issue of Management of Care
handoffs between caregivers. 127
18. The Answer is 2 and 4 illustrate quality care delivery by the nurse? Select
The nurse is preparing to perform an admission all that apply.
assessment on a 28-year-old man being admitted for Category: Referrals
Crohn’s disease. The nurse knows that according to (1) This is not an appropriate answer to a new mother
the Patients’ Bill of Rights, this client is responsible expressing concerns about her ability to cope.
for which of the following? Select all that apply. (2) CORRECT: A referral to home health care provides
Category: Client rights the client with opportunities for support
(1) Consenting to treatment is not a patient and assistance during this transition.
responsibility (3) CORRECT: A referral to support groups provides
delineated in the Patients’ Bill of Rights; the client with opportunities for support
it is a patient right. and assistance during this transition.
(2) CORRECT: According to the American Hospital (4) This is not an appropriate answer to a new
Association, patients’ responsibilities include mother expressing concerns about her ability to
(among other things) providing information cope.
about medications. 22. The Answer is 2
(3) Providing proof of insurance is not a patient After responding to a code, several staff nurses
responsibility delineated in the Patients’ Bill of express concerns over their confidence levels and
Rights. performance to the nurse in charge of the hospital’s
(4) CORRECT: According to the American Hospital performance improvement program. The nurse in
Association, patients’ responsibilities include charge knows the BEST way to evaluate and improve
(among other things) providing information performance is to implement which of the following?
about past illnesses. Category: Performance improvement (quality
19. The Answer is 1 improvement)
The nurse is caring for a 41-year-old man with a new (1) Although collecting and analyzing performance
colostomy. As part of the care planning for this client, data can be helpful in understanding performance
the nurse knows a referral to which of the following issues, it is not the best way to improve
will be the priority? performance.
Category: Referral (2) CORRECT: Mock codes can improve performance
(1) CORRECT: A referral to a certified wound, by encouraging teamwork, improving
ostomy, and continence nurse (CWOCN), if communication and skill-building, and enhancing
available, is important to the management confidence of caregivers.
of a client with a colostomy during and after (3) Studies suggest that, although important for
hospitalization. learning, training courses are not the best way
(2) Although a referral to social services might be to improve performance.
necessary based on other factors, it is not the priority (4) A written competency exam is not the best way
in the situation described. to evaluate and improve performance because it
(3) Although a referral to physical therapy might tests knowledge rather than performance.
be necessary based on other factors, it is not the 23. The Answer is 2
priority in the situation described. A client is being treated for uncontrolled hypertension.
(4) Although a referral to occupational therapy The nurse knows that the involvement of nursing,
might be necessary based on other factors, it is pharmacy, cardiology, and nutritional services
not the priority in the situation described. is an example of which of the following approaches?
20. The Answer is 1 Category: Collaboration with interdisciplinary team
An RN is in charge of a team on a medical/surgical (1) The concept of managed care is not related to a
unit that includes an LPN. The RN understands that multidisciplinary approach.
which of the following is an activity that falls within (2) CORRECT: A multidisciplinary approach
the scope of practice of an LPN? involves members from nursing, medicine, and
Category: Supervision other health care teams in the management of a
(1) CORRECT: Administering oral medications is particular disorder, in order to provide a more
an appropriate activity for the LPN. comprehensive and individualized approach.
(2) Collaborating with social services to develop a (3) Case management is not related to a
discharge plan is an activity that falls within registered multidisciplinary
nurses’ scope of practice. approach.
(3) Formulating a nursing diagnosis is an activity (4) The concept of performance improvement is not
that falls within registered nurses’ scope of related to a multidisciplinary approach.
24. The Answer is 2, 4, 1, 3 chapter 5
The nurse is caring for a client newly diagnosed with 130
diabetes, and performs the following tasks. Place the NCLEX-RN® Exam Content Review and Practice
tasks the nurse would perform in the appropriate Safety Background
order. All options must be used. Begin your review of safety issues by making sure you
Category: Establishing priorities understand the various elements that
(1) Establishing outcomes/planning is the third step are involved in client safety and accident prevention,
in the nursing process. including developmental- or age-related
(2) Assessment is the first step in the nursing process. risks specific to infants, toddlers, school-age children,
(3) Evaluation is the last step in the nursing process. adolescents, adults, and older adults
(4) Diagnosis is the second step in the nursing (geriatric clients), as follows:
process. • Infants: Educate parents or caretakers regarding
25. The Answer is 1 infant safety and their responsibility to
The nurse administers the first dose of chemotherapy take proper precautions to prevent injury. Infants
to a client on an oncology unit. The nurse knows should be placed on their backs after
that which of the following activities is appropriate eating and while sleeping, and transported using car
to delegate to an LPN? seats. This age group has a high
Category: Delegation risk for falls and burns.
(1) CORRECT: An LPN may obtain the client’s • Toddlers: Mobility and curiosity create safety issues
blood pressure. including poisoning, choking, and
(2) Providing teaching about the side effects of drowning. Keep medications, poisons, and cleaning
chemotherapy supplies in locked cabinets. Toddlers
is not an activity that should be performed should be transported only in car seats.
by an LPN. • School-age children: Time spent in school and
(3) Administering a dose of chemotherapy is not an playing with friends creates new safety
activity that should be performed by an LPN. risks. Emphasize traffic safety, water safety, fire
(4) Flushing the client’s central line is not an activity safety, and the dangers of strangers.
that should be performed by an LPN. Car seats and/or booster seats should be used for
children until adult seat belts fit correctly,
129 which typically does not occur until the child reaches
Safety and infection control are closely linked areas 4´9˝, weighs at least 80 lb.,
that are particularly important in keeping and is between ages 8 and 12. (Age and height/weight
clients healthy or helping them get well. Both home requirements vary by state.)
safety and safety in a hospital setting • Adolescents: Their sense of independence and
are covered in this topic area. An important part of invincibility, and access to cars, creates
hospital safety is the control of infections risk. Emphasize driver education, alcohol and
that clients might acquire while they are in the substance abuse education, and sexual
hospital (called nosocomial infections). These health information.
infections might not be related to their original • Adults: Safety risks for this age group include home,
condition or reason for admission but may workplace, and leisure activities.
have tremendous impact on their ability to heal. Educate adults about motor vehicle, fire, and firearm
On the NCLEX-RN® exam, you can expect safety.
approximately 12 percent of the questions to • Older adults: Aging issues, both physical and
relate to Safety and Infection Control. This cognitive, impact safety, particularly
subcategory focuses on protecting clients and regarding falls and side effects of medication.
health care personnel from health and environmental Possibilities of elder abuse and motor
hazards. vehicle accidents also increase for older adults.
Exam content related to the Safety and Infection You also need to understand the elements that are
Control subcategory includes, but is not involved in client safety and accident prevention
limited to, the following areas: related to the care environment. For example, in a
• Accident/injury prevention hospital setting, fall risks are most
• Emergency response plan common in infants and geriatric clients. Know the
• Ergonomic principles elements of a fall prevention program,
• Error prevention including the different steps taken based on the age
• Handling hazardous and infectious materials of the client. Safety also involves the use
• Home safety of restraints to limit mobility, and taking proper
• Reporting of incident/event/irregular seizure precautions. You should be able to
occurrence/variance explain these precautions, which include the use of
• Safe use of equipment physical restraints, and the need for suction
• Security plan and oxygen equipment.
• Standard precautions/transmission-based Infection Control Background
precautions/surgical asepsis To correctly answer questions about infection control,
• Use of restraints/safety devices begin by making sure you understand
Safe and Effective Care some basic information about etiologic agents and the
EN vironment: chain of infection.
Safety and IN fection control 131
5: Safe and Effective Care Environment: Safety You must understand ergonomic principles when
and Infection Control caring for clients. This includes using assistive
An etiologic agent is any pathogen that can cause an devices and proper lifting techniques. Assess a client’s
infection. Etiologic agents include bacteria, ability to balance and use assistive
fungi, protozoa, rickettsiae, and helminthes. devices, such as crutches or a walker, and use that
There are six elements in the chain of infection: information to help develop an appropriate
1. Pathogen: An infectious agent, like a bacteria or care plan.
virus. For clients with repetitive stress injuries, provide
2. Reservoirs: Any environment that is favorable for instruction and information about body
growth and reproduction of infectious positions that can minimize or prevent these injuries.
agents. A reservoir may be animate or inanimate. For clients with conditions that cause
Human systems that can act as reservoirs stress to specific skeletal or muscular groups,
include blood, respiratory, gastrointestinal, understand and educate the clients about necessary
reproductive, and urinary. modifications. These may include changing positions
3. Portal of exit: A place where the infectious frequently, and performing routine
organisms get out of a host. Any of the stretching exercises for the shoulders, neck, arms,
abovementioned hands, and fingers.
systems may be portals of exit. Error Prevention
4. Method of transmission: The way an infectious Medication and allergies are primary areas for error.
organism is transferred from reservoir to Error prevention, therefore, begins
host. This happens in one of three ways: direct with proper identification of the client. You should be
contact, indirect contact via a vector, or able to identify client allergies and
through the air (airborne). intervene appropriately, know how to verify
5. Portal of entry: A place where an infectious agent appropriateness and/or accuracy of a treatment
enters the susceptible host. A portal of or medication order, and be able to prevent treatment
entry may also be through a system that can act as a errors using critical thinking and by
reservoir. following policies.
6. Susceptible host: A client, staff member, or other Handling Hazardous and Infectious Materials
individual at risk for infection. It is important to be aware of the elements of
Now let’s review the most important concepts covered employee safety. These include the safe use of
by the Safety and Infection Control equipment, safe handling of hazardous chemicals, and
subcategory on the NCLEX-RN® exam. the use of Material Safety Data Sheets
Accident/Injury Prevention (MSDS), which are Occupational Safety and Health
To help protect clients from accident and injury, you Administration (OSHA)-required handouts
should assess risk factors upon the client’s that describe all chemical agents in an employment
admission and identify appropriate methods to setting.
minimize risk of injury. This includes Know the standard precautions to protect against
knowledge of the developmental stages mentioned blood-borne pathogen exposure. (OSHA
previously in the Safety Background section, has written standards that include recommendations
the client’s lifestyle, and his or her knowledge of from the Centers for Disease Control
safety precautions. and Prevention [CDC], including the use of gloves and
You should know how to identify specific deficits, such face and eye protection.) You must
as sight, hearing, and other sensory know what to do in case of a needlestick, the
perceptions that may impact client safety. It’s also standards for environmental infection control,
important to be able to teach families how and necessary information related to latex allergies
to properly install and use infant and child car seats. for both staff and clients. Be sure latexfree
Emergency Response Plan gloves and latex-free carts are available and used as
The Joint Commission requires hospitals to have a necessary.
disaster plan and periodically practice 133
response to the plan. You are responsible for knowing 5: Safe and Effective Care Environment: Safety
your role in disaster response. and Infection Control
Know all of the steps involved in fire safety in a You also need to be able to identify biohazardous,
hospital setting. If a fire occurs, first get flammable, and infectious materials; know
clients out of danger, then work to contain the fire, how to control the spread of infectious agents; follow
and finally determine the order in which procedures for handling biohazardous
to evacuate clients, including identification of clients materials; and be able to demonstrate safe handling
who must be evacuated in beds or on techniques to staff and clients.
132 The Needlestick Safety and Prevention Act is
NCLEX-RN® Exam Content Review and Practice significant legislation that was enacted to
stretchers (horizontally). You must also know how to protect health care workers. Do not re-cap needles or
teach clients about fire safety at home, bend or break them before disposal.
such as knowing emergency numbers, installing and Ensure that sharps containers are in each client room
testing smoke alarms, acquiring fire and medication area.
extinguishers, and so on. Home Safety
Ergonomic Principles Home safety includes evaluating the client’s home
care environment for fire risk, environmental
hazards, and other elements that present a risk of newborn nursery security event, and bomb threat.
accident or injury to the client. It also Nurses often participate in developing
involves working with the client and the client’s family security and emergency plans, so you should be
and significant others to recommend prepared to do so. Clinical decision-making
modifications, such as lighting or handrails. skills and critical thinking are important components
Home safety also includes teaching clients self-care, of the development and successful
and teaching parents how to care for execution of a security plan.
children. It also includes teaching preventive Standard Precautions/Transmission-based
measures for home care, such as encouraging Precautions/Surgical Asepsis
the client to use protective equipment when using There are a variety of different precautions that
devices that can cause injury. should be used to prevent the spread of infection.
Reporting of Incident/Event/Irregular These include “standard” precautions that should
Occurrence/ always be used, precautions specifically
Variance aimed at the transmission of pathogenic
Incident reports are tools designed to provide microorganisms, and surgical asepsis (sterile
information about potential areas of exposure techniques).
to liability, and are also used to identify problems and Standard Precautions
develop solutions to prevent the same In addition to understanding the chain of infection,
incident from happening again. Being able to you should be able to apply standard precautions
accurately identify situations requiring completion (such as handwashing, wearing gloves and gowns,
of an incident or unusual occurrence report is an and using face protection, such as
important skill. Although each hospital masks, goggles, and face shields) with respect to
has its own procedures, the most important thing is hand hygiene, blood, bodily fluids, excretions,
to prevent further injury. In addition 135
to reporting, you need to evaluate the response to the 5: Safe and Effective Care Environment: Safety
event to ensure it helped to correct and Infection Control
the situation and to prevent further errors. Record the and secretions. These principles apply whether or not
facts of the incident in the medical the skin and mucous membranes are
record, but do not include a copy of the incident report intact, and should always be used in caring for clients
or make reference to its existence in across all diagnoses and all care settings.
the medical record. Be aware of how, and in what order, to correctly put
Safe Use of Equipment on and remove personal protective
You must make sure that equipment needed to equipment (PPE). Perform hand hygiene first. Then
perform client care procedures and treatments before making contact with the client,
is used safely and properly. This includes inspecting and preferably outside the room, put on PPE: gown
equipment to make sure it is safe to use. first, then the mask, then eye protection,
If a client needs to use equipment at home, you must and gloves last. The steps reverse for removing PPE:
teach the client how to use the equipment remove gloves first, then eye
safely and properly. protection, then mask, then gown, with hand hygiene
134 coming last.
NCLEX-RN® Exam Content Review and Practice Transmission-based Precautions
If equipment is not safe, or if it malfunctions, you Transmission-based precautions limit the spread of
should stop using it, label it as broken, pathogenic microorganisms. You should
remove it from any possible use, put it in a designated be able to compare and contrast airborne, droplet,
area for broken equipment (if available), and contact precautions; know when to
and report the problem to the appropriate person. use each; and know when multiple precautions may
Security Plan be needed. For example, when small
You may be asked to triage injured or ill clients in an (< 5 mcm) pathogen-infected droplets remain
emergency, and to identify those in need suspended in the air over time and travel distances
of urgent care. The exam focuses on airway, greater than 3 feet, use airborne precautions.
breathing, circulation, and neurological deficits. Pathogens may include measles (rubeola),
The order is: chickenpox (varicella), and tuberculosis, among
1. Clear and open the airway. others. Use droplet precautions for larger
2. Assess for respiratory distress. (> 5 mcm) pathogen-infected droplets that travel 3
3. Assess quality of breathing (rate, and color of skin, feet or less via coughing, sneezing, and so
lips, and fingernails) and auscultate on. An example of this type of pathogen is
lungs. Haemophilus influenzae. Use contact precautions
4. Check pulse. with known or suspected microorganisms transmitted
5. Assess for external bleeding. by direct hand-to-skin contact or indirect
6. Take blood pressure. contact with surfaces (Clostridium difficile, herpes
7. Assess the level of consciousness and papillary simplex, impetigo, etc.).
response, and the weakness or paralysis You should also be able to identify infectious agents
of extremities. that require transmission-based precautions,
You should also be aware of your facility’s procedures and specific precautions used in cases of drug-
and protocols during an evacuation, resistant infections.
Surgical Asepsis
You need to understand the principles of surgical 4. The nurse is preparing to test a client who
asepsis—the practices necessary to maintain has allergies from an unknown cause. Which
objects and areas free of microorganisms—also known of the following tests should the nurse
as sterile techniques. Know how perform?
to use these techniques in implementing a variety of 1. Tzanck test
procedures, including IV therapy and 2. Patch test
urinary catheterization. 3. Rinne test
The basic principles of surgical asepsis are: 4. Stress test
• Every object used in a sterile field must be sterile. 5. The nurse is preparing a client with acquired
• If a sterile object touches an unsterile object, it is immunodeficiency syndrome (AIDS) for
no longer sterile. discharge to home. Which of the following
• If a sterile object is out of view, or below waist level, instructions should the nurse include?
it is considered unsterile. 1. “Avoid sharing articles such as razors and
• A sterile object can become unsterile through toothbrushes.”
exposure to airborne microorganisms. 2. “Do not share eating utensils with family
• Fluids flow in the direction of gravity. members.”
• Moisture passing through a sterile object can draw 3. “Limit the time you spend in public
microorganisms from unsterile surfaces places.”
above or below through capillary action. 4. “Avoid eating food from serving dishes
• The edges of a sterile field are unsterile. shared with others.”
• The skin cannot be sterilized. 6. The nurse is preparing to administer a
You need to understand the difference between tuberculin (Mantoux) skin test to a client
chemical (medication) and physical restraints suspected of having tuberculosis (TB). The
(bedside rails, jacket, and extremity strap restraints). nurse knows that the test will reveal which of
In addition, you need to know how to the following?
utilize restraints safely, effectively, and only when 1. How long the client has been infected
necessary, as well as how and with what with TB
frequency to monitor clients who are restrained. It’s 2. Active TB infection
also important to understand the legal 3. Latent TB infection
implications of restraining clients, as well as agency- 4. Whether the client has been infected with
specific policies and procedures. This TB bacteria
includes understanding that seizures may necessitate 7. An older adult has been admitted with
restraint. diagnosis of stroke and a history of
5: Safe and Effective Care Environment: Safety dementia. Which of the following nursing
and Infection Control diagnoses has the highest priority for this
137 client?
1. The physician orders an MRI of the brain for 1. Bathing/hygiene self-care deficit
an adult male client. Which of the following 2. Risk for injury
findings in the client’s history should the 3. Impaired physical mobility
nurse report to the physician? 4. Disturbed thought processes
1. Allergy to contrast dye Chapter Quiz
2. Implanted cardiac pacemaker NCLEX-RN® Exam Content Review and Practice
3. Chronic obstructive pulmonary disease 138
(COPD) 8. The nurse has just administered insulin to
4. Hernia repair a diabetic client. In which of the following
2. The nurse is developing a care plan for a ways should the nurse dispose of the needle?
client with hepatitis C. The nurse knows that 1. Re-cap the needle and discard it in the
the primary route of transmission of this nearest puncture-resistant container.
hepatitis virus is which of the following? 2. Re-cap the needle and discard it in the
1. Contaminated food nearest biohazard container.
2. Feces 3. Discard the needle in a punctureresistant
3. Blood container.
4. Sputum 4. Break the needle and discard it in the
3. The nurse is preparing to discharge a nearest puncture-resistant container.
client with rheumatic heart disease who is 9. The nurse is preparing to administer
recovering from endocarditis. Which of the packed red blood cells (PRBCs) to a client.
following statements from the client indicates Arrange the following steps in the order
that the client understands the teaching? the nurse should perform them. All options
1. “I’m so glad I don’t need any more must be used.
antibiotics now that I’m feeling better.” 1. Explain the procedure to the client.
2. “I can restart my exercise program in a 2. Obtain the client’s vital signs.
day or two.” 3. Assess that the client has a blood bank
3. “I will watch for signs of relapse the first identification armband.
few days after discharge.” 4. Obtain the PRBCs from the blood
4. “I will inform my dentist should I ever bank according to hospital policy and
need any dental work.” perform a visual check of the blood.
5. Perform a bedside identification and risk of falls. Which of the following should
blood product verification by two the nurse collect?
licensed individuals. 1. The facility’s restraint policy
6. Verify the physician order. 2. Gait, balance, and visual impairment
7. Prime the transfusion tubing with a 0.9% information
sodium chloride solution. 3. Psychosocial history
10. Two nurses are preparing to lift a client up 4. The facility’s environmental safety plan
in bed. Which of the following should the 16. The nurse is administering nightly
nurses do to help avoid injuring their backs? medications, which include an
1. Bend from the waist. anticoagulant and a stool softener. Which
2. Lift with the back, not with the legs. of the following should the nurse do FIRST
3. Lower the head of the bed to about 30 before administering the medications?
degrees, if the client can tolerate it. 1. Scan the medication label and the
4. Make certain the bed is in a reasonably client’s wristband.
high position. 2. Ask the client his or her name to
11. In the emergency room, the nurse assesses properly identify this client as the one
a 4-year-old child suspected of having for whom the medications were ordered.
measles. Which of the following kinds of 3. Match the client’s date of birth and
precautions should the nurse initiate? name on the client’s wristband with the
1. Contact precautions same information on the medication
2. Droplet precautions order.
3. Airborne precautions 4. Match the client’s name and room
4. Reverse isolation number with the medication order.
12. A female client comes to the Emergency 17. The physician verbally orders a medication
Department complaining of vaginal for a client during an emergency code.
discharge, irritation of the vagina, and the Which of the following should the nurse do?
need to urinate often. The nurse suspects a 1. Repeat the order back to the physician
sexually transmitted disease (STD), and the for confirmation and administer it.
physician orders diagnostic testing of the 2. Retrieve the medication and administer
vaginal discharge. Which of the following it.
STDs does the nurse know must be reported 3. Write the order down, retrieve the
to the Department of Public Health? medication, and administer it.
1. Genital herpes 4. Read the order to another nurse, have
2. Human papillomavirus infection that nurse retrieve the medication, and
3. Gonorrhea stay with the client.
4. Trichomoniasis 18. The client has a new order for placement of a
13. An elderly client, who is not oriented to Foley catheter due to urinary retention. Which
time, place, or person, had a total hip of the following should the nurse do before
replacement. The client is attempting to get starting the procedure? Select all that apply.
out of bed and pull out the IV line that is 1. The nurse should confirm the client’s
infusing antibiotics. The client has bilateral identity, because a procedure requires
soft wrist restraints and a vest restraint. proper identification.
Which of the following interventions by the 2. The nurse should confirm the client’s
nurse are appropriate? Select all that apply. medical record number via the
1. Ask the client if he needs to use the wristband and order.
bathroom, and provide range-of-motion 3. Ask the client his or her name only,
exercises every 2 hours. because this is a procedure and not a
2. Document the type of restraint used and medication administration.
assess the need for continued use. 4. The nurse should confirm the client’s
3. Tie the restraints to the side rails of the name via the wristband and order.
bed. 19. Which of the following actions by the nurse
4. Obtain a new physician order for the is the MOST effective means of preventing
restraint every 12 hours. infection?
5. Observe for correct placement of 1. Washing hands after client contact
restraints. 2. Washing hands after removing gloves
6. Tie the restraints in a quick-release knot. 3. Hand hygiene between clients
14. The nurse is preparing to administer a 4. Hand hygiene before entry to a client’s
unit of PRBCs to an anemic client. After room and upon exit of a client’s room
obtaining the blood from the blood bank, 20. The client is an obese male with decubitus
the nurse must begin administering it ulcers. Treatment of the ulcers requires
within which of the following time periods? frequent turning and repositioning. The
1. 15 minutes nursing unit has a special lift that allows
2. 30 minutes for turning of clients and placement onto
3. 45 minutes a bedpan without any lifting on the part
4. 60 minutes of the staff. The client urgently requests
15. The nurse is assessing an elderly client for the bedpan. Because the lift apparatus
takes a few minutes to set up, which of the 1. File an incident report.
following should the nurse do? 2. Put the bed alarm back on.
1. Quickly assist the client onto the bedpan 3. Institute a client observer to sit with the
without the lift because he needs to use client and prevent further falls.
it urgentl 4. Notify the nurse manager.
2. Encourage the client to try to be patient, 25. The hospitalized client is receiving an
and set up the apparatus. infusion and the pump has malfunctioned.
3. Get the assistance of an aide to help lift Which of the following actions by the nurse
the client. is MOST appropriate once the infusion
4. Encourage the client to wear an has been stopped and restarted with a
incontinence brief. functioning pump?
21. The client has experienced multiple 1. Place a “Broken” sticker on the
episodes of hyperglycemia not manageable malfunctioning pump according to
by subcutaneous insulin injections. The hospital policy, and place the pump
client has an active order for infusion of in the designated malfunctioning
an insulin drip for glycemic management equipment area.
to be discontinued at bedtime, after which 2. Place the malfunctioning pump in the
the client is NPO. The client’s most recent utility room.
blood sugar level, taken at 3 p.m., was 60. 5: Safe and Effective Care Environment: Safety
Which of the following actions by the nurse and Infection Control
is the MOST appropriate? 141
1. The nurse should follow the order and 3. Remove the malfunctioning pump from
allow the insulin to infuse until bedtime. the client’s room and place with other
2. The nurse should recheck the client’s pumps.
blood sugar. 4. Place the malfunctioning pump to the
3. The nurse should bring this blood sugar side in the client’s room.
level to the physician’s attention and 26. The nurse completes a peripherally inserted
discuss stopping the infusion. central catheter (PICC) line dressing change
4. The nurse should seek advice from other for a home care client. When removing
nurses. the PPE, the nurse should do which of the
22. The adult children of a hospice home care following?
client inquire about whether it is safe to 1. Remove the mask and then the gloves.
hug their mother, because she has had a 2. Remove the gloves and then the mask.
methicillin-resistant Staphylococcus aureus 3. Remove only the gloves; there is no need
(MRSA) infection in the past. Which of the to wear a mask.
following statements by the children would 4. Remove only the mask; there is no need
indicate a need for further teaching by the to wear gloves.
nurse? 27. The client is found on the floor by the
1. “We should wash our hands frequently.” nursing assistive personnel (NAP). Once the
2. “We should use hand sanitizer.” client is safe, which of the following should
3. “Those of us with poor immune systems the nurse do next?
should be extra careful.” 1. Document the event in the client’s
4. “We should wear gowns and gloves at medical record and file an incident
all times when having contact with our report.
mother.” 2. File an incident report only.
23. The nurse witnesses another nurse, wearing 3. Document the event in the client’s
a gown and gloves, enter a client room medical record and have the NAP file an
labeled “Airborne Precautions.” Which incident report.
of the following actions by the witnessing 4. Document the event in the client’s
nurse is MOST appropriate? medical record only.
1. Notify the nurse manager to discuss 28. The nurse is making a home visit to an
policies with the other nurse. elderly client during the winter. The nurse
2. Ask a physician to give a presentation notices upon arrival that the client has the
on which precautions require which oven turned on with the oven door open,
types of personal protective equipment and is using it as a form of heat. Which of
(PPE). the following actions by the nurse is MOST
3. Remind the other nurse that she needs a appropriate?
mask in addition to a gown and gloves 1. Take care of the client’s medical needs
for airborne-type precautions. and do not get involved in the client’s
4. Ask the other nurse to look up the policy private matters.
about precautions. 2. Shut the oven off and continue with the
24. The nurse discovers a client on the floor in home visit.
the client’s hospital room. After examining 3. Report the event to the local Fire
the client and assisting him safely back to Department.
bed, which of the following should the nurse 4. Have a meeting with the client and
do FIRST? family and warn them of the fire and
safety risks of using the oven for heat. from the client indicates that the client understands
29. The medical center encounters a bomb the teaching?
threat. The emergency response team Category: Standard precautions/transmission-based
informs the staff that the threat is legitimate precautions/surgical asepsis
and that clients should start being (1) The client must take the full course of prescribed
evacuated. Which of the following clients antibiotics even if feeling better.
should the nurse begin evacuating FIRST to (2) The client must restrict activity as directed by the
the safe designated area? physician.
1. Ambulatory clients (3) Relapse may occur, but not until about 2 weeks
2. Bedridden clients after treatment stops.
3. ICU clients (4) CORRECT: Susceptible clients must understand
4. Infant clients the need for prophylactic antibiotics before, during,
30. The nurse discovers that the last dose of and after dental work.
intravenous antibiotic administered to a 4. The Answer is 2
client was the wrong dose. Which of the The nurse is preparing to test a client who has
following should the nurse do? allergies
1. Document the event in the client’s from an unknown cause. Which of the following
medical record only. tests should the nurse perform?
2. File an incident report, and document Category: Error prevention
the event in the client’s medical record. (1) The Tzanck test is used to detect the herpes
3. Document in the client’s medical record virus.
that an incident report was filed. (2) CORRECT: The patch test identifies the cause of
4. File an incident report, but don’t allergic contact sensitization and is indicated in
document the event in the client’s clients with suspected allergies or allergies from
medical record, because information an unknown cause.
about the incident is protected. (3) The Rinne test compares bone conduction to air
NCLEX-RN ® Exam Content Review and Practice conduction in the ears.
142 (4) A stress test assesses cardiovascular response to
NCLE X-1. The Answer is 2 increased workload.
The physician orders an MRI of the brain for an 5. The Answer is 1
adult male client. Which of the following findings The nurse is preparing a client with acquired
in the client’s history should the nurse report to the immunodeficiency
physician? syndrome (AIDS) for discharge to
Category: Accident/injury prevention; Safe use of home. Which of the following instructions should
equipment the nurse include?
(1) Allergy to contrast dye is contraindicated in CT Category: Standard precautions/transmission-based
scans with contrast, not MRI. precautions/surgical asepsis
(2) CORRECT: Metallic items, including metallic (1) CORRECT: The human immunodeficiency
implants such as a cardiac pacemaker, are virus (HIV), which causes AIDS, is concentrated
contraindicated mostly in blood and semen. The client should not
in MRI. share articles that may be contaminated with
(3) COPD is not a contraindication for MRI. blood, such as razors and toothbrushes.
(4) Hernia repair is not a contraindication for MRI. Chapter Quiz Answers and Explanations
2. The Answer is 3 5: Safe and Effective Care Environment: Safety
The nurse is developing a care plan for a client with and Infection Control
hepatitis C. The nurse knows that the primary route 143
of transmission of this hepatitis virus is which of the (2) HIV is not transmitted by sharing eating utensils.
following? (3) Someone with HIV does not need to limit time
Category: Standard precautions/transmission-based in public places.
precautions/surgical asepsis (4) HIV is not transmitted by sharing food from
(1) The hepatitis A (not hepatitis C) virus is serving dishes used by someone with AIDS.
transmitted 6. The Answer is 4
through the fecal-oral route, primarily The nurse is preparing to administer a tuberculin
through ingestion of contaminated food. (Mantoux) skin test to a client suspected of having
(2) The hepatitis A (not hepatitis C) virus is tuberculosis (TB). The nurse knows that the test will
transmitted reveal which of the following?
through the fecal-oral route, primarily Category: Standard precautions/transmission-based
through ingestion of contaminated food. precautions/surgical asepsis
(3) CORRECT: The hepatitis C virus is transmitted (1) The test cannot detect how long a person has
through blood and parenteral routes. been infected.
(4) The hepatitis C virus is not transmitted through (2) The test cannot detect whether the infection is
sputum. latent (inactive) or active.
3. The Answer is 4 (3) The test cannot detect whether the infection can
The nurse is preparing to discharge a client with be passed on to others.
rheumatic heart disease who is recovering from (4) CORRECT: A tuberculin skin test is performed
endocarditis. Which of the following statements to determine if a person has ever had TB.
7. The Answer is 2 it.
An older adult has been admitted with diagnosis of (4) CORRECT: The bed should be in a reasonably
stroke and a history of dementia. Which of the high position so the nurses do not have to lean.
following 11. The Answer is 3
nursing diagnoses has the highest priority for In the emergency room, the nurse assesses a 4-
this client? yearold
Category: Accident/injury prevention child suspected of having measles. Which of
(1) A bathing/hygiene self-care deficit would not be the following kinds of precautions should the nurse
the highest priority. initiate?
(2) CORRECT: Older adults with dementia are at Category: Standard precautions/transmission-based
risk for injury due to increased risk for falls, precautions/surgical asepsis
because they may not recognize their limitations, (1) Contact precautions are not used for measles.
despite immobility related to stroke. (2) Droplet precautions are not used for measles.
(3) Impaired physical mobility would not be the (3) CORRECT: Airborne precautions are used to
highest priority. prevent the transmission of infectious agents
(4) Disturbed thought processes would not be the that remain infectious over long distances when
highest priority. suspended in the air.
8. The Answer is 3 (4) Reverse isolation is not used for measles.
The nurse has just administered insulin to a diabetic 12. The Answer is 3
client. In which of the following ways should A female client comes to the Emergency Department
the nurse dispose of the needle? complaining of vaginal discharge, irritation of
Category: Handling hazardous and infectious the vagina, and the need to urinate often. The nurse
materials suspects a sexually transmitted disease (STD), and
(1) Needles should not be re-capped. the physician orders diagnostic testing of the vaginal
(2) Needles should not be re-capped and should be discharge. Which of the following STDs does the
placed in puncture-resistant containers, not just nurse know must be reported to the Department of
any biohazard container. Public Health?
(3) CORRECT: Needles and sharps should be Category: Standard precautions/transmission-based
placed in the nearest puncture-resistant container. precautions/surgical asepsis
(4) Needles should not be broken. (1) Genital herpes is not a reportable disease.
9. The Answer is 6, 3, 1, 2, 7, 4, 5 (2) Human papillomavirus infection is not a reportable
The nurse is preparing to administer packed red disease.
blood cells (PRBCs) to a client. Arrange the following (3) CORRECT: Gonorrhea must be reported to the
steps in the order the nurse should perform them. Department of Public Health.
All options must be used. (4) Trichomoniasis is not a reportable disease.
Category: Error prevention 13. The Answer is 1, 2, 5, and 6
(1) The third step is to explain the procedure to the An elderly client, who is not oriented to time, place,
client. or person, had a total hip replacement. The client is
(2) The fourth step is to obtain the client’s vital attempting to get out of bed and pull out the IV line
signs. that is infusing antibiotics. The client has bilateral
(3) The second step is to assess that the client has a soft wrist restraints and a vest restraint. Which of the
blood bank identification armband. following interventions by the nurse are appropriate?
(4) The sixth step is to obtain the PRBCs from the Select all that apply.
blood bank according to hospital policy and perform Category: Use of restraints/safety devices
a visual check of the blood. (1) CORRECT: Toileting and range-of-motion
(5) The last step is to perform a bedside identification exercises
and blood product verification by two should be provided every 2 hours while a
licensed individuals. client is in restraints.
(6) The first step is to verify the physician order. (2) CORRECT: The client must be assessed frequently
(7) The fifth step is to prime the transfusion tubing to ascertain when restraints can
with a 0.9% sodium chloride solution. be removed, and this information must be
10. The Answer is 4 documented.
Two nurses are preparing to lift a client up in bed. (3) Restraints should never be tied to the side rails,
Which of the following should the nurses do to help because this can cause injury if the side rail is
avoid injuring their backs? lowered without untying the restraint.
Category: Accident/injury prevention; Ergonomic (4) A new physician’s order must be obtained every
principles 24 hours if restraints are continued.
NCLEX-RN® Exam Content Review and Practice (5) CORRECT: The nurse should observe for correct
144 placement of restraints.
(1) When lifting or moving a client, nurses should (6) CORRECT: Restraints should be tied in knots
maintain the natural curve of the spine and not that can be released quickly and easily.
bend at the waist. 14. The Answer is 2
(2) When lifting or moving a client, nurses should The nurse is preparing to administer a unit of PRBCs
lift with the legs and not the back. to an anemic client. After obtaining the blood from
(3) When lifting or moving a client, place the bed in the blood bank, the nurse must begin administering
the Trendelenburg position if the client can tolerate it within which of the following time periods?
Category: Error prevention identity, because a procedure requires
(1) The nurse has up to 30 minutes to begin proper identification.
administering (2) CORRECT: The nurse should confirm the client’s
the blood product. medical record number via the wristband
(2) CORRECT: After obtaining the blood product and order.
from the blood bank, the nurse must begin (3) The nurse must always properly identify clients
administering the product within 30 minutes. for any and all treatments, not just for medication
(3) This time period is too long. administration.
(4) This time period is too long. (4) CORRECT: The nurse should confirm the client’s
5: Safe and Effective Care Environment: Safety name via the wristband and order.
and Infection Control 19. The Answer is 4
145 Which of the following actions by the nurse is the
15. The Answer is 2 MOST effective means of preventing infection?
The nurse is assessing an elderly client for risk of Category: Standard precautions/transmission-based
falls. Which of the following should the nurse collect? precautions/surgical asepsis
Category: Accident/injury prevention (1) Washing hands after client contact is appropriate
(1) The facility’s restraint policy is not relevant to a but not the most effective means of preventing an
fall risk assessment. infection, and alone, is not enough.
(2) CORRECT: Fall risk should include assessment (2) Washing hands after removing gloves is
of gait, balance, and visual impairment. appropriate
(3) The client’s psychosocial history is important but not the most effective means of preventing
but not in relation to risk for falls. an infection, and alone, is not enough.
(4) The facility’s environmental safety plan is not (3) Hand hygiene between clients is appropriate but
relevant to a fall risk assessment. not the most effective means of preventing an
16. The Answer is 3 infection, and alone, is not enough.
The nurse is administering nightly medications, (4) CORRECT: Hand hygiene should occur before
which include an anticoagulant and a stool softener. entry and upon exit of all client care transactions.
Which of the following should the nurse do FIRST NCLEX-RN® Exam Content Review and Practice
before administering the medications? 146
Category: Error prevention 20. The Answer is 2
(1) Scanning the medication label and the client’s The client is an obese male with decubitus ulcers.
wristband might be correct if the institution has Treatment of the ulcers requires frequent turning
a bar coding system, but it is not the first thing and repositioning. The nursing unit has a special lift
you would do. that allows for turning of clients and placement onto
(2) Asking the client his or her name might be correct, a bedpan without any lifting on the part of the staff.
but it is not the most complete answer. The client urgently requests the bedpan. Because the
(3) CORRECT: The 2012 National Patient Safety lift apparatus takes a few minutes to set up, which of
Goals require using a minimum of two patient the following should the nurse do?
identifiers as a means to promote the safest care Category: Ergonomic principles
and to prevent medication errors. (1) Quickly assisting the client onto the bedpan is a
(4) The room number should never be used as a client tempting answer and might happen frequently
identifier. in real life. However, it is not the best or safest
17. The Answer is 1 option for the client or the nurse.
The physician verbally orders a medication for a client (2) CORRECT: Encourage the client to wait while
during an emergency code. Which of the following the apparatus is set up. It is more important to
should the nurse do? prevent potential injury to the nurse. Nurses are
Category: Error prevention commonly affected by ergonomic injuries related
(1) CORRECT: In an emergency code situation, the to lifting and moving clients.
order can be repeated back to the physician for (3) This is not the best or safest option for the client
confirmation and given, as there is another nurse or the aide.
recording events of the code. (4) Encouraging the client to wear an incontinence
(2) The medication order should be confirmed with brief is inappropriate.
the physician first. 21. The Answer is 3
(3) The order should be repeated back to the The client has experienced multiple episodes of
physician hyperglycemia not manageable by subcutaneous
for verification before it is administered. insulin injections. The client has an active order for
(4) The nurse should confirm the order with the infusion of an insulin drip for glycemic management
physician first. to be discontinued at bedtime, after which the
18. The Answer is 1, 2, and 4 client is NPO. The client’s most recent blood sugar
The client has a new order for placement of a Foley level, taken at 3 p.m., was 60. Which of the following
catheter due to urinary retention. Which of the actions by the nurse is the MOST appropriate?
following Category: Error prevention
should the nurse do before starting the procedure? (1) The nurse has a duty to verify the order, given
Select all that apply. the change in circumstances. The blood sugar is
Category: Error prevention now low, and continuing an insulin drip has the
(1) CORRECT: The nurse should confirm the client’s potential to drop it to a dangerous level.
(2) The nurse would recheck the client’s blood sugar meeting the patient’s care needs.
level only if there was reason to believe it might (2) CORRECT: Putting the bed alarm back on is the
be in error. most appropriate first step to promote immediate
(3) CORRECT: The most appropriate action is to safety of the client.
contact the physician and discuss stopping the (3) Instituting a client observer might be appropriate,
infusion, based on the last blood sugar level. but not enough information about the
(4) The nurse might ask a colleague for advice, but circumstances of the client and the manner in
the most appropriate action is to discuss the situation which he got to the floor is given in the question
with the physician. stem.
22. The Answer is 4 (4) Notifying the nurse manager might be
The adult children of a hospice home care client appropriate,
inquire about whether it is safe to hug their mother, but not enough information about the
because she has had a methicillin-resistant circumstances of the client and the manner in
Staphylococcus which he got to the floor is given in the question
aureus (MRSA) infection in the past. Which stem.
of the following statements by the children would 25. The Answer is 1
indicate a need for further teaching by the nurse? The hospitalized client is receiving an infusion and
Category: Standard precautions/transmission-based the pump has malfunctioned. Which of the following
precautions/surgical asepsis actions by the nurse is MOST appropriate once
(1) A statement that “we should wash our hands the infusion has been stopped and restarted with a
frequently” functioning pump?
is accurate. Category: Safe use of equipment
(2) A statement that “we should use hand sanitizer” (1) CORRECT: The malfunctioning equipment
is accurate. should be labeled clearly and put in a separate
(3) A statement that “those of us with poor immune area to be reviewed by the equipment department.
systems should be extra careful” is accurate. (2) Placing the malfunctioning pump in the utility
(4) CORRECT: The family does not have to wear room may inadvertently allow the pump to reenter
gowns and gloves when interacting with their circulation and have the potential to lead to
mother. The infection occurred in the past; even an infusion error.
if it was active, gowns and gloves would not be (3) Placing the pump with other pumps may
required. Staff wear PPE to prevent spreading inadvertently
these types of infections to other clients. allow the pump to reenter circulation
23. The Answer is 3 and have the potential to lead to an infusion
The nurse witnesses another nurse, wearing a gown error.
and gloves, enter a client room labeled “Airborne (4) Placing the pump to the side in the client’s room
Precautions.” Which of the following actions by the may inadvertently allow the pump to reenter
witnessing nurse is MOST appropriate? circulation and have the potential to lead to an
Category: Standard precautions/transmission-based infusion error.
precautions/surgical asepsis 26. The Answer is 2
(1) The nurse manager does not need to be notified The nurse completes a peripherally inserted central
about this event unless it was recurring behavior catheter (PICC) line dressing change for a home care
endangering clients and staff. The witnessing client. When removing the PPE, the nurse should do
nurse may still notify the manager, but it is not which of the following?
the most appropriate priority action. Category: Standard precautions/transmission-based
(2) A presentation about which precautions require precautions/surgical asepsis
which types of PPE does not need to be delivered (1) Removing the mask with gloves on could transfer
by a physician. Precautions are within the realm contamination from the gloves to the mask
of nursing practice. and potentially to the nurse’s head.
5: Safe and Effective Care Environment: Safety (2) CORRECT: Gloves are removed first.
and Infection Control (3) Both gloves and a mask should be worn when
147 changing a PICC line dressing.
(3) CORRECT: Remind the other nurse that she (4) Both gloves and a mask should be worn when
needs a mask in addition to a gown and gloves changing a PICC line dressing.
for airborne-type precautions. 27. The Answer is 3
(4) The other nurse might need to review the policy, The client is found on the floor by the nursing assistive
but a gentle reminder to use a mask is the most personnel (NAP). Once the client is safe, which
professionally appropriate act by the witnessing of the following should the nurse do next?
nurse. Category: Reporting of incident/event/irregular
24. The Answer is 2 occurrence/variance
The nurse discovers a client on the floor in the client’s (1) The nurse should not file the incident report—
hospital room. After examining the client and the one who discovers the event (the NAP) should
assisting him safely back to bed, which of the document it.
following (2) An incident report needs to be filed for internal
should the nurse do FIRST? purposes of learning from occurrences, but the
Category: Accident/injury prevention event must also be documented for purposes of
(1) The nurse would file an incident report after client care.
(3) CORRECT: The event should be documented in 149
the client’s medical record and the NAP should Health promotion and maintenance involves helping
file an incident report. your clients achieve and continue to
(4) An incident report needs to be filed for internal enjoy optimal health. You help people to identify that
purposes of learning from occurrences. target state, discover their strengths
148 and their needs, and then support their path to full
NCLEX-RN ® Exam Content Review and Practice health and wellness potential. Putting
28. The Answer is 4 your enthusiasm into screening, education, and
The nurse is making a home visit to an elderly client treatment efforts can make a significant difference
during the winter. The nurse notices upon arrival in successful outcomes.
that the client has the oven turned on with the oven On the NCLEX-RN® exam, you can expect
door open, and is using it as a form of heat. Which approximately 9 percent of the questions to relate
of the following actions by the nurse is MOST to Health Promotion and Maintenance. This category
appropriate? focuses on the knowledge of expected
Category: Home safety growth and development principles, prevention
(1) As the home care nurse, it is the nurse’s obligation and/or early detection of health problems,
to promote client safety and to prevent and strategies to achieve optimal health. Exam
hazards. content related to Health Promotion and
(2) Shutting the oven off and continuing with the Maintenance includes, but is not limited to, the
home visit might be a tempting choice. The nurse following areas:
might do this, too, but it doesn’t solve the problem • Aging process
if no education is done with the client and • Ante/intra/postpartum and newborn care
family. • Developmental stages and transitions
(3) Reporting the event to the local Fire Department • Health and wellness
is not necessary unless the nurse has a fear that • Health promotion/disease prevention
this activity will be continued. • Health screening
(4) CORRECT: Have a meeting with the client and • High risk behaviors
family and warn them of the fire and safety risks • Lifestyle choices
of using the oven for heat. • Principles of teaching/learning
29. The Answer is 1 • Self-care
The medical center encounters a bomb threat. The • Techniques of physical assessment
emergency response team informs the staff that the Let’s now review the most important concepts covered
threat is legitimate and that clients should start being by the Health Promotion and Maintenance
evacuated. Which of the following clients should the category on the NCLEX-RN® exam.
nurse begin evacuating FIRST to the safe designated Health Promotion and
area? Maintenance
Category: Emergency response plan chapter 6
(1) CORRECT: Ambulatory clients have the potential 150
to wander and end up in an unsafe place if NCLEX-RN® Exam Content Review and Practice
not directed correctly. Aging Process
(2) Bedridden clients cannot leave without assistance; The aging process unfolds gradually, starting with
therefore they would be evacuated subsequent infancy (the first year of life). After that,
to the ambulatory clients. school becomes the dividing marker. Thus
(3) ICU clients cannot leave without assistance; preadolescent stages are divided into two: preschool
therefore they would be evacuated subsequent (1–4 years) and school-age (5–12 years). Puberty
to the ambulatory clients. marks the onset of the adolescent
(4) Infant clients cannot leave without assistance; stage (13–18 years). Adulthood is divided into three
therefore they would be evacuated subsequent parts: the working years (19–64 years),
to the ambulatory clients. the retirement years (65–85 years), and the elderly
30. The Answer is 2 years (over 85 years). As you review for the
The nurse discovers that the last dose of intravenous NCLEX-RN® exam, make sure you understand the
antibiotic administered to a client was the wrong special needs of each of these age groups
dose. Which of the following should the nurse do? so that you can provide the necessary care and
Category: Reporting of incident/event/irregular education required.
occurrence/variance Whichever stage your clients are in, you need to be
(1) The event should be filed both in an incident able to assess their reactions to expected
report and in the client’s medical record. age-related changes. For example, an adolescent and
(2) CORRECT: The event should be filed in an incident an elderly person are going to react
report and in the client’s medical record. differently to a change in their residential location. A
(3) Nurses should not document in the client’s medical teenager will probably make that
record that an incident report was filed. The transition more easily than an elderly client who is
incident report is for internal purposes of learning coping with other physical and cognitive
for the institution. losses.
(4) The event should be filed in both an incident Ante/Intra/Postpartum and Newborn Care
report and in the client’s medical record. To ensure the health of both mother and baby,
NCLE X-® pregnancies are now closely monitored from
the moment a woman knows she is expecting to Nutrition is an important part of prenatal care and
several weeks after the baby is born. education. An estimated 50 percent of
Antepartum Care pregnancies are unplanned, and the mother-to-be
Antepartum care is care given to the mother and baby might not have been getting adequate
before birth. It is also known as prenatal nutrients. Pregnant teenagers need more protein,
care. Antepartum care involves keeping track of the calcium, and phosphorus than pregnant
client’s history and includes a number adults, because their bones are still growing.
of important examinations. Weight gain should be limited to between 22 and 27
Calculating Expected Delivery Date pounds—somewhat less if overweight,
Every mother wants to know her estimated date of somewhat more if underweight. Substantial weight
delivery. A simple way to calculate this gain is deleterious to both mother and
is to add 7 days and 9 months to the first day of the baby because it increases risk of preeclampsia. If the
last menstrual period. Only 4 percent of mother does not lose the extra pounds
women actually give birth that day. after childbirth, she increases her risk of diabetes and
A pregnancy is considered full term between weeks 37 high blood pressure, which are linked
and 42. Birth occurring prior to week to a greater risk of coronary artery disease, among
37 is considered a premature birth, and later than other conditions.
week 42 is considered to be overdue. 152
Documenting the Mother’s Current Health and NCLEX-RN® Exam Content Review and Practice
Previous Health History You also need to be able to provide prenatal education
Documenting the mother’s current health and about normal pregnancy events, such
previous health history is an important part as quickening (the first perceptible fetal movement,
of prenatal care. You should obtain data about blood typically at 17–19 weeks, but in some
pressure, weight, lifestyle, and family instances as early as 13 weeks or as late as 25
and genetic history; and ask about support systems, weeks). Some women might have some Braxton
perception of pregnancy, and previous Hicks contractions after the 20th week.
coping mechanisms. The absence of an in-place It is equally important to educate about possible
support system can be countered by putting danger signals. Examples include:
151 • Vaginal bleeding
6: Health Promotion and Maintenance • Continuous headaches during the last three months
the client in touch with a prenatal support group, for • Marked or sudden swelling of extremities during the
example. A referral is also appropriate last three months
if the client sees pregnancy as an illness, or if she has • Dimness or blurring of vision during the last three
previously used denial or fantasy as months
coping mechanisms. • Severe, unrelenting abdominal pain
You also need to know which medications the client is • Decreased fetal movement after 24 weeks
using—prescribed, alternative, and Recognizing Cultural Differences
over-the-counter. Category X medications have such Be aware of cultural differences in childbearing
a harmful effect on the developing fetus practices. Chinese Confucian women value
that they are contraindicated in pregnancy. These modesty and self-control, so such women may remain
include: stoic during pregnancy, asking few
• Birth control pills questions. For Mormon women, pregnancy is viewed
• Accutane as a time of personal and family growth,
• Some hyperlipidemia medications as it creates a connection with eternity. The Orthodox
• Warfarin (Coumadin) Jewish woman is considered ritually
• Ulcer drug (Cytotec) impure after her water breaks, so her Orthodox Jewish
• Vaccines for measles, mumps, and smallpox husband is unlikely to be in the delivery
You also need to test for the Rh factor, unless the room. Instead, he prays in the waiting area.
mother is Rh-positive (has the factor) or Intrapartum Care and Education
both parents are Rh-negative (lack the factor). If the Intrapartum care is defined as care that is given
mother is Rh-negative and the father during labor and birth.
is Rh-positive, the mother needs to have Rho (D) Identifying Onset of Labor
immune globulin (RhoGAM) in the 28th The three main factors that may cause labor to begin
week. are the effect of hormones, the distension
Ultrasounds are used to noninvasively confirm fetal of the uterus, and the effect of oxytocin. Two
viability, gestational age, fetal anatomy, recognizable signs of impending labor are
and location of the placenta. the passage of a thick mucus plug from the cervix and
Sometimes an amniocentesis—withdrawing amniotic rupture of the amniotic membranes.
fluid for analysis—is done after the On average, the entire process from onset to birth
14th week. The test is indicated for women over age lasts about 12–14 hours for a first baby.
35 and those with a family history of Subsequent labors tend to be shorter in length.
genetic or metabolic problems. Care During Labor
Documenting Fetal Health Nursing care mirrors labor’s four stages:
Fetal heart rate during routine prenatal exams should 1. From 4–10 cm: Assess cervical effacement and
be between 120–160 beats per minute. dilation, and need for analgesia.
Educating a New Mother-to-Be
2. From complete dilation to delivery of baby: Assess • Physical: May have swollen genitals and breasts, a
newborn. misshapen head, milia (white spots)
3. From delivery of baby to expulsion of placenta: on face; exhibits sucking, grasping reflexes; able to
Usually within 5–20 minutes after birth; focus; learns to grasp with thumb
assess umbilical cord for two arteries and one vein. and finger
153 • Cognitive and psychosocial: Vocalizes sounds
6: Health Promotion and Maintenance (coos); begins to respond selectively to
4. Immediate recovery and observation: words
Approximately two hours after birth; assess maternal Deviations
vital signs, uterine fundal height, vaginal discharge, • Not rolling from tummy to side at 10 months
and bladder distention; assist breastfeeding • Not transferring toys from hand to hand at 9 months
efforts if indicated. Special Needs
Postpartum Care and Education • Parent-infant bonding
The mother must be carefully observed after birth to Preschool-Age Children
identify serious complications, including Preschool-age children are 1–4 years old.
the following: Expected Development
• Hemorrhage: Report heavy clots or spurts of • Physical: Enjoys physical activities; has increasing
bleeding. Expect some blood in vaginal bladder and bowel control; can
discharge for 3–6 weeks. manipulate small objects with hands; is able to dress
• Infection or other illnesses: Watch for a temperature and undress self; has refined coordination
over 100.4° F (38° C); sudden increase • Cognitive and psychosocial: Becomes aware of
in perineal pain; unusually heavy or foul-smelling limits; says “no” often; has a limited
vaginal discharge; hot, tender, or red vocabulary of 500–3,000 words in very short
breast; dysuria; pain or swelling in the legs; and chest sentences (3–4 words); believes that adults
pain or cough. know everything; can use a pencil to draw shapes; is
Newborn Care and Education eager to learn; has a strong desire
One minute after birth, the physician rates five to please adults
factors: Deviations
1. Appearance (color) • Does not walk at 18 months
2. Pulse (heart rate) • Does not speak at least 15 words
3. Grimace (reflex irritability) • Does not imitate actions or words or follow simple
4. Activity (muscle tone) instructions
5. Respiration (respiratory effort) 155
This is known as the APGAR score. The value of each 6: Health Promotion and Maintenance
factor is 0 (not good), 1 (OK), or 2 • Talks excessively about violence or other mature
(good). A total score of 10 is optimum. topics
Inform the mother of the warning signs of • Not interested in “pretend” play or other children
complications with her newborn, and explain when Special Needs
to call a doctor or take the baby to an emergency • Security and consistency of environment
room. Those complications include: • Protection from harmful situations caused by natural
• Has sunken or swollen soft spots on the head curiosity
• Has a fever higher than 100.4° F (38° C) • Some allowance for independence and playtime
• Vomits more than once in 24 hours School-Age Children
• Is unable to keep down food or water School-age children are 5–12 years old.
• Is not breathing easily Expected Development
It is also important to assist the mother in performing • Physical: Able to do a series of motions to perform
newborn care. This is an ideal time to activities, such as skipping or jumping
answer questions about parent-infant bonding. This is rope
also the best time to provide contraception • Cognitive and psychosocial: Able to follow two-step
education, if needed. The client’s menstrual cycle directions; knows full name, age, and
should begin in 6–8 weeks after giving address; tends to identify with parent of the same sex
birth, unless she is breastfeeding. Make sure your Deviations
client knows about normal emotional • Bed-wetting late into childhood
stress (the blues) during her second or third • Verbal or outward expression of anxiety about
postpartum week. Tell her to contact her physician school or home
if she experiences significant negative mood changes. Special Needs
154 • Developing scoliosis (sideways curvature of spine)
NCLEX-RN® Exam Content Review and Practice • Vision and hearing problems: important to discover
Developmental Stages and Transitions at earliest stages
The following sections provide an overview of life’s Adolescents
milestones to review for the NCLEX-RN® Adolescents are 13–18 years old.
exam. Expected Development
Infants • Physical: Shows increased interest in personal
Infants are 1–12 months old. attractiveness; develops secondary sexual
Expected Development characteristics
• Cognitive and psychosocial: Struggles with sense of During the continuum of life from infancy to old age,
identity; forms strong peer allegiances; you need to be able to educate clients
engages in risk-taking due to a sense of immortality about their health and help them make changes to
Deviations increase their wellness. Your approach is
• Persistent misbehavior, especially in school straightforward:
• Aggression 1. Assess the client’s perception of his or her own
156 health status.
NCLEX-RN® Exam Content Review and Practice 2. Identify the client’s health-oriented behaviors.
Special Needs 3. At regular intervals, evaluate the client’s
• Understanding of puberty’s effect on disposition and understanding of health and wellness activities.
personality 4. Encourage client participation in behavior
Adults modification programs, as needed.
Adults are 19–64 years old. Health Promotion/Disease Prevention
Expected Development Health promotion concerns helping people to increase
• Physical: Peak reached between 25 and 35 years control over and to improve their
old; might live for many years with a health. Health promotion activities seek to empower
chronic condition individuals and their communities to
• Cognitive and psychosocial: From 19–34 years old— organize, prioritize, and act on health issues. Disease
Erikson’s stage of intimacy versus prevention, on the other hand, involves
isolation; from 35–64 years old—Erikson’s stage of efforts to stop the onset of a specific illness or
generativity versus stagnation condition, such a cancer.
Deviations You should be able to identify the important risk
• Feeling that life is meaningless factors for disease/illness. Table 1 lists the
Special Needs top three leading causes of death by age group, as
• Learning lessons of workplace, long-term identified by the Centers for Disease Control
relationships, and parenting and Prevention.
Older Adults 158
Older adults are 65–85 years old. NCLEX-RN® Exam Content Review and Practice
Expected Development Cause of death
• Physical: General slowing of physical functioning Under
• Cognitive and psychosocial: General slowing of 1 1–4 5–9 10–14 15–24 25–34 35–44 45–54 55–64
cognitive functioning; Erikson’s stage of Over
ego integrity versus despair; interpersonal 65
relationships continue despite changes and Birth defects 1 2 3
losses Disorders related to
Deviations premature birth 2
• Despair can arise from remorse for what might have Sudden infant death
been syndrome (SIDS) 3
Special Needs Unintentional
• Learning lessons of successfully retiring from the injuries 1 1 1 1 1 1 3 3
workplace Cancer 2 2 2 1 1 2
• Keeping or losing long-term relationships Homicide 3 3 3
Very Old Adults Suicide 3 2 2
Very old adults are over 85 years old. Heart disease 3 2 2 1
157 Chronic low
6: Health Promotion and Maintenance respiratory disease 3
Expected Development Stroke
• Physical: Continued decline of physical functioning * Adapted from the Centers for Disease Control and
• Cognitive and psychosocial: Continued decline of Prevention’s “10 Leading Causes of Death, United
cognitive functioning; marked increase States,
in changes and losses in relationships 2011.”
Deviations Table 1
• Suicidal thoughts and behavior Health Promotion/Disease Prevention Programs
Special Needs Health promotion/disease prevention programs
• Acceptance of life’s accomplishments and declines include using community intervention techniques,
Health and Wellness such as holding health fairs or doing on-site education
Traditionally, health is the absence of disease and at elementary and high schools.
disability. Known as the medical model, Health promotion topics might include the following:
that philosophy has changed with the recognition that • Healthy weight management: The client’s current
people can enjoy life even while experiencing weight should be assessed in comparison
challenges. The World Health Organization defines to a desirable weight. Know that a person with type 2
health as a state of physical, diabetes can improve glucose
mental, and social well-being. Thus the phrase “health control by losing only 10–20 pounds.
and wellness” points to your helping • Smoking cessation: Factors associated with
each client achieve optimal functioning regardless of continued smoking include the strength of
current health status or disability.
the nicotine addiction, continued exposure to during crises.
smoking-associated stimuli (at work or • Stroke and heart disease prevention: Clients should
in social settings), stress, depression, and habit. monitor blood pressure regularly,
Continued smoking is more prevalent especially if they have a positive family history of
among those with low incomes, low levels of hypertension.
education, and psychosocial problems. • Healthy joints: Teach clients to do weight-bearing
Multiple factors often require multiple strategies. and stretching exercises regularly.
• Stress management: Studies show a cause-and- • Bone health: Diets need to include vitamin D and
effect relationship between stress and calcium to prevent osteoporosis.
events including infectious diseases, traumatic • Skin cancer prevention: Teach clients to counteract
injuries (e.g., motor vehicle crashes), the negative impacts of excessive sun
159 exposure by using sunscreen, wearing protective
6: Health Promotion and Maintenance clothing, or limiting time outdoors.
and some chronic illnesses. Teaching stress reduction 160
prevents other additional negative NCLEX-RN® Exam Content Review and Practice
consequences. Health Screening
• Exercise: Benefits include improved circulatory and Health screening requires you to apply your
respiratory systems; decreased cholesterol; knowledge of pathophysiology and risk factors
lower body weight; delayed osteoporosis; and more linked to ethnicity and known population or
flexibility, strength, and community characteristics. Screening examples
endurance. include:
• Special diets: Clients with hypertension should avoid • Blood sugar check: Levels more than 199 mg/dL
foods high in sodium, such as processed, without fasting or more than 125 mg/dL
canned foods. Clients with high cholesterol should with fasting for 8 hours signal the need for a more
avoid saturated fatty acids complete workup.
(found in fatty meats) and trans fatty acids (found in • Blood pressure check: One-third of people whose
deep-fried fast foods). blood pressures exceed 140/90 mm Hg
• Complementary, alternative, or homeopathic do not know it. Incidence of the silent killer is higher
therapies: Examples include hypnosis, acupuncture, in the southeastern United States,
and massage. Some clients use over-the-counter especially among African Americans. Other risk
remedies, vitamins, minerals, factors are age over 60 years, inactive
herbal medicines, or other approaches, such as a lifestyle, and hyperlipidemia.
shaman. • Fasting lipid profile: Adults should have a fasting
• Breast self-examination (BSE): Beginning at lipid profile done at least once every 5
puberty, women should examine their breasts years. The total cholesterol value should be under 200
monthly, between day 5 and day 7 of their menstrual mg/dL, triglycerides (fatty acids)
period. In menopause, BSE should should be under 150 mg/dL, the low-density
continue monthly. lipoprotein (LDL, the “bad” cholesterol
• Testicular self-examination: Testicular cancer is the that accelerates atherosclerosis) value should be
most common cancer in men ages under 100 mg/dL, and the high-density
15–35, and one of the most curable solid tumors. lipoprotein (HDL, the “good” cholesterol that removes
Teach clients that the best time to cholesterol) value should be
check is after bathing, when the scrotum is more greater than 40 mg/dL for men and 50 mg/dL for
relaxed. Any evidence of a lump or women.
swelling should be reported to a physician. • Colorectal screening: Regular screening, beginning
• Hormone replacement therapy (HRT) information at age 50, is the key to preventing
updates: HRT lowers the risk of osteoporosis- colorectal cancer. This screening can include fecal
related bone fractures, but increases the risk for occult blood test (FOBT), sigmoidoscopy,
coronary artery disease (CAD), colonoscopy, double-contrast barium enema (DCBE),
breast cancer, deep vein thrombosis (DVT), and or digital rectal exam
stroke. (DRE).
• Immunizations: Hepatitis B vaccine is given to • Prostate screening: Men should get a prostate-
newborns. Infants get most immunizations specific antigen (PSA) test beginning at
from 2–12 months. Annual flu shots can start at 6 age 50.
months. Meningococcal • Mammograms: Women should get a baseline
vaccine is recommended for previously unvaccinated mammogram between ages 40 and 50, after
college freshmen living in dormitories. considering risk factors.
Seniors over age 60 need vaccinations to prevent High-Risk Behaviors
shingles (herpes zoster) and High-risk behaviors are those lifestyle practices that
pneumonia. increase the likelihood of illness, disease,
• Oral health: Gum disease can allow bacteria to enter or death. For example, in the case of HIV/AIDS, those
the body. Clients should schedule activities include unprotected sex
regular visits to dentists every 6 months beginning at (anal, vaginal, or oral), using contaminated needles
age 2. or sharing syringes, and coming in contact
• Mental health: Teach ways to deal with stress and with bodily fluids (blood, semen, vaginal fluids, and
encourage seeking professional help saliva). Unprotected sex can also
lead to other consequences, such as sexually e. Low literacy and comprehension skills
transmitted diseases (STDs). Most safe sexual NCLEX-RN ® Exam Content Review and Practice
practices take some planning, such as having 162
condoms available. An unplanned pregnancy NCLE X-f. Cultural/ethnic background and language
can be avoided by taking birth control pills regularly. barriers
Promote accident awareness to reduce deaths due to g. Lack of motivation
unintentional injuries. This includes h. Environment
using seat belts in automobiles, wearing helmets while i. Negative past experiences
biking, and using crosswalks. j. Denial of personal responsibility
161 Self-Care
6: Health Promotion and Maintenance Self-care includes all activities that promote and
Lifestyle Choices maintain personal well-being without medical,
Lifestyle is a characteristic set of behaviors and professional, or other assistance or oversight. For
practices that range from habits and conventional developmentally delayed or elderly
ways of doing things to reasoned actions. Examples of people, an inability to perform these tasks can curtail
lifestyle choices include being their ability to live independently. Your
child-free; living in urban, rural, or suburban knowledge of in-home community resources might
environments; educating children in public or enable them to live in that environment
private schools or home-schooling; and using longer. Your care plan might also need to involve
alternative or homeopathic health care practices. clients (if able to give input), family members,
Any of these choices might have an impact on your friends, or paid staff inside or outside an institution.
clients’ health. Techniques of Physical Assessment
Principles of Teaching/Learning You should know the four methods or techniques of
Principles of teaching and learning are techniques that performing a physical assessment:
allow you to share medical and health 1. Inspection or purposeful observation: Pay attention
information with clients. You have been in school for to outward details about the client,
some time, so it is second nature for you noting any deviations from expected age-related
to absorb new information. For clients, you need to do development. Note posture and stature,
the following: body movements, nutritional status by appearance,
1. Use an organized approach to assess readiness and speech pattern, and vital signs.
ability to learn. Individualize your approach. For example, with an
a. Consider age and developmental stage when obese young person, use an adult-size
teaching clients. For example, teaching blood pressure cuff to get an accurate reading while
adolescents might best be done by pointing them to assuring client comfort.
trusted Internet sites, so 2. Palpation: Use fingers and palms to apply a light
that they can have a sense of autonomy in discovering touch or deeper pressure to gather data
health advice for themselves. about the health of superficial blood vessels, lymph
b. Take into account clients’ living situations. An nodes, the thyroid, and the organs
example is an elderly person who is of the abdomen and pelvis.
socially isolated due to decreasing sight and hearing 3. Percussion: Tap a part of the body and listen for
or geographically isolated due the returned sound. This technique is
to family and friends living far away. often used on the chest and abdominal walls.
c. Encourage clients to establish their own goals and 4. Auscultation: Use a stethoscope to listen to sounds
evaluate their own progress. caused by movement of air or fluid
d. Let clients demonstrate their understanding of within the client’s body. This provides information
information and practice their about breath sounds, the spoken
skills. voice, bowel sounds, cardiac murmurs, and heart
e. After teaching, evaluate the results. sounds. The stethoscope’s bell (hollow
2. Account for learning preference. cup) part of the endpiece can assess very-low-
a. Visual learners think in pictures, so use visual aids frequency sound, such as heart murmurs.
such as diagrams, videotapes, The diaphragm (disc) part of the endpiece can assess
and handouts. high-frequency sounds from the
b. Auditory learners best understand material through heart and lungs.
listening. Tell them about This chapter reviewed aspects of childbirth to features
community lectures, discussions, and recordings. of old age, and looked at health and
c. Tactile or kinesthetic learners prefer to learn via wellness across the life span. Whatever your clinical
experience—moving, touching, setting, and whatever the reason for clients
and doing. Let the client hold a scale model of body seeking medical help, you can rely on your grasp of
organs to illustrate anatomy, basic health promotion and maintenance
for example. concepts. That knowledge will be evident when you
3. Identify barriers to client learning. successfully take the NCLEX-RN® exam.
a. Physical condition, such as decreased sight or 6: Health Promotion and Maintenance
hearing 163
b. Financial considerations 1. A 20-year-old client has just given birth.
c. Lack of support systems The baby looks healthy, with the exception
d. Misconceptions about disease and treatment of giving a grimace instead of a cry. Which
of the following would the nurse expect the 4. Low blood sugar
obstetrician to say? Chapter Quiz
1. “The APGAR score is 3.” NCLEX-RN® Exam Content Review and Practice
2. “The APGAR score is 6.” 164
3. “The APGAR score is 9.” 7. The nurse is providing education at a senior
4. “The APGAR score is 12.” center. Which of the following measures
2. The outpatient client is postmenopausal. In will the nurse say is MOST effective in
discussing breast self-examination, which of attaining normal blood sugar levels in a
the following should the nurse let the client client with type 2 diabetes?
know that she can do? 1. Decreasing sodium intake
1. Switch to an annual schedule, because she 2. Increasing potassium and calcium
does not have periods. intake
2. Discontinue self-examination, because 3. Reaching recommended weight
hormone changes decrease her risks. 4. Decreasing daily exercise
3. Wait until her mammogram shows some 8. A local high school is having a health fair.
findings. Which of the following main courses should
4. Continue to palpate monthly, picking her the nurse recommend as most healthful for
own meaningful date. a teenager whose cholesterol level is 300 mg/
3. A client with acne has been using dL?
isotretinoin (Accutane). She tells the nurse 1. Medium-rare hamburger with only one
that she recently learned she is pregnant. She slice of cheese
asks “Will my pregnancy interfere with the 2. Vegetarian New York–style pizza
medication’s effectiveness?” Which of the 3. Grilled chicken breast
following is the appropriate response by the 4. Salad with extra dressing
nurse? 9. The nurse is talking to a client who is still
1. “The medication is contraindicated for grieving the loss of a parent to stomach
pregnant women.” cancer. The nurse knows that which of the
2. “You will have to change the route following would increase the client’s risk of
of administration, because you are cancer?
pregnant.” 1. Keeping a strict high-protein diet
3. “There is no reason you can’t continue 2. Following a low-fat, low-carbohydrate
taking it.” diet
4. “If the medication helps you look better, 3. Using considerable spices when cooking
that will help feel better about yourself.” 4. Smoking cigarettes
4. The nurse is preparing for a women’s health 10. A 3-month-old child accompanies her
fair. The nurse knows that which of the parents to a seasonal flu clinic. Assuming
following is correct when teaching about the that the child does not have a fever, can the
risks and benefits of hormone replacement nurse give the child a flu shot?
therapy (HRT)? 1. Yes, if regular immunizations are up to
1. HRT is related to a decreased risk of deep date.
vein thrombosis (DVT). 2. No, because the child is not old enough.
2. HRT is related to an increased risk for 3. Yes, because then the child won’t get
coronary artery disease (CAD). sick later.
3. HRT is related to an increased risk for 4. No, because it would interfere with
osteoporosis-related bone fractures. regular immunizations.
4. HRT is related to a decreased risk of 11. The nurse gives a 35-year-old primigravida
breast cancer. client a RhoGAM injection in her 28th
5. The nurse has been working with a 45-yearold week of pregnancy. Which of the following
African American who bicycles to client situations requires the nurse to take
work. Lab tests show low serum lipids. The this action?
nurse knows that the client’s risk factors for 1. Rh-positive mother and Rh-negative
primary (essential) hypertension include father
which of the following? 2. Rh-positive mother and Rh-positive
1. Being under the age of 65 father
2. Race 3. Rh-negative mother and Rh-negative
3. Low serum lipids father
4. Active lifestyle 4. Rh-negative mother and Rh-positive
6. The nurse is designing a diet plan for a father
70-year-old with poorly fitting dentures 12. The nurse is teaching a young male client
who has been recently diagnosed with type to recognize the most common early sign of
2 diabetes. The nurse knows that which of testicular cancer. The nurse emphasizes the
the following is the LEAST likely risk to the fact that he should be aware of which of the
client? following?
1. Malnutrition 1. Lumbar pain
2. Dehydration 2. Urinary frequency
3. Hyperglycemia 3. Urinary urgency
4. Painless testicular enlargement 1. “We routinely do an amniocentesis
13. New parents are concerned about an on all our clients to check the child’s
unexpected characteristic of their newborn gender.”
baby. Which of the following would cause 2. “An amniocentesis is not invasive, so there
the nurse to initiate contact with the is less risk than doing an ultrasound.”
physician? 3. “The standard for doing an amniocentesis
1. Swollen genitals and breast is motherhood over age 35.”
2. High-pitched crying 4. “If we know the baby’s size, you can
3. Misshapen head better count on having a vaginal birth.”
4. Milia 19. The nurse is educating a mother-to-be
14. A public health nurse visits a client at home about possible danger signs during the
three days after the client gave birth. In last three months of pregnancy. Which of
which of the following situations should the following would NOT cause the nurse
the nurse instruct the client to report to a concern about danger signs?
clinician? 1. Rectal bleeding
6: Health Promotion and Maintenance 2. Continuous headaches
165 3. Marked swelling of hands
1. Vaginal drainage with streaks of bright 4. Blurred vision
red blood 20. A first-time parent is discussing
2. Some discomfort at the site of her developmental milestones with the nurse.
episiotomy The nurse tells the client that she can
3. Feelings of fatigue late in the afternoon reasonably expect her child to achieve
and evening which of the following by the time the child
4. An elevated temperature without other is 1 year old? Select all that apply.
symptoms 1. Walking
15. The pediatric nurse is providing discharge 2. Rolling from tummy to side
instructions to the parents of a newborn. In 3. Transferring toys from hand to hand
which of the following situations would the 4. Beginning to respond selectively to words
nurse advise the parents to call a physician? 5. Vocalizing sounds (coos)
Select all that apply. NCLEX-RN® Exam Content Review and Practice
1. The infant has a temperature higher 166
than 100.4º F (38º C). 21. A parent is discussing the behavior of her
2. The infant vomits more than once in 24 3-year-old child with the nurse. At 3 years,
hours. the nurse would expect the client’s child
3. The infant’s respirations are even and to be doing all of the following EXCEPT
unlabored. which activity?
4. The infant is unable to keep down food 1. Saying “no” often
or water. 2. Using a limited vocabulary of 500–3,000
5. The infant has sunken or swollen soft words
spots on the head. 3. Speaking in 10-word sentences
16. The client’s first day of her last period was 4. Believing that adults know everything
February 1. Which of the following should 22. The nurse is teaching a group of mothers
the nurse tell the client is her expected date of toddlers how to prevent accidental
of delivery? poisoning from medications. The nurse
1. November 8 teaches the mothers to store medications in
2. October 8 which of the following locations?
3. December 1 1. In a secure, locked place
4. November 20 2. In vials with childproof caps
17. The client is 7 months pregnant with her 3. On the highest shelf in the room
first child. She is anxious because she feels 4. Disguised in different containers
some mild contractions at times. The nurse 23. The nurse is assessing an elderly couple,
tells her which of the following? both 80 years old, to determine if they
1. She should increase her bed rest to can safely continue to live independently.
prevent those contractions. They insist they are getting along fine
2. The contractions are normal unless they but need help with grocery shopping and
increase in severity. housekeeping. The nurse determines that
3. The contractions are a way of her body they have difficulty in doing which of the
asking for more exercise. following?
4. She should avoid getting constipated and 1. Activities of daily living (ADLs)
having gas as a result. 2. Instrumental activities of daily living
18. The client is 40 years old and pregnant (IADLs)
with her first child. Her obstetrician has 3. Daily living milestones (DLMs)
asked the nurse to schedule her for an 4. Preventive health activities (PHAs)
amniocentesis. The client inquires why she 24. The nurse is giving a lecture at the senior
needs that test. The nurse says which of the center about preventative health activities
following as an explanation? for people over age 60. The nurse tells the
clients that the Centers for Disease Control Match the appropriate part of the profile below on the
and Prevention (CDC) now recommends left to the values on the right. All options
which of the following vaccines for this age must be used.
group? 1. Total cholesterol A. More than 40 mg/dL
1. Shingles (herpes zoster) 2. HDL cholesterol for men B. More than 50 mg/dL
2. Diphtheria 3. HDL cholesterol for women C. Less than 100 mg/dL
3. Pertussis (whooping cough) 4. LDL cholesterol D. Less than 150 mg/dL
4. Meningitis 5. Triglycerides E. Less than 200 mg/dL
25. The nurse is teaching about the challenges 30. The nurse is assessing the best approach to
of smoking cessation. Which of the prepare three clients for surgery. Each has a different
following factors will the nurse identify as learning preference. Match the learning preference to
known challenges that clients face when the appropriate approach. All options must
attempting to quit smoking? Select all that be used.
apply. 1. Brochures about preparation activities A. Auditory
1. Stress and depression 2. Models of the relevant anatomy B. Visual
2. Low level of income 3. Discussions about the surgery C. Tactile
3. High level of education NCLEX-RN ® Exam Content Review and Practice
4. Psychosocial problems 168
5. Continued exposure to smokingassociated NCLE X-1. The Answer is 3
stimuli The 20-year-old client has just given birth. The baby
26. Stress reduction techniques include looks healthy, with the exception of giving a grimace
biofeedback and meditation. The nurse instead of a cry. Which of the following would the
conducting classes on these methods knows nurse expect the obstetrician to say?
that studies have shown a cause-and-effect Category: Ante/intra/postpartum and newborn
relationship between stress and which of the care
following? Select all that apply. (1) An APGAR score of 3 indicates a baby in poor
1. Adverse medication effects health.
2. Infectious diseases (2) An APGAR score of 6 indicates a less healthy
3. Traumatic injuries, such as motor baby.
vehicle accidents (3) CORRECT: In 4 of the 5 categories of rating, the
4. Some chronic illnesses baby scored a 2. In the category of reflex irritability,
27. The nurse is performing the initial the baby scored a 1, for a total APGAR
assessment of an adult from a culture the score of 9.
nurse is not familiar with, and asks about (4) An APGAR score of 12 does not exist; the highest
the client’s use of alternative therapies. The score is 10.
client says, irritably, “Do you have to ask 2. The Answer is 4
all these questions?” Which of the following The outpatient client is postmenopausal. In discussing
is the BEST explanation for what the nurse breast self-examination, which of the following
should do in response? should the nurse let the client know that
1. Ask the question, because the nurse she can do?
might learn about therapies used by a Category: Aging process
different culture. (1) Although menopause itself is not associated with
6: Health Promotion and Maintenance increased risk of breast cancer, the rate does
167 increase with age. The client should continue
2. Ask the question, because knowledge with breast self-examination.
about actual use of other therapies is (2) Although menopause itself is not associated with
imperative. increased risk of breast cancer, the rate does
3. Don’t ask the question, because it is increase with age. The client should continue
important to not upset the irritable with breast self-examination.
client any further. (3) Although menopause itself is not associated with
4. Don’t ask the question, because the increased risk of breast cancer, the rate does
client needs to choose to initiate increase with age. The client should continue
discussion of other therapies. with breast self-examination.
28. The nurse is preparing a community (4) CORRECT: Breast self-examination is extremely
educational presentation. The topic is the important for a client in this soon-to-be high risk
leading cause of death for people from ages group. About 70 percent of new diagnoses come
1–44. The nurse knows that which of the after age 50.
following is the leading cause? 3. The Answer is 1
1. Cancer A client with acne has been using isotretinoin
2. Heart disease (Accutane).
3. Unintentional injuries She tells the nurse that she recently learned
4. Diabetes she is pregnant. She asks “Will my pregnancy
29. The nurse is reviewing the client’s lipid profile to interfere
determine if education is needed to reduce the with the medication’s effectiveness?” Which
risk of heart disease. The nurse knows how to match of the following is the appropriate response by the
healthy target values with lab descriptions. nurse?
Category: Ante/intra/postpartum and newborn care Category: Health and wellness
(1) CORRECT: Severe fetal abnormalities may (1) Decreasing sodium intake is not an effective way
occur if Accutane is used during pregnancy. The to attaining normal blood sugar levels in a client
nurse should stress that the priority is the high with type 2 diabetes.
risk of fetal abnormalities that the medication (2) More potassium and calcium will not affect
can cause rather than the effectiveness of the blood glucose.
medication. (3) CORRECT: Losing only as much as 10–20
(2) The nurse would not tell the client to continue pounds improves blood glucose control.
taking this drug. (4) The client needs to increase, not decrease, daily
(3) The nurse would not tell the client to continue exercise.
taking this drug. 8. The Answer is 3
(4) The nurse would not tell the client to continue A local high school is having a health fair. Which of
taking this drug. the following main courses should the nurse
4. The Answer is 2 recommend
The nurse is preparing for a women’s health fair. The as most healthful for a teenager whose cholesterol
nurse knows that which of the following is correct level is 300 mg/dL?
when teaching about the risks and benefits of Category: Health and wellness; Health promotion/
hormone disease prevention
replacement therapy (HRT)? (1) The fat content of the main course (hamburger)
Category: Health promotion/disease prevention needs to be lower due to the teenager’s known
(1) HRT causes an increased risk of DVT. elevated cholesterol level.
(2) CORRECT: Current research counteracts earlier (2) The fat content of the main course (pizza) needs
theories of a decreased risk of CAD. to be lower due to the teenager’s known elevated
(3) HRT causes a decreased risk of cholesterol level.
osteoporosisrelated (3) CORRECT: The fat content of a grilled chicken
bone fractures. breast is the lowest of the choices.
(4) HRT causes an increased risk of breast cancer. (4) The fat content of the main course (salad with
Chapter Quiz Answers and Explanations extra dressing) needs to be lower due to the
6: Health Promotion and Maintenance teenager’s
169 known elevated cholesterol level.
5. The Answer is 2 9. The Answer is 4
The nurse has been working with a 45-year-old The nurse is talking to a client who is still grieving
African the loss of a parent to stomach cancer. The nurse
American who bicycles to work. Lab tests show knows that which of the following would increase
low serum lipids. The nurse knows that the client’s the client’s risk of cancer?
risk factors for primary (essential) hypertension Category: Health promotion/disease prevention
include which of the following? (1) High-protein diets have not been shown to be a
Category: Health promotion/disease prevention; risk for cancer.
Health screening (2) Low-fat, low-carbohydrate diets have not been
(1) Being under the age of 65 is associated with lower shown to be a risk for cancer.
risk. (3) Spicy food has not been shown to be a risk for
(2) CORRECT: African Americans have an cancer.
increased risk for hypertension. (4) CORRECT: Tobacco use has been shown to be
(3) Low serum lipids are associated with lower risk. a risk for cancer.
(4) An active lifestyle is associated with lower risk. NCLEX-RN® Exam Content Review and Practice
6. The Answer is 3 170
The nurse is designing a diet plan for a 70-year-old 10. The Answer is 2
with poorly fitting dentures who has been recently A 3-month-old child accompanies her parents to a
diagnosed with type 2 diabetes. The nurse knows seasonal flu clinic. Assuming that the child does not
that which of the following is the LEAST likely risk have a fever, can the nurse give the child a flu shot?
to the client? Category: Aging process
Category: Health promotion/disease prevention (1) The minimum age to receive a flu shot is 6
(1) Malnutrition is a possibility due to difficulty in months; therefore the nurse cannot give the child
eating. the shot.
(2) Dehydration is a possibility. (2) CORRECT: The minimum age to receive a flu
(3) CORRECT: Hypoglycemia is more likely than shot is 6 months.
hyperglycemia. Often a client with denture problems (3) The minimum age to receive a flu shot is 6
will only be able to tolerate liquid or pureed months; therefore the nurse cannot give the child
foods eaten slowly. This decreases the chances of the shot.
adequate nutrition. (4) The minimum age to receive a flu shot is 6
(4) Low blood sugar is a possibility. months; therefore the nurse cannot give the child
7. The Answer is 3 the shot.
The nurse is providing education at a senior center. 11. The Answer is 4
Which of the following measures will the nurse say The nurse gives a 35-year-old primigravida client a
is MOST effective in attaining normal blood sugar RhoGAM injection in her 28th week of pregnancy.
levels in a client with type 2 diabetes? Which of the following client situations requires the
nurse to take this action? (2) CORRECT: If an infant vomits more than once
Category: Ante/intra/postpartum and newborn care in 24 hours, the parents should call the physician.
(1) An Rh-positive mother does not need to worry (3) There would be no need to call the physician in
about the Rh factor of the father. this instance.
(2) An Rh-positive mother does not need to worry (4) CORRECT: If an infant is unable to keep
about the Rh factor of the father. down food or water, the parents should call the
(3) An Rh-negative mother does not need to worry physician.
about the Rh factor of the father, if it is the same (5) CORRECT: A physician should evaluate the
as her status. infant immediately if the infant has sunken or
(4) CORRECT: An Rh-negative mother and Rhpositive swollen soft spots on the head.
father is the combined Rh status in 16. The Answer is 1
which the mother could develop harmful antibodies. The client’s first day of her last period was February
12. The Answer is 4 1. Which of the following should the nurse tell the
The nurse is teaching a young male client to recognize client is her expected date of delivery?
the most common early sign of testicular cancer. Category: Ante/intra/postpartum and newborn care
The nurse emphasizes the fact that he should be (1) CORRECT: November 8 is 9 months and 7 days
aware of which of the following? later.
Category: Health promotion/disease prevention (2) October 8 is one month too early.
(1) Among other serious causes, lumbar pain could (3) By December 1, the baby would be overdue.
be a sign of metastasis. (4) By November 20, the baby would be overdue.
(2) Urinary frequency is not an early sign of testicular 17. The Answer is 2
cancer. The client is 7 months pregnant with her first child.
(3) Urinary urgency is not an early sign of testicular She is anxious because she feels some mild
cancer. contractions
(4) CORRECT: Painless testicular enlargement is a at times. The nurse tells her which of the following?
common early sign of testicular cancer. Category: Ante/intra/postpartum and newborn care
13. The Answer is 2 (1) Increasing bed rest is not necessary; Braxton
New parents are concerned about an unexpected Hicks contractions are normal at this stage in
characteristic of their newborn baby. Which of the the pregnancy.
following would cause the nurse to initiate contact (2) CORRECT: Braxton Hicks contractions are
with the physician? normal at this stage in the pregnancy.
Category: Ante/intra/postpartum and newborn care (3) More exercise is not necessary: Braxton Hicks
(1) Swollen genitals and breast are normal due to contractions are normal at this stage in the
maternal hormones. pregnancy.
(2) CORRECT: High-pitched crying is not normal (4) Gas is not likely to be the cause of the
and could be due to a neurological problem. contractions;
(3) A misshapen head is normal due to descent Braxton Hicks contractions are normal at
through the birth canal. this stage in the pregnancy.
(4) Milia is normal due to blocked sebaceous glands. 18. The Answer is 3
14. The Answer is 4 The client is 40 years old and pregnant with her first
A public health nurse visits a client at home three child. Her obstetrician has asked the nurse to schedule
days after the client gave birth. In which of the her for an amniocentesis. The client inquires why
following she needs that test. The nurse says which of the
situations should the nurse instruct the client following
to report to a clinician? as an explanation?
Category: Ante/intra/postpartum and newborn care Category: Ante/intra/postpartum and newborn care
(1) Vaginal drainage with streaks of bright red blood (1) The most common reason for an amniocentesis
is normal for the first 3–6 weeks. is to check chromosomal abnormalities, not to
(2) The area will continue to heal and is not a cause check the child’s gender.
for concern, unless the discomfort rises to the (2) The ultrasound is not invasive; the amniocentesis
level of persistent or increasing pain. is invasive.
(3) Feelings of fatigue are normal after giving birth. (3) CORRECT: After age 35, the risk of infant
(4) CORRECT: A fever above 100.4º F (38º C) is chromosomal
reason abnormality is greater than the risk
to call the physician. associated with the procedure.
6: Health Promotion and Maintenance (4) The most common reason for an amniocentesis
171 is to check chromosomal abnormalities, not to
15. The Answer is 1, 2, 4, 5 check the baby’s size.
The pediatric nurse is providing discharge instructions 19. The Answer is 1
to the parents of a newborn. In which of the following The nurse is educating a mother-to-be about possible
situations would the nurse advise the parents danger signs during the last three months of
to call a physician? Select all that apply. pregnancy.
Category: Ante/intra/postpartum and newborn care Which of the following would NOT cause the
(1) CORRECT: If an infant has a fever higher than nurse concern about danger signs?
100.4º F (38º C), the parents should call the Category: Ante/intra/postpartum and newborn care
physician. (1) CORRECT: Although hemorrhoids could cause
rectal bleeding, it is vaginal bleeding that would The nurse is assessing an elderly couple, both 80
concern the nurse. years old, to determine if they can safely continue
(2) Continuous headaches is a symptom that would to live independently. They insist they are getting
concern the nurse. along fine but need help with grocery shopping and
(3) Marked swelling of hands would concern the housekeeping. The nurse determines that they have
nurse. difficulty in doing which of the following?
(4) Blurred vision would concern the nurse. Category: Aging process; Self-care
NCLEX-RN® Exam Content Review and Practice (1) ADLs are basic functions of self-care, such as
172 feeding, dressing, and bathing.
20. The Answer is 2, 3, 4, and 5 (2) CORRECT: Grocery shopping and housekeeping
A first-time parent is discussing developmental are two important IADL functions.
milestones (3) Grocery shopping and housekeeping are not
with the nurse. The nurse tells the client that milestones.
she can reasonably expect her child to achieve which (4) Grocery shopping and housekeeping are not
of the following by the time the child is 1 year old? prevention
Select all that apply. activities.
Category: Developmental stages and transitions 24. The Answer is 1
(1) The parent should not become concerned unless The nurse is giving a lecture at the senior center
the child cannot walk at 18 months. about preventative health activities for people over
(2) CORRECT: Rolling from tummy to side is a age 60. The nurse tells the clients that the Centers for
developmental milestone that the client can Disease Control and Prevention (CDC) now
expect the child to reach by age 1. recommends
(3) CORRECT: Transferring toys from hand to which of the following vaccines for this age
hand is a developmental milestone that the client group?
can expect the child to reach by age 1. Category: Health promotion/disease prevention
(4) CORRECT: Beginning to respond selectively to (1) CORRECT: The shingles vaccine reduces the
words is a developmental milestone that the client risk of shingles by about half and the risk of
can expect the child to reach by age 1. postherpetic neuralgia by two-thirds.
(5) CORRECT: Vocalizing sounds (coos) is a 6: Health Promotion and Maintenance
developmental 173
milestone that the client can expect the (2) The diphtheria vaccine is given much earlier in
child to reach by age 1. life.
21. The Answer is 3 (3) The pertussis (whooping cough) vaccine is given
A parent is discussing the behavior of her 3-yearold much earlier in life.
child with the nurse. At 3 years, the nurse would (4) The CDC recommends that college freshmen living
expect the client’s child to be doing all of the following in dormitories get the meningitis vaccine, but
EXCEPT which activity? this is unlikely to apply to those over age 60.
Category: Developmental stages and transitions 25. The Answer is 1, 2, 4, and 5
(1) Saying “no” often is an appropriate behavior at The nurse is teaching about the challenges of smoking
this age. cessation. Which of the following factors will
(2) Using a limited vocabulary of 500–3,000 words the nurse identify as known challenges that clients
is an appropriate behavior at this age. face when attempting to quit smoking? Select all
(3) CORRECT: Only three- or four-word sentences that
can be expected at this age. apply.
(4) Believing adults know everything is an appropriate Category: Health promotion/disease prevention;
behavior for this age. High risk behaviors
22. The Answer is 1 (1) CORRECT: Stress and depression are known
The nurse is teaching a group of mothers of toddlers challenges to smoking cessation.
how to prevent accidental poisoning from (2) CORRECT: Continued smoking is more prevalent
medications. among those with a low level of income.
The nurse teaches the mothers to store medications (3) A low, not high, level of education has been
in which of the following locations? found to be associated with continued smoking.
Category: Aging process; Developmental stages and (4) CORRECT: Continued smoking is more prevalent
transitions among those with psychosocial problems.
(1) CORRECT: A secure, locked place is the only (5) CORRECT: Continued exposure to
safe place. smokingassociated
(2) Children have been known to pull childproof stimuli is a known challenge to smoking
caps off, especially if the cap is not fully engaged. cessation.
(3) Children have been known to climb up on counters 26. The Answer is 2, 3, and 4
and other surfaces, so placing medications Stress reduction techniques include biofeedback and
on a high shelf is not necessarily safe. meditation. The nurse conducting classes on these
(4) The problem is the toddler’s natural curiosity, methods knows that studies have shown a causeand-
not whether the toddler recognizes the item as a effect relationship between stress and which of
medication vial. If containers are disguised, this the following? Select all that apply.
might also cause a medication error. Category: Health promotion/disease prevention
23. The Answer is 2
(1) No association between stress and adverse three clients for surgery. Each has a different learning
medication preference. Match the learning preference to the
effects is known at present. appropriate approach. All options must be used.
(2) CORRECT: Research shows a relationship Category: Principles of teaching/learning
between stress and infectious diseases. 1. (B): Brochures about preparation activities are
(3) CORRECT: Research shows a relationship visual: the client needs to see words and pictures.
between stress and traumatic injuries, such as 2. (C): Models of the relevant anatomy are tactile:
motor vehicle accidents. the client needs to touch the model.
(4) CORRECT: Research shows a relationship 3. (A): Discussions about the surgery are auditory:
between stress and some chronic illnesses. the client needs to hear the words.
27. The Answer is 2 175
The nurse is performing the initial assessment of an Psychosocial integrity, along with physiological
adult from a culture the nurse is not familiar with, integrity, is a basic health need for all clients.
and asks about the client’s use of alternative It is the state of dynamic psychological and
therapies. sociological homeostasis, which may be affected
The client says, irritably, “Do you have to ask during periods of stress, illness, or crisis. Any threats
all these questions?” Which of the following is the to a person’s emotional, mental, and
BEST explanation for what the nurse should do in social well-being can disrupt this homeostasis. Any
response? change in adaptive and coping responses
Category: Health screening may result in counterproductive ways of thinking,
(1) The client is the focus, not the nurse’s education. communicating, feeling, and acting. When
(2) CORRECT: The need to discuss the use of these assisting clients with psychosocial needs, you must be
adjunct therapies with clients in all settings is able to anticipate, recognize, and
imperative. This is important because it could analyze these types of responses.
affect or interfere with other treatment modalities. On the NCLEX-RN® exam, you can expect
(3) The client might become impatient, but that does approximately 9 percent of the questions to relate
not mean that the nurse shortens her clinical to Psychosocial Integrity. This category focuses on
review. promoting and supporting the emotional,
(4) The nurse needs to ask critical questions to get mental, and social well-being of clients experiencing
the complete clinical picture. stressful events, as well as clients with
28. The Answer is 3 acute or chronic mental illness.
The nurse is preparing a community educational Exam content related to Psychosocial Integrity
presentation. The topic is the leading cause of death includes, but is not limited to, the following
for people from ages 1–44. The nurse knows that areas:
which of the following is the leading cause? • Abuse/neglect
Category: Health promotion/disease prevention • Behavioral interventions
(1) Cancer is not the leading cause of death for people • Chemical and other dependencies
from ages 1–44, according to the CDC. • Coping mechanisms
(2) Heart disease is not the leading cause of death for • Crisis intervention
people from ages 1–44, according to the CDC. • Cultural diversity
(3) CORRECT: Unintentional injuries are the leading • End of life care
cause of death for people ages 1–44, according • Family dynamics
to the CDC. • Grief and loss
(4) Diabetes is not the leading cause of death for • Mental health concepts
people from ages 1– 44, according to the CDC. • Religious and spiritual influences on health
NCLEX-RN ® Exam Content R NCLE X-Review • Sensory/perceptual alterations
and Practice • Stress management
29. The Answer is 1 (E), 2 (A), 3 (B), 4 (C), 5 (D) Psychosocial Integrity
The nurse is reviewing the client’s lipid profile to chapter 7
determine if education is needed to reduce the risk 176
of heart disease. The nurse knows how to match NCLEX-RN® Exam Content Review and Practice
healthy target values with lab descriptions. Match • Support systems
the appropriate part of the profile below on the left • Therapeutic communications
to the values on the right. All options must be used. • Therapeutic environment
Category: Health promotion/disease prevention Now let’s review the most important concepts covered
1. (E): Total cholesterol should be less than 200 mg/ by the Psychosocial Integrity category
dL. on the NCLEX-RN® exam.
2. (A): HDL cholesterol for men should be more Abuse/Neglect
than 40 mg/dL. Abuse includes physical abuse, physical neglect,
3. (B): HDL cholesterol for women should be more sexual abuse, and emotional abuse and
than 50 mg/dL. neglect. You should be familiar with your state’s laws
4. (C): LDL cholesterol should be less than 100 mg/ for reporting suspected or known
dL. abuse. In addition, you must be able to identify risk
5. (D): Triglycerides should be less than 150 mg/dL. factors and recognize signs of possible
30. The Answer is 1 (B), 2 (C), 3 (A) abuse and neglect and their roles in follow-up care.
The nurse is assessing the best approach to prepare
All suspected cases of child abuse must be reported to symptom management. You should be able to
the appropriate agency or authority. evaluate the client’s response to the treatment
It is not sufficient just to document the suspected plan.
abuse in the medical record. Risk factors Nursing interventions that you should be familiar with
for child abuse include: include:
• Past or present spousal abuse • Maintaining routine interactions, activities, and close
• Perception of stress observation
• Life changes • Developing an open and honest relationship with
• Age at birth of first child respectful and clearly verbalized
• Education expectations
• Little or no prenatal care • Verbalizing acceptance of the client despite
• Having an unlisted phone/not having a phone inappropriate behavior
• Low income • Providing role modeling through appropriate social
• Current unemployment and professional interactions with
• Evidence of harsh discipline other clients and staff
Elder abuse can affect either sex, but usually the • Encouraging the client to assume responsibility for
victims are women who are over 75 years his or her own behavior but verbalizing
of age, physically or mentally impaired, and willingness to assist
dependent on the abuser for their care. Nurses 178
can intervene by educating caregivers about the NCLEX-RN® Exam Content Review and Practice
needs of older adults and making resources • Providing positive reinforcement
available to provide support. A legally competent • Orienting the client to reality
adult, however, cannot be forced to leave • Encouraging the client to attend group therapy
the abusive situation. sessions, if appropriate
Domestic/spousal abuse affects families at all You should also know how to help the client achieve
socioeconomic levels. Risk factors for domestic and maintain behavioral self-control,
abuse include: including strategies that the client can use to decrease
• Planning to leave or having recently left an abusive anxiety.
relationship Chemical and Other Dependencies
• Having been in an abusive relationship in the past Substance abuse is the harmful use of psychoactive
• Poverty or poor living situation substances, including alcohol and illicit
• Unemployment drugs. A history of substance abuse may reflect
177 several risk factors for health problems. Substance
7: Psychosocial Integrity use may coexist with other psychiatric,
• Physical or mental disability developmental, or cognitive problems, and is
• Separation or divorce closely related to certain medical complications such
• Abuse as a child as pancreatitis and ulcers. Substance
• Social isolation from family and friends abuse also affects the client’s relationship with the
• Having witnessed domestic violence as a child environment, family, and society. The client
• Pregnancy, especially if unplanned may deny the problem. Non-substance-related
• Being younger than 30 years old dependencies include gambling addiction,
• Being stalked by a partner sexual addiction, and addiction to pornography,
In any abuse situation, you should communicate among others.
openly, encourage victims to share their Nursing priorities when dealing with a client with
problems, provide counseling and information about chemical and other dependencies include:
resources and coping strategies, provide • Maintaining the physiological stability of clients
support, and educate your clients. In addition, you experiencing substance-related withdrawal
should know how to plan interventions or toxicity by providing symptom management. For
for victims and suspected victims, and help direct example, benzodiazepines
them to a safe environment. It is also are often part of treating alcohol withdrawal, with its
important to evaluate a client’s response to symptoms of tremors, diaphoresis,
interventions. and elevated heart rate.
Behavioral Interventions • Promoting client safety. This might include using
Nurses can intervene, helping to restore a client’s restraining devices, even against a
ability to evaluate reality correctly. Characteristics client’s wishes, to ensure that the client does not get
of altered mental processes that you should be hurt.
familiar with include: • Educating the client about chemical and other
• Disorientation dependency complications and dangers.
• Altered behavioral patterns • Providing appropriate referral and follow-up.
• Altered mood states • Encouraging and supporting involvement in an
• Impaired ability to perform self-maintenance intervention process (counseling).
activities • Teaching friends and family members how to
• Altered sleep patterns provide ongoing support, and encouraging
• Altered perceptions of surroundings their participation in support groups.
The treatment plan should respond to the specific • Evaluating the client’s response to the treatment
needs of the client for structure, safety, and plan.
Coping Mechanisms provide teaching on methods, support systems, and
How a client responds to life’s stressors depends on available resources to cope with stress
the client’s coping resources—for example, and tension.
social support networks and problem-solving skills. Crisis Intervention
Sociocultural and religious factors A crisis is an emotionally significant event or radical
can also influence how a client handles problems. change of status in a person’s life. It is
Some clients may not have the resources an unstable and/or crucial time with the possibility of
or skills to cope with stressors. You should be able to an undesirable outcome—a situation
assess these client support systems, that has reached a critical stage. During a crisis, you
resources, and skills, as well as a client’s response to should:
illness and the emotional reaction of a • Identify the client’s history of the present problem.
family to a client’s illness. • Identify the client’s current feelings.
179 • Assess the client’s support systems.
7: Psychosocial Integrity • Teach crisis intervention techniques to assist the
Characteristics of the inability to cope that you should client in coping.
be familiar with include: • Assess the client’s potential for self-harm or harm to
• Verbalization of the inability to cope others.
• Inability to make decisions or ask for help Goal planning is based on nursing assessment and
• Destructive behavior toward self or others diagnosis, and outcomes are compared to
• Physical symptoms goals and the client’s response.
• Emotional tensions Goals of crisis intervention include:
• General irritability • Decreasing emotional stress and protecting the
Factors related to the inability to cope include, but are client from additional stress
not limited to: • Assisting the client in organizing and mobilizing
• Diagnosis of a serious illness resources or support systems to meet
• Change in health status the client’s needs, and reaching a solution for that
• Unsatisfactory support system situation
• Inadequate psychological resources • Returning the client to a pre-crisis level of
• Situational crises functioning
You should also be familiar with the variety of different Assessing the risk for suicide includes asking
defense mechanisms your client may questions (from general to specific, as well as
employ, and be able to evaluate whether uses of these about plans and lethality), obtaining a history,
mechanisms are constructive or not, assessing mental status, and assessing the signals
such as: given by the client that may indicate that he or she is
• Denial: Completely rejecting a thought or feeling at high risk for suicide. The highest
• Suppression: Vaguely aware of a thought or feeling priority for patients at risk for suicide is safety. Thus,
but trying to hide it arrangements might have to be made to
• Projection: Thinking someone else has the same provide constant observation of the high risk client.
thought or feeling Cultural Diversity
• Acting out: Performing an extreme behavior in order Caring varies among different racial and ethnic groups
to express thoughts or feelings the in its expressions, processes, and
person feels incapable of otherwise expressing patterns. Cultural competence requires you to
• Displacement: Redirecting feelings to another target understand the client’s world views as well
• Isolation of affect: “Thinking” the feeling but not as your own, while avoiding stereotyping. You can
really feeling it obtain cultural information by asking
• Intellectualization: Avoiding the emotion of an act or questions, and then apply the knowledge to improve
feeling by substituting a rational the quality of client care and outcomes.
explanation. This requires flexibility on your part and respect for
• Regression: Reverting to an old, usually immature other viewpoints. To do so, you should:
behavior to ventilate a person’s feeling • Listen carefully to the client.
• Reaction formation: Turning the feeling into its • Learn about the client’s beliefs regarding health and
opposite illness.
• Rationalization: Coming up with various 181
explanations to justify the situation (while 7: Psychosocial Integrity
denying personal feelings) • Show respect, understanding, and tolerance of the
• Sublimation: Directing the feeling into a socially client’s cultural background and
productive activity practices.
• Dissociation: Losing track of time and/or person, • Provide culturally appropriate care.
and instead finding another representation • Identify language needs and use appropriate
of self in order to continue in the moment interpreters, as necessary. Avoid bias and
Additionally, you need to provide clients with subjectivity by arranging for nonfamily translation
opportunities to express their thoughts and assistance.
feelings, help them set realistic goals, assist them in • Document how the client’s language needs were
constructive problem solving, and met.
180 End of Life Care
NCLEX-RN® Exam Content Review and Practice
A client has the right to make informed choices about about legal issues surrounding death, such as advance
his or her end of life care that reflects directives, autopsies, organ
personal, cultural, and religious values. donation, and do-not-resuscitate (DNR) orders.
Nurses provide support, education, and impartial You also need to take the time to analyze your own
interpretation of medical information in a feelings about death before you can
way that clients and families can understand, which effectively help others.
may include treatment options as well as Additional nursing responsibilities include:
the right to refuse treatment. This requires open, • Brainstorming ways to provide relief from loneliness,
honest, sensitive communication and effective fear, and depression
teamwork. You should encourage clients and families • Helping clients maintain a sense of security
to express their goals and wishes, • Helping clients and families accept the loss
and then tailor the care plan to the needs of each • Providing physical comfort measures
client and family. As a nurse, you have an • Providing emotional support, structure, and
ethical and legal duty to respect the client’s wishes, continuity
choices, and priorities. • Allowing expression of thoughts and feelings
You also need to prepare the client and family for what Mental Health Concepts
to expect during the final phase of Mental health is a positive state in which one is
a terminal illness, which includes the physical aspects responsible, displays self-awareness, is selfdirective,
of a deteriorating condition and the is reasonably worry-free, and can cope with usual
act of dying. As the client’s death approaches, the daily tensions. Such individuals
family may become more anxious. It is function well in society, are accepted within a group,
important for you to teach the family about the signs and are generally satisfied with their lives.
and symptoms of impending death, as Influences on mental health include inherited
well as reassure the family that the health care characteristics, nurturing during childhood,
providers are making the client as comfortable and life circumstances. Influences on maintaining
as possible. After the death, you acknowledge the mental health include interpersonal communication,
loss, express sympathy, and provide the use of ego defense mechanisms, and the presence
the opportunity for the family to view the body, but of support people.
only after asking if they wish to do so. Nurses focus on different aspects of care based on the
Family Dynamics identified needs or presenting problems
Family members ideally support each other by of patients. You should also be able to apply your
listening, empathizing, and reaching out to knowledge of client psychopathology
one another. When communication patterns are to mental health concepts.
dysfunctional, the result can be gross 183
misunderstanding, 7: Psychosocial Integrity
which may lead to hostility, anger, or silence. Religious and Spiritual Influences on Health
You need to be able to assess a family’s dynamics and Religion and spirituality have a great influence on the
ability to function constructively by health of clients and how they cope,
closely observing how well family members and make a difference in physical and psychosocial
communicate. You should also assess coping outcomes. You should promote your
mechanisms that determine how families relate to clients’ physical, emotional, and spiritual health,
stress, and evaluate resources and support because this balance of well-being is essential
systems available to the family. to a client’s overall health. You must strive to be an
Family units may be vulnerable to health problems empathetic listener and attempt to
based on various factors, such as heredity, identify your clients’ spiritual needs. To accomplish
developmental level, and lifestyle practices. You this, the nurse should understand how
should plan interventions, such as encouraging spirituality influences clinical care.
participation in group/family therapy. That Nurses should be knowledgeable about religious
intervention can assist the family with traditions and spiritual expressions other
182 than their own. You should approach each client based
NCLEX-RN® Exam Content Review and Practice on that client’s distinct need, because
realistic strategies that enhance family functioning, people develop and nurture their own spirituality in
such as improving communication skills different ways. Each client’s spiritual
and identifying and utilizing support systems. beliefs or religious practice should influence how you
Grief and Loss care for that client. Clients have the
Grieving is a normal, subjective emotional response right to receive care that respects their religious and
to loss and is essential for mental and spiritual values. At the same time, they
physical health. How a client or family responds to have the right to refuse care on religious grounds.
loss, and how they express grief, varies Sensory/Perceptual Alterations
widely. Factors that influence the process of grieving A disruption in a client’s cognitive processes can lead
include age, stage of development, gender, to faulty interpretations of their surroundings.
culture, and personal reserves and strengths. Alterations in sensory perception, or altered thought
You should know the different stages of grieving and processes, affect a client’s ability to
factors that influence how clients and function within his or her environment, which may
families react to death to understand their responses place the client at risk for harm. You
and needs. You must also be knowledgeable
should assist the client to function safely in health care role strain.
settings. Therapeutic Communications
Some factors that influence sensory function include You use therapeutic communication techniques to
developmental stage, culture, stress, promote understanding and establish
medications, illness, lifestyle, and personality. a constructive relationship with the client. Therapeutic
You need to identify clients at risk for communication is planned, and is
sensory/perceptual alterations so you can initiate client- and goal-directed. It means listening to and
preventive understanding the client while promoting
measures. Examples of clients at risk include those 185
who: 7: Psychosocial Integrity
• are confined in a non-stimulating environment. clarification and insight. It enables the nurse to form
• have impaired vision or hearing. a working relationship with the client
• have mobility restrictions. and peers, using both verbal and nonverbal
• have emotional disorders. communication. Remember that nonverbal
• have limited social contact. communication
• are experiencing pain or discomfort. is the most accurate reflection of attitude.
• are acutely ill. You should be familiar with the foundations for a
• are closely monitored (such as in the ICU). therapeutic relationship, which include:
• have decreased cognitive ability (as in a head • An understanding of the factors influencing
injury). communication
184 • Realization of the importance of nonverbal
NCLEX-RN® Exam Content Review and Practice communication
When dealing with such a client, you should organize • Development of effective communication skills
nursing care to reduce unessential • Recognition of the causes of ineffective
stimuli; orient the client to person, place, and time communication
during every contact; and explain all • Ability to participate in a therapeutic communication
nursing care. process
Stress Management You should also be familiar with the conditions
Everyone experiences stress, which can result from essential for a therapeutic relationship, which
both positive and negative experiences. include:
A person’s response to any change in homeostasis • Empathy
results in stress. Stress indicators can be • Respect
physiologic (increased heart rate or respirations, • Genuineness
muscle tension), psychological (anxiety, • Self-disclosure
fear, anger), and/or cognitive (thinking responses). • Concreteness and specificity
Consequences of stress may be physical, • Confrontation (limited to a well-established
emotional, intellectual, social, spiritual, or any nurse/client relationship with an accepting,
combination of these. gentle manner)
To minimize stress in a client, you should help the It is important to understand the client’s views and
client to do the following: feelings before responding. You also need
• Determine situations that precipitate anxiety. to recognize barriers to effective communication, such
• Verbalize feelings, perceptions, and fears, as as:
appropriate. • Failure to listen
• Identify personal strengths. • Improperly decoding the client’s intended message
• Recognize usual coping patterns. • Placing the nurse’s needs above the client’s needs
• Identify new strategies. • Stereotyping, challenging, probing, and/or rejecting
You should also listen attentively, provide an • Being defensive
atmosphere of warmth and trust, provide factual • Changing topics and subjects
information as needed, encourage clients to • Passing judgment
participate in the plan of care, promote Effective therapeutic responses include:
safety and security, and provide education. Responses • Using silence: Allows the client time to think and
to stress are called coping mechanisms. reflect; conveys acceptance; allows the
Support Systems client to take the lead in the conversation
A support system is a network of personal contacts • Using general leads or a broad opening: Encourages
that are available to clients for practical, the client to talk; indicates your interest
emotional, or moral support when needed. Support in the client; allows the client to choose the subject
systems are important to clients in • Clarification: Encourages recall and details of a
that they enhance client learning, offer support, help particular experience; encourages
the client perform required skills, and description of feelings; seeks explanation; pinpoints
help the client maintain required lifestyle changes. specifics
Exploring the client’s support system is a • Reflecting: Paraphrases what client says
component of the initial assessment. NCLEX-RN ® Exam Content Review and Practice
Caregivers might also need to be connected to outside 186
resources. For example, community NCLE X-Therapeutic Environment
support groups are appropriate interventions for Nurses provide care for clients who constantly interact
family members suffering from caregiver with their environment. Clients may
have unmet needs, be unable to care for themselves, by the nurse?
or be unable to adapt to the environment 1. The nurse allows the young man to
due to health problems. You provide therapeutic care refuse, because clients do have a right to
so clients can adapt to their refuse care.
environment. 2. The nurse implements the intervention,
The Nursing Process and Psychosocial Integrity because protecting the client’s safety
You utilize the nursing process (assess, diagnose, trumps the client’s right to refuse care.
plan, implement, and evaluate) to promote 3. The nurse checks on the client every hour
a client’s psychosocial integrity by conveying to be sure he is safe.
understanding, sensitivity, and compassion to a 4. The nurse asks the NAP to check on the
client who is experiencing stress, illness, or crisis. client every 30 minutes to be sure he is
Promoting a client’s psychosocial integrity safe.
is not just for the mental health client, but for all 4. A client is scheduled to have surgery the
clients. The nursing process respects the following day. The client tells the nurse,
client’s autonomy, freedom to make decisions, and “I’m very scared. I have never had surgery
involvement in nursing care. before and am afraid that I might not make it
Although you need to identify emotional disorders and through.” Which of the following responses
behaviors that indicate mental illness, by the nurse is the MOST appropriate?
a client does not need to be mentally ill for you to 1. “Why do you feel this way?”
include psychosocial integrity in the care 2. “Don’t worry, you will be fine.”
plan. You must possess sound knowledge and focused 3. “Why don’t we take some time to explore
clinical experiences to be prepared to why you feel this way?”
recognize and effectively intervene with any client 4. “It’s completely normal to be scared. You
whose state of dynamic psychological and will be taken care of. Tell me how you are
sociological homeostasis is being threatened— feeling.”
whether or not the client has a mental illness. 5. The nurse is working on a pediatric unit.
7: Psychosocial Integrity The client is a 13-month-old child diagnosed
187 with failure to thrive. The parents report that
1. The nurse cares for an elderly client who the child cries frequently, does not like to be
appears fully alert and oriented. As it held, and will not eat. The nurse learns that
gets later in the day, the nurse notices the the child’s uncle lives in the house with the
client becoming increasingly confused and family. When the uncle visits in the hospital,
agitated. It would be MOST appropriate the nurse notices the child acting differently
for the nurse to take which of the following and turning away from the uncle. Sometimes
actions? the child’s heart rate increases when the
1. Reorient the client, and then turn on the uncle is present. The nurse should take which
lights and television to distract the client of the following actions FIRST?
from his confusion. 1. Immediately report the possible situation
2. Encourage the client’s alert roommate to of abuse to the authorities.
talk with the client. Chapter Quiz
3. Tell the client he is at home in his own bed NCLEX-RN® Exam Content Review and Practice
to get him to settle down and go to sleep. 188
4. Reorient the client, pull the shades down, 2. Call the physician, who will probably
shut the lights and television off, and have more long-term knowledge.
promote a quiet environment. 3. Discuss it with other nurses to see which
2. On the evening shift, the nurse is caring for a approaches they have taken.
client who will be undergoing a mastectomy in 4. Encourage the team that’s caring for the
the morning. A call from the front desk alerts client to have a family meeting including
the nurse that the client’s family has arrived. It the parents, but not the uncle, to gather
would be MOST appropriate for the nurse to more information.
take which of the following actions? 6. The nurse learns that the client’s sibling has
1. Tell the family that they cannot come in passed away during his hospitalization, and
because visiting hours are over. the client is distraught by this news. Which
2. Tell the client you want to make sure she of the following should the nurse do FIRST?
has some alone time to relax. 1. Allow the client an opportunity to
3. Invite the family in to offer support after verbalize feelings, and inquire if the client
confirming with the client. would like to be visited by social services,
4. Tell the nursing assistive personnel (NAP) chaplaincy, or psychiatry for support.
to sit with the client who needs company. 2. Provide alone time by not going into the
3. The nurse is caring for a young man who has client’s room unless absolutely necessary.
expressed his desire to commit suicide. He 3. Call psychiatry services to arrange for
has informed the nurse of plans to pursue them to see the client as soon as possible.
this. The nurse requests a sitter to stay with 4. Find out which religion the client
the client around the clock, but the client practices by viewing the chart and then
says he does not want this. Which of the request a chaplain from that religion to
following is the MOST appropriate response see the client.
7. The nurse is working on a busy locked 4. “The goal of the medication is to reduce
psychiatric unit. The alarm gets tripped when symptoms associated with bipolar
somebody tries to go through the locked disorder and to hopefully help with the
doors without permission from the front desk. mood swings.”
Which of the following actions should the 11. The home care nurse makes a visit to the
nurse take after the alarm is tripped? home of an elderly client who has episodic
1. Reset the alarm from the front desk confusion but who has remained safe at
after verifying that everybody is safe and home while occasionally alone. The nurse
nobody has escaped from the unit. finds the client disheveled, confused, and
2. Reset the alarm from the location where agitated, and the home is messy. This
the alarm was tripped after verifying that degree of confusion is unusual for this
everybody is safe and nobody has escaped client. The nurse takes the client’s vital
from the unit. signs, which are BP 115/70, HR 70, RR 16,
3. Reset the alarm from a client’s room after and temperature 98.7º F (37º C). Which of
doing a quick scan of the hallway. the following actions should the nurse take
4. Reset the alarm from the front desk FIRST?
once the receptionist says everybody is 1. Nothing, because the client’s vital signs
accounted for. are stable.
8. The client is an intoxicated male on the 2. Plan to come back the following day to
medical/surgical unit who attempts to get out reevaluate the client.
of bed every few minutes. He is unsteady on 3. Encourage the client to verbalize his or
his feet, and the nurse is concerned that he her feelings.
will fall if he does get out of bed. The doctor 4. Call the client’s family to take the client
writes an order for the nurse to place wrist to be evaluated by a physician because
restraints to maintain the client’s safety and the client is not safe to be alone right
prevent him from falling. The man refuses now.
the restraints. The nurse should take which 12. The nurse is on an Alzheimer’s unit. A
of the following actions? client is agitated and pulling at things.
1. Place the restraints in compliance with Which of the following should the nurse do?
hospital policy. 1. Provide the client with therapeutic
2. Refrain from placing restraints to honor sensory devices.
the client’s wishes, because he has the 2. Cohort the client with another client
right to refuse care. who is agitated, because they will calm
3. Call the physician for advice on how to each other.
proceed. 3. Place the client in a room with several
4. Check on the client every hour to ensure other clients.
his safety. 4. Leave the client alone for a period of
9. The nurse is working in an outpatient time to reduce stimulation.
clinic. The nurse has a client who 13. The nurse is caring for a terminally ill
appears intoxicated and who drove to the client who has agreed to enter hospice care.
appointment. The nurse is concerned about Which of the following statements by the
the client’s ability to drive home. Which of spouse indicates a need for further teaching
the following should the nurse do FIRST? by the nurse?
1. Call the police immediately. 1. “You will help to make my spouse as
2. Ask the client’s permission to call a comfortable as possible while in hospice
family member or friend for a ride. care.”
3. Give the client a ride home to protect his 2. “You will help my spouse get better so
privacy. we can get back to our old life.”
4. Call clinic security to detain the client to 3. “The goal is to make the end of my
protect his safety. spouse’s life as comfortable as possible.”
10. The mother of a teenage client who has 4. “You will provide me with support
permission to be involved in the plan of during this difficult time.”
care is asking the nurse questions, after it 14. The nurse is caring for a male client. The
has been explained to her that her child has client has exhibited some signs of anxiety
bipolar disorder. Which of the following and hostility. The nurse is aware that the
statements by the mother indicates that client is a recently returned combat veteran.
further teaching is needed? The nurse should assess the client for which
1. “My child will be cured after being on of the following conditions?
medications for a few months.” 1. Post-traumatic stress disorder (PTSD)
7: Psychosocial Integrity 2. Bipolar disorder
189 3. Schizophrenia
2. “My child will require support and 4. Borderline personality disorder (BPD)
encouragement.” 15. The nurse is caring for a client with a
3. “My child will be on psychiatric known past medical history for intravenous
medications probably for the rest of her substance abuse. The client requests to
life.” go outside for a few minutes to smoke a
cigarette and promises to come right back. would be MOST appropriate?
The client has a peripheral intravenous 1. “It may seem to you that there are bugs
line in. The nurse should take which of the crawling on the floor, but I do not see
following actions? any bugs.”
1. Allow the client to go outside but set a 2. “I see them too. How should I kill
time limit in which to return. them?”
2. Call security to escort the client to an 3. “Can you tell me more about these
approved smoking area. bugs?”
NCLEX-RN® Exam Content Review and Practice 4. “What do the bugs look like?”
190 20. The client has had a depressed mood,
3. Make a behavioral contract with the decreased sleep, poor concentration, and
client that includes an agreement to have poor appetite for the past 4 months. Which
the NAP accompany the client outside. of the following does the nurse expect the
4. Watch the client from the window to physician to prescribe?
make sure the IV line stays open. 1. Quetiapine
16. The client is a non-English-speaking 2. Haloperidol
elderly woman who is being admitted to 3. Mirtazapine
the hospital for worrisome symptoms. She 4. Clonazepam
is accompanied by family members who 21. A client is experiencing a manic episode.
speak English. The nurse admitting the It would be MOST appropriate for the
client needs to ask some general admission nurse to perform which of the following
questions. It would be MOST appropriate interventions?
for the nurse to take which of the following 7: Psychosocial Integrity
actions? 191
1. Call the hospital’s interpreter services to 1. Give the client materials to make a
assist with asking the client questions in collage.
her native language. 2. Encourage the client to use an exercise
2. Ask family members the questions and bike.
document their responses. 3. Encourage the client to attend a group
3. Ask family members to translate and ask about managing feelings.
the questions for the nurse. 4. Ask the client to play a board game with
4. Document “Unable to obtain answers, other clients.
patient does not speak English.” 22. A client with bipolar disorder makes a
17. The client has a medical history of alcohol sexually inappropriate comment to the
abuse and had a drink yesterday. The nurse nurse. The nurse should take which of the
notes tremors, diaphoresis, and an elevated following actions?
heart rate. The nurse should perform which 1. Ignore the comment because the client
of the following actions FIRST? has a mental health disorder and cannot
1. Call the physician to report help it.
the symptoms and administer 2. Report the comment to the nurse
hydromorphone per the alcohol manager.
withdrawal pathway. 3. Ignore the comment, but tell the
2. Assess the client every hour to monitor incoming nurse to be aware of
symptoms. the client’s propensity to make
3. Call the family and administer inappropriate comments.
meperidine per the alcohol withdrawal 4. Tell the client that it is inappropriate for
pathway. clients to speak to any nurse that way.
4. Administer lorazepam per the alcohol 23. The nurse makes a home visit to a child
withdrawal pathway. with a G-tube. Upon arrival, the nurse
18. A client with post-traumatic stress disorder notices that the client’s sibling is wearing
(PTSD) appears to be having a flashback. dirty clothes that are too small. The
It would be MOST appropriate for the nurse also notices that there is no food
nurse to perform which of the following in the refrigerator or in the kitchen
interventions? cabinets. Which of the following MOST
1. Encourage the client to tell the nurse appropriately describes how the nurse
how the client is feeling in that moment. should respond to these observations?
2. Calmly reorient the client to the current 1. The nurse should not be concerned
situation. because the sibling is not her client and
3. Assist the client in acting out the event. the client is being fed through a G-tube
4. Tell the client loudly that what the client appropriately.
is experiencing is not real. 2. The nurse should not be concerned
19. An elderly client asks the nurse to kill the because there are no signs of physical
bugs that are crawling on the floor of her abuse.
room. The nurse does not see any bugs and 3. The nurse should be concerned and
suspects the client is hallucinating. Which take action because there is no food or
of the following statements by the nurse appropriate clothing available to the
sibling. ideation. The 24-hour observer calls the
4. The nurse should not be concerned nurse to report that the client took off down
because her client is well cared for. the hall. The nurse is unable to immediately
24. The nurse is caring for a hospice client locate the client. Arrange the following
who lives at home with an attentive spouse. actions by the nurse in the order that is
The client’s spouse quit work to care for MOST appropriate. All options must be
the client. During the nurse’s visit, the used.
spouse expresses frustration and hostility 1. Notify security that the client has
toward the nurse. Which of the following eloped, and provide a description of the
are appropriate interventions by the nurse? client.
Select all that apply. 2. Notify the nurse manager.
1. The nurse should encourage the spouse 3. Notify other staff on the unit.
to verbalize feelings. 4. Ask the observer in what direction the
2. The nurse should encourage the spouse client headed.
to attend a caregiver support group. 29. The nurse discovers a hospice client
3. The nurse should encourage the spouse has expired. The family members are
to go back to work part-time. regrouping in the facility’s waiting room.
4. The nurse should encourage the spouse Which of the following actions by the nurse
not to verbalize negative feelings that would be the MOST appropriate?
may upset the client. 1. Tell the family it would not be in their
25. The nurse is taking a history from a client best interests to see their loved one.
in an outpatient clinic. The client has been 2. Encourage the family to view the body
taking lorazepam for 6 months. Which of to help accept the situation.
the following is the MOST likely side effect 3. Provide condolences to the family and
that the nurse would expect to see as a offer them viewing time.
result of the client using Ativan for this time 4. Tell the family “I will give you some time
period? to spend with your loved one. Let me
1. Excessive appetite know if you need anything.”
2. Physical dependence 30. The nurse is caring for a newly admitted
3. Suicidal ideation client in a hospital setting. The client was
4. Seizure activity recently diagnosed with cancer but is
26. A client requires a lifesaving blood alert and oriented. The client is a Greek
transfusion per hospital guidelines. The immigrant, but does speak English. During
client refuses based on religious beliefs. It the admission process, the nurse inquires
would be MOST appropriate for the nurse about advance directives with the client.
to take which of the following actions? The client tells the nurse: “I do not want
NCLEX-RN® Exam Content Review and Practice to make any medical decisions. I want
192 my daughter to make these decisions for
1. Confirm with the client that the client me.” The nurse should take which of the
understands the potential risks of not following actions?
having the blood transfusion. 1. Make sure that the written advance
2. Tell the client that, regardless of directives document the client’s wishes.
personal beliefs, the client has to have 2. Tell the client that, being alert and
the lifesaving transfusion. oriented, the client should make his or
3. Call the Legal Department of the her own medical decisions.
hospital immediately. 3. Tell the client that due to confidentiality,
4. Try to gently encourage the client to the daughter will not be informed of
change his or her mind. details of the client’s care.
27. The nurse monitors clients’ medications in 4. Encourage both the daughter and the
a day program for clients with disabilities. client to work together on making
The nurse notices a teenage client who is medical decisions.
frequently alone and often quiet. It would 7: Psychosocial Integrity
be MOST appropriate for the nurse to take 193
which of the following actions? 1. The Answer is 4
1. Allow the client alone time since the The nurse cares for an elderly client who appears
client seems to prefer this. The client has fully alert and oriented. As it gets later in the day, the
the right to make that choice. nurse notices the client becoming increasingly
2. Make an effort to interact with the client confused
periodically. and agitated. It would be MOST appropriate
3. Encourage the client to join a youth for the nurse to take which of the following actions?
group. Category: Therapeutic environment
4. Encourage other clients in the program (1) Although the nurse would reorient the client, the
to interact more frequently with the nurse would not turn on the lights and television
client. in an attempt to distract. Clients with confusion
28. The nurse on the inpatient psychiatric ward can become increasingly more agitated with
is caring for a client with known suicidal stimulation, such as lights and television.
(2) Encouraging the client’s roommate to talk with responses by the nurse is the MOST appropriate?
the client is another inappropriate attempt at Category: Therapeutic communications
distraction. Chapter Quiz Answers and Explanations
(3) Reassuring clients is usually a good practice, but NCLEX-RN® Exam Content Review and Practice
it is not appropriate unless the nurse is honest 194
with the attempt to reorient to an accurate location, (1) Avoid asking “Why” questions because they
time, and place. imply disapproval with what the client is saying.
(4) CORRECT: Promoting a quiet environment (2) Telling the client not to worry, and that the client
decreases stimulation to prevent agitation. It will be fine, dismisses the client’s feelings and
also promotes the normal sleep-wake cycle, provides for false reassurances.
consistent (3) The nurse must remain within the nursing scope
with it being “later in the day.” of practice. The nurse is not a therapist, so asking
2. The Answer is 3 the client to explore his feelings with the
On the evening shift, the nurse is caring for a client nurse would not be appropriate.
who will be undergoing a mastectomy in the morning. (4) CORRECT: A response that tells the client that
A call from the front desk alerts the nurse that it is normal to be scared, and that he will be
the client’s family has arrived. It would be MOST taken care of, and asks how he is feeling, normalizes
appropriate for the nurse to take which of the the client’s experience, provides some
following reassurance, and allows for him to verbalize.
actions? 5. The Answer is 4
Category: Support systems The nurse is working on a pediatric unit. The client
(1) Enforcing nursing unit rules, in this instance is a 13-month-old child diagnosed with failure
by telling the family they cannot come because to thrive. The parents report that the child cries
visiting hours are over, is not considered a best frequently,
answer on the NCLEX-RN® exam. On the exam, does not like to be held, and will not eat.
the nurse should do what is in the best interests The nurse learns that the child’s uncle lives in the
of the client. house with the family. When the uncle visits in the
(2) The client may need some time alone, but the hospital, the nurse notices the child acting differently
nurse would include the client in making that and turning away from the uncle. Sometimes
decision. the child’s heart rate increases when the uncle is
(3) CORRECT: During times of stress and anxiety, present. The nurse should take which of the following
such as undergoing surgery, nurses should actions FIRST?
promote family support. The answer choice also Category: Abuse/neglect
states that the nurse would ask the client first. (1) Although nurses are mandated to report child
This supports including clients in their care. abuse in almost every state, the question stem
(4) Telling the NAP to sit with client may be does not present enough solid, verifiable facts to
appropriate know whether abuse should be suspected. Thus
at times, but it is not the best option here. the nurse should use the support of other colleagues
3. The Answer is 2 and the interdisciplinary team to make
The nurse is caring for a young man who has this decision.
expressed his desire to commit suicide. He has (2) Although the nurse might want to eventually
informed the nurse of plans to pursue this. The notify the physician of abuse suspicions, this is
nurse requests a sitter to stay with the client around not the first step. Most importantly, the NCLEXRN
the clock, but the client says he does not want this. ® exam wants to see what the test taker would
Which of the following is the MOST appropriate do rather than passing the responsibility to
response by the nurse? someone else.
Category: Crisis intervention (3) Although the nurse might ask for advice from
(1) Although clients do have the right to refuse peers on the client’s care team, the nurse should
care, in certain high risk-to-safety situations (for not discuss a client’s information with those who
example, suicide), nurses put measures in place are not part of the care team.
to prevent harm. (4) CORRECT: The nurse should utilize other
(2) CORRECT: Protecting the client’s safety trumps disciplines
the client’s right to refuse care. in a team fashion and attempt to gather
(3) The nurse could check in on the client every hour more facts before deciding appropriate further
in combination with other interventions. However, steps.
clients at high risk for suicide cannot be left 6. The Answer is 1
alone for any time period. The nurse learns that the client’s sibling has passed
(4) This answer is incorrect for the same reasons as away during his hospitalization, and the client is
answer choice (3): clients at high risk for suicide distraught
cannot be left alone for any time period, even for by this news. Which of the following should
30 minutes. the nurse do FIRST?
4. The Answer is 4 Category: Grief and loss
A client is scheduled to have surgery the following (1) CORRECT: Allowing the client an opportunity
day. The client tells the nurse, “I’m very scared. I to verbalize feelings, and inquiring if the client
have never had surgery before and am afraid that I would like to be visited by social services, chaplaincy,
might not make it through.” Which of the following or psychiatry are all appropriate for the
nurse to do. 9. The Answer is 2
(2) Ignoring the client is not best practice. The client The nurse is working in an outpatient clinic. The
may want to be alone, but the nurse first needs nurse has a client who appears intoxicated and who
to assess the situation and check with the client drove to the appointment. The nurse is concerned
about his needs. about the client’s ability to drive home. Which of the
(3) The nurse would not call psychiatry services following should the nurse do FIRST?
without first evaluating the client’s emotional Category: Chemical and other dependencies
status and needs. (1) The nurse’s goal is to protect the client (and in
(4) It would be important to know if the client is this scenario, potentially the public as well), but
practicing that religion and has current spiritual calling the police immediately is not the best
needs. But the client would dictate that and not first option. The nurse may end up doing this but
the nurse. should first take the time to review other options.
7. The Answer is 2 (2) CORRECT: Asking the client’s permission to
The nurse is working on a busy locked psychiatric call a family member is a better option because
unit. The alarm gets tripped when somebody tries it includes the client in the choice. An intoxicated
to go through the locked doors without permission client may not make good choices, but the client
from the front desk. Which of the following actions may be amenable to good suggestions. Ideally,
should the nurse take after the alarm is tripped? the nurse would find somebody (not the police)
Category: Therapeutic environment to get the client home safely. That would allow
(1) Resetting the alarm from the front desk is not maintaining a trusting nurse-client relationship.
proper procedure. (3) The nurse should not overstep the boundaries
(2) CORRECT: An alarm is a safety mechanism and drive the client home.
meant to alert staff to somebody at risk attempting (4) Calling clinic security to detain the client sounds
to leave. When an alarm is activated, the less threatening than calling the police and might
nurse should first make sure that all clients are be done eventually, but the first option would be
accounted for and safe, and then reset the alarm answer choice 2.
by going to the place where it was tripped. 10. The Answer is 1
7: Psychosocial Integrity The mother of a teenage client who has permission
195 to be involved in the plan of care is asking the nurse
(3) The nurse must be sure, based on firsthand questions, after it has been explained to her that
knowledge, that all clients are safe. Resetting the her child has bipolar disorder. Which of the following
alarm without doing so would not be appropriate. statements by the mother indicates that further
(4) The nurse must be sure, based on firsthand teaching is needed?
knowledge, that all clients are safe. Resetting the Category: Mental health concepts
alarm without doing so would not be appropriate. (1) CORRECT: Bipolar disorder is not curable. Clients
8. The Answer is 1 can suffer from bipolar disorder throughout
The client is an intoxicated male on the their entire lives. The mother’s statement that
medical/surgical the child will be cured after being on medication
unit who attempts to get out of bed every few indicates further teaching about the disorder is
minutes. He is unsteady on his feet, and the nurse needed.
is concerned that he will fall if he does get out of (2) This is an accurate statement. Somebody suffering
bed. The doctor writes an order for the nurse to from this mental illness would need support
place wrist restraints to maintain the client’s safety and encouragement.
and prevent him from falling. The man refuses the (3) This is an accurate statement. Psychotropic
restraints. The nurse should take which of the medications are used to treat bipolar disorder,
following usually for life.
actions? (4) This is an accurate statement. The goal of the
Category: Chemical and other dependencies medication is to reduce symptoms associated
(1) CORRECT: The nurse should place the restraints with bipolar disorder and to lessen mood swings.
in compliance with hospital policy. This is a NCLEX-RN® Exam Content Review and Practice
circumstance 196
where the client’s risk of harm and 11. The Answer is 4
promotion of safety trumps the client’s right to The home care nurse makes a visit to the home of an
refuse. elderly client who has episodic confusion, but who
(2) The client is at risk and intoxicated, so the nurse has remained safe at home while occasionally alone.
should place the restraints. The nurse finds the client disheveled, confused, and
(3) The nurse, at some point, may call the physician agitated, and the home is messy. This degree of
for further assistance, but the NCLEX-RN® confusion
exam wants to know what the test taker would do is unusual for this client. The nurse takes the
rather than passing the responsibility to someone client’s vital signs, which are BP 115/70, HR 70, RR
else. 16, and temperature 98.7º F (37º C). Which of the
(4) The nurse could check on the client every hour, following actions should the nurse do FIRST?
but only in addition to the needed ongoing safety Category: Crisis intervention
measure of restraints or constant observation. A (1) Although the vital signs are within normal limits,
client could fall within minutes; an hour is too the onset of worsening symptoms could be an
long to leave an at-risk client alone. indication of something more serious, so doing
nothing would not be correct. schizophrenia, PTSD is the best answer choice
(2) The nurse may plan to come back the following because of the common link.
day depending on what happens to the client in 7: Psychosocial Integrity
the next 24 hours, but as a first choice, this is not 197
correct. (4) Although veterans can potentially suffer from
(3) Encouraging the client to verbalize his or her borderline personality disorder, PTSD is the best
feelings is not an inappropriate intervention answer choice because of the common link.
given the presenting symptoms. 15. The Answer is 3
(4) CORRECT: These are new symptoms, and the The nurse is caring for a client with a known past
client does not appear safe to be alone. By contacting medical history for intravenous substance abuse.
the family, the nurse is performing an The client requests to go outside for a few minutes to
intervention based on the assessment of the client. smoke a cigarette and promises to come right back.
In the home care setting, assessing safety is The client has a peripheral intravenous line in. The
prioritized, especially with new symptoms. nurse should take which of the following actions?
12. The Answer is 1 Category: Chemical and other dependencies
The nurse is on an Alzheimer’s unit. A client is agitated (1) Allowing the client to go outside for a set time
and pulling at things. Which of the following could potentially be a part of an agreement with
should the nurse do? the client, but review the other choices first.
Category: Sensory/perceptual alterations (2) The client has not shown any signs of eloping and
(1) CORRECT: Alzheimer’s clients often pick at has not threatened anyone. If the client tried to
items, such as buttons on clothing or medical elope, the nurse might then call security. At this
devices, which poses a danger to them. Providing point, the client has merely requested to go outside.
them with safely designed sensory devices serves (3) CORRECT: Contracting with the client is the
the need of stimulating the senses as well as their best choice. The nurse makes a compromise that
urge to pick. the client can go outside but must be supervised
(2) Cohorting the client with another agitated client while doing so. The client is a known abuser of
can worsen the problem due to increased stimulation. intravenous substances, so sending the client
(3) Placing the client in a room with several other outside alone could be a safety risk.
clients can worsen the problem due to increased (4) Watching the client from the window is not an
stimulation. appropriate form of medical supervision.
(4) Leaving the client alone could lead to injuries 16. The Answer is 1
related to the agitation and picking. The client is a non-English-speaking elderly woman
13. The Answer is 2 who is being admitted to the hospital for worrisome
The nurse is caring for a terminally ill client who has symptoms. She is accompanied by family members
agreed to enter hospice care. Which of the following who speak English. The nurse admitting the client
statements by the spouse indicates a need for further needs to ask some general admission questions. It
teaching by the nurse? would be MOST appropriate for the nurse to take
Category: End of life care which of the following actions?
(1) This is an accurate statement. The goal of hospice Category: Cultural diversity
is to make clients as comfortable as possible (1) CORRECT: The only way to avoid bias and
during the remainder of their life. interjection by family members is by utilizing
(2) CORRECT: This is an inaccurate statement. The interpreter services at your hospital.
philosophy of hospice care is not to help a client (2) Asking family members the questions and
recover, but to promote comfort and peace during documenting
the end of life. The presumption is that the their responses will result in obtaining
client will not improve. answers to the questions and being able to create
(3) This is an accurate statement. The goal of hospice some documentation, but the responses should
is to make the end of life as comfortable as come straight from the client.
possible. (3) Asking family members to translate and ask
(4) This is an accurate statement. Hospice care the questions is not appropriate. The responses
involves the family as well as the client. should come straight from the client.
14. The Answer is 1 (4) Documenting “unable to obtain answers, patient
The nurse is caring for a male client. The client has does not speak English” is a poor choice without
exhibited some signs of anxiety and hostility. The trying another more appropriate method.
nurse is aware that the client is a recently returned 17. The Answer is 4
combat veteran. The nurse should assess the client The client has a medical history of alcohol abuse
for which of the following conditions? and had a drink yesterday. The nurse notes tremors,
Category: Mental health concepts diaphoresis, and an elevated heart rate. The
(1) CORRECT: PTSD is a known disorder from nurse should perform which of the following actions
which veterans of war can suffer. Any thorough FIRST?
evaluation of symptoms would include one for Category: Chemical and other dependencies
PTSD. (1) The nurse might call the physician at some point
(2) Although veterans can potentially suffer from to report unmanageable symptoms, but
bipolar disorder, PTSD is the best answer choice hydromorphone
because of the common link. is for pain and not for management
(3) Although veterans can potentially suffer from of alcohol withdrawal.
(2) The nurse might assess the client every hour, but for depression.
it (4) Clonazepam is more typically given for panic
is not the first thing the nurse would do. The nurse disorders.
needs to intervene to prevent acute withdrawal. 21. The Answer is 2
(3) The nurse would not call the family unless the A client is experiencing a manic episode. It would be
nurse had permission of the client, and would MOST appropriate for the nurse to perform which
not give meperidine for withdrawal; it is for pain. of the following interventions?
(4) CORRECT: Benzodiazepines such as lorazepam Category: Coping mechanisms
are often given as part of an alcohol withdrawal (1) Manic energy does not lend itself well to the
pathway; this client is clearly beginning to exhibit patience and organization needed for a collage.
symptoms of withdrawal by having tremors, (2) CORRECT: The exercise bike would allow an
diaphoresis, outlet for the client’s excessive energy.
and an elevated heart rate. (3) During the manic phase, clients do not have the
18. The Answer is 2 patience to sit in a group and discuss feelings.
A client with post-traumatic stress disorder (PTSD) This is not an appropriate intervention.
appears to be having a flashback. It would be MOST (4) During the manic phase, clients do not have the
appropriate for the nurse to perform which of the patience play a board game. This is not an appropriate
following interventions? intervention.
Category: Crisis intervention 22. The Answer is 4
(1) The patient is in crisis mode. Encouraging the A client with bipolar disorder makes a sexually
client to verbalize feelings is not going to bring inappropriate
the client back to reality. comment to the nurse. The nurse should
(2) CORRECT: The nurse wants to calmly orient take which of the following actions?
the client back to the reality of the moment, to Category: Mental health concepts; Behavioral
the actual safe environment. interventions
NCLEX-RN® Exam Content Review and Practice (1) Clients have to be accountable for their own
198 actions even if they have bipolar disorder. It is
(3) Assisting the client in acting out the event is not important to correct inappropriate behavior,
an appropriate intervention. The nurse wants to and to encourage clients to interact socially in
encourage the client back to reality and not go an acceptable way.
further into the flashback. (2) The nurse’s priority is to first communicate with
(4) Although the nurse wants to orient the client to the client; the nurse might want to report the
reality, this would not be done loudly. This could incident to the nurse manager later.
possibly cause more hostility or violence if the (3) The nurse should not ignore the comment.
client feels a sense of heightened danger. (4) CORRECT: The nurse should notify the client
19. The Answer is 1 that this is inappropriate behavior and set up
An elderly client asks the nurse to kill the bugs that appropriate boundaries.
are crawling on the floor of her room. The nurse 23. The Answer is 3
does not see any bugs and suspects the client is The nurse makes a home visit to a child with a
hallucinating. G-tube. Upon arrival, the nurse notices that the
Which of the following statements by the client’s sibling is wearing dirty clothes that are too
nurse would be MOST appropriate? small. The nurse also notices that there is no food
Category: Crisis intervention in the refrigerator or in the kitchen cabinets. Which
(1) CORRECT: This response validates what the client of the following MOST appropriately describes how
is seeing. To the client, a hallucination is real. the nurse should respond to these observations?
However, the nurse must reorient the client to the 7: Psychosocial Integrity
appropriate reality and try to restore the client’s 199
feelings of safety. Category: Abuse/neglect
(2) The nurse should not reinforce the hallucination. (1) Nurses are mandated reporters of child abuse
(3) The nurse should not encourage verbalizing whether or not it is their client.
feelings (2) Although there are no signs of physical abuse,
during an active hallucination. neglect is considered abuse even without violence
(4) It is not helpful to question or imply that the client and is reportable.
is not seeing real bugs. (3) CORRECT: As a mandated reporter, the nurse
20. The Answer is 3 needs to investigate to determine if there is a
The client has had a depressed mood, decreased reasonable
sleep, poor concentration, and poor appetite for the explanation: for example, the sibling just
past 4 months. Which of the following does the nurse came in from playing and the parents are on their
expect the physician to prescribe? way to buy food.
Category: Mental health concepts (4) Nurses are mandated reporters for any suspected
(1) Quetiapine is not typically given for depression child abuse whether or not it is their client.
symptoms. It is usually given for bipolar disorder. 24. The Answer is 1 and 2
(2) Haloperidol is given for symptoms of The nurse is caring for a hospice client who lives at
schizophrenia, home with an attentive spouse. The client’s spouse
not depression. quit work to care for the client. During the nurse’s
(3) CORRECT: Mirtazapine is typically prescribed visit, the spouse expresses frustration and hostility
toward the nurse. Which of the following are which could stunt the client’s social growth. It
appropriate could also defeat the purpose of a day program,
interventions by the nurse? Select all that which is to promote interaction among clients.
apply. NCLEX-RN® Exam Content Review and Practice
Category: Support systems (2) Making an effort to interact with the client
(1) CORRECT: Verbalizing feelings is an appropriate periodically
intervention for family members suffering does not lead to the client’s personal
from caregiver role strain. growth. Therefore, it is not the best option.
(2) CORRECT: Attending a support group is an (3) CORRECT: Participating in a youth group can
appropriate intervention for family members help a teenage client with a disability develop
suffering from caregiver role strain. social skills, use support systems, and feel more
(3) It may not be possible or practical for the spouse like a typical teenager.
to go back to work part time. (4) It would not be appropriate to talk about one
(4) Encouraging the spouse not to verbalize negative client with other clients, for reasons of confidentiality
feelings interferes with natural expression and privacy.
and personal family conversations. 28. The Answer is 4, 3, 1, 2
25. The Answer is 2 The nurse on the inpatient psychiatric unit is caring
The nursing is taking a history from a client in an for a client with known suicidal ideation. The
outpatient clinic. The client has been taking lorazepam 24-hour observer calls the nurse to report that the
for 6 months. Which of the following is the client took off down the hall. The nurse is unable to
MOST likely side effect that the nurse would expect immediately locate the client. Arrange the following
to see as a result of the client using Ativan for this actions by the nurse in the order that is MOST
time period? appropriate. All options must be used.
Category: Mental health concepts Category: Crisis intervention
(1) Excessive appetite is a possibility, but not the (1) Security is the third step because, although they
most likely. are not immediately on hand, they can have multiple
(2) CORRECT: Clients can experience all types of people search from different directions.
side effects from benzodiazepines, but the most (2) Notifying your nurse manager is the last step,
likely side effect from prolonged use is physical because the manager may not be readily available.
dependence. Your priority is locating client and ensuring
(3) Suicidal ideation is a possibility, but not the most the client’s safety.
likely. (3) Notifying other staff is the second step because
(4) Seizure activity is a withdrawal effect the nurse they know the client and are readily available to
would monitor for if the client discontinued search locally.
lorazepam abruptly. (4) Asking the observer which direction the client
26. The Answer is 1 headed is the first step. This enables the nurse to
A client requires a lifesaving blood transfusion per give accurate information to staff, and if necessary,
hospital guidelines. The client refuses based on security to help locate the client.
religious 29. The Answer is 3
beliefs. It would be MOST appropriate for the The nurse discovers a hospice client has expired.
nurse to take which of the following actions? The family members are regrouping in the facility’s
Category: Religious and spiritual influences on waiting room. Which of the following actions by the
health nurse would be the MOST appropriate?
(1) CORRECT: The nurse must be sure the client Category: Grief and loss
understands the potential risks of not receiving (1) It is not the nurse’s decision whether a family
the transfusion. wants to view a body or not. This is a paternalistic
(2) Clients do have the right to refuse care on religious attitude to be avoided in this setting.
grounds. (2) The nurse should react to that particular family’s
(3) Although the nurse may call the Legal Department needs or wishes, and not encourage or discourage
at some future time, this would not be the in either direction.
first course of action in this situation. (3) CORRECT: The nurse acknowledges the loss,
(4) The nurse must be sure that the client expresses sympathy, and offers the viewing
comprehends opportunity.
the choice he or she is making, including (4) This statement assumes the family wants to view
risks and benefits. However, the nurse does not the body without the nurse inquiring first.
want to coerce the client into changing his or her 30. The Answer is 1
mind. The nurse is caring for a newly admitted client in a
27. The Answer is 3 hospital setting. The client was recently diagnosed
The nurse monitors clients’ medications in a day with cancer but is alert and oriented. The client is
program a Greek immigrant, but does speak English. During
for clients with disabilities. The nurse notices the admission process, the nurse inquires about
a teenage client who is frequently alone and often advance directives with the client. The client tells
quiet. It would be MOST appropriate for the nurse the nurse: “I do not want to make any medical
to take which of the following actions? decisions.
Category: Support systems I want my daughter to make these decisions
(1) It appears that the client has enough alone time, for me.” The nurse should take which of the following
actions? • Use a warm, pleasant speaking voice; do not speak
Category: Cultural diversity loudly.
(1) CORRECT: As long as the client is not pressured • Explain procedures before starting them.
into this decision and the nurse believes that it is • Announce when you are leaving the room.
being made of the client’s free will, it is acceptable Communicating with the Client with Auditory Deficits
for the daughter to take over medical decision • Move where you can be seen by the client, or touch
making for the ill parent. the client gently so the client knows
(2) The client is entitled to have her daughter make where you are standing, before starting a
the medical decisions for the client, if that is what conversation.
the client wishes to do. • Keep background noise to a minimum.
(3) The client is entitled to allow her daughter to be • Speak in a normal voice; do not shout.
informed of the details of the client’s care. • Look at the client when speaking so he or she can
(4) The client is entitled to have her daughter make see your face/mouth for lip reading.
the medical decisions for the client, and the nurse • Mime, write, or spell words, if needed.
should not encourage her to do otherwise. • Pronounce words carefully.
201 • When changing the subject, slow down or use key
Providing basic care and comfort for your clients is one words to indicate the change.
of your most important roles. Ensuring Elimination
that your clients have adequate nutrition and Clients’ elimination needs are important to their basic
hydration, personal hygiene, and rest and care and comfort, as well to as their
sleep, and that their elimination needs are being health. You need to provide appropriate interventions
properly attended to, are important priorities. for a client who has an alteration in
Being able to help your clients with non- elimination.
pharmacological comfort interventions, mobility Urinary Issues
issues, and assistive devices are also part of providing One of the most common urinary problems is a urinary
them with basic care and comfort. tract infection (UTI).
On the NCLEX-RN® exam, approximately 9 percent of Lower Urinary Tract Issues
the questions will relate to Basic Care • Urethritis, inflammation of urethra
and Comfort. Exam content related to this • Cystitis, inflammation of the bladder
subcategory includes, but is not limited to, the • Prostatitis, inflammation of the prostate
following topics: Upper Urinary Tract Issues
• Assistive devices • Pyelonephritis, inflammation of the pelvis and
• Elimination parenchyma
• Mobility/immobility 203
• Non-pharmacological comfort interventions 8: Physiological Integrity: Basic Care and
• Nutrition and oral hydration Comfort
• Personal hygiene • Incontinence
• Rest and sleep ‚ . Stress
Let’s now review the most important concepts covered ‚ . Reflex
by these subtopics on the NCLEXRN ‚ . Urge
® exam. ‚ . Functional
Assistive Devices Other Urinary Issues
It is important to assess your clients for • Urgency
communication, speech, vision, and hearing issues, • Pain or difficulty (dysuria)
and help them learn how to compensate for deficits by • Frequency
using appropriate strengthening exercises, • Hesitancy
assistive devices, positioning, and/or other • Polyuria (large volume at one time)
compensatory techniques. You will help • Nocturia (excessive at night, interrupting sleep)
clients select and learn how to use appropriate • Hematuria (red blood cells in urine)
assistive devices, such as crutches, walkers, • Retention
canes, hearing aids, and prosthetics, and evaluate Be familiar with common urinary tests, including the
whether the client is using them correctly. bladder scan at the bedside. It is also
PHysiological Integrity: Basic important to teach clients how to maintain a healthy
Ca re and Comfort urinary tract—provide information
chapter 8 and instruction about adequate hydration (1,500–
202 2,000 mL/day), emptying the bladder completely,
NCLEX-RN® Exam Content Review and Practice the impact of caffeine and alcohol, proper personal
Being able to communicate with clients who have hygiene, and Kegel exercises. In
visual and auditory deficits is also important. addition, teach clients to recognize the signs of a UTI.
The following summarizes some techniques that you Foley catheters are used to drain urine. Catheters can
can employ. cause infection, so it is highly important
Communicating with the Client with Visual Deficits to use proper sterile techniques when inserting,
• Announce yourself and say your name when maintaining, and removing them. You
entering a client room. should also know how to perform irrigations of the
• Stay in the client’s field of vision, if possible. bladder, eyes, and ears.
Bowel Issues
Be able to recognize potential bowel issues based on • Pneumonia and pulmonary embolisms
the age and health of a client. Common • Decreased peristalsis, constipation
bowel problems include constipation (hard, dry stools • Kidney stones
that are difficult to pass), impaction (an 205
accumulated mass of stool that cannot be passed), 8: Physiological Integrity: Basic Care and
diarrhea (frequent passage of unformed/ Comfort
liquid stool), incontinence (inability to retain urine or Psychological complications can include body image
stool), flatulence, and hemorrhoids. issues, lack of social interaction, sensory
Bowel problems are diagnosed by abdominal x-ray, deprivation, and depression.
upper gastrointestinal (GI) barium test, Interventions should be implemented to counteract
barium enema, and upper (oral) and lower (rectal) physiological and psychological complications.
endoscopy. Active and passive range of motion exercises,
Treatments include the following: positioning, and mobilization can be
• Constipation: Increase fluid intake, including hot used to promote circulation. Turning, repositioning,
liquids and fruit juices; advise a highfiber and pressure-relieving support surfaces
diet. can be used to maintain skin integrity and prevent
• Diarrhea: Understand and treat underlying cause skin breakdown. Anti-embolic stockings
(which may be a virus, reaction to and sequential compression devices can be used to
certain foods or medications, GI tract infection, etc.); promote venous return.
typically, advise bland foods and It is also important to know when orthopedic and
204 assistive devices, such as crutches, walkers,
NCLEX-RN® Exam Content Review and Practice canes, splints, traction, braces, or casts, are needed;
a low-fiber diet, as well as to avoid spicy foods, you should be able to teach the client how
alcohol, and caffeine while symptoms to use them properly to maintain correct body
continue. alignment.
• Flatulence: Limit gum, carbonated beverages, Nonpharmacological Comfort Interventions
cabbage, cauliflower, beans, and onions. It is important to be able to apply your knowledge of
Care for ostomies (a surgically created opening in the client pathophysiology to nonpharmacological
abdominal wall through which feces can interventions. Assess the client’s need for pain
pass) is also an important part of basic care and management and implement comfort
comfort. Types and locations are as follows: measures, as needed.
• Ileostomy: An opening into the distal end of the Therapies for comfort and treatment of
small intestine inflammation/swelling can include heat, cold, or
• Colostomy: An opening into the colon elevation of limbs. Use the pain scale and verbal
Ostomy care includes regularly assessing the reports to assess the effectiveness of the
condition of the stoma (the opening), making intervention.
sure the skin around the stoma is clean and dry, and Palliative Care
teaching the client how to care for the Nurses have an important role to play in palliative
ostomy, including proper diet, fluid intake, and care, particularly in relation to pain and
hygiene, and how to remove a food blockage. symptom management and the coordination of care.
It is also important to use proper skin care for clients Assess a client’s need for palliative care
who are incontinent, including the use and provide counseling, as needed. Call in specialists
of barrier creams and ointments. You should also be from other disciplines, including doctors,
able to evaluate whether client elimination psychologists, social workers, and clergy, as
is restored to normal and whether it’s maintained. appropriate.
Mobility/Immobility You should be able determine whether interventions
It is a nurse’s responsibility to assess a client’s are working and whether they are meeting
mobility, gait, strength, motor skills, and use the client’s goals. The client’s care may include pain
of assistive devices. You should be able to identify management to improve comfort and
common causes of immobility, and the quality of life, but may exclude painful treatments or
complications associated with each. The main causes heroic interventions.
are: You must respect a client’s palliative care choices, and
• Pain review those choices with the client
• Motor/nervous system impairment periodically because they may change during the
• Functional problems course of a client’s disease. Assisting a client
• Generalized weakness in receiving appropriate end-of-life symptom
• Psychological problems management, particularly as the client enters
• Side effect of medication the active dying phase, is also important.
Complications of immobility can be physiological Nutrition and Oral Hydration
and/or psychological in nature. Physical It is important to know the principles of nutrition, such
complications can include: as the basic food groups, their functions,
• Atrophy, joint contracture and which foods fall within those groups. These
• Disuse osteoporosis include:
• Pressure ulcers 206
• Orthostatic hypotension NCLEX-RN® Exam Content Review and Practice
• Deep vein thrombosis
• Carbohydrates: Are converted to glucose, which the • Clogging
body uses for energy. Sources of • Aspiration
carbohydrates include grain products (bread, pasta, Monitor the client’s underlying condition to ensure the
and rice), fruits, milk, and products right dietary/feeding choices are
with high sugar content. made. Factors you must monitor include weight,
• Proteins: Are used to build and repair body tissue, protein measures, TLC, blood urea nitrogen
such as muscles, and also for many (BUN), and creatinine levels, making adjustments as
essential body processes, such as nutrient transport needed.
and muscle contraction. Sources of Personal Hygiene
protein include meat, poultry, fish, eggs, nuts, beans, It is important to assess your clients’ personal hygiene
peas, and lentils. and assist them in performance of both
• Fats: Are used to insulate the body, provide energy, activities of daily living (ADLs) and instrumental
and store certain vitamins such as activities of daily living (IADLs). Provide
A, D, E, and K, which are soluble in fats and insoluble information on adaptations, such as shower chairs and
in water. Sources of fat include hand rails.
whole milk and milk products, oils, nuts, and certain Personal hygiene topics to know include care of skin,
meats. eyes, ears, nose, mouth, feet, nails, hair
Be familiar with general dietary guidelines, key and scalp, perineal area, and prostheses. Care of the
nutritional concepts across a client’s life skin is particularly important. Know
span, and types of diets appropriate for specific the measures to keep skin clean and moist, and how
conditions, for example, which foods would to prevent pressure points. Keeping skin
be appropriate for a client with heart disease (foods clean can help prevent skin breakdowns and
with low fat and low cholesterol) or inappropriate infections.
(foods with high fat and high cholesterol). You should You should also know how to perform post-mortem
also be able to apply your care. After the patient is pronounced dead,
knowledge of mathematics to nutrition (e.g., body nurses prepare the body for viewing by the family and
mass index [BMI] calculations). transport to the morgue or funeral
You can use the following to assess a client’s ability to home. Family members should be given the option of
eat: seeing their loved one before or after
• Documented history post-mortem care is provided, or not at all, if that is
‚ . From patient their choice.
‚ . Nutritional screening initiatives (NSIs) Rest and Sleep
• Anthropomorphic measures It is important to know the physiology of sleep, the
‚ . Height, weight, and body size phases, normal sleep patterns, and how
‚ . BMI sleep differs at each developmental stage. Your
‚ . Basal metabolic rate (BMR) knowledge of each client’s pathophysiology
‚ . Distribution of body fat (obesity) will help you to provide the appropriate interventions,
• Lab/diagnostic measures which could include the following:
‚ . Albumin levels • Keeping the environment conducive to quiet
‚ . Total lymphocyte count (TLC) relaxation
‚ . Hemoglobin levels • Promoting bedtime routines
• Ability to chew and swallow • Promoting comfort
Assess clients for specific food/medication • Avoiding heavy meals before bedtime
interactions, and consider client choices regarding • Promoting appropriate activity
nutritional requirements and dietary restrictions. Also • Providing pharmaceutical aids (sedatives or
monitor client hydration status. hypnotics) as needed
For example, be familiar with the signs and symptoms NCLEX-RN ® Exam Content Review and Practice
of both edema (excess fluid) and 208
dehydration. NCLE X-1. The nurse is assessing an irritable 6-
For clients unable to eat on their own, nutrition can be monthold
provided through continuous or intermittent infant during a well-baby checkup.
tube feedings. This includes nasogastric, enterostomy The infant’s weight is 19 lb., 6.4 oz. (8.8
(surgical), or percutaneous kg). The infant does not have an elevated
tubes. You should know how to maintain the tube temperature, the heart rate is 102, and the
insertion site, monitor it for infection and respiratory rate is 32. The mother states
proper function, as well as ensure that the proper that the infant wakes every hour or two
volume of formula is getting through. You throughout the night. The infant wants a
207 bottle, and falls asleep while eating, but
8: Physiological Integrity: Basic Care and doesn’t stay asleep. Which of the following
Comfort instructions should the nurse give the
should also recognize mechanical or metabolic parents?
problems and intervene, as needed. These 1. Instruct the parents to offer
include: acetaminophen 325 mg orally for
• Formula selection comfort, and diphenhydramine 25 mg
• Formula adjustment orally for sleep.
• Skin irritation 2. Instruct the parents to offer high-calorie
solid foods during daytime hours so the 5. The nurse has been assigned to an adult
infant does not wake up hungry during male client who is less than 24 hours post-op.
the night. In report, the nurse learns that he rings his
3. Instruct the parents to offer the last call light frequently, is anxious, and has had
feeding as late as possible, and put the pain medication as ordered. Which of the
infant to bed awake without a bottle. following nondrug nursing interventions
4. Suggest using pacifiers, taking the infant should the nurse include when caring for this
to the parent’s bed, or rocking the infant client?
to sleep. 1. Assure the client his anxiety is
2. The nurse caring for a child burned over understandable, because the pain
20% of her body assists the physician in medication needs time to take effect.
performing dressing changes on day 5 2. Assess other clients first, giving this client
after the initial injury. The child appears time to relax before evaluating his level of
disoriented, has a fever of 101º F (38.3º C), pain.
and is crying in pain. Which of the following 3. Call the client’s physician to increase the
nursing interventions would be the MOST amount or frequency of pain medications
appropriate in caring for this client? ordered.
1. Gather equipment for the dressing change 4. Provide a quiet environment, offer
and explain the procedure to the child. repositioning, straighten the bed linens,
2. Do a complete physical assessment and offer fluids, and assess his pain level.
notify the physician of the findings. 6. The nurse is taking care of an adult male
3. Administer appropriate analgesics and with bilateral leg fractures. He has a long
gather equipment for the dressing change. leg cast on his right leg as well as traction
4. Offer the child an enticing distraction applied to the left femur. Which of the
from pain, such as a video, music, or toy. following is the MAIN purpose served by the
3. The nurse is taking care of a young child a cast for this client?
few hours after a tonsillectomy. Which of 1. Immobilizes the tibia and fibula and
the following nursing interventions would be corrects deformities
appropriate to promote adequate nutrition 2. Keeps the client, who is in traction, more
and oral hydration for this child? comfortable
1. Offer the child warm soup, watch for 3. Immobilizes the pelvic bones for better
signs of bleeding, and suction vigorously healing
to remove old blood. 4. Encircles the trunk and stabilizes the
2. Offer ice chips after the child awakens; spine
advance to cool, clear liquids; and suction 7. The nurse is taking care of an elderly male
gently to remove oral secretions without client who has shortness of breath, cough,
causing the child to cough or gag. and fluid in his pleural space. The physician
3. Maintain the intravenous fluids asks the nurse to assist in the performance of
appropriate for the child’s weight for the a therapeutic and diagnostic thoracentesis.
next 24 hours and keep the child NPO. Which of the following nursing interventions
4. Offer soft, warm foods so the child will should the nurse perform to assist this client?
not be hungry; orange juice to provide 1. Make certain the consents are signed,
vitamin C; and milk shakes for calories. witnessed, and filed in the chart.
4. The nurse is caring for a child who had 2. Offer oral fluids, because the client will
an adenoidectomy and tonsillectomy 10 not be able to take a drink during the
hours ago. The parents are in the room and procedure.
preparing the child for bedtime. Which of 3. Help the client to lie flat with a pillow
the following nursing interventions would under his feet for comfort during the
be helpful to promote rest and sleep for this procedure.
client? 4. Help the client to sit up and place his
1. Provide a cool water rinse, adjust the arms over a bedside table, encouraging
head of the bed to a 30–45-degree angle, him to remain still during the procedure.
and offer an ice collar for comfort. 8. The nurse has been assigned to a 2-dayold
2. Encourage the parents to leave so the male infant on the mother/baby unit
child can sleep. of an acute care facility. The infant will
3. Suction vigorously before the child falls undergo a circumcision procedure in the
asleep to ensure the child has a patent afternoon, before being discharged the
airway. following morning. Which of the following
Chapter Quiz non-pharmacologic interventions should the
8: Physiological Integrity: Basic Care and nurse teach the parents to keep this infant
Comfort comfortable while the circumcision heals?
209 1. Fasten his diaper tightly to avoid having
4. Provide a water rinse, offer an ice collar it move around the wound.
for discomfort, and assist the child in 2. Apply petroleum jelly to gauze and place
finding a position of comfort while over the end of the penis when changing
promoting a patent airway for sleep. the diaper, leaving the diaper slightly
loose when fastening. injury. He will be required to be nonweight
3. Offer feedings more often to soothe the bearing for 4–6 weeks. Which of
child who is in pain. the following crutch gaits should the nurse
4. Wash the end of the penis vigorously to teach this client for safe ambulation?
prevent infection. 1. The two-point gait
NCLEX-RN® Exam Content Review and Practice 2. The three-point gait
210 3. The four-point gait
9. The nurse is taking care of a quadriplegic 4. None, there is no special gait for crutch
young man who suffers from a C2-C3 training
fracture after an auto accident 3 months 13. The nurse is working in an extended
prior. He has a tracheotomy, is ventilatordependent, care facility when a nursing assistive
and has been discharged to home personnel (NAP) reports that an elderly
with skilled home nursing care. The nurse client is crying in pain. The nurse finds
knows that this client is at risk for autonomic the client in the bathroom complaining of
dysreflexia. Which of the following measures severe constipation. What would be the
should this nurse take to keep the client appropriate order of nursing interventions
comfortable, manage his elimination needs, to assist this client with his immediate
and prevent common causes of autonomic elimination needs? All options must be used.
dysreflexia? 1. Offer oral fluids to ease the constipation.
1. Turn the client at least every two hours 2. Notify the physician.
and look for skin breakdown. 3. Offer PRN medications orally, if
2. Allow the client to sleep 8–10 hours ordered.
without interruption each night to 8: Physiological Integrity: Basic Care and
promote rest. Comfort
3. Offer appetizing fluids at least every 211
two hours during the day to promote 4. Use a gloved hand with lubricant to
hydration. manually assess for fecal impaction and
4. Straight catheterize the client to prevent to stimulate the rectal wall to loosen the
bladder distention and maintain a regular fecal matter.
bowel program to prevent impaction. 14. The nurse is caring for a young child who
10. The nurse is taking care of a child after an has recently had a vesicostomy. Which of
open reduction of the radius and ulna of the following nursing interventions should
her right arm. The child is now immobilized the nurse undertake to assist this child with
in a plaster cast splint reinforced with an basic comfort and elimination?
Ace wrap. Which of the following nonpharmacological 1. Offer fluids, apply an absorbent diaper
nursing interventions will or incontinence pads, and dilate the
promote comfort for this child? opening once or twice a day as ordered
1. Apply a heat pack to the approximate by the physician.
area of the surgical incision. 2. Double-diapering the area is the only
2. Position the child so the cast is flat on intervention needed.
the mattress for firm support. 3. Apply a urine bag and change it daily.
3. Elevate the cast on a pillow, apply an 4. Double-diaper the area after applying a
ice pack to the approximate area of the urine bag.
surgical incision, and reposition the 15. A client who has chronic pain asks
child every two hours. the nurse about alternative therapy in
4. Do not move any part of the child’s arm conjunction with traditional treatment.
until the physician orders a specific Which of the following forms of alternative
position. therapy could the nurse provide for this
11. The nurse is taking care of an elderly client client?
with left-sided heart failure. Which of 1. Music therapy or guided imagery
the following are the MOST appropriate 2. Acupuncture
nursing interventions to reduce the 3. Kegel exercises
workload of the heart and to promote 4. None, nurses do not participate in
comfort and rest? Select all that apply. providing alternative treatments
1. Assist the client on short walks at 16. The nurse is taking care of an adult client
least two times per shift to increase with a fractured femur who must be
circulation. maintained in traction for several days
2. Provide a comfortable armchair or before surgical interventions can take place.
raise the head of the bed to increase the The client has several abrasions, his hair
reserve of the heart and to decrease the is dirty, and he has healing wounds in his
work of breathing. mouth. Which of the following nursing
3. Allow the client to lie flat to sleep. interventions should the nurse use in caring
4. Help the client walk to the bathroom for the personal hygiene of this client?
rather than using a bedside commode. 1. Place everything within the reach of the
12. The nurse is instructing a male client on client so he can bathe himself.
the proper use of crutches for an ankle 2. Assist with a bed bath, with teeth
brushing, and by washing his hair with about nutrition and hydration. Which of
soap and water or a non-shampoo the following suggestions might the nurse
product for bed-bound clients. include when providing education to this
3. Allow a family member to bathe the client?
client. 1. Drink clear water, progress diet rapidly
4. Offer an oral rinse for hygiene, but as tolerated, and weigh daily.
postpone the bath until a later time due 2. Puree foods, choose low-protein foods
to the traction. for easier digestion, and weigh weekly.
17. The nurse is taking care of an adult client 3. Take herbal therapies, avoid vitamins,
with a long-bone fracture. The nurse and don’t monitor weight.
encourages the client to move fingers and 4. Use spices to stimulate taste buds, eat
toes hourly, to change positions slightly cool foods to decrease odor, and eat
every hour, and to eat high-iron foods small but frequent high-protein and
as part of a balanced diet. Which of the high-carbohydrate meals.
following foods or beverages should the 22. The nurse is caring for an elderly client
nurse advise the client to avoid while on bed who has been on long-term nutritional
rest? support. The nurse is reviewing the infusion
1. Fruit juices procedure with the client’s daughter. The
2. Large amounts of milk or milk products nurse states which of the following as the
3. Cranberry juice cocktail rationale for removing the formula from
4. No need to avoid any foods while on bed the refrigerator and infusing it through the
rest gastrostomy tube at room temperature?
18. The nurse working in an outpatient clinic 1. “The formula tastes better at room
has the opportunity to teach an insulindependent temperature.”
client. Which of the following 2. “This method will be the least likely to
topics would be MOST appropriate for the give your father gastric discomfort.”
nurse to include when teaching personal 3. “There is no need to bring the formula
hygiene? to room temperature.”
1. Oral care is not a top priority. 4. “Room-temperature prepared formula
2. Hair care is the most important part of reduces aspiration.”
personal hygiene for the diabetic client. 23. The nurse is working with a middle-aged
3. It is most important to keep skin clean female after a knee injury. Ambulation is
and dry, especially the feet. still difficult for the client, and the physical
4. Personal hygiene is not included in therapist has suggested the client use a cane.
diabetic teaching because it is an The nurse states which of the following with
individual choice. respect to using a cane rather than a walker
NCLEX-RN® Exam Content Review and Practice for this injury?
212 1. “The cane is just a reminder to use good
19. The nurse is taking care of a child in posture.”
the ambulatory care clinic. The parents 2. “The cane can be more dangerous than
relate a 24-hour period of gastrointestinal helpful, and another type of assistive
complaints, including vomiting several device should be considered for this
times and 3 watery stools. Which of the client.”
following should the nurse do to assist in 3. “The cane will help with fatigue while
maintaining nutrition for this child? assisting the client with balance and
1. Educate the parents on the signs of support.”
dehydration and the slow introduction of 4. “A cane does not offer any relief on
fluids to rehydrate the child. weight-bearing joints.”
2. Offer no advice to the parents other than 8: Physiological Integrity: Basic Care and
to suggest parents offer whatever foods Comfort
the child feels like taking. 213
3. Encourage the parents to offer the child 24. The nurse is preparing for a pediatric trauma
milk products for the vitamins and admission in which traction will be applied to
rehydration. immobilize a femur fracture for a child. The nurse
4. Encourage the parents to offer solid reviews the forms of traction and the purposes
foods to improve the nutritional status for each before gathering equipment prior to the
quickly. child’s arrival. Match the type of traction on the
20. An 11-lb. (5-kg) infant is NPO after a minor left with the type of injury or indication on the right.
surgical procedure. What would be the All options must be used.
appropriate rate of infusion of intravenous 1. Bryant’s traction A. Stabilizes a spinal fracture or
fluids if the physician ordered fluids to run muscle spasm
at 15 mL/kg/day? Record your answer using 2. Russell’s traction B. Used on the femur if skin
one decimal place. traction isn’t
mL/hr suitable
21. An adult diagnosed with pancreatic cancer 3. 90-degree traction C. Temporarily immobilizes a
is having a consultation with the nurse fractured leg
4. Buck’s traction D. May reduce fractures of the hip (4) Distractions may be offered after the assessment
or femur but they do not take priority over notifying
5. Cervical traction E. Used in children younger than the physician regarding the findings about
age 2 to the source of fever and pain.
reduce femur fractures or stabilize hips 3. The Answer is 2
25. It is important to evaluate pain in the neonate. The nurse is taking care of a young child a few hours
Look at the chart below. What would the pain after a tonsillectomy. Which of the following nursing
score be for an infant with a high-pitched cry, O2 interventions would be appropriate to promote
saturation of 96%, a grimace, and frequent adequate nutrition and oral hydration for this child?
periods of wakefulness? Category: Nutrition and oral hydration
Score (1) Warm liquids may increase bleeding and should
012 be avoided the first few hours after surgery.
Crying No High-pitched Inconsolable (2) CORRECT: The child may first take ice chips
Requires O2 No < 30% > 30% 1–2 hours after awakening, followed by cool,
Expression None Grimace Grimace/grunt clear liquids without pulp or ice pops. Gentle
Sleepless No Wakes frequently Always awake suctioning may be necessary to remove secretions
1. Score of 0 in the mouth and to keep the child from
2. Score of 2 gagging. Suctioning should be kept to a minimum
3. Score of 3 to avoid traumatizing the oropharynx.
4. Not enough information (3) The physician may maintain an intravenous
NCLEX-RN ® Exam Content Review and Practice infusion postoperatively, but it is not necessary
214 to keep the child NPO after the surgery. Ice chips
NCLE X-1. The Answer is 3 or cool, clear liquids are soothing.
The nurse is assessing an irritable 6-month-old (4) Soft foods are not given in the first few hours
infant during a well-baby checkup. The infant’s after surgery to prevent emesis. Orange juice
weight is 19 lb., 6.4 oz. (8.8 kg). The infant does not is acidic, and juices should be alkaline when
have an elevated temperature, the heart rate is 102, offered to a postoperative child. Milk products
and the respiratory rate is 32. The mother states that are controversial because they coat the throat
the infant wakes every hour or two throughout the and may cause the child to cough.
night. The infant wants a bottle and falls asleep while 4. The Answer is 4
eating, but doesn’t stay asleep. Which of the following The nurse is caring for a child who had an
instructions should the nurse give the parents? adenoidectomy
Category: Rest and sleep and tonsillectomy 10 hours ago. The parents
(1) Tylenol may be appropriate for teething pain, are in the room and preparing the child for bedtime.
and Benadryl is an antihistamine that may cause Chapter Quiz Answers and Explanations
drowsiness, but the doses as given are for adults. 8: Physiological Integrity: Basic Care and
(2) The infant’s weight is within normal limits, so Comfort
high-calorie foods may not be appropriate. 215
(3) CORRECT: The infant is having sleep disturbances Which of the following nursing interventions would
related to nighttime feeding. Feeding be helpful to promote rest and sleep for this client?
late and putting the infant to bed awake help the Category: Rest and sleep
infant learn to recognize bedtime and to selfsoothe (1) Semi-Fowler’s may not be a position comfortable
to fall asleep. for some children, so other positions may need to
(4) The Academy of Pediatrics does not promote be considered.
putting infants to bed with parents. Rocking the (2) The parents should be encouraged to stay with
infant will not help learning to self-soothe. the child and to participate in the care and comfort
2. The Answer is 2 of the child, if possible.
The nurse caring for a child burned over 20% of her (3) Suctioning should not be vigorous after an
body assists the physician in performing dressing adenoidectomy
changes on day 5 after the initial injury. The child or a tonsillectomy.
appears disoriented, has a fever of 101º F (38.3º C), (4) CORRECT: Assist the child in finding a position
and is crying in pain. Which of the following nursing of comfort. This may be prone, semi-prone, or
interventions would be the MOST appropriate in semi-Fowler’s. An ice collar and a cool oral rinse
caring for this client? will also aid in comfort.
Category: Non-pharmacological comfort interventions 5. The Answer is 4
(1) The nurse would gather equipment, but not The nurse has been assigned to an adult male client
before addressing the crying child. who is less than 24 hours post-op. In report, the
(2) CORRECT: The child may be suffering from an nurse learns that he rings his call light frequently, is
infection. The nurse recognizes that disorientation anxious, and has had pain medication as ordered.
and fever are the first signs of sepsis in burn Which of the following nondrug nursing interventions
clients. It would be most appropriate to assess should the nurse include when caring for this
for the causes of fever and pain and notify the client?
physician before proceeding. Category: Non-pharmacological comfort interventions
(3) Analgesics may be appropriate but not before (1) The client will probably be more reassured if
assessing the pain and source of fever and physical comfort measures are taken, rather than
disorientation. just verbal assurances.
(2) Prioritizing is necessary, but avoiding an already cause emesis and is not the best way to soothe an
anxious client may cause the nurse to overlook a infant.
serious symptom. (4) The end of the penis has a yellow exudate that is
(3) Call a physician, if needed, AFTER offering part of the healing process and should not be
basic comfort measures and doing an assessment. vigorously
(4) CORRECT: Changing the client’s position, washed off. It will disappear with healing.
removing wrinkles in the bed linen, helping the 9. The Answer is 4
client to take a drink, or limiting noise can help The nurse is taking care of a quadriplegic young
the client to rest and may reduce pain. man who suffers from a C2-C3 fracture after an auto
6. The Answer is 1 accident 3 months prior. He has a tracheotomy, is
The nurse is taking care of an adult male with bilateral ventilator-dependent, and has been discharged to
leg fractures. He has a long leg cast on his right home with skilled home nursing care. The nurse
leg as well as traction applied to the left femur. knows that this client is at risk for autonomic
Which of the following is the MAIN purpose served dysreflexia.
by the cast for this client? Which of the following measures should this
Category: Mobility/immobility nurse take to keep the client comfortable, manage
(1) CORRECT: A long leg cast serves to immobilize his elimination needs, and prevent common causes
the tibia and fibula by being placed above and of autonomic dysreflexia?
below the knee and ankle joints. Category: Elimination
(2) A long leg cast is not used for comfort for a client (1) Turning is necessary to prevent decubitus ulcers
in traction. and promote comfort, but it does not necessarily
(3) A long leg cast does not immobilize the pelvis. prevent an increase in blood pressure as seen
(4) A body cast, not a long leg cast, encircles the with autonomic dysreflexia.
trunk. (2) Sleeping 8–10 hours is not related to autonomic
7. The Answer is 4 dysreflexia.
The nurse is taking care of an elderly male client who (3) Offering fluids is a nursing measure but may not
has shortness of breath, cough, and fluid in his pleural be related to autonomic dysreflexia because a
space. The physician asks the nurse to assist in client with a spinal cord injury may have a fluid
the performance of a therapeutic and diagnostic restriction to help control blood pressure.
thoracentesis. (4) CORRECT: Bladder distension and bowel
Which of the following nursing interventions impaction can result in autonomic dysreflexia,
should the nurse perform to assist this client? causing a critical increase in blood pressure.
Category: Non-pharmacological comfort interventions 10. The Answer is 3
(1) The nurse should make certain that consents are The nurse is taking care of a child after an open
signed before the start of a procedure, but that reduction of the radius and ulna of her right arm.
does not affect the client’s comfort. The child is now immobilized in a plaster cast splint
(2) Fluids should not be offered right before a reinforced with an Ace wrap. Which of the following
procedure non-pharmacological nursing interventions will
to avoid nausea and vomiting if pain is promote comfort for this child?
experienced. Category: Non-pharmacological comfort
(3) Lying flat with feet elevated is not the position of interventions;
choice for a thoracentesis. Mobility/immobility
(4) CORRECT: Placing the client in a sitting position (1) Heat would not be appropriate, because it could
over a bedside table is the most comfortable cause, rather than reduce, swelling.
and allows the best opportunity to remove fluid (2) The cast should be elevated for the first 24–48
at the base of the chest. hours and not be left flat on the mattress.
8. The Answer is 2 (3) CORRECT: Elevating the extremity and applying
The nurse has been assigned to a 2-day-old male an ice pack will help to reduce swelling and
infant on the mother/baby unit of an acute care may reduce pain. Repositioning is a comfort
facility. The infant will undergo a circumcision intervention.
procedure (4) The child should not be totally immobile because
in the afternoon, before being discharged it can lead to post-op respiratory complications.
the following morning. Which of the following 11. The Answer is 1 and 2
nonpharmacologic The nurse is taking care of an elderly client with
interventions should the nurse teach leftsided
the parents to keep this infant comfortable while the heart failure. Which of the following are the
circumcision heals? MOST appropriate nursing interventions to reduce
NCLEX-RN® Exam Content Review and Practice the workload of the heart and to promote comfort
216 and rest? Select all that apply.
Category: Non-pharmacological comfort interventions Category: Rest and sleep
(1) Leaving the diaper slightly loose when fastening (1) CORRECT: Taking short walks may provide
will be more comfortable. distraction and increase mobility, circulation,
(2) CORRECT: Petroleum jelly offers lubrication and overall well-being if tolerated.
and helps stop the friction of the diaper over the (2) CORRECT: Allowing the client to sit in an armchair
raw area. makes it easier to breathe and is a safe
(3) Offering feedings more often than necessary may alternative to an armless chair. It is also helpful
to have the client raise the head of the bed when chronic neurogenic bladder and frequent urinary
sleeping or napping. These are appropriate for a tract infections become problematic. Hydration,
client with left-sided heart failure. cleansing and drying of the area, absorbent diapers,
(3) A client in left-sided heart failure most likely will and daily dilation of the opening are all
not tolerate lying flat, so this would not promote appropriate care to prevent infection and to provide
sleep and rest in this position. comfort.
(4) A bedside commode would reduce the work of (2) Double diapers alone are not enough to keep the
getting to the bathroom and should be used. child comfortable and free from infection.
12. The Answer is 2 (3) It is not customary to apply a urine bag over the
The nurse is instructing a male client on the proper opening of a vesicostomy.
use of crutches for an ankle injury. He will be (4) The addition of a urine bag to double diapers
required to be non-weight bearing for 4–6 weeks. will not keep the child comfortable and free from
Which of the following crutch gaits should the nurse infection.
teach this client for safe ambulation? 15. The Answer is 1
8: Physiological Integrity: Basic Care and A client who has chronic pain asks the nurse about
Comfort alternative therapy in conjunction with traditional
217 treatment. Which of the following forms of alternative
Category: Assistive devices therapy could the nurse provide for this client?
(1) The two-point gait is an advanced four-point gait Category: Alternative therapy; Non-pharmacological
and allows for faster ambulation with minimal comfort interventions
support. (1) CORRECT: Music therapy and guided imagery
(2) CORRECT: The three-point gait is the safest to have been proven to increase a client’s ability to
use when one leg is injured. Both crutches and perform activities of daily living by helping to
the injured leg move forward, followed by swinging focus on something other than pain.
the stronger lower extremity as the rest of the (2) Acupuncture must be performed by a skilled
body weight is placed on the crutches. practitioner and is not done by a nurse.
(3) The four-point gait is used as a slow and stable (3) Kegel exercises are done independently by the
gait for those who can bear weight on each leg. client to tighten the muscles of the pelvic floor.
(4) Gait training is part of client education when They do not provide pain relief.
crutches or adaptive equipment is used for (4) Nurses may participate in many forms of
ambulation. alternative
13. The Answer is 4, 3, 2, 1 therapies as nursing interventions when
The nurse is working in an extended care facility trained properly.
when a nursing assistive personnel (NAP) reports NCLEX-RN® Exam Content Review and Practice
that an elderly client is crying in pain. The nurse 218
finds the client in the bathroom complaining of 16. The Answer is 2
severe constipation. What would be the appropriate The nurse is taking care of an adult client with a
order of nursing interventions to assist this client fractured femur who must be maintained in traction
with his immediate elimination needs? All options for several days before surgical interventions can
must be used. take place. The client has several abrasions, his hair
Category: Elimination is dirty, and he has healing wounds in his mouth.
(1) This is last in the appropriate order of nursing Which of the following nursing interventions should
interventions. Oral fluids should be increased the nurse use in caring for the personal hygiene of
but will not impact the immediate pain and this client?
constipation. Category: Personal hygiene
(2) Relief of the immediate pain is the priority. After (1) The client may be able to do some of his bath,
an attempt to manually remove the impaction, but it would not be possible for him to cleanse
and offering a PRN medication, the physician his own back and other areas while maintaining
should be notified. traction.
(3) PRN medications do not offer immediate relief (2) CORRECT: Assisting with the bath allows
and may not be effective if the impaction is solid. inspection of the skin for any pressure areas;
After a manual exam assessment, and an attempt gentle teeth brushing and hair cleansing are
to remove the stool, it would be appropriate to nursing measures and promote comfort while
offer a PRN medication orally, if ordered, to prevent maintaining the traction.
a repeat incident. (3) A family member should not be responsible for
(4) The first nursing intervention should be manual inspecting the skin and maintaining the traction.
assessment and removal of the fecal impaction. These are nursing responsibilities.
This will offer immediate relief while helping to (4) Oral care is important but the bath should not be
assess what needs to be relayed to the physician. postponed and can easily be done with the client
14. The Answer is 1 in traction. It will promote comfort and healing.
The nurse is caring for a young child who has recently 17. The Answer is 2
had a vesicostomy. Which of the following nursing The nurse is taking care of an adult client with a
interventions should the nurse undertake to assist long-bone fracture. The nurse encourages the client
this child with basic comfort and elimination? to move fingers and toes hourly, to change positions
Category: Elimination slightly every hour, and to eat high-iron foods as part
(1) CORRECT: A vesicostomy is performed when of a balanced diet. Which of the following foods or
beverages should the nurse advise the client to avoid Category: Nutrition and oral hydration
while on bed rest? Multiply 5 kg by 15 mL/kg. This equals 75 mL. Then
Category: Nutrition and oral hydration; Mobility/ divide 75 mL by 24 hours in a day to arrive at the
immobility answer: 3.1 mL/hr. The nurse would run the IV at 3.1
(1) Fruit juices can be taken while on bed rest. mL over 24 hours to get the ordered amount of fluid.
(2) CORRECT: Too much milk increases the 21. The Answer is 4
demand on the kidneys to excrete calcium and An adult diagnosed with pancreatic cancer is having
can lead to kidney stones. a consultation with the nurse about nutrition and
(3) Cranberry juice can be taken while on bed rest hydration. Which of the following suggestions might
and also aids in prevention of urinary tract infections. the nurse include when providing education to this
(4) Some foods should be avoided or limited while client?
on bed rest. For instance, milk and milk products Category: Nutrition and oral hydration
should be avoided or limited while on bed (1) It is more appropriate to progress the diet slowly
rest to avoid kidney stone formation. to avoid nausea and vomiting.
18. The Answer is 3 (2) Pureed foods may cause nausea and gagging,
The nurse working in an outpatient clinic has the low-protein foods do not offer enough nutrients,
opportunity to teach an insulin-dependent client. and daily weights are the norm.
Which of the following topics would be MOST (3) Herbal therapies have not been researched
appropriate for the nurse to include when teaching enough to be certain that they would not interfere
personal hygiene? or compromise cancer treatments when
Category: Personal hygiene ingested. Topical herbal treatments may be of
(1) Oral care is an important part of diabetic hygiene use for comfort.
to prevent cavities and infections. (4) CORRECT: Flavored foods high in both protein
(2) Hair care is not the most important part of and carbohydrates will help to increase calorie
personal intake. Foods that have less odor, and small, frequent
hygiene, although it is important for selfesteem. meals help ward off nausea.
(3) CORRECT: Skin care is essential to prevent 22. The Answer is 2
infection and skin breakdown. This is especially The nurse is caring for an elderly client who has
true for the feet, where a client may not see or feel been on long-term nutritional support. The nurse
problem areas. is reviewing the infusion procedure with the client’s
(4) Personal hygiene is definitely a part of self-care daughter. The nurse states which of the following
teaching for an insulin-dependent client. as the rationale for removing the formula from the
19. The Answer is 1 refrigerator and infusing it through the gastrostomy
The nurse is taking care of a child in the ambulatory tube at room temperature?
care clinic. The parents relate a 24-hour period Category: Nutrition and oral hydration
of gastrointestinal complaints, including vomiting (1) There would not be a taste to formula given
several times and 3 watery stools. Which of the through the G-tube.
following (2) CORRECT: Cold formula through the G-tube
should the nurse do to assist in maintaining can cause discomfort and cramping.
nutrition for this child? (3) It is most appropriate for the comfort of the client
Category: Nutrition and oral hydration to bring the formula to room temperature
(1) CORRECT: Signs of dehydration would be part before administering.
of parental teaching, and a slow introduction of (4) Temperature has nothing to do with the risk of
clear liquids advancing to other liquids is appropriate. aspiration.
(2) It would not be appropriate for the nurse to 23. The Answer is 3
suggest The nurse is working with a middle-aged female
that the parents offer whatever foods the after a knee injury. Ambulation is still difficult for
child feels like taking, without first educating the client, and the physical therapist has suggested
the parents about the signs of dehydration. the client use a cane. The nurse states which of the
(3) Milk products would not be the first type of fluids following as the rationale for using a cane rather
offered for a child who has been vomiting, than a walker for this injury?
due to how irritating milk can be on the digestive Category: Assistive devices
system. (1) A cane is not used as a reminder for good posture;
8: Physiological Integrity: Basic Care and it is used for comfort and support.
Comfort (2) A cane is safe when used properly.
219 (3) CORRECT: A cane offers support and can give
(4) Solid foods are introduced later, after liquids are the client relief of joint pain and fatigue, and
offered over several hours, once vomiting has promote a safe way to ambulate when a lower
stopped. extremity is injured.
20. The Answer is 3.1 mL/hour (4) A cane does offer relief on weight-bearing joints
An 11-lb. (5-kg) infant is NPO after a minor surgical when used properly.
procedure. What would be the appropriate rate NCLEX-RN® Exam Content Review and Practice
of infusion of intravenous fluids if the physician 24. The Answer is 1 (E), 2 (D), 3 (B), 4 (C), 5 (A)
ordered fluids to run at 15 mL/kg/day? Record your The nurse is preparing for a pediatric trauma
answer using one decimal place. admission
mL/hr in which traction will be applied to immobilize
a femur fracture for a child. The nurse reviews the • Central venous access devices
forms of traction and the purposes for each before • Dosage calculation
gathering equipment prior to the child’s arrival. • Expected actions/outcomes
Match the type of traction on the left with the type • Medication administration
of injury or indication on the right. All options must • Parenteral/intravenous therapies
be used. • Pharmacological pain management
Category: Mobility/immobility • Total parenteral nutrition
(1) (E): Bryant’s traction is used in children younger Now let’s review some of the most important concepts
than age 2 to reduce femur fractures or stabilize related to these subtopics.
hips. Adverse Effects/Contraindications/Side
(2) (D): Russell’s traction may reduce fractures of Effects/
the hip or femur. Interactions
(3) (B): 90-degree traction is used on the femur if It is important to assess clients for actual and
skin traction isn’t suitable. potential side effects and adverse effects of
(4) (C): Buck’s traction is used to temporarily medications, including prescription, over-the-counter,
immobilize and herbal medications. This requires
a fractured leg. knowledge of all medications a client is taking, and
(5) (A): Cervical traction is used to stabilize a spinal information on preexisting conditions.
fracture or muscle spasm. PHysi ologica l Integrity:
25. The Answer is 3 Pharmac ologica l and Parenteral
It is important to evaluate pain in the neonate. Look The rapies
at the chart in the exhibit below. What would the pain chap ter 9
score be for an infant with a high-pitched cry, O2 222
saturation of 96%, a grimace, and frequent periods of NCLEX-RN® Exam Content Review and Practice
wakefulness? Provide clients with information on common side
Score effects and how to manage them. This
012 includes letting clients know when to call or notify
Crying No High pitched Inconsolable their primary health care provider regarding
Requires O2 No < 30% > 30% side effects. Also know when to contact the client’s
Expression None Grimace Grimace/grunt primary health care provider regarding
Sleepless No Wakes frequently Always awake side effects of medication or parenteral therapy for
Category: Rest and sleep clients who are hospitalized.
(1) A score of 0 is incorrect, because the infant has a Be able to identify signs and symptoms of an allergic
grimace (1), periods of wakefulness (1), and a high- reaction, which include the following:
pitched • Skin: Redness, itching, swelling, blistering, weeping,
cry (1). crusting, rash, eruptions, or hives
(2) A score of 2 is incorrect, because the infant has a (itchy bumps or welts)
grimace (1), periods of wakefulness (1), and a high- • Lungs: Wheezing, tightness, cough, or shortness of
pitched breath
cry (1). • Head: Swelling of the face, eyelids, lips, tongue, or
(3) CORRECT: This is the closest evaluation with the throat; headache
information given. The infant has a high-pitched cry • Nose: Stuffy nose, runny nose (clear, thin
(1), discharge), or sneezing
an adequate O2 saturation (0), a grimace (1), and • Eyes: Red (bloodshot), itchy, swollen, or watery
periods of wakefulness (1). • Stomach: Pain, nausea, vomiting, diarrhea, or
(4) Enough information is provided to answer the bloody diarrhea
question. In addition, you must know which procedures are
221 appropriate for counteracting adverse
Pharmacological and parenteral therapies involve the effects due to medication or parenteral therapy, and
provision of care related to the administration how to implement them. And of course,
of all forms of medication as well as parenteral/IV document client response to actions taken to
therapy. Generic names of medications counteract adverse effects.
are used in a fairly consistent manner, while the Blood and Blood Products
brand/trade name may vary. Therefore, One of the most important aspects of dealing with
you should expect to see the use of generic blood products is the correct identification
medication names only on the NCLEX-RN® exam. of clients to ensure the right products are used.
Some test items may also refer more broadly to Identify the client according to facility/agency
general classifications of medications. policy prior to administration of red blood cells/blood
On the NCLEX-RN® exam, you can expect 15 percent products. The steps involved include
of the questions to relate to the Pharmacological reviewing the prescription for administration,
and Parenteral Therapies subcategory. Exam content ensuring the blood is the correct type, ensuring
includes, but is not limited the identity of the client, checking that crossmatching
to, the following areas: is complete, and ensuring client consent.
• Adverse effects/contraindications/side Before administering any blood products, check the
effects/interactions client for appropriate venous access for
• Blood and blood products
product administration, select the correct needle infections, electrolyte imbalance, and iron overload. If
gauge, and check the integrity of the access complications occur, they must
site. Understand when it is appropriate for a client to be documented in the client’s medical record.
be an autologous donor (i.e., use the Central Venous Access Devices
client’s own blood), and the procedures for autologous Provide information to clients regarding reasons for
blood donation: and care of central venous access devices
• Four to six weeks prior to surgery (CVADs). Types of CVADs include the following:
• Every three days if hemoglobin levels are 224
satisfactory NCLEX-RN® Exam Content Review and Practice
• Good for rare blood types, transfusion reactions, • Tunneled catheter: A tunneled catheter is placed in
prevention of blood-borne disease a central vein, tunneled under the
transmission skin, and then brought out through the skin. Examples
• Not good if client has an acute infection, a low include Hickman and Broviac.
hemoglobin count, or cardiovascular • Implanted port: A port is inserted under
disease subcutaneous tissue and attached to a catheter,
Know the different blood types (ABO and Rh blood which is threaded into the superior vena cava.
group systems) and compatibilities Examples include Mediport and Port-a-
based on blood type, Rh factors, antibody screening, Cath.
and crossmatching. Be familiar with • Peripherally inserted central catheter (PICC): PICCs
the procedures employed after blood is drawn for are inserted into a basilic or cephalic
typing, including the use of special client vein just above or below the antecubital space of the
223 client’s right arm by a doctor or
9: Physiological Integrity: Pharmacological and specially trained IV therapy nurse. The catheter
Parenteral Therapies terminates in the superior vena cava.
identification bracelets, and how to match the PICCs often remain in place for long periods of time.
bracelets with the unique blood donor number Know how to access an implanted CVAD to provide
on a sample or identification tag on any unit of blood medication and/or nutrition for a client,
the client receives. as well as how to care for a client with a CVAD. This
It is also important to know the various blood includes:
components and what they are used for: • Maintaining strict sterile procedures to minimize risk
• Whole blood: Not normally used; mainly situations of infection
of major hemorrhage • Flushing line periodically with normal saline solution
• Red blood cells (RBCs): Anemia, blood loss • Checking port placement
• Fresh frozen plasma (FFP): Coagulation deficiency • Changing dressing
• Platelets: Thrombocytopenia Dosage Calculation
• Albumin: Shock, blood loss, low protein levels due Medications are prescribed in specific amounts or
to surgery or liver failure weights per volume for liquids. You should
• Cryoprecipitate: Blood loss or immediately prior to be able to perform the calculations needed for proper
an invasive procedure in clients with medication administration. The common
significant hypofibrinogenemia formulas for calculating dosages include the following:
To administer blood products safely, and evaluate • Ratio and proportion
client response to administered products, • “Desired over have”
follow the procedure detailed below: • Dimensional analysis
1. Verify client consent. Be aware of rounding rules when calculating dosages,
2. Check client’s baseline vital signs. as well.
3. Check physician’s order. Dosages are calculated using body weight in
4. Identify a stable vein, and then choose a needle kilograms, so you convert between pounds and
with the proper gauge. kilograms. Most often, you multiply the body weight
5. Set up equipment and start IV. by the dosage order per kilogram. You
6. Obtain correct component from blood bank. can also calculate volume using standard
7. Verify client identification and related information pharmaceutical math calculations. To calculate
(use second nurse to double-check). single dosages, divide the total daily dose by the
8. Hang blood. number of doses per day. You can also use a
9. Begin transfusion at a slow rate (2 mL per minute). nomogram (a type of graph) to calculate dosages
10. Monitor client vital signs after the first 15 minutes based on body surface area.
and thereafter in accordance with In addition to dosage calculation for medications for
facility policy. adults, it is important to know the differences
11. After 15 minutes, increase rate of infusion. between adult and pediatric dosages and how to
12. Monitor client vital signs and lung sounds for one calculate pediatric dosages. It’s also
hour after transfusion is complete. important to know how to help children swallow pills
13. Document all activities in the client’s medical and how to give medications to infants.
record. 225
It is important to know how to respond to common 9: Physiological Integrity: Pharmacological and
complications from blood transfusions, Parenteral Therapies
including transfusion reactions (allergic, febrile, or Oral Medications
hemolytic), circulatory overload, bloodborne
When tablets are scored, they may be broken and the formulary and consulting the pharmacist, as
given as partial doses. Do not break or needed. You must also understand the
crush extended release tablets. Abbreviations to know likely effects and outcomes for any oral, intradermal,
include the following: subcutaneous, intramuscular, or topical
• CR: Controlled release medications prescribed for your client.
• CRT: Controlled release tablet Evaluate and document a client’s use of medications
• LA: Long acting over time, including prescriptions, overthe-
• SA: Sustained action counter medications, and home remedies. This
• SR: Sustained release includes explaining effects and outcomes
• TR: Timed release to clients and families.
• XL: Extended length Medication Administration
• XR: Extended release It is important to understand the general principles of
Enteral Medications medication administration, including
Enteral medications are administered through a tube. how medications are named (generic versus brand
Know the correct tube placement for name or trade name). Use the six “rights”
the following types of tubes: when administering client medications, as follows:
• Nasogastric (through the nose and into the stomach) 1. Right client: Identify the client in two ways, such
• Nasointestinal (through the nose, past the stomach, as checking the client’s armband and
and into the small intestine) asking the client to state his or her name, if able. Do
• Percutaneous (through the skin directly into the not use the room number as a
stomach) method to identify the client.
It is important to know how to care for a client 227
receiving enteral medication. Flush the tube 9: Physiological Integrity: Pharmacological and
with 30 mL water before administering the Parenteral Therapies
medication. Use a solution/elixir form of medication, 2. Right drug: Know both the generic name and its
when available. brand equivalent; also double-check the
Injectable Medications medication order.
The following steps comprise the procedure for 3. Right dose: Make sure the dose that is administered
injecting medications: is a safe amount.
1. Choose a needle based on volume and type of 4. Right route: Check the medication order to verify
medication, destination site, client size, the route of administration, such as
and viscosity of medication. oral, IV, or suppository.
2. Maintain sterility when assembling the syringe and 5. Right time: Verify that the medication is being
needle. given at the proper time (with meal, a.m./
3. Withdraw medication from the vial/ampule. p.m., etc.)
4. Use anatomical landmarks (intramuscular, 6. Right documentation: Document details
intravenous, and/or subcutaneous). immediately after the medication is administered.
5. Wash hands and put on gloves. Review pertinent data prior to administration of
6. Cleanse area with alcohol swabs and wait for it to medication. This includes vital signs, lab
dry. results, allergies, potential medication interactions,
7. Inject medication. medical history, and current diagnosis.
226 Know the drug name, dosage, route, frequency, and
NCLEX-RN® Exam Content Review and Practice special parameters for withholding doses
8. Discard the syringe and needle into a sharps or administering additional doses. Check each medical
container. order for accuracy: ensure that it
9. Remove gloves. includes the date, time, and client’s last name, and
10. Wash hands. that it is signed by the prescribing physician.
Topical Medications This is important because you are responsible if you
Understand how to administer the following types of administer a drug based on an
topical medications: incorrect order.
• Skin It is important to understand the basic concepts of
• Nasal pharmacology, including:
• Optical • Pharmacokinetics: How the body absorbs,
• Otic (ear) distributes, and metabolizes medications
• Vaginal • Absorption routes: GI tract, respiratory tract, and
• Rectal skin
Inhaled Medications • Distribution: How a drug moves through the body
You should be able to explain how to use a metered- from absorption site to action site
dose inhaled (MDI) medication to your • Metabolism: Conversion of a drug by enzymes into
clients. A spacer is a device that attaches to the MDI a less-active, excretable substance
to help deliver the medicine to the lungs • Excretion: Elimination of drug and metabolites from
instead of the mouth. the body
Expected Actions/Outcomes The basic principles of medication administration are:
You are expected to obtain information on prescribed • Make sure the medication order is accurate.
medications for clients by reviewing • Check for client allergies.
• Assess the client to be sure the medication makes images of faces with different expressions. Be aware
sense. of nonverbal indicators, such as facial
• Check all other medications the client is taking. expressions or sounds.
• Calculate the proper dosage. Know how to provide pain management appropriate
• Check the expiration date of the medication. for client age and different diagnoses
• Label all medications. (pregnant women, children, and older adults).
You are responsible not only for preparing and 229
administering but also for documenting medications 9: Physiological Integrity: Pharmacological and
given by common routes (oral or topical), as well as Parenteral Therapies
by parenteral routes (IV, IM, or Document pain mediation administration according to
subcutaneous). This may include mixing medications facility/agency policy, and comply
from two vials when necessary, such as with regulations governing controlled substances
when administering a mixed dose of insulin. (such as counting narcotics and wasting
228 narcotics), and evaluate and document client use and
NCLEX-RN® Exam Content Review and Practice response to pain medications.
You are expected to be able to adjust/titrate dosages Total Parenteral Nutrition
of medication based on the assessment of Total parenteral nutrition (TPN) is nutrition provided
physiologic parameters of each client. This includes intravenously for clients who are unable
giving insulin according to blood glucose to tolerate oral or enteral feedings. It may be used in
levels and titrating medication to maintain a specific both home and hospital environments.
blood pressure. Know how to administer, maintain, and discontinue
Nurses must properly dispose of unused medications TPN. This includes knowing the ingredients
according to facility/agency policy. In of the solution: amino acids, dextrose for
addition, it is your responsibility to educate clients carbohydrates, vitamins and minerals, trace
about medications, including their potential elements, electrolytes, and water, and sometimes
side effects, how to take them, and how to handle side lipids, insulin, and heparin. You should
effects and/or allergic reactions. also know the components of different solutions that
Parenteral/Intravenous Therapies need to be used depending on the client’s
It is important to know the basics of intravenous nutritional needs and disease state. Clients who may
therapy, including the indicators, types of need TPN are those with GI tract
fluids used (isotonic solutions, hypertonic solutions, issues, who are recovering from GI surgery, or who
and hypotonic solutions), and equipment have experienced trauma. Clients with
(catheters and needles, infusion pumps, electronic high nutritional needs may also require TPN.
delivery devices, regulators, controllers, Access sites for TPN include peripheral lines through
mechanical infusion devices, and tubing). There are veins (for supplements only, not when
four types of infusion therapy: a client needs nutrition replacement; these should
peripheral, central, continuous, and intermittent. only be used for two weeks or less) and
Know when each should be used. central lines, which are more typical (often a PICC
As with medication dosages, apply mathematic line).
concepts when administering intravenous Be aware of the following as you manage a client
and parenteral therapy. To calculate an IV drip rate, receiving TPN:
use the following formula to calculate • There is a risk of pneumothorax during catheter
drops per minute: insertion for a PICC line.
(Total number of milliliters divided by total number of • Examine the IV insertion site during each shift for
minutes) × drip factor = gtt/minute signs of infection.
You will be provided with the drip factor in the • Do not use the IV line for anything other than TPN.
question stem on the NCLEX-RN® exam. • Inspect the bag of solution for particles prior to
Know which veins to access for various therapies; you hanging.
should be able to prepare clients for • Monitor the client’s blood glucose level.
intravenous catheter insertion, insert and remove a • Measure daily weight to determine/adjust fluid
peripheral intravenous line, and monitor balance.
the use of an infusion pump, whether it’s intravenous • Monitor other lab results, such as electrolytes,
or patient controlled analgesia (PCA). protein, prealbumin/albumin, creatinine,
You should also be able to maintain an epidural lymphocytic count, and liver function.
infusion. Know the rates of administration of TPN, how to
If a client needs intermittent parenteral fluid therapy monitor clients for adverse effects, and
for nutritional purposes, educate the how to taper down use of TPN. (Do not discontinue
client and evaluate the client’s response. You should TPN abruptly.) Possible complications
also be able to monitor and maintain include fluid overload, air embolism, infection/sepsis,
infusion sites and track the rates of infusion to ensure hyperglycemia, and hypoglycemia.
they are correct. Finally, you should be able to evaluate outcomes of
Pharmacological Pain Management TPN, including satisfactory weight gain
To determine client need for administration of a PRN and fluids, and electrolytes within normal limits.
pain medication, question the client NCLEX-RN ® Exam Content Review and Practice
about his or her level of pain using a pain rating scale 230
from 1–10, or a visual scale using 1. The nurse is conducting a home visit with a
client who has a history of angina. Which 1. Nausea and vomiting
of the following BEST demonstrates that 2. Difficulty swallowing
further teaching about nitroglycerin therapy 3. Neutropenia
is required? 4. Fever
1. “I take a tablet about 10 minutes before I CHAPTER QUIZ
walk up the stairs.” 9: Physiological Integrity: Pharmacological and
2. “I take no more than 3 doses in a Parenteral Therapies
15-minute period of time.” 231
3. “I keep the tablets in a glass dish on the 9: Physiological Integrity: Pharmacological and
windowsill so they are readily available.” Parenteral Therapies
4. “I will call my doctor immediately if I 7. A client is admitted for gastrointestinal
experience blurred vision.” bleeding. He has a platelet count of 15,000/
2. The nurse assesses the peripheral IV site of mm and platelets have been ordered from
a client receiving a doxorubicin infusion and the blood bank. Which of the following does
suspects extravasation. After stopping the the nurse know are required for platelet
infusion and disconnecting the IV tubing, transfusions? Select all that apply.
which of the following should the nurse do 1. ABO compatibility
next? 2. Rh compatibility
1. Apply a hot compress to the IV site. 3. Crossmatching
2. Apply a cold compress to the IV site. 4. A specialized platelet filter
3. Elevate the affected extremity. 8. A client’s red blood cell transfusion was
4. Attempt to aspirate the residual drug. discontinued due to an acute hemolytic
3. The nurse is preparing to discharge a transfusion reaction. Which of the following
72-year-old man on warfarin therapy for a strategies should the nurse use to BEST
pulmonary embolism. The nurse’s discharge minimize the risk of such a reaction?
teaching should include which of the 1. The nurse ensures the client’s temperature
following instructions? does not increase more than 1.8º F during
1. Follow a healthy diet by increasing the transfusion.
ingestion of green, leafy vegetables. 2. The nurse verifies all client-identifying
2. Take herbal remedies to manage cold information according to hospital
symptoms. protocol prior to hanging the unit of
3. Avoid alcohol due to enhanced blood.
anticoagulant effect. 3. The nurse administers meperidine for
4. Take Coumadin only on an empty severe rigors.
stomach. 4. The nurse administers acetaminophen
4. A 75-year-old woman has been prescribed prior to the transfusion.
amitriptyline hydrochloride to manage 9. A client is receiving a blood transfusion. The
neuropathic pain associated with diabetic nurse observes that the client is experiencing
neuropathy. She reports to the nurse that her diarrhea, abdominal pain, and chills. Which
pain level has decreased from a 7 to a 3 on a of the following actions should the nurse take
scale of 1–10. However, she is experiencing FIRST?
severe xerostomia. Which of the following 1. Assist the client to the bathroom.
strategies should the nurse choose to help 2. Stop the transfusion.
relieve this symptom? 3. Administer meperidine.
1. Increase caffeine intake. 4. Get a warming blanket.
2. Decrease fluid intake. 10. The nurse aspirates a central venous
3. Increase dietary sodium. catheter prior to drug administration
4. Chew sugar-free gum. but is not able to verify blood return.
5. Prior to administering digoxin 0.125 mg PO The nurse does not feel resistance when
to a client with chronic heart failure, the flushing or see any fluid leakage, swelling,
nurse determines that the apical pulse is 56. or redness around the catheter site. Which
Which of the following should the nurse do of the following does the nurse know are
FIRST? appropriate steps? Select all that apply.
1. Administer the drug and recheck the 1. Flush the catheter with saline, using
pulse in one hour. a 10-mL syringe and a push-pull
2. Withhold the drug and notify the technique.
physician. 2. Request that the client cough and
3. Obtain an EKG. reattempt aspiration.
4. Send a blood sample to the laboratory for 3. Administer IV medication and observe
a digoxin level. for signs and symptoms of catheter
6. A 65-year-old man with metastatic colon malfunction.
cancer has been prescribed hydromorphone 4. Follow institutional protocol to initiate a
PO/PRN to help manage his pain. The nurse declotting protocol.
knows that the rectal route of administration 11. A client is admitted for pulmonary
is contraindicated when which of the embolism and is receiving heparin 1,500
following is present? units/hour IV. In case of a serious bleeding
reaction, the nurse has which of the abruptly stopped
following drugs readily available? 2. Withdrawal symptoms when the drug
1. Vitamin K dose is reduced
2. Protamine sulfate 3. Habitual and compulsive use of a drug
3. Promethazine hydrochloride 4. A state of adaptation
4. Protamine 18. A client is admitted with severe back pain
12. A client with known heparin-induced and is requesting pain medication. During
thrombocytopenia (HIT) is undergoing her assessment, the nurse notes the client
chemotherapy and is having a central has been taking acetaminophen 650 mg
venous access device placed. Which of every 4 hours at home with minimal relief.
the following types of central venous Based on this information, which of the
access device does the nurse know following PRN-ordered drug(s) should the
BEST minimizes the risk of HIT-related nurse consider administering?
complication? 1. Hydrocodone with acetaminophen
1. Hickman 2. Acetaminophen
2. Broviac 3. Ibuprofen
3. Groshong 4. Acetaminophen with oxycodone
4. Port 19. A 14-year-old boy has been prescribed
NCLEX-RN® Exam Content Review and Practice amphetamine and dextroamphetamine for
232 attention-deficit/hyperactivity disorder
13. A client has been instructed by his (ADHD). The nurse explains that the client
physician to increase his warfarin sodium should be alert for which of the following
dose from 5 mg to 7.5 mg. He only has 5-mg adverse drug effects?
tablets available. How many tablets should 1. Weight gain
the nurse instruct him to take? 2. Depression
1. 0.5 3. Somnolence
2. 1 4. Bradycardia
3. 1.5 9: Physiological Integrity: Pharmacological and
4. 2 Parenteral Therapies
14. The nurse is preparing to set up an 233
intravenous infusion of normal saline 1,000 9: Physiological Integrity: Pharmacological and
mL over a 6-hour period. The tubing drop Parenteral Therapies
factor is 10 gtt/mL. Which of the following 20. The nurse is administering a drug by
rates of infusion should the nurse choose? Z-track and must follow the proper
1. 12 gtt/min technique. Place the following steps in the
2. 28 gtt/min appropriate order. All options must be
3. 33 gtt/min used.
4. 36 gtt/min 1. Withdraw the needle.
15. A man weighs 165 lb. and is being treated 2. Administer the drug intramuscularly
for shock. The nurse is preparing a (IM) in the dorsogluteal site.
dopamine hydrochloride infusion to start 3. Release the skin.
at 5 mcg/kg/min. The nurse has prepared 4. Displace the skin lateral to the injection
the following to infuse: dopamine 400 mg in site.
250 mL D5W. Which of the following rates 21. A client admitted with chronic heart
of infusion should the nurse choose? failure is taking furosemide. Which of
1. 14 mL/hr the following statements, if made by the
2. 16 mL/hr client, BEST demonstrates to the nurse
3. 22.5 mL/hr that the client understands the side effects
4. 37.5 mL/hr associated with this drug?
16. A 45-year-old woman with breast cancer is 1. “My blood pressure might be
receiving doxorubicin 60 mg/m2 as part of abnormally high.”
her cancer therapy. She is 5 ft. 6 in. tall and 2. “I should include more foods such as
weighs 145 lb. Her body surface area is 1.75 bananas, apricots, and legumes in my
m2. What is the correct dose that the nurse diet.”
should administer? Record your answer 3. “I should take the drug before bedtime.”
using one decimal place. 4. “I should not take the pill with food.”
mg 22. The nurse is administering vancomycin 1 g
17. A client is admitted with sickle-cell anemia every 12 hours for a soft tissue infection.
and voices concerns about becoming The nurse reminds the client to report
addicted to pain medicine. The nurse symptoms associated with one of the
explains the difference between physical serious side effects of the drug, ototoxicity.
dependence, tolerance, and addiction. Which of the following statements by the
Which of the following symptoms or client indicates to the nurse that the client
behaviors does the nurse know is BEST may be experiencing this adverse reaction?
associated with addiction? 1. “I hear ringing in my ear.”
1. Withdrawal symptoms when the drug is 2. “The IV is burning.”
3. “My skin is very itchy.” effects/interactions
4. “I have a bad taste in my mouth.” (1) Hot compresses should not be applied in an
23. A client is leaving the clinic with a new doxorubicin-associated extravasation.
prescription for lisinopril. Which of the (2) Although a cold compress is recommended in an
following suggestions can the nurse make doxorubicin-associated extravasation, it should
to minimize one of the major effects of not be applied until residual drug removal has
lisinopril? been attempted.
1. Eat fruits and vegetables high in iron. (3) Although elevating the arm for 48 hours is
2. Rise slowly from a lying to a sitting recommended,
position. this should not be done until after
3. Increase fluid intake. the residual drug has been removed.
4. Avoid aspirin-containing drugs. (4) CORRECT: The first step the nurse should take
24. The nurse is administering a doxorubicin is to attempt to remove any residual drug using
IV push to a client with breast cancer. a 1–3 mL syringe.
Which of the following should the nurse 3. The Answer is 3
explain is to be expected during therapy The nurse is preparing to discharge a 72-year-old
with this drug? man on warfarin therapy for a pulmonary embolism.
1. Burning at the IV site during The nurse’s discharge teaching should include which
administration of the following instructions?
2. Red-colored urine Category: Adverse effects/contraindications/side
3. Permanent alopecia effects/interactions
4. Teeth discoloration (1) The intake of foods containing vitamin K should
25. A 60-year-old woman with anorexia nervosa not be altered from baseline.
is having an indwelling central venous (2) Herbal medications may interfere with the
access device placed in preparation for total effectiveness
parenteral nutrition (TPN) administration. of warfarin.
Which of the following factors does the (3) CORRECT: Alcohol can increase the anticoagulant
nurse know accounts for the client’s effect of warfarin and should be avoided.
increased risk of thrombophlebitis with a (4) Warfarin can be taken without regard to food
peripheral intravenous line? Select all that intake, although gastrointestinal upset may be
apply. diminished if taken with food.
1. Age 4. The Answer is 4
2. Hypertonicity of the TPN A 75-year-old woman has been prescribed
3. Hypotonicity of the TPN amitriptyline
4. Poor peripheral venous access hydrochloride to manage neuropathic pain
NCLEX-RN ® Exam Content Review and Practice associated with diabetic neuropathy. She reports to
234 the nurse that her pain level has decreased from a 7
1. The Answer is 3 to a 3 on a scale of 1–10. However, she is
The nurse is conducting a home visit with a client experiencing
who has a history of angina. Which of the following severe xerostomia. Which of the following strategies
BEST demonstrates that further teaching about should the nurse choose to help relieve this symptom?
nitroglycerin therapy is required? Category: Adverse effects/contraindications/side
Category: Adverse effects/contraindications/side effects/interactions
effects/interactions (1) Increasing caffeine intake will not relieve
(1) Taking a nitroglycerin tablet prior to exertion xerostomia.
is an appropriate way to help prevent anginarelated (2) Decreasing fluid intake will not relieve xerostomia.
symptoms induced by activity. (3) Increasing dietary sodium will not relieve
(2) Taking no more than 3 doses in a 15-minute xerostomia.
period of time is appropriate nitroglycerin dosing (4) CORRECT: Strategies to reduce xerostomia (dry
instructions. mouth) include increasing fluid intake and chewing
(3) CORRECT: Nitroglycerin tablets may lose sugar-free gum.
effectiveness Chapter Quiz Answers and Explanations
if not protected from light. Therefore, 9: Physiological Integrity: Pharmacological and
they should be stored in dark containers. Parenteral Therapies
(4) Blurred vision is a significant side effect of 235
nitroglycerin 9: Physiological Integrity: Pharmacological and
therapy that should be immediately Parenteral Therapies
reported to the physician. 5. The Answer is 2
2. The Answer is 4 Prior to administering digoxin 0.125 mg PO to a client
The nurse assesses the peripheral IV site of a client with chronic heart failure, the nurse determines
receiving a doxorubicin infusion and suspects that the apical pulse is 56. Which of the following
extravasation. After stopping the infusion and should the nurse do FIRST?
disconnecting Category: Adverse effects/contraindications/side
the IV tubing, which of the following effects/interactions
should the nurse do next? (1) Unless the physician’s order specifies otherwise,
Category: Adverse effects/contraindications/side when the client’s apical pulse drops below 60, the
nurse should hold the dose and notify the physician. this action will not minimize the risk of an acute
(2) CORRECT: Unless the physician’s order specifies hemolytic transfusion reaction from taking
otherwise, when the client’s apical pulse place.
drops below 60, the nurse should hold the dose 9. The Answer is 2
and notify the physician. A client is receiving a blood transfusion. The nurse
(3) Although an EKG may be indicated, it is not observes that the client is experiencing diarrhea,
generally the first course of action. abdominal pain, and chills. Which of the following
(4) Although obtaining a digoxin level may be actions should the nurse take FIRST?
indicated, Category: Blood and blood products
it is not generally the first course of action. NCLEX-RN® Exam Content Review and Practice
6. The Answer is 3 236
A 65-year-old man with metastatic colon cancer has (1) Assisting the client to the bathroom may be an
been prescribed hydromorphone PO/PRN to help appropriate comfort measure but should not be
manage his pain. The nurse knows that the rectal performed first.
route of administration is contraindicated when (2) CORRECT: Signs and symptoms of a transfusion
which of the following is present? reaction may include chills, diarrhea, fever,
Category: Adverse effects/contraindications/side hives, pruritus, flushing, and abdominal or back
effects/interactions pain. The nurse’s first action should be to stop
(1) The rectal route of administration may be the transfusion.
preferred (3) Meperidine may alleviate rigors, which the client
when a client has nausea and vomiting. was not experiencing.
(2) The rectal route of administration may be (4) Getting a warming blanket may be an appropriate
preferred comfort measure but should not be performed
when a client has difficulty swallowing. first.
(3) CORRECT: The rectal route of administration 10. The Answer is 1, 2, and 4
should NOT be used in clients who have anal or The nurse aspirates a central venous catheter prior
rectal lesions, mucositis, thrombocytopenia, or to drug administration but is not able to verify blood
neutropenia. return. The nurse does not feel resistance when
(4) The rectal route of administration may also be flushing or see any fluid leakage, swelling, or redness
appropriate for a client who has a fever. around the catheter site. Which of the following
7. The Answer is 1, 2, and 4 does the nurse know are appropriate steps? Select
A client is admitted for gastrointestinal bleeding. all
He has a platelet count of 15,000/mm and platelets that apply.
have been ordered from the blood bank. Which of Category: Central venous access devices
the following does the nurse know are required for (1) CORRECT: Flushing the catheter with saline
platelet transfusions? Select all that apply. using a 10-mL syringe and a push-pull technique
Category: Blood and blood products are appropriate steps to try to verify blood
(1) CORRECT: The donor and recipient should be return in a central venous catheter.
ABO-compatible. (2) CORRECT: Instructing the client to cough
(2) CORRECT: The donor and recipient should be before reattempting aspiration is an appropriate
Rh-compatible. step to try to verify blood return in a central
(3) Crossmatching is not required for platelet venous catheter.
transfusions. (3) Administering IV medication (particularly
(4) CORRECT: Platelets are administered using cytotoxic medications) and fluids should not be
specialized platelet filters. performed until other steps are taken to verify
8. The Answer is 2 proper placement of the catheter by assessing for
A client’s red blood cell transfusion was discontinued patency and blood return.
due to an acute hemolytic transfusion reaction. (4) CORRECT: Initiating a declotting protocol
Which of the following strategies should the nurse per policy is an appropriate step to try to verify
use to BEST minimize the risk of such a reaction? blood return in a central venous catheter.
Category: Blood and blood products 11. The Answer is 2
(1) Monitoring the client’s temperature may help to A client is admitted for pulmonary embolism and is
promptly alert the nurse to a reaction but does receiving heparin 1,500 units/hour IV. In case of a
not prevent it from occurring. serious bleeding reaction, the nurse has which of the
(2) CORRECT: The most common cause of an acute following drugs readily available?
hemolytic transfusion reaction is the administration Category: Central venous access devices
of ABO-incompatible blood. By verifying (1) Vitamin K is not an antidote for heparin and
client-identifying information according to hospital does not reverse the effects of the drug.
policy, the nurse can minimize the risk of a (2) CORRECT: The antidote for heparin is protamine
client being transfused with ABO-incompatible sulfate.
blood. (3) Promethazine hydrochloride is not an antidote
(3) Administering meperidine may alleviate symptoms for heparin and does not reverse the effects of the
associated with a reaction but does not prevent drug.
it from developing. (4) Protamine is not an antidote for heparin and
(4) Administering acetaminophen may be indicated does not reverse the effects of the drug.
to prevent hypersensitivity reactions, but 12. The Answer is 3
A client with known heparin-induced the weight (75 kg) by the ordered dose (5 mcg/
thrombocytopenia kg/min), and multiply the result (375 mcg/min)
(HIT) is undergoing chemotherapy and is by 60. This equals 22,500 mcg/hr. Calculate mL/
having a central venous access device placed. Which hr by dividing 22,500 mcg/hr by 1,600 mcg/mL.
of the following types of central venous access device The appropriate rate is 14 mL/hr.
does the nurse know BEST minimizes the risk of (2) A rate of infusion of 16 mL/hour is not correct.
HIT-related complication? (3) A rate of infusion of 22.5 mL/hour is not correct.
Category: Central venous access devices (4) A rate of infusion of 37.5 mL/hour is not correct.
(1) A Hickman does not contain valves and is routinely 16. The Answer is 105 mg
flushed with heparin. A 45-year-old woman with breast cancer is receiving
(2) A Broviac does not contain valves and is routinely doxorubicin 60 mg/m2 as part of her cancer therapy.
flushed with heparin. She is 5 ft. 6 in. tall and weighs 145 lb. Her body
(3) CORRECT: A Groshong is a valved catheter surface
that does not require heparin flushing. area is 1.75 m2. What is the correct dose that the
(4) A port does not contain valves and is routinely nurse should administer? Record your answer using
flushed with heparin. one decimal place.
13. The Answer is 3 Category: Dose calculation
A client has been instructed by his physician to Answer: 60 mg/m2 × 1.75 m2 = 105 mg
increase his warfarin sodium dose from 5 mg to 17. The Answer is 3
7.5 mg. He only has 5-mg tablets available. How A client is admitted with sickle-cell anemia and
many tablets should the nurse instruct him to take? voices concerns about becoming addicted to pain
Category: Dose calculation medicine. The nurse explains the difference between
(1) Taking half a tablet would only provide 2.5 mg physical dependence, tolerance, and addiction.
of warfarin sodium. Which of the following symptoms or behaviors does
(2) Taking one tablet would only provide 5 mg of the nurse know is BEST associated with addiction?
warfarin sodium. Category: Pharmacological pain management
(3) CORRECT: Taking one and a half tablets (1) Withdrawal symptoms when the drug is abruptly
containing 5 mg of warfarin sodium each will stopped are associated with physical dependence
achieve a total dose of 7.5 mg. on a particular drug, not addiction.
(4) Taking two tablets would provide 10 mg of (2) Withdrawal symptoms when the drug dose is
warfarin reduced are associated with physical dependence
sodium. on a particular drug, not addiction.
9: Physiological Integrity: Pharmacological and (3) CORRECT: Addiction is characterized by
Parenteral Therapies compulsive
237 use of a drug for reasons other than therapeutic
9: Physiological Integrity: Pharmacological and benefit.
Parenteral Therapies (4) A state of adaptation is associated with tolerance
14. The Answer is 2 to a particular drug, not addiction.
The nurse is preparing to set up an intravenous 18. The Answer is 3
infusion A client is admitted with severe back pain and is
of normal saline 1,000 mL over a 6-hour period. requesting pain medication. During her assessment,
The tubing drop factor is 10 gtt/mL. Which of the the nurse notes the client has been taking
following rates of infusion should the nurse choose? acetaminophen
Category: Dose calculation 650 mg every 4 hours at home with minimal
(1) 12 gtt/min is not the correct rate of infusion. relief. Based on this information, which of the
(2) CORRECT: 28 gtt/min is the correct rate of following
infusion, PRN-ordered drug(s) should the nurse consider
arrived at as follows: 1,000 mL/6 hours × 10 administering?
gtt/mL/60 min/hour = 27.8 or 28 gtt/min. Category: Pharmacological pain management
(3) 33 gtt/min is not the correct rate of infusion. (1) Hydrocodone with acetaminophen would
(4) 36 gtt/min is not the correct of infusion. increase the client’s intake of acetaminophen.
15. The Answer is 1 The maximum recommended dose of acetaminophen
A man weighs 165 lb. and is being treated for shock. in a 24 hour period is 4 g.
The nurse is preparing a dopamine hydrochloride (2) Giving the client more acetaminophen would
infusion to start at 5 mcg/kg/min. The nurse has increase intake above the maximum recommended
prepared dose of 4 g in a 24-hour period.
the following to infuse: dopamine 400 mg in NCLEX-RN® Exam Content Review and Practice
250 mL D5W. Which of the following rates of infusion 238
should the nurse choose? (3) CORRECT: Ibuprofen is the only pain relief
Category: Dose calculation medication listed that does not contain
(1) CORRECT: The correct rate of infusion is 14 acetaminophen.
mL/hour, arrived at as follows: First convert (4) Acetaminophen with oxycodone would increase
165 lb. to kg by dividing by 2.2 (75 kg). Then, the client’s intake of acetaminophen. The maximum
convert 400 mg/250 mL to mcg/mL by dividing recommended dose of acetaminophen in a
400 mg/250 mL and multiplying the result (1.6 24-hour period is 4 g.
mg/mL) by 1,000 (1,600 mcg/m). Next, multiply 19. The Answer is 2
A 14-year-old boy has been prescribed amphetamine (1) Eating fruits and vegetables high in iron will not
and dextroamphetamine for attention- minimize the side effects of lisinopril.
deficit/hyperactivity (2) CORRECT: The hypotensive effect of lisinopril
disorder (ADHD). The nurse explains that may be reduced by rising slowly from a lying to
the client should be alert for which of the following a sitting position.
adverse drug effects? (3) Increasing fluid intake will not minimize the side
Category: Medication administration effects of lisinopril.
(1) Adderall may be associated with weight loss, not (4) Avoiding aspirin-containing drugs will not
weight gain. minimize
(2) CORRECT: Adderall may be associated with the side effects of lisinopril.
depression. 9: Physiological Integrity: Pharmacological 9:
(3) Adderall may be associated with agitation or Physiological Integrity: Pharmacological
restlessness, not somnolence. aanndd PPaarreenntteerraall
(4) Adderall may be associated with tachycardia, TThheerraappiieess
not bradycardia. 24. The Answer is 2
20. The Answer is 4, 2, 1, 3 The nurse is administering a doxorubicin IV push
The nurse is administering a drug by Z-track and to a client with breast cancer. Which of the following
must follow the proper technique. Place the following should the nurse explain is to be expected during
steps in the appropriate order. All options must therapy with this drug?
be used. Category: Expected actions/outcomes
Category: Medication administration (1) Burning at the IV site during administration is
(1) The third step in proper Z-track technique is to not a side effect of doxorubicin.
withdraw the needle. (2) CORRECT: A common side effect of doxorubicin
(2) The second step in proper Z-track technique is is red-colored urine.
to administer the drug IM. (3) Permanent alopecia is not a side effect of
(3) The last step in proper Z-track technique is the doxorubicin.
release the skin. (4) Teeth discoloration is not a side effect of
(4) The first step in proper Z-track technique is to doxorubicin.
displace the skin lateral to the injection site. 25. The Answer is 1, 2, and 4
21. The Answer is 2 A 60-year-old woman with anorexia nervosa is having
A client admitted with chronic heart failure is taking an indwelling central venous access device placed
furosemide. Which of the following statements, if in preparation for total parenteral nutrition (TPN)
made by the client, BEST demonstrates to the nurse administration. Which of the following factors does
that the client understands the side effects associated the nurse know accounts for the client’s increased
with this drug? risk of thrombophlebitis with a peripheral intravenous
Category: Medication administration line? Select all that apply.
(1) Lasix may be associated with hypotension. Category: Total parenteral nutrition
(2) CORRECT: Furosemide may decrease potassium. (1) CORRECT: The risk of thrombophlebitis is
Eating foods rich in potassium is advised. increased in individuals over the age of 60.
(3) Lasix may be associated with nocturia. (2) CORRECT: The risk of thrombophlebitis is
(4) Lasix does not have to be taken with food. increased in individuals undergoing treatment
22. The Answer is 1 with hypertonic fluids.
The nurse is administering vancomycin 1 g every (3) The risk of thrombophlebitis is increased with
12 hours for a soft tissue infection. The nurse reminds hypertonic, not hypotonic, IV therapy.
the client to report symptoms associated with one (4) CORRECT: The risk of thrombophlebitis is
of the serious side effects of the drug, ototoxicity. increased in individuals with poor peripheral
Which of the following statements by the client venous access.
indicates
to the nurse that the client may be experiencing 241
this adverse reaction? Reduction of risk potential involves ways in which you
Category: Medication administration can help to reduce the likelihood that
(1) CORRECT: Tinnitus may indicate that ototoxicity clients will develop complications or health problems
is developing. related to existing conditions, diagnostic
(2) A feeling that the IV is burning is not related to tests, treatments, or other procedures.
the development of ototoxicity. On the NCLEX-RN® exam, you can expect 12 percent
(3) Itchiness of the skin is not related to the of the questions to relate to Reduction
development of Risk Potential. Exam content for this category
of ototoxicity. includes, but is not limited to, the following
(4) The sensation of a bad taste in the mouth is not areas:
related to the development of ototoxicity. • Changes/abnormalities in vital signs
23. The Answer is 2 • Diagnostic tests
A client is leaving the clinic with a new prescription • Laboratory values
for lisinopril. Which of the following suggestions can • Potential for alterations in body systems
the nurse make to minimize one of the major effects • Potential for complications of diagnostic
of lisinopril? tests/treatments/procedures
Category: Expected actions/outcomes
• Potential for complications from surgical procedures • Cardiovascular
and health alterations ‚ . Angiography (angiogram)
• System specific assessments ‚ . Cardiac catheterization
• Therapeutic procedures ‚ . Echocardiography (echocardiogram)
Now let’s review some of the most important concepts ‚ . Electrocardiography (electrocardiogram or ECG,
related to these subtopics. EKG)
Changes/Abnormalities in Vital Signs ‚ . Holter monitoring
You must be able to assess client vital signs and 243
intervene when those vital signs are abnormal. 10: Physiological Integrity: Reduction of Risk
Abnormal vital signs include fever, hypertension, Potential
bradycardia, and tachypnea. ‚ . Stress/exercise tests
In order to properly assess vital signs and recognize ‚ . Venography (venogram), also called phlebography
abnormalities, apply your knowledge of • Renal/Urinary
the client’s pathophysiology. Evaluate invasive ‚ . Cystoscopy and cystography (cystogram)
monitoring data, such as pulmonary artery ‚ . Intravenous pyelography (IVP)
pressure and intracranial pressure. ‚ . Retrograde pyelography (retrograde pyelogram)
PHysiologica l Integrity: • Neurological
Reduction of Risk Potential ‚ . Electroencephalography (electroencephalogram or
chap ter 10 EEG)
242 ‚ . Myelography (myelogram)
NCLEX-RN® Exam Content Review and Practice • Musculoskeletal
Diagnostic Tests ‚ . Arthroscopy
It is important to understand the general principles of ‚ . Bone densitometry
specimen collection. Ideally, routine • Gastrointestinal
specimen collection should take place early morning ‚ . Barium enema
before a client has any food or fluids. If ‚ . Cholangiography
fasting is required, it is usually for an 8–12-hour ‚ . Cholecystography (oral)
period prior to the test. Use standard precautions ‚ . Colonoscopy
and aseptic techniques to protect yourself and your ‚ . Endoscopic retrograde cholangiopancreatography
clients from infection. (ERCP)
Label specimens with the client’s name, date, exact ‚ . Esophagogastroduodenoscopy
time of collection, and type of specimen. ‚ . Gastric analysis
On the laboratory requisition slip, include the client’s ‚ . Gastrointestinal ( GI) series
name, age, gender, room number, physician’s • Reproductive
name, possible diagnosis, tests requested, and any ‚ . Fetal nonstress test
factors that might interfere with ‚ . Amniocentesis
the test results. To avoid hemolysis, do not shake ‚ . Hysteroscopy
blood specimens unless instructed to do so. ‚ . Mammography
All specimens should be sent to the lab promptly. ‚ . Papanicolaou smear (Pap smear)
Values, or test results, that fall within predetermined • Integumentary
laboratory reference ranges are considered normal. ‚ . Tuberculin skin test
Abnormal values are outside ‚ . Other skin tests (allergy)
the reference range, and critical values are far enough Know how to compare client diagnostic findings with
outside of the reference range that they pretest results, and how to perform a
can cause immediate risk to the client. Critical values variety of diagnostic tests, including:
are called in to the nurse’s station and • Oxygen saturation
should be acted on immediately. You may be • Glucose monitoring
responsible for informing the client’s physician • Testing for occult blood
about these critical lab values. • Gastric pH
You should understand the purpose of and preparation • Urine specific gravity
for a variety of diagnostic tests, such 244
as the following: NCLEX-RN® Exam Content Review and Practice
• General • Arterial blood gases
‚ . Biopsy • Serum electrolytes
‚ . Computed tomography (CT) scan You should also know how to perform an
‚ . Fluoroscopy electrocardiogram. This test measures electrical
‚ . Magnetic resonance imaging (MRI) activity of the heart and detects cardiac dysrhythmias
‚ . Nuclear scan (radionuclide imaging or radioisotope and electrolyte imbalances. Electrodes
scan) are placed on the client’s extremities and chest, and
‚ . Positron emission tomography (PET) scan the electrical activity of the heart is
‚ . Ultrasonography recorded with each heartbeat. Cardiac waveforms are
‚ . X-rays recorded in 12 leads. There are no
• Respiratory food and fluid restrictions on clients getting an
‚ . Bronchoscopy electrocardiogram, and no preconsent is
‚ . Pulmonary function tests needed for the test. The client should be asked to lie
‚ . Ventilation scan (pulmonary ventilation scan) down and to expose arms and legs for
lead placement. Men should be bare-chested, women is at its lowest, right before the next scheduled drug
given gowns. It is your responsibility to administration. Peak levels are drawn
make note of any medications the client is taking that when the dose is at its highest (30 minutes after
might impact the test results. The client infusion). You must make sure drug levels
should be told to relax his or her muscles and to remain within the proper therapeutic range. If you find
breathe normally during the procedure, an abnormal level, alert the prescribing
which is painless. physician immediately.
In addition to performing an electrocardiogram on an In addition to knowing laboratory values, and how to
adult, you should know how to perform measure drug levels, you should be able
fetal heart monitoring using computer-assisted to recognize deviations from normal values of the
auditory assessment. This involves following:
inserting a fetal scalp electrode through the client’s • Albumin (blood)
cervix and attaching it to the epidermis • ALT (SGPT) (liver enzyme test)
of the fetus. You should also be able to monitor the • Ammonia
results of additional maternal and fetal • AST (SGOT) (liver enzyme test)
diagnostic tests, including nonstress tests, an • Bilirubin
amniocentesis, and an ultrasound. • Bleeding time
Laboratory Values • Calcium (total)
You must be familiar with a wide range of laboratory • Cholesterol (HDL and LDL)
values, which include the following: • Creatinine
• Arterial blood gases, including pH, pO2, pCO2, • Digoxin
SaO2, and HCO3 • Erythrocyte sedimentation rate (ESR), to diagnose
• Serum electrolytes conditions associated with inflammation
• Glucose studies, such as fasting blood glucose, • Lithium
random blood glucose, two-hour postprandial • Magnesium
blood glucose, glucose tolerance test (GTT), and • Partial thromboplastin time (PTT) and APTT
glycosylated hemoglobin (HgbA1C) • INR
• Coagulation studies, such as prothrombin time (PT), • Phosphorous/phosphate
international normalized ratio • Protein (total)
(INR), and activated partial thromboplastin time • PT (clotting)
(APTT) • Urine (albumin, pH, WBC count, differential)
• Complete blood count (CBC), which includes 246
hematocrit (Hct), hemoglobin (Hgb), NCLEX-RN® Exam Content Review and Practice
RBC count and index, platelet count and mean Know how to obtain blood specimens peripherally or
volume, and white blood cell (WBC) through a central line. Also know how
count and differential to obtain specimens other than blood for diagnostic
• Cardiovascular function studies, which include testing. This includes procedures for
serum lipids, creatine kinase (CK) or getting specimens from wound cultures and stool and
creatine phosphokinase (CPK), lactic dehydrogenase urine samples.
(LDH), and troponins Monitor client laboratory values and provide clients
• Thyroid function studies, such as thyroxine (T4), with information about the purpose and
triiodothyronine (T3), and thyroidstimulating procedures for prescribed laboratory tests.
hormone (TSH) Potential for Alterations in Body Systems
• Renal function studies, such as blood, urea, nitrogen It is important to be able to compare current client
(BUN) and serum creatinine data to baseline client data, particularly
• Urinalysis, which includes the detection of nitrites to evaluate symptoms of illness/disease. Identify
and leukocyte esterase client potential for aspiration (e.g., feeding
245 tube, sedation, and swallowing difficulties), skin
10: Physiological Integrity: Reduction of Risk breakdown potential due to immobility,
Potential nutritional status or incontinence, and clients with an
• Liver function studies, such as alanine increased risk for insufficient vascular
aminotransferase (ALT) or serum glutamicpyruvic perfusion (such as clients with immobilized limbs, who
transaminase (SGPT), aspartate aminotransferase are postsurgery, or who have diabetes).
(AST) or serum glutamicoxaloacetic You should also be able to provide treatments and/or
transaminase (SGOT), bilirubin, and ammonia care in response.
• Pancreatic enzymes, such as amylase and lipase Monitor client output for changes from baseline
• GI function studies, such as albumin, alkaline (nasogastric tube, emesis, stools, and urine)
phosphatase, total protein, and uric acid and educate clients about methods to prevent
• Immune function studies, such as human complications associated with activity level
immunodeficiency virus (HIV) test, CD4 T or diagnosed illness/disease (such as contractures,
cell counts, CD4 to CD8 ratios, and viral load testing and foot care for client with diabetes
You should know how to measure the amount of drug mellitus).
circulating in the client’s bloodstream, Potential for Complications of Diagnostic Tests/
usually before the scheduled daily dose of the drug. Treatments/Procedures
Trough levels are drawn when the dose You must assess a client for complications or
abnormal responses following a diagnostic
test or procedure, such as monitoring the client for Educate clients about treatments and procedures, and
signs of bleeding. Know how to position home management and care. The education
clients to prevent complications following tests, may include preoperative and/or postoperative
treatments, and procedures, by, for example, instructions to clients and families.
elevating the head of the bed or immobilizing an Monitor a client before, during, and after a procedure
extremity. When you see a complication, or surgery, and provide preoperative
it is important to recommend a change in tests, and intraoperative care (positioning, maintaining
procedures, and/or treatment prescriptions sterile field, and operative assessment). To
based on the client’s response to the initial testing and prevent further injury while moving a client with a
treatment. musculoskeletal condition, for example,
You should be able to insert an oral/nasogastric tube, use the log-rolling technique or an abduction pillow.
and maintain tube patency. Be able NCLEX-RN ® Exam Content Review and Practice
to recognize potential circulatory complications (such 248
as hemorrhage, embolus, and shock) NCLE X-1. The nurse is reviewing the chart of an
and know how to intervene to manage them. older
Examples of measures you can take to manage, adult male client after surgery for removal of
prevent, or lessen possible complications include the parathyroid glands. The client complains
restricting fluids or sodium, raising side of difficulty swallowing and a feeling of
rails of the client’s bed, or implementing suicide “pins and needles.” The nurse expects which
precautions. of the following laboratory values to be
You also need to know how to provide care for clients abnormal?
undergoing electroconvulsive therapy. 1. Calcium
This includes monitoring the airway, assessing for side 2. Lipase
effects, and teaching the client 3. Potassium
about the procedure. You should be able to intervene 4. Sodium
to prevent aspiration, and to prevent 2. The nurse is assessing a young-adult
potential neurological complications. Signs of pregnant client with no allergies who has
neurological complications include foot drop, tested positive for gonorrhea. Which of
247 the following medications should the nurse
10: Physiological Integrity: Reduction of Risk expect to be part of the treatment plan?
Potential 1. Tetracycline
numbness, and tingling. Make sure to evaluate and 2. Ciprofloxacin
document responses for all procedures 3. Azithromycin
and treatments. 4. Ceftriaxone
Potential for Complications from Surgical 3. A client is one day post-op for abdominal
Procedures and Health Alterations surgery. The nurse is teaching the client
Apply your knowledge of pathophysiology to monitor techniques to reduce pain when he moves,
for complications from surgical procedures coughs, or breathes deeply. Which of the
and health alterations. For example, you should following statements from the client indicates
recognize signs of thrombocytopenia. that the client understands the teaching?
You should also evaluate the client’s response to 1. “I can start exercising my limbs as soon
postoperative interventions aimed at preventing as you medicate me.”
complications, such as reducing the risk of aspiration 2. “I will just lie here for a few days until the
and promoting venous return pain goes away.”
and mobility. 3. “I will use the side rail for support when I
System-Specific Assessments move or turn.”
Assess clients for abnormal peripheral pulses and 4. “I will ask for pain medication only when
neurological status after a procedure or absolutely necessary.”
treatment. Neurological status can be assessed by 4. A 36-year-old primigravid client with
checking level of consciousness and evaluating a history of diabetes is admitted with
muscle strength and mobility. You should also be able preeclampsia. Which of the following actions
to assess clients for peripheral should the nurse take FIRST?
edema, hypoglycemia, and hyperglycemia. 1. Administer low-dose aspirin as ordered.
It is also important to identify factors that could result 2. Ask the physician for an order for calcium
in delayed wound healing and to supplements.
implement appropriate treatment in response, and/or 3. Monitor the client’s blood pressure.
to notify the primary care provider. 4. Prepare the client for delivery.
Perform a risk assessment for sensory impairment, 5. The nurse has just answered a call light for a
falls, level of mobility, and skin integrity. client who is two days post-op for abdominal
Once initial assessments are complete, perform surgery. The client states, “I coughed and
focused assessments and reassessments based heard this pop.” The nurse assesses the
on initial findings. surgical site and observes dehiscence of the
Therapeutic Procedures wound. Which of the following should the
When caring for clients undergoing therapeutic nurse do FIRST?
procedures, assess client response to recovery 1. Stay with the client and have a colleague
from local, regional, or general anesthesia. notify the physician.
2. Help the client to lie with his head slightly should the nurse also expect to see? Select
elevated and with knees bent. all that apply.
3. Apply warm, sterile normal saline soaks. 1. Elevated serum albumin
4. Help the client to sit up, which will reduce 2. Low serum globulin
the harmful effects of further coughing. 3. Elevated serum transaminate (ALT and
6. An elderly man is admitted to the hospital AST)
from the Emergency Department during the 4. Prolonged prothrombin time (PT)
night shift. The nurse is assessing the client’s 5. Low urine bilirubin
cerebellar function. Which of the following 12. The nurse is assessing a client with
questions should the nurse ask the client? Addison’s disease. The nurse expects to note
1. “Who is the current president of the which of the following?
United States?” 1. Anorexia
2. “Do you have trouble swallowing fluids 2. Weight gain
or foods?” 3. Yellow skin coloration
3. “Do you have any muscle pain?” 4. A craving for sweets
4. “Do you have problems with balance?” 13. A client is having a tonic-clonic seizure.
Chapter Quiz Which of the following should the nurse do
10: Physiological Integrity: Reduction of Risk FIRST?
Potential 1. Check the client’s breathing.
249 2. Remove objects from the client’s
7. An older adult male client with a history surroundings.
of myasthenia gravis is admitted to the 3. Place a tongue blade in the client’s
medical/surgical unit. Which of the mouth.
following tests should the nurse expect to 4. Restrain the client.
see ordered? Select all that apply. NCLEX-RN® Exam Content Review and Practice
1. Tensilon test 250
2. Nerve conduction studies 14. A client is recovering from a bout with
3. Lumbar puncture chronic glomerulonephritis. The nurse
4. EEG prepares the client for discharge and home
5. Electromyography management. Which of the following
8. A middle-aged female client with a statements indicates the client understands
history of atherosclerosis is admitted his condition and how to control it?
with complaints of abdominal tenderness 1. “I should stop taking my blood pressure
during deep palpation. The nurse notices medication if I feel better or have side
a pulsating mass in the periumbilical area. effects.”
Which of the following does the nurse 2. “I will take my furosemide medications
suspect? as ordered every morning.”
1. Appendicitis 3. “I will keep my negative feelings to
2. Abdominal aortic aneurysm myself, so I don’t get stressed.”
3. Acute cholecystitis 4. “I don’t need a follow-up examination
4. Paralytic ileus unless I’m feeling poorly.”
9. An older adult client with a history of 15. A nursing home client is admitted to the
blood clots is in the emergency room with hospital with a pressure ulcer involving fullthickness
suspected deep vein thrombosis (DVT) of loss extending to the bone. The
the left leg. The nurse starts IV heparin as nurse documents the pressure ulcer as being
ordered. Which of the following is LEAST at which of the following stages?
likely to be included in the care plan? 1. Stage I
1. Ambulation as tolerated 2. Stage II
2. Warm, moist soaks applied to the 3. Stage III
affected area 4. Stage IV
3. Analgesics as ordered 16. A client with Raynaud’s disease is
4. Anti-embolism stockings experiencing an acute attack. The nurse
10. The nurse is caring for a client with a should anticipate which of the following
history of chronic liver disease and cirrhosis assessment findings?
of the liver. Lab values reveal rising 1. Involuntary muscle contractions and
ammonia levels. Which of the following twitching
treatments should the nurse question? 2. Unilateral facial weakness and drooping
1. Calorie intake 1,800–2,400 cal/day in the mouth
form of glucose or carbohydrates 3. Numbness and tingling of fingers and
2. Protein 100 g/day blanching of the skin at the fingertips
3. An order to administer neomycin 4. Photophobia
4. Potassium supplements 17. The physician orders a CT scan of the
11. The laboratory values of an adult male client’s chest with IV contrast. Which of
client reveal the presence of hepatitis B the following findings in the client’s history
surface antigens and hepatitis B antibodies. should the nurse report to the physician?
Which of the following laboratory results 1. Hypertension
2. Allergy to shellfish blood components from the blood bank.
3. Urinary tract infection (UTI) 4. The only solution that should be added
4. Allergy to penicillin to blood or blood components is 0.45%
18. The oncologist examines a client in the sodium chloride (half normal saline
clinic and subsequently admits the client solution).
to the hospital with severe bone marrow 24. The nurse is providing discharge teaching
depression. The client’s therapy included to a client stabilized after an acute attack of
radiation and chemotherapy. Which of the primary gout. Which of the following foods
following nursing diagnoses takes priority should the nurse instruct the client to avoid
in the client’s care plan? to prevent future attacks?
1. Imbalanced nutrition: less than body 1. Cauliflower, asparagus, and mushrooms
requirements 2. Anchovies, liver, and lentils
2. Risk for infection 3. Cherries, strawberries, and blueberries
3. Pain 4. Cereal, pasta, and rice
4. Risk for injury 25. In the emergency room, the nurse is caring
19. The nurse is preparing to discharge a client for a client with complaints of substernal
who is stable after a sickle-cell anemia pain radiating to the arm and jaw, shortness
crisis. Which of the following instructions of breath, and a feeling of impending doom.
should the nurse provide to the client to The client had a stroke one month ago. The
avoid future crises? Select all that apply. client’s vital signs are blood pressure 146/72,
1. Limit your fluid intake. pulse 128, and respirations 36. The 12-lead
2. Avoid strenuous exercise. ECG reveals evolving acute myocardial
3. Apply cold compresses to painful areas. infarction (MI). Which of the following
4. Take pain medications as ordered. physician orders should the nurse question?
5. Avoid tight clothing. 1. Beta-adrenergic blocker
20. The clinic nurse is updating the medications 2. Morphine for pain
being taken by an anxious middleaged 3. IV nitroglycerin
client, and sees that the physician 4. Thrombolytic therapy
prescribed an antidiuretic hormone. The 26. An older adult female, newly diagnosed
nurse knows the medication has which of with type 2 diabetes, is ready for discharge.
the following effects on the kidneys? When providing discharge instructions,
1. Increases water reabsorption and urine the nurse teaches the client that the key to
concentration preventing diabetic foot complications is
2. Decreases water reabsorption and which of the following?
dilutes the urine 1. Taking the medication as ordered
3. Regulates sodium retention 2. Following the recommended diet
4. Controls potassium secretion 3. Surgical intervention
10: Physiological Integrity: Reduction of Risk 4. Regular evaluation of the look and feel
Potential of her feet
251 27. The nurse knows that the physician is
21. The nurse is performing an assessment on a most likely to order which of the following
client who has developed cirrhosis. Which laboratory tests to evaluate a client for
of the following signs and symptoms should hypoxia?
the nurse expect to see? Select all that apply. 1. Hematocrit
1. Dull abdominal ache 2. Sputum analysis
2. Cyanosis 3. Arterial blood gas (ABG) analysis
3. Poor tissue turgor 4. Total hemoglobin
4. Bruises NCLEX-RN® Exam Content Review and Practice
5. Fruity breath 252
22. The physician orders 0.5 mg of digoxin for a 28. The nurse is performing a 12-lead ECG on a
client with atrial fibrillation. The pharmacy client who has come to the emergency room
has 250-mcg tablets available. How many complaining of chest pain. Where should
tablets will the nurse give? the nurse place lead V1?
23. The nurse is preparing to administer a red A
blood cell transfusion to a client with a B
low hemoglobin level and low hematocrit. C
The nurse knows which of the following D
statements about blood transfusion practice 1. A
is true? 2. B
1. The client should be monitored for at 3. C
least one hour after the start of the 4. D
transfusion. 29. The nurse is assessing a client admitted
2. The transfusion should be completed with a cerebrovascular accident (CVA).
within 2 hours. The physician has ordered a swallow study.
3. The transfusion should be started within The nurse knows which of the following
30 minutes of removing the blood or lobes of the cerebral hemisphere is
involved in the control of voluntary muscle (2) The client should frequently move the parts of
movement, including those necessary for his body not affected by surgery to prevent stiffness
the production of speech and swallowing? and soreness.
1. Frontal (3) CORRECT: The client should use the side rail
2. Parietal for support and move slowly and smoothly without
3. Temporal sudden movement.
4. Occipital (4) The client should ask for pain medication as
30. The nurse is preparing to do the Heimlich needed so that he can move comfortably.
maneuver on a choking middle-aged adult 4. The Answer is 4
male client. Arrange the following steps in A 36-year-old primigravid client with a history of
the order the nurse should perform them. diabetes is admitted with preeclampsia. Which of
All options must be used. the following actions should the nurse take FIRST?
1. Make a fist with one hand. Category: Potential for complications of diagnostic
2. Stand behind the client. tests/treatments/procedures
3. Wrap your other arm around the client (1) Using low-dose aspirin has not been successful
and place that hand on top of your fist. and is not recommended for routine use in pregnancy.
4. Place your thumb toward the client, (2) Using calcium supplements has not been
below the rib cage and above the waist, successful
and wrap one arm around the client. and is not recommended for routine use
5. Ask the client if he is choking. in pregnancy.
6. Thrust upward 6–10 times. (3) Although frequent monitoring of blood pressure
10: Physiological Integrity: Reduction of Risk is a part of the management of preeclampsia, this
Potential is not the first thing the nurse should do.
253 (4) CORRECT: The nurse should prepare the client
1. The Answer is 1 for delivery, which is the most effective treatment
The nurse is reviewing the chart of an older adult for preeclampsia.
male client after surgery for removal of the 5. The Answer is 1
parathyroid The nurse has just answered a call light for a client
glands. The client complains of difficulty swallowing who is two days post-op for abdominal surgery. The
and a feeling of “pins and needles.” The nurse client states, “I coughed and heard this pop.” The
expects which of the following laboratory values to nurse assesses the surgical site and observes
be abnormal? dehiscence
Category: Laboratory values of the wound. Which of the following should
(1) CORRECT: Hypocalcemia is an indication of the nurse do FIRST?
hypoparathyroidism; symptoms include dysphagia Category: Therapeutic procedures; Potential for
and paresthesia. complications of diagnostic
(2) Lipase levels are not indicators of tests/treatments/procedures;
hypoparathyroidism. Potential for complications from surgical
(3) Potassium levels are not indicators of procedures and health alterations
hypoparathyroidism. Chapter Quiz Answers and Explanations
(4) Sodium levels are not indicators of NCLEX-RN® Exam Content Review and Practice
hypoparathyroidism. 254
2. The Answer is 4 (1) CORRECT: The nurse should stay with the client
The nurse is assessing a young-adult pregnant client and have a colleague notify the physician
with no allergies who has tested positive for first.
gonorrhea. (2) The second thing the nurse should do is help
Which of the following medications should the the client lie with his head slightly elevated (low
nurse expect to be part of the treatment plan? Fowler’s position) with knees bent in to decrease
Category: Potential for complications of diagnostic abdominal tension and monitor the client’s vital
tests/treatments/procedures signs.
(1) Tetracyclines are contraindicated in pregnancy. (3) The nurse should not place anything on the
(2) Fluoroquinolones are contraindicated in wound unless it has eviscerated, and then cover
pregnancy. the extruding wound contents with warm, sterile
(3) Azithromycin is the treatment for chlamydia. normal saline soaks.
(4) CORRECT: Ceftriaxone, a third-generation (4) The nurse would not help the client to sit up.
cephalosporin, is the recommended treatment Instead, the nurse would help the client to a low
for gonorrhea in pregnancy. Fowler’s position with knees bent in to decrease
3. The Answer is 3 abdominal tension and monitor the client’s vital
A client is one day post-op for abdominal surgery. signs.
The nurse is teaching the client techniques to reduce 6. The Answer is 4
pain when he moves, coughs, or breathes deeply. An elderly man is admitted to the hospital from the
Which of the following statements from the client Emergency Department during the night shift. The
indicates that the client understands the teaching? nurse is assessing the client’s cerebellar function.
Category: Therapeutic procedures Which of the following questions should the nurse
(1) The client should wait until the medication has ask the client?
taken effect. Category: System specific assessments
(1) This question will not help the nurse assess the An older adult client with a history of blood clots
client’s cerebellar function, which is related to is in the emergency room with suspected deep vein
balance and coordination. thrombosis (DVT) of the left leg. The nurse starts
(2) Trouble swallowing fluids or foods is not related IV heparin as ordered. Which of the following is
to cerebellar function. LEAST likely to be included in the care plan?
(3) Muscle pain is not related to cerebellar function. Category: Potential for complications of diagnostic
(4) CORRECT: The nurse evaluates cerebellar function tests/treatments/procedures
by testing the client’s balance and coordination. (1) CORRECT: Treatment aims to prevent
7. The Answer is 1, 2, and 5 complications,
An older adult male client with a history of myasthenia relieve pain, and prevent recurrence.
gravis is admitted to the medical/surgical unit. Ambulation is not allowed until the physician
Which of the following tests should the nurse expect approves walking.
to see ordered? Select all that apply. (2) Warm, moist soaks is an appropriate action to
Category: Diagnostic tests; Potential for complications take.
of diagnostic tests/treatments/procedures (3) Analgesics are an appropriate action to take.
(1) CORRECT: Myasthenia gravis produces sporadic (4) When the acute episode of DVT subsides, the
but progressive weakness and abnormal client may begin to walk while wearing antiembolism
fatigue in skeletal muscles. The Tensilon test stockings.
confirms the diagnosis by temporarily improving 10. The Answer is 2
muscle function after an IV injection of edrophonium The nurse is caring for a client with a history of
or neostigmine. chronic liver disease and cirrhosis of the liver. Lab
(2) CORRECT: Nerve conduction studies test for values reveal rising ammonia levels. Which of the
receptor antibodies. following treatments should the nurse question?
(3) Lumbar puncture is a test used to diagnose Category: Potential for complications of diagnostic
multiple tests/treatments/procedures
sclerosis, a result of progressive demyelination (1) Adequate calorie intake in the form of glucose or
of the white matter of the brain and spinal carbohydrates helps prevent protein catabolism.
cord. (2) CORRECT: Rising blood ammonia levels can
(4) An EEG is a test used to diagnose multiple result from cirrhosis, and hepatic encephalopathy
sclerosis, follows. Protein is restricted to 40 g/day and
a result of progressive demyelination of the increased up to 100 g/day as symptoms improve.
white matter of the brain and spinal cord. (3) Neomycin is administered to remove
(5) CORRECT: Electromyography helps differentiate ammoniaproducing
nerve disorders from muscle disorders. substances from the GI tract and suppress
8. The Answer is 2 bacterial ammonia production.
A middle-aged female client with a history of (4) Potassium supplements are administered to help
atherosclerosis correct alkalosis from increased ammonia levels.
is admitted with complaints of abdominal 11. The Answer is 3 and 4
tenderness during deep palpation. The nurse notices The laboratory values of an adult male client reveal
a pulsating mass in the periumbilical area. Which of the presence of hepatitis B surface antigens and
the following does the nurse suspect? hepatitis B antibodies. Which of the following
Category: System specific assessments; Potential for laboratory
complications of diagnostic results should the nurse also expect to see?
tests/treatments/procedures; Select all that apply.
Potential for complications from surgical Category: Laboratory values
procedures and health alterations (1) In viral hepatitis, serum albumin levels are low.
(1) Signs of appendicitis include loss of appetite, (2) In viral hepatitis, serum globulin levels are high.
nausea, vomiting, fever, board-like abdominal (3) CORRECT: In viral hepatitis, serum transaminate
rigidity, and increasingly severe abdominal levels are elevated.
spasm. (4) CORRECT: In viral hepatitis, prothrombin time
(2) CORRECT: Signs of abdominal aortic aneurysm is prolonged.
include asymptomatic pulsating mass in (5) In viral hepatitis, urine bilirubin levels are
the periumbilical area, possible systolic bruit elevated.
over the aorta on auscultation, possible abdominal 12. The Answer is 1
tenderness on deep palpation, and lumbar The nurse is assessing a client with Addison’s disease.
pain that radiates to the flank and groin (imminent The nurse expects to note which of the following?
rupture). Category: System specific assessments; Potential for
(3) Signs of acute cholecystitis include midepigastric alterations in body systems
or right upper quadrant pain radiating to the (1) CORRECT: Anorexia is associated with Addison’s
back or referred to the right scapula. disease.
(4) Signs of paralytic ileus include severe abdominal (2) Weight loss, not weight gain, is a sign of Addison’s
distention, vomiting, and severe constipation. disease.
10: Physiological Integrity: Reduction of Risk (3) Bronze skin coloration (not yellow skin coloration)
Potential is a sign of Addison’s disease.
255 (4) A craving for salty foods (not sweets) is a sign of
9. The Answer is 1 Addison’s disease.
13. The Answer is 2 (ALS).
A client is having a tonic-clonic seizure. Which of (2) Unilateral facial weakness and drooping mouth
the following should the nurse do FIRST? are signs of Bell’s palsy.
Category: Potential for complications of diagnostic (3) CORRECT: The cause of Raynaud’s disease is
tests/treatments/procedures; Therapeutic procedures unknown; however, after exposure to cold or
(1) This is not the first thing the nurse should do. stress, the client typically experiences blanching
When the seizure stops, the nurse should check of the skin at the fingertips and numbness and
for breathing and, if necessary, initiate rescue tingling of the fingers.
breathing. (4) Photophobia is not a symptom of Raynaud’s
(2) CORRECT: The nurse’s first priority during a disease.
seizure is to protect the client from injury. To do 17. The Answer is 2
this, the nurse must first remove objects from The physician orders a CT scan of the client’s chest
the surroundings and pad objects that cannot be with IV contrast. Which of the following findings
removed. in the client’s history should the nurse report to the
(3) Placing an object in the client’s mouth can cause physician?
injury. Category: Potential for complications of diagnostic
(4) Restraining the client can cause injury. tests/treatments/procedures
NCLEX-RN® Exam Content Review and Practice (1) Hypertension is not a contraindication for a CT
256 scan with IV contrast.
14. The Answer is 2 (2) CORRECT: A client with an allergy to iodine
A client is recovering from a bout with chronic or shellfish may have an adverse reaction to the
glomerulonephritis. contrast medium.
The nurse prepares the client for (3) A UTI is not a contraindication for a CT scan
discharge and home management. Which of the with IV contrast.
following (4) An allergy to penicillin is not a contraindication
statements indicates the client understands for a CT scan with IV contrast.
his condition and how to control it? 18. The Answer is 2
Category: Therapeutic procedures The oncologist examines a client in the clinic and
(1) The nurse should teach the client to take his subsequently admits the client to the hospital with
prescribed severe bone marrow depression. The client’s therapy
antihypertensive medication as scheduled, included radiation and chemotherapy. Which of the
even if he’s feeling better. following nursing diagnoses takes priority in the
(2) CORRECT: The nurse should teach the client to client’s
take diuretics, such as furosemide (Lasix), in the care plan?
morning so that sleep won’t be disrupted by the Category: Potential for complications of diagnostic
need to void. tests/treatments/procedures
(3) The nurse should teach the client to report any (1) Imbalanced nutrition is a health-threatening but
adverse side effects and encourage the client to not life-threatening problem.
express his feelings to help him adjust to this illness. 10: Physiological Integrity: Reduction of Risk
(4) The nurse should urge the client to schedule Potential
follow- 257
up examinations to assess renal function. (2) CORRECT: Because clients with bone marrow
15. The Answer is 4 depression have a decrease in white blood cells—
A nursing home client is admitted to the hospital those cells that fight infection—risk for infection
with a pressure ulcer involving full-thickness loss takes priority. Nursing diagnoses should be
extending to the bone. The nurse documents the categorized
pressure ulcer as being at which of the following in order of priority, with life-threatening
stages? problems addressed first, followed by
Category: System specific assessments; Potential for healththreatening
complications of diagnostic concerns.
tests/treatments/procedures (3) Pain may be a health-threatening problem, but
(1) In Stage I, the skin is intact with non-blanchable typically is not life-threatening.
redness over a localized area. (4) Risk for injury is a health-threatening but not
(2) Stage II involves partial-thickness loss of the life-threatening problem.
dermis. 19. The Answer is 2, 4, and 5
(3) Stage III involves full-thickness loss, but bone, The nurse is preparing to discharge a client who is
tendon, and muscle are not exposed. stable after a sickle-cell anemia crisis. Which of the
(4) CORRECT: Stage IV involves full-thickness loss following instructions should the nurse provide to
with exposed bone, tendon, and muscle. the client to avoid future crises? Select all that
16. The Answer is 3 apply.
A client with Raynaud’s disease is experiencing an Category: Therapeutic procedures
acute attack. The nurse should anticipate which of (1) Clients should maintain a high fluid intake to
the following assessment findings? prevent dehydration.
Category: System specific assessments (2) CORRECT: Sickle-cell anemia clients should
(1) Involuntary muscle contractions and twitching avoid strenuous exercise, which could provoke
may be signs of amyotrophic lateral sclerosis hypoxia.
(3) Clients should apply warm compresses to painful or blood components is 0.9% sodium chloride
areas. (normal saline solution).
(4) CORRECT: Sickle-cell anemia clients should 24. The Answer is 2
take pain medications as ordered to provide The nurse is providing discharge teaching to a client
effective pain management. stabilized after an acute attack of primary gout.
(5) CORRECT: Sickle-cell anemia clients should Which of the following foods should the nurse
avoid tight clothing that restricts circulation. instruct the client to avoid to prevent future attacks?
20. The Answer is 1 Category: Therapeutic procedures
The clinic nurse is updating the medications being (1) Cauliflower, asparagus, and mushrooms can be
taken by an anxious middle-aged client, and sees consumed unless contraindicated for a comorbid
that the physician prescribed an antidiuretic hormone. condition.
The nurse knows the medication has which (2) CORRECT: A client with gout should avoid
of the following effects on the kidneys? high-purine foods, such as anchovies, liver, sardines,
Category: Laboratory values and lentils.
(1) CORRECT: Antidiuretic hormone (ADH) is (3) Cherries, strawberries, and blueberries can be
produced by the pituitary gland, and acts in the consumed unless contraindicated for a comorbid
distal tubule and collecting ducts to increase condition.
water reabsorption and urine concentration. (4) Cereal, pasta, and rice can be consumed unless
(2) ADH deficiency decreases water reabsorption, contraindicated for a comorbid condition, such
causing dilute urine. as diabetes.
(3) Aldosterone, produced by the adrenal gland, 25. The Answer is 4
regulates sodium retention. In the emergency room, the nurse is caring for a client
(4) Aldosterone also controls potassium secretion. with complaints of substernal pain radiating to
21. The Answer is 1, 3, and 4 the arm and jaw, shortness of breath, and a feeling of
The nurse is performing an assessment on a client impending doom. The client had a stroke one month
who has developed cirrhosis. Which of the following ago. The client’s vital signs are blood pressure
signs and symptoms should the nurse expect to see? 146/72,
Select all that apply. pulse 128, and respirations 36. The 12-lead ECG
Category: System specific assessments; Potential for reveals evolving acute myocardial infarction (MI).
alterations in body systems Which of the following physician orders should the
(1) CORRECT: Signs and symptoms of cirrhosis nurse question?
include dull abdominal ache. Category: Changes/abnormalities in vital signs;
(2) Jaundice, not cyanosis, is a sign of cirrhosis. Potential for complications of diagnostic
(3) CORRECT: Signs and symptoms of cirrhosis tests/treatments/
include poor tissue turgor. procedures
(4) CORRECT: Signs and symptoms of cirrhosis (1) Beta-adrenergic blockers are an accepted
include bruises due to bleeding tendencies. treatment
(5) Musty breath, not fruity breath, is a sign of for evolving acute MI.
cirrhosis. (2) Morphine for pain is an accepted treatment for
22. The Answer is 2 evolving acute MI.
The physician orders 0.5 mg of digoxin for a client (3) IV nitroglycerin (in clients without hypotension
with atrial fibrillation. The pharmacy has 250-mcg or bradycardia) is an accepted treatment for
tablets available. How many tablets will the nurse evolving acute MI.
give? (4) CORRECT: Thrombolytic therapy is
Category: Potential for complications of diagnostic contraindicated
tests/treatments/procedures in clients with a history of recent stroke
Answer: The nurse will give 2 tablets, arrived at as (within the past 2 months).
follows: 500 mcg (=0.5 mg) divided by 250 = 2 26. The Answer is 2
23. The Answer is 3 An older adult female, newly diagnosed with type 2
The nurse is preparing to administer a red blood cell diabetes, is ready for discharge. When providing
transfusion to a client with a low hemoglobin level discharge
and low hematocrit. The nurse knows which of the instructions, the nurse teaches the client that
following statements about blood transfusion practice the key to preventing diabetic foot complications is
is true? which of the following?
Category: Diagnostic tests; Potential for complications Category: Therapeutic procedures
of diagnostic tests/treatments/procedures (1) Although taking medication as ordered is
NCLEX-RN® Exam Content Review and Practice important,
258 following the recommended diet is key to
(1) The client should be monitored for at least 15 preventing diabetic foot complications.
minutes after the start of the transfusion. (2) CORRECT: Following the recommended diet is
(2) The transfusion needs to be completed within 4 key to preventing diabetic foot complications.
hours, not 2 hours. (3) Surgical intervention may be a choice in
(3) CORRECT: The transfusion should be started preventing
within 30 minutes of removing the blood or further complications.
blood components from the blood bank. (4) Although any foot problems should be evaluated,
(4) The only solution that should be added to blood following the recommended diet is key to
preventing diabetic foot complications. tests/treatments/procedures
27. The Answer is 3 (1) The third step is to make a fist with one hand.
The nurse knows that the physician is most likely (2) The second step is to stand behind the client.
to order which of the following laboratory tests to (3) The fifth step is to wrap your other arm around
evaluate a client for hypoxia? the client and place that hand on top of your fist.
Category: Laboratory values (4) The fourth step is to place your thumb toward
(1) Hematocrit does not assess gas exchange. the client, below the rib cage and above the waist,
(2) Sputum analysis helps diagnose respiratory and wrap one arm around the client.
infection. (5) The first step is to ask the client if he is choking.
(3) CORRECT: Hypoxia deprives the body of adequate (6) The last step is to thrust upward 6–10 times.
oxygen supply. ABGs assess respiratory
status by helping to evaluate gas exchange in the 261
lungs. Physiological adaptation involves managing and
(4) Total hemoglobin does not assess gas exchange. providing care for clients who may have a
10: Physiological Integrity: Reduction of Risk variety of acute, chronic, or life-threatening health
Potential conditions. To provide the proper care,
259 you need to understand the client’s stable/normal
28. The Answer is 1 state (homeostasis), understand the internal
The nurse is performing a 12-lead ECG on a client and external factors that can influence or change it,
who has come to the emergency room complaining and know how to help the client
of chest pain. Where should the nurse place lead V1? return to a stable state.
A Understanding “normal” involves knowing the basics
B about all bodily systems, and knowing
C about the fluids and chemicals that keep the body
D functioning properly. In addition to
Category: Diagnostic tests knowing what a normal state looks like—in general
(1) CORRECT: Location A is correct. The V1 lead and for each client—you also must know
is placed at the fourth intercostal space to the proper fluid and electrolyte balances and pH balance
right of the sternum. A 12-lead ECG measures (water, sodium, potassium, calcium,
electrical potential and helps make a definitive magnesium, chloride, etc.). This is also a good time
diagnosis of acute myocardial infarction. The six to remember the six elements of infection:
precordial leads—V1-V6—in combination with 1. Susceptible host
other leads, record potential in the horizontal 2. Portal of entry
plane. 3. Cause
(2) Location B is incorrect for the V1 lead. 4. Reservoir
(3) Location C is incorrect for the V1 lead. 5. Portal of exit
(4) Location D is incorrect for the V1 lead. 6. Modes of transmission
29. The Answer is 1 It’s important to know how to protect yourself and
The nurse is assessing a client admitted with a your client from infection and understand
cerebrovascular how to intervene to break the chain of infection. When
accident (CVA). The physician has a client is ill or injured, his or her
ordered a swallow study. The nurse knows which body cannot respond quickly enough to internal or
of the following lobes of the cerebral hemisphere is external events. Between your powers of
involved in the control of voluntary muscle movement, observation and your understanding of
including those necessary for the production pathophysiology, you should know how to determine
of speech and swallowing? whether there is a problem, identify the problem, and
Category: System specific assessments respond appropriately. This includes
(1) CORRECT: The frontal lobe deals with higher recognizing which body systems can be affected by
levels of cognitive functions, such as reasoning the client’s condition, being aware of each
and judgment. It also contains several cortical client’s usual baselines and preexisting conditions,
areas involved in the control of voluntary muscle and incorporating that information to
movement, including those necessary for the determine which care measures to try and whether
production of speech and swallowing. they are effective.
(2) The parietal lobe is associated with sensation, PHysiological Integrity:
and is involved in writing and some aspects of Physiological Adaptation
reading. chapter 11
(3) The temporal lobe is associated with auditory 262
processing, olfaction, and word meaning. NCLEX-RN® Exam Content Review and Practice
(4) The occipital lobe is involved in vision. On the NCLEX-RN® exam, you can expect
30. The Answer is 5, 2, 1, 4, 3, 6 approximately 13 percent of the questions to
The nurse is preparing to do the Heimlich maneuver relate to Physiological Adaptation. Exam content for
on a choking middle-aged adult male client. Arrange this subcategory includes, but is not
the following steps in the order the nurse should limited to, the following areas:
perform • Alterations in body systems
them. All options must be used. • Fluid and electrolyte imbalances
Category: Potential for complications of diagnostic • Hemodynamics
• Illness management the chemical regulation of fluid and electrolyte
• Medical emergencies balances (hormones and peptides).
• Pathophysiology One of the most important elements is being able to
• Unexpected response to therapies identify the signs and symptoms of fluid
Now let’s review some of the most important concepts or electrolyte imbalance in a client. In terms of fluids,
related to these subtopics. this means identifying both dehydration
Alterations in Body Systems and edema, knowing how to treat each one, and being
Clients can experience a variety of alterations in body able to teach the client how to
systems when they are ill. You should prevent recurrence.
be able to monitor and assess these changes, and For example, a dehydrated client needs fluids, with no
implement and explain appropriate interventions sugar, salt, or caffeine. If the client can
to clients. take fluids orally, they should be delivered that way,
You should understand the most common therapeutic but you should know when parenteral
activities, which include: (IV) therapy is the right choice. Clients may also
• Assessing tube drainage when a client has an retain excess fluid; risk factors include age,
alteration in a body system (e.g., whether surgery, cardiac or renal failure, and medications. A
the amount of fluid increased or decreased, whether client with excess fluid should have fluid
the color of the fluid changed) intake limited, protein intake increased, and excretion
• Monitoring and maintaining a client on a ventilator promoted, and should be carefully
• Maintaining desired temperature using external monitored for overcorrection.
devices Implement interventions to restore client fluid and/or
• Implementing and monitoring phototherapy electrolyte balance. Common electrolyte
• Providing ostomy care imbalances include the following:
• Providing care to clients who have experienced a • Hyponatremia and hypernatremia (sodium)
seizure • Hypokalemia and hyperkalemia (potassium)
• Assisting with invasive procedures (central line • Hypocalcemia and hypercalcemia (calcium)
placement, biopsy, debridement) • Hypomagnesemia and hypermagnesemia
• Performing peritoneal dialysis (magnesium)
• Providing pulmonary hygiene (chest physiotherapy, • Hypochloremia and hyperchloremia (chlorine)
spirometry) • Hypophosphatemia and hyperphosphatemia
• Performing oral nasopharyngeal suctioning (phosphates)
• Suctioning via an endotracheal or a tracheostomy 264
tube NCLEX-RN® Exam Content Review and Practice
• Performing tracheostomy care Hemodynamics
• Providing care for clients experiencing increased Your responsibility in hemodynamic monitoring is to
intracranial pressure position the transducer at the level of
Providing wound care includes assisting in or the right atrium, level central venous pressure (CVP)
performing dressing changes and removal of of the pulmonary artery catheter transducer
sutures or staples, monitoring wounds for signs and into this point during each shift and before each
symptoms of infection, and promoting measurement, and maintain patency
263 of the catheter with a constant small amount of fluid
11: Physiological Integrity: Physiological delivered under pressure.
Adaptation Assess clients for decreased cardiac output, and
client wound healing through turning, hydration, identify cardiac rhythm strip abnormalities,
nutrition, and skin care. In surgical cases, such as sinus bradycardia, premature ventricular
wound care may also include monitoring and contractions, ventricular tachycardia, and
maintaining devices and equipment used for fibrillation.
drainage, such as chest tube suction. Monitor and maintain arterial lines, and connect and
It is important to identify signs of potential prenatal maintain pacing devices, including
complications, and to provide care for pacemakers, biventricular pacemakers, and
clients experiencing complications from pregnancy, implantable cardioverter defibrillators. You
labor, and delivery (such as eclampsia, should also be able to initiate, maintain, and evaluate
precipitous labor, or hemorrhage). You should also be telemetry monitoring.
able to assess a client’s response to In addition to monitoring pacemakers and
surgery and provide postoperative care. defibrillators, you should be able to intervene to
In a more general sense, you should be able to improve client cardiovascular status through
educate clients about managing their health modifying an activity schedule and initiating a
problem, whether it’s a chronic illness such as protocol to manage cardiac arrhythmias.
diabetes or appropriate post-stroke care. Your You should be able to provide care for clients with
efforts should promote progress toward recovery, and vascular access for hemodialysis, such as
you should be able to evaluate whether via an arteriovenous shunt, a fistula, or a graft.
the client has successfully achieved treatment goals. Finally, apply your knowledge of pathophysiology to
Fluid and Electrolyte Imbalances interventions in response to client
It is important to understand the concepts of fluid abnormal hemodynamics, and provide clients with
transport, capillary fluid movement, and strategies to manage decreased cardiac
output, such as frequent rest periods and limiting 266
activities. NCLE X-1. The nurse has just completed setting up
Illness Management an
In addition to recognizing symptoms and helping to external warming device (Bear Hugger) for
identify client health issues, it is important a 48-year-old client and is ready to initiate
to implement interventions that help a client manage therapy. The core temperature taken with
recovery from an illness. This a rectal probe is currently 91.4° F (33° C).
includes applying your knowledge of each client’s Which of the following actions should the
pathophysiology when determining which nurse perform?
interventions are best. 1. Active rewarming to increase the core
When examining a client who is ill, examine and temperature no more than 0.9° F (0.5° C)
interpret data and know what information per hour
should be reported to the physician immediately. This 2. Active rewarming to increase the core
means knowing a particular client’s temperature as quickly as possible
baseline values, identifying abnormal values or test 3. Active rewarming to increase the core
results, and identifying critical values. temperature to 96.8° F (36° C)
You play an important role in teaching clients how to 4. Active rewarming to increase the core
manage their illnesses, so communicate temperature to 100.4° F (38° C)
appropriate and helpful information to clients with 2. The nurse is cleansing a simple surgical
infectious illnesses such as AIDS, as well wound. The client is two days postoperative,
as chronic conditions such as asthma and diabetes. and the incision has well-approximated edges
Evaluate and document client response with no sign of infection. A Jackson-Pratt
to interventions, and promote continuity of care in drain is adjacent to the incision site. Which
illness management activities. of the following should the nurse do?
In terms of specific care, you should be able to 1. Cleanse the incision and drain sites while
perform gastric lavage and to administer wearing standard clean gloves.
oxygen therapy and evaluate client response. 2. Cleanse in a back-and-forth motion
265 across the incision line and in a circular
11: Physiological Integrity: Physiological motion around the drain site.
Adaptation 3. Cleanse the incision site and drain site
Medical Emergencies together.
When a client appears to be experiencing a medical 4. Cleanse the incision and drain sites using
emergency, you should know how best a sterile saline solution.
to intervene. Although things are likely to happen very 3. The nurse is emptying an evacuator of a
quickly, you should also be able to Jackson-Pratt drain. The nurse has drained
explain emergency interventions to the client, despite the fluid into a calibrated container and has
being in a pressure situation. placed the container on a level flat surface.
You should be able to perform a variety of emergency The nurse measures 20 mL of bloody fluid.
care procedures, including CPR, the Arrange the following actions the nurse
Heimlich maneuver (abdominal thrusts), respiratory should take in sequential order. All options
support, and use of an automated external must be used.
defibrillator. You should also know how to provide 1. Dispose of the bloody drainage.
emergency care for a wound disruption 2. Compress the evacuator completely.
(evisceration or dehiscence). You should also be able 3. Replace the plug in the evacuator.
to monitor and maintain a client 4. Cleanse the plug with an alcohol wipe.
on a ventilator. Once emergency procedures are 5. Document the amount, odor, and
implemented, evaluate and document client consistency of the drainage.
responses, such as restoration of breathing and return 4. The nurse in an outpatient clinic has received
to normal pulse rate. an order from the physician to remove the
Nurses are typically expected to notify the clinician client’s sutures. The nurse should do which
about unexpected responses and/or of the following?
emergency situations. 1. Use gloves when removing sutures.
Pathophysiology 2. Apply hydrogen peroxide gauze pads to
It is important to understand the general principles of cleanse the area first, then remove the
pathophysiology, including injury and sutures.
repair, immunity, and cellular structure. You should 3. Use sterile technique when removing
also be able to identify and determine a sutures.
client’s health status based on pathophysiology. 4. Nothing, suture removal is outside of the
Unexpected Response to Therapies nurse’s scope of practice.
Most clients will respond to therapies in a predictable 5. The medical floor nurse receives report from
manner, but some will not. Assess the Emergency Department on a 42-yearold
clients for unexpected adverse responses to therapy client who is admitted to the hospital
(e.g., increased intracranial pressure or for hyperphosphatemia related to end-stage
hemorrhage) and know how to intervene to renal disease. The client receives continuous
counteract such complications. ambulatory peritoneal dialysis (CAPD), and
NCLEX-RN ® Exam Content Review and Practice the physician has ordered continuation of
treatment during hospitalization. The nurse the 8-hour shift. The client drank 5 oz.
should do which of the following? of juice at breakfast, 2 oz. of water with
Chapter Quiz medications, 8 oz. of soup at lunch, and 6
11: Physiological Integrity: Physiological oz. of milk with lunch. Intravenous fluids,
Adaptation flushes, and intravenous antibiotics for the
267 shift were 400 mL. Urinary output was 300
1. Maintain a permanent peritoneal catheter mL, 100 mL, and 250 mL. What should the
with flushes of 0.9% normal saline (0.9% nurse document, in milliliters, as the total
NS) every 4–6 hours. fluid intake for the shift?
2. Obtain a pump in preparation for mL
dialysate infusion. 10. A 70-year-old male presented to the
3. Ensure the dialysate is refrigerated until Emergency Department with shortness of
ready to infuse, and obtain a warming breath, crackles in the bases and middle
pad or a warming machine to warm the of the lung fields bilaterally, +2 pitting
dialysate to body temperature prior to edema bilaterally of the lower extremities,
exchange. and a weight increase of 6 lb. in one
4. Weigh the client at the same time every week. His heart rate is 82 and his blood
day, and use sterile technique while pressure is 162/90. Per physician’s order,
working with a permanent peritoneal the nurse administers 40 mg of furosemide
catheter. intravenously. The nurse knows that which
6. An 8-year-old girl is discharged from the of the following indicates effectiveness of
hospital with a new tracheostomy. The the medication?
parents have received initial teaching in NCLEX-RN® Exam Content Review and Practice
the hospital, and the home health nurse 268
will reinforce this teaching. Per report, the 1. A heart rate of 58
parents are willing to learn and are grasping 2. A blood pressure of 100/52
the concepts well. The home health nurse 3. Urine output increase of 200 mL over
would expect the parents to verbalize and the next hour
demonstrate which of the following? 4. Diminished lung sounds bilaterally with
1. “The cleansing and dressing of the stoma crackles in the bases
will be done at least every 24 hours.” 11. The nurse takes report on a client returning
2. “It is not always necessary to suction from left-sided cardiac catheterization.
before tracheostomy care.” The client also underwent a percutaneous
3. “The inner cannula should be changed by transluminal coronary angioplasty (PTCA),
the physician or home health nurse.” with drug-eluding stents placed in the right
4. “Hydrogen peroxide is used to cleanse the coronary artery and left coronary artery,
stoma area.” and the site was closed with a collagen plug.
7. The nurse is preparing to suction a client The nurse would expect to assess the entry
with an endotracheal tube. After ventilating, site on the client at which of the following
which is the correct sequence of actions for locations?
the nurse to follow during suctioning? A
1. Apply suction, insert a sterile catheter, B
and withdraw while rotating the catheter. C
2. Insert a sterile catheter, begin to D
withdraw, apply suction, and continue to 1. A
withdraw while rotating the catheter. 2. B
3. Apply suction, insert a sterile catheter, and 3. C
withdraw without rotating the catheter. 4. D
4. Insert a sterile catheter, begin to 12. The progressive care unit nurse is assessing
withdraw, apply suction, and continue to the following cardiac rhythm. Using the
withdraw without rotating the catheter. following exhibit, the nurse should identify
8. The nurse assesses a client with a diagnosis this rhythm as which of the following?
of parathyroid disease. The client is having 1. Atrial fibrillation
abdominal cramping, positive Chovstek’s 2. Atrial flutter
and Trousseau’s signs, and tingling in the 3. Ventricular fibrillation
extremities. The nurse knows that these 4. Third-degree atrioventricular block
findings could be signs and symptoms of 13. The critical care nurse is caring for a client
which of the following? with an arterial line (A-line). The nurse can
1. Hypermagnesemia utilize this line for which of the following?
2. Hypomagnesemia 1. Monitoring blood pressure and heart
3. Hypercalcemia rate, and infusing medications
4. Hypocalcemia 2. Monitoring blood pressure and heart
9. The physician has ordered a 2-L daily rate, and obtaining blood gases and
fluid restriction for a client diagnosed other laboratory samples
with congestive heart failure. The nurse 3. Monitoring heart rate, obtaining blood
is totaling the client’s fluid intake for gases and other laboratory samples, and
infusing medications client on the floor during rounds. The client
4. Obtaining blood gases and other is not responsive. Vital signs have been
laboratory samples, and infusing taken by the certified nursing assistant:
medications blood pressure 98/52, heart rate 120,
14. An 82-year-old woman is admitted with a respirations 28, and oxygen saturation
diagnosis of rapid atrial fibrillation. The 94%. The client has a history of falls,
nurse has initiated telemetry monitoring hypertension, and an extensive cardiac
per the physician’s order. Two hours after history. The client’s chart indicates a
initiation of monitoring, an alarm sounds signed physician order that states “Do not
at the central monitoring station: the resuscitate” and “Do not intubate” (DNR/
client is in what appears to be ventricular DNI). Which of the following should the
tachycardia. Which of the following actions nurse do?
should the nurse take FIRST? 1. Stay with the client and have another
1. Call a code blue. staff member call 911.
2. Silence the alarm and change the alarm 2. Begin CPR and have another staff
parameters. member call 911.
3. Notify the physician of a change in 3. Move the client into the bed and call the
rhythm. physician.
4. Assess the client and check lead 4. Call the family and ask what they would
placement. like to have done for the client.
11: Physiological Integrity: Physiological 19. The surgical floor nurse is working with a
Adaptation client on coughing and deep breathing. The
269 mildly obese client is six days postoperative,
15. A 39-year-old client has been diagnosed and has a large midline abdominal incision
with end-stage renal disease and is on the that is not well approximated. The client
transplant waiting list. The client has been stops the exercise and states she felt a
receiving dialysis through a subclavian popping sensation in her abdominal area.
central vein catheter while an arteriovenous Upon assessment, the nurse finds a small
fistula is maturing. Besides dialysis access, portion of the viscera to be protruding
the surgical floor nurse can utilize this through the incision. Which of the
subclavian central vein catheter for which of following actions should the nurse take
the following? FIRST?
1. Nothing 1. Do nothing; this is a normal finding for
2. Blood draws only a large midline abdominal incision.
3. Infusion of normal saline (0.9% NS) and 2. Call the surgeon who operated on the
obtaining blood draws client and inform the physician of the
4. Infusion of medications, all intravenous finding.
fluids, and obtaining blood draws NCLEX-RN® Exam Content Review and Practice
16. A 76-year-old man is brought into the 270
Emergency Department by his spouse. The 3. Place sterile dressings moistened with
client’s spouse tells the nurse he is confused, sterile normal saline (0.9% NS) over the
disoriented, and weak, and has not been viscera and hold in place with a sterile
eating well. The nurse obtains blood work gloved hand.
as ordered by the physician, including 4. Place an abdominal binder on the area,
a complete blood count (CBC) and a elevate the head of the bed no more than
comprehensive metabolic panel (CMP). For 20 degrees, and have the client recline
which result should the nurse immediately with her knees bent.
notify the physician? 20. The intensive care nurse is caring for a
1. Potassium (K+) 3.8 mEq/L client requiring mechanical ventilation.
2. Sodium (Na) 122 mEq/L Which of the following are interventions the
3. Magnesium (Mag+) 1.9 mg/dL nurse should take to help prevent ventilatorassociated
4. Hemoglobin (Hgb) 12 g/dL pneumonia (VAP)? Select all that
17. The nurse is providing discharge apply.
instructions to a client going home on 1. Reposition the client at least every 2
enoxaparin. Which of the following hours and maintain the head of the bed
responses by the client indicates to the upright at 30–45 degrees.
nurse that the teaching was effective? 2. Promote nutrition with the use of a
1. “Prior to injection, I will rub the site nasogastric tube and high-calorie
with an alcohol wipe.” feedings.
2. “I will use the same site for each 3. Suction oral and pharynx secretions,
injection.” and provide thorough oral care at least
3. “I will not pull back the plunger after every 2 hours.
inserting the needle into the site.” 4. Assess the client for sedation reduction
4. “After injection, I will massage the site and weaning/extubation readiness.
to increase absorption.” 5. Perform hand hygiene before and
18. The nursing home nurse finds a 92-year-old after care of the client, and implement
prophylactic intravenous antibiotic 4. Administer PRN pain medication as
therapy. ordered, apply oxygen at 2 L/min, and
21. The night nurse on a medical floor has just provide an additional blanket.
received report. On which of the following 25. The nurse receives report on a client with a
clients should the nurse make rounds right total knee arthroplasty who developed
FIRST? methicillin-resistant Staphylococcus aureus
1. The 52-year-old female with pancreatitis (MRSA) in the surgical incision. The
who is experiencing abdominal pain incision was cultured and showed sensitivity
rated 4 on a 1–10 scale to vancomycin. The client’s blood urea
2. The 70-year-old male who underwent a nitrogen (BUN) is 14 mg/dL and serum
transurethral resection of the prostate creatinine (Cr) is 0.9 mg/dL. Intake and
(TURP) yesterday and is having a output are balanced. A peak and trough
burning sensation during urination have been ordered. The third dose of
3. The 78-year-old male with diagnosis vancomycin is to be given on the nurse’s
of left-sided heart failure who has shift. The nurse should do which of the
developed a new nonproductive cough following?
and is restless 1. Draw a trough 30 minutes prior to
4. The 37-year-old female diagnosed with dose and draw a peak 60 minutes after
cellulitis of the left leg yesterday who is infusion.
experiencing redness and warmth of the 2. Draw a peak 30 minutes prior to dose
left leg and draw a trough 60 minutes after
22. The nurse knows which of the following infusion.
body systems is responsible for the 3. Hold the dose of vancomycin and notify
production of erythropoietin? the physician of the BUN and Cr levels.
1. Urinary system 4. Give the dose of vancomycin as ordered
2. Cardiovascular system and draw the peak and trough with
3. Lymphatic system other evening labs.
4. Endocrine system NCLEX-RN ® Exam Content Review and Practice
23. The nurse is initiating cefazolin therapy 272
following a physician’s order. The nurse NCLE X-1. The Answer is 1
notes that the client has an allergy to The nurse has just completed setting up an external
penicillin. The client states he becomes warming device (Bear Hugger) for a 48-year-old client
a little short of breath and itches after and is ready to initiate therapy. The core temperature
receiving penicillin. The nurse should do taken with a rectal probe is currently 91.4°
which of the following? F (33° C). Which of the following actions should the
1. Call the pharmacy to therapeutically nurse perform?
change the medication and notify the Category: Alterations in body systems
physician of this change. (1) CORRECT: The client is in moderate hypothermia
2. Hold the medication and call the with a core temperature of 91.4° F (33° C).
physician to double-check the order. This would indicate the need for an external
3. Give the medication as ordered— warming device and other measures to increase
cefazolin is not a penicillin. core temperature.
4. After asking another nurse, give the (2) The core temperature should be brought up by
medication as ordered. no more than 0.9° F (0.5° C) per hour for treatment
11: Physiological Integrity: Physiological of moderate hypothermia.
Adaptation (3) Active rewarming would be discontinued when
271 the core temperature is greater than 95° F (35° C)
24. A 52-year-old woman is admitted with to prevent hyperthermia.
a new diagnosis of gastrointestinal (GI) (4) Active rewarming would be discontinued when
bleed. The physician has ordered the client the core temperature is greater than 95° F (35° C)
to receive 2 units of packed red blood cells to prevent hyperthermia.
(PRBCs) for a hemoglobin (Hgb) of 6.8 g/ 2. The Answer is 4
dL. The nurse begins the infusion of the The nurse is cleansing a simple surgical wound. The
first unit at 100 mL/hr. Fifteen minutes client is two days postoperative, and the incision has
after the start of the infusion, the client well-approximated edges with no sign of infection. A
complains that she is feeling chilled, is Jackson-Pratt drain is adjacent to the incision site.
short of breath, and is experiencing lumbar Which of the following should the nurse do?
pain rated 8 on a 1–10 scale. Which of the Category: Alterations in body systems
following should be the nurse’s FIRST (1) The nurse should use a sterile/aseptic technique;
action? standard clean gloves are not sufficient.
1. Obtain vital signs and notify the (2) The nurse will use a small circular motion along
physician of potential reaction. the wound edges, but cleanse from one end of the
2. Slow the infusion to 75 mL/hr and incision to the other. This is to prevent contamination
reassess in 15 minutes. and trauma to the wound.
3. Stop the infusion and run normal saline (3) The drain site should be cleansed last, separately
(NS) to keep the vein open (KVO). from the primary incision site, to prevent the risk
of cross-contamination. peritoneal catheter with normal saline solution.
(4) CORRECT: The nurse will use a sterile/aseptic (2) The dialysate bag is raised to shoulder level and
technique while cleansing and dressing the is infused by gravity into the peritoneal cavity
wound, including using sterile gloves, cottontipped after the dwell dialysate solution is drained.
applicators, sterile saline, and sterile (3) Dialysate for CAPD is not refrigerated but
dressings. should be warmed to body temperature prior to
3. The Answer is 4, 2, 3, 1, 5 infusion, if a warmer is available. Never use a
The nurse is emptying an evacuator of a Jackson- microwave to warm the dialysate; this method
Pratt drain. The nurse has drained the fluid into a creates an unpredictable temperature.
calibrated container and has placed the container (4) CORRECT: The nurse would weigh the client at
on a level flat surface. The nurse measures 20 mL the same time daily. The nurse would use sterile
of bloody fluid. Arrange the following actions the technique and equipment when working with the
nurse should take in sequential order. All options peritoneal catheter to infuse and drain the dialysate,
must be used. including having the client and nurse wear a
Category: Alterations in body systems surgical mask while the peritoneal catheter and
(1) The fourth thing the nurse would do is dispose of hub are exposed.
the bloody drainage, typically into a toilet. This 6. The Answer is 4
would be done after closing the Jackson-Pratt An 8-year-old girl is discharged from the hospital
drain to potentially infective agents and after with a new tracheostomy. The parents have received
resuming negative pressure (suction). initial teaching in the hospital, and the home health
(2) The second thing the nurse would do is compress nurse will reinforce this teaching. Per report, the
the evacuator completely. parents are willing to learn and are grasping the
(3) The third thing the nurse would do is replace concepts well. The home health nurse would expect
the plug while the evacuator is compressed. the parents to verbalize and demonstrate which of
This creates a negative pressure as the evacuator the following?
expands. Category: Alterations in body systems
(4) The first thing the nurse would do is cleanse the (1) The stoma should be cleaned and dressed at
plug with an alcohol wipe. This is to reduce the least every 8 hours and the ties every 24 hours;
risk of infection. cleaning and dressing changes may be done more
(5) The last thing the nurse would do is document frequently to keep the dressing and ties dry to
the process and drainage, including amount, prevent infection.
consistency, color, odor, date, time, and client’s (2) Suctioning the trachea and pharynx thoroughly
tolerance of the procedure. before tracheostomy care keeps the area clean
4. The Answer is 3 longer.
The nurse in an outpatient clinic has received an (3) The inner cannula can be cleaned and changed
order from the physician to remove the client’s by the parents, and should be removed and
sutures. The nurse should do which of the following? cleaned at least every 8 hours.
Category: Alterations in body systems (4) CORRECT: Hydrogen peroxide-soaked gauze
Chapter Quiz Answers and Explanations pads or cotton-tipped applicators are used to
11: Physiological Integrity: Physiological clean the stoma area, followed by the use of sterile
Adaptation water-soaked gauze pads and cotton-tipped
273 applicators to remove the hydrogen peroxide.
(1) To prevent incision contamination, this is should The stoma area would then be dried using sterile
be a sterile procedure. Wearing regular gloves is gauze pads to reduce the risk of infection and
not sufficient. irritation.
(2) To prevent incision contamination, this should 7. The Answer is 2
be a sterile procedure. This answer choice does The nurse is preparing to suction a client with an
not provide enough information to determine if endotracheal tube. After ventilating, which is the
proper sterile procedures are being followed. correct sequence of actions for the nurse to follow
(3) CORRECT: A sterile field is maintained, a sterile during suctioning?
suture removal tray is used, and sterile gloves Category: Alterations in body systems
are applied. (1) Suctioning on insertion unnecessarily decreases
(4) In many facilities, nurses do remove sutures and oxygen in the airway. Most clients will cough
staples following a physician’s order. when the suction catheter touches the carina.
5. The Answer is 4 (2) CORRECT: The nurse would ventilate the client
The medical floor nurse receives report from the and insert the sterile catheter without applying
Emergency Department on a 42-year-old client who suction. The nurse would then withdraw the
is admitted to the hospital for hyperphosphatemia catheter about 1 inch and apply suction while
related to end-stage renal disease. The client receives rotating the catheter.
continuous ambulatory peritoneal dialysis (CAPD), (3) Suctioning on insertion unnecessarily decreases
and the physician has ordered continuation of oxygen in the airway. Most clients will cough
treatment when the suction catheter touches the carina.
during hospitalization. The nurse should do (4) Failure to withdraw and rotate the catheter may
which of the following? result in damage to the tracheal mucosa.
Category: Alterations in body systems NCLEX-RN® Exam Content Review and Practice
(1) The nurse would not flush a CAPD permanent 274
8. The Answer is 4 10. The Answer is 3
The nurse assesses a client with a diagnosis of A 70-year-old male presented to the Emergency
parathyroid Department with shortness of breath, crackles in
disease. The client is having abdominal the bases and middle of the lung fields bilaterally,
cramping, positive Chovstek’s and Trousseau’s signs, +2 pitting edema bilaterally of the lower extremities,
and tingling in the extremities. The nurse knows that and a weight increase of 6 lb. in one week. His
these findings could be signs and symptoms of which heart rate is 82 and his blood pressure is 162/90. Per
of the following? physician’s order, the nurse administers 40 mg of
Category: Fluid and electrolyte imbalances furosemide intravenously. The nurse knows that
(1) A person with hypermagnesemia would not which of the following indicates effectiveness of the
exhibit all of these symptoms. medication?
(2) A person with hypomagnesemia might exhibit Category: Fluid and electrolyte imbalances
a positive Chovstek’s sign but not the rest of the (1) A heart rate of 58 could indicate a side effect of
symptoms listed. the
(3) A person with hypercalcemia would not exhibit medication rather than effectiveness. This significant
all of these symptoms. drop in heart rate would be cause for alarm,
(4) CORRECT: Hypocalcemia can be demonstrated especially after the administration of furosemide.
by abdominal cramping, tingling of the (2) A blood pressure of 100/52 could indicate a side
extremities, and tetany. Chovstek’s sign refers to effect of the medication rather than effectiveness.
an abnormal reaction to the stimulation of the This significant drop in blood pressure would be
facial nerve such that, when tapped at the masseter cause for alarm, especially after the administration
muscle, the facial muscles on the same side of of furosemide.
the face contract, causing a brief twitching of the (3) CORRECT: The nurse would expect an increase
nose or lips. Chovstek’s sign can be seen in in urine output after the administration of furosemide.
hypomagnesemia The client presented with signs and
and hypocalcemia. Trousseau’s sign symptoms of hypervolemia or fluid overload,
of latent tetany is more sensitive than Chovstek’s including shortness of breath, crackles in lung
sign in hypocalcemia, and may be positive before 11: Physiological Integrity: Physiological
gross manifestations of hypocalcemia, specifically Adaptation
tetany and hyperreflexia. A blood pressure 275
cuff is inflated to a pressure greater than the systolic bases, and edema. Weight gain and hypertension
pressure and held in place for 3 minutes. can also be indicative of hypervolemia. The goal
This causes the occlusion of the brachial artery, of treatment using furosemide is diuresis, with
and the hypocalcemia and subsequent neuromuscular care not to send the client into hypovolemia.
irritability will induce a muscle spasm (4) The nurse would expect to auscultate a reduction,
of the client’s hand and forearm. if not elimination, of crackles in the lung
9. The Answer is 1,030 mL bases. The nurse would also not expect diminished
The physician has ordered a 2-L daily fluid restriction lung sounds, because this could indicate
for a client diagnosed with congestive heart atelectasis and/or decreased air flow through the
failure. The nurse is totaling the client’s fluid intake lungs. The goal would be baseline or clear lung
for the 8-hour shift. The client drank 5 oz. of juice sounds bilaterally, with minimal to no crackles
at breakfast, 2 oz. of water with medications, 8 oz. in the lung bases upon auscultation.
of soup at lunch, and 6 oz. of milk with lunch. 11. The Answer is 2
Intravenous The nurse takes report on a client returning from
fluids, flushes, and intravenous antibiotics left-sided cardiac catheterization. The client also
for the shift were 400 mL. Urinary output was 300 underwent a percutaneous transluminal coronary
mL, 100 mL, and 250 mL. What should the nurse angioplasty (PTCA), with drug-eluding stents placed
document, in milliliters, as the total fluid intake for in the right coronary artery and left coronary artery,
the shift? and the site was closed with a collagen plug. The
Category: Fluid and electrolyte imbalances nurse would expect to assess the entry site on the
Answer: The answer can be calculated as follows: client at which of the following locations?
1 ounce is equal to 30 milliliters. Oral intake is 150 A
mL (because 5 oz. × 30 mL = 150 mL) + 60 mL B
(because 2 oz. × 30 mL = 60 mL) + 240 mL (because C
8 oz. × 30 mL = 240 mL) + 180 mL (because 6 oz. D
× 30 mL = 180 mL) to equal 630 mL of oral intake. Category: Hemodynamics
Add that to the intravenous fluids and antibiotics (1) A: This is not the correct location.
for a total intake of 1,030 mL (630 mL + 400 mL). (2) CORRECT: B is the correct answer. The nurse
Intake consists of oral intake, intravenous intake, would expect to assess the entry site in the left
intake through any feeding tube, intravenous blood femoral artery. This is the preferred site for leftsided
products, liquid medications, and flushes of any cardiac catheterization and PTCA.
tubes or IV accesses. The nurse would not include (3) C: This is not the correct location.
urinary output in intake totals, because that is (4) D: This is not the correct location.
totaled separately under output. Output includes 12. The Answer is 2
urine, diarrhea, emesis, wound drainage, and any The progressive care unit nurse is assessing the
gastric suction. following
cardiac rhythm. Using the following exhibit, (2) Verify alarm limits with the physician, and only
the nurse should identify this rhythm as which of the change parameters following an order from the
following? physician.
Category: Hemodynamics (3) The nurse would first check the lead wires and
(1) The atrial rhythm would be irregular in atrial assess the client to ensure that information given
fibrillation. Coarse, chaotic, asynchronous to the physician is accurate.
waves would be present, and the atrial and ventricular (4) CORRECT: Assess the client first, then the
rhythms would be grossly irregular and equipment for disconnections or malfunctions.
barely discernible with rates that vary. Check lead placement to determine if the monitoring
(2) CORRECT: This is an atrial flutter rhythm— results are indeed accurate, and not due to
there are no identifiable P waves, and characteristic interference or an artifact. If assessment of the
sawtooth flutter waves are present. The client reveals true ventricular tachycardia, follow
PR interval is not measurable, and the atrial rate advance directives as established by the client,
is regular and greater than the ventricular rate. including, but not limited to, calling a code.
This is expressed in a ratio; the example is 3:1, 15. The Answer is 1
because there are 3 atrial beats for every 1 ventricular A 39-year-old client has been diagnosed with
beat. The flutter waves should be able endstage
to be mapped across the rhythm strip. Flutter renal disease and is on the transplant wait list.
waves would mostly be visible with few occurring The client has been receiving dialysis through a
within the QRS and T waves. The subsequent subclavian
flutter waves would occur on time. central vein catheter while an arteriovenous
(3) Ventricular fibrillation (VF) is characterized by fistula is maturing. Besides dialysis access, the
atrial and ventricular rates and rhythms that cannot surgical
be determined, coarse and chaotic waves in floor nurse can utilize this subclavian central
coarse VF, and fine and chaotic waves in fine VF. vein catheter for which of the following?
(4) Third-degree atrioventricular (AV) block, also Category: Hemodynamics
known as complete heart block, is characterized (1) CORRECT: The nurse is not to access the
by a regular rhythm, an independent atrial rate subclavian
that is faster than the ventricular rate, an identifiable central vein catheter that is being used
P wave that is normal and occurring without for dialysis for blood draws, for infusions, or for
a QRS complex, and no PR interval. any reason other than dialysis. Only in the event
13. The Answer is 2 of a life-threatening emergency may the access
The critical care nurse is caring for a client with an be used for anything other than dialysis, and that
arterial line (A-line). The nurse can utilize this line is only under the physician’s direct order.
for which of the following? (2) Any other use could jeopardize the access that
Category: Hemodynamics must be patent for dialysis until the fistula
NCLEX-RN® Exam Content Review and Practice matures.
276 (3) Any other use could jeopardize the access that
(1) Medications should never be infused through an must be patent for dialysis until the fistula
arterial line. matures.
(2) CORRECT: Arterial lines are used for monitoring (4) Any other use could jeopardize the access that
blood pressure and heart rate, especially in must be patent for dialysis until the fistula
clients requiring the use of vasopressor medications matures.
intravenously. They are also used for clients 16. The Answer is 2
requiring frequent blood draws. The nurse may A 76-year-old man is brought into the Emergency
also draw arterial blood gases and other laboratory Department by his spouse. The client’s spouse tells
samples from the line, following the proper the nurse he is confused, disoriented, and weak, and
procedure. This saves the client from frequent has not been eating well. The nurse obtains blood
arterial and venous draws. work as ordered by the physician, including a
(3) Medications should never be infused through an complete
arterial line. blood count (CBC) and a comprehensive metabolic
(4) Medications should never be infused through an panel (CMP). For which result should the nurse
arterial line. immediately notify the physician?
14. The Answer is 4 Category: Illness management
An 82-year-old woman is admitted with a diagnosis (1) This lab value is within normal range.
of rapid atrial fibrillation. The nurse has initiated (2) CORRECT: Symptoms of hyponatremia include
telemetry monitoring per the physician’s order. Two confusion, disorientation, weakness, and poor
hours after initiation of monitoring, an alarm sounds appetite. The physician should be notified immediately
at the central monitoring station: the client is in what for this critical level of sodium.
appears to be ventricular tachycardia. Which of the (3) This lab value is within normal range.
following actions should the nurse take FIRST? (4) This lab value is within normal range.
Category: Hemodynamics 11: Physiological Integrity: Physiological
(1) This would be premature on the part of the Adaptation
nurse—assessment of the client may yield different 277
information than what is reported by the 17. The Answer is 3
monitor. The nurse is providing discharge instructions to a
client going home on enoxaparin. Which of the The client stops the exercise and states she felt a
following popping
responses by the client indicates to the nurse sensation in her abdominal area. Upon assessment,
that the teaching was effective? the nurse finds a small portion of the viscera
Category: Illness management to be protruding through the incision. Which of the
(1) The area would be cleansed with an alcohol wipe, following actions should the nurse take FIRST?
with care not to rub. Rubbing may cause damage Category: Medical emergencies
to the skin and could contribute to formation of (1) This is not a normal finding for a large midline
a hematoma. abdominal incision.
(2) Sites for injection should be rotated, focusing on (2) It is very important for someone to stay with the
areas that have an easily accessible, fatty, client due to the anxiety that the client will be
subcutaneous feeling. The surgeon must be notified, and possible
layer. This is also minimizes tissue damage surgery could ensue, but this is not the first
from repeated injections, which may affect thing the nurse would do.
absorption. (3) CORRECT: This medical emergency is known
(3) CORRECT: Aspiration, or pulling back the as wound dehiscence with evisceration. The
plunger after needle insertion, can cause damage nurse would saturate sterile dressings with normal
to small capillaries and blood vessels and can saline and hold the dressings over the viscera,
lead to hematoma formation and bleeding. which is most likely part of the bowel loop.
(4) Massaging the area postinjection may cause (4) The nurse should attempt to minimize any
damage to the skin and could contribute to additional
hematoma formation. stress on the incision by having the client
18. The Answer is 1 lie in a low Fowler’s position with knees bent.
The nursing home nurse finds a 92-year-old client on An abdominal binder is used in the prevention
the floor during rounds. The client is not responsive. of dehiscence and not in the treatment of
Vital signs have been taken by the certified nursing evisceration.
assistant: blood pressure 98/52, heart rate 120, NCLEX-RN® Exam Content Review and Practice
respirations 278
28, and oxygen saturation 94%. The client 20. The Answer is 1, 3 and 4
has a history of falls, hypertension, and an extensive The intensive care nurse is caring for a client requiring
cardiac history. The client’s chart indicates a signed mechanical ventilation. Which of the following
physician order that states “Do not resuscitate” and are interventions the nurse should take to help
“Do not intubate” (DNR/DNI). Which of the following prevent
should the nurse do? ventilator-associated pneumonia (VAP)? Select
Category: Medical emergencies all that apply.
(1) CORRECT: The nurse should have another staff Category: Medical emergencies
member call 911, gather paperwork, and contact (1) CORRECT: The standard of care is to reposition
the primary physician to notify of the transfer the client at least every 2 hours using lateral and
while the nurse stays with the client to continue horizontal positioning techniques. The head of
to assess for any change in condition. the bed should be raised 30–45 degrees unless
(2) The client has a pulse and is breathing contraindicated. This helps reduce aspiration of
spontaneously both secretions and gastric contents.
at this point, so initiation of CPR would (2) A nasogastric tube can lead to sinusitis, which
be contraindicated. If the client would no longer increases the likelihood of the client developing
have a pulse and/or stop breathing, CPR would VAP. The use of an orogastric tube to aid in
not be initiated due to the DNR/DNI status of feeding and/or gastric decompression is
the client. Unless the client has advance directives recommended
that indicate no emergency treatment or over the use of a nasogastric tube.
no hospitalization, the nurse should continue (3) CORRECT: Oral care should be done at least
reasonable and necessary treatment and nursing every 2 hours. The removal of excess secretions
care up to the point of resuscitation and intubation. is also an important element in the reduction of
This includes calling 911 for emergency VAP. These secretions can cause aspiration, and
assistance. can also be a perfect moist breeding ground for
(3) The nurse would not move the client from the infection.
floor because the client may have experienced a (4) CORRECT: A reduction in the duration of
fracture or head trauma during the unwitnessed mechanical ventilation and/or a reduction in
fall. sedation to assess readiness of weaning have
(4) Family notification would take place after been shown to decrease the development and
emergency incidence of VAP. No alteration in medication
services are requested (or ordered). or weaning/extubation should be attempted
19. The Answer is 3 without an order from the physician. The nurse
The surgical floor nurse is working with a client on can be proactive and encourage the progression
coughing and deep breathing. The mildly obese client of weaning through assessment and subsequent
is six days postoperative, and has a large midline discussions with the physician.
abdominal incision that is not well approximated. (5) Although proper hand hygiene and the use of
gloves have been shown to reduce the risk of VAP,
prophylactic intravenous antibiotic therapy is not a little short of breath and itches after receiving
recommended. A broad-spectrum antibacterial penicillin.
oral rinse (chlorhexidine) has been used in The nurse should do which of the following?
conjunction Category: Unexpected response to therapies
with thorough oral care with good results. (1) The physician should be made aware of this
21. The Answer is 3 allergy prior to the client receiving the medication.
The night nurse on a medical floor has just received In some instances, the physician will confirm
report. On which of the following clients should the the order and not change the medication,
nurse make rounds FIRST? depending on the severity of the past or prior
Category: Pathophysiology reaction to penicillin or cephalosporins. More
(1) The client with mild to moderate pain related to often, though, the physician will change the
pancreatitis is not as critical as the client with antibiotic to a different family, but that is for the
potential acute pulmonary edema. physician to decide and not the pharmacist or
(2) The client with the burning sensation post-TURP the nurse.
is not as critical. In fact, this is a common finding (2) CORRECT: The nurse would call the physician
with this procedure/diagnosis. and double-check this order.
(3) CORRECT: This new nonproductive cough and (3) Cefazolin is not a penicillin; it’s a first-generation
restlessness could indicate acute pulmonary cephalosporin, which can cause a reaction
edema. The nurse would assess this client first, in clients with penicillin allergies.
focusing on lung sounds and heart sounds. Acute (4) It is for the physician to decide to change the
pulmonary edema is seen in clients that have medication, not the nurse.
heart disease, circulatory overload (from transfusions 24. The Answer is 3
and infusions), or lung injuries; that are A 52-year-old woman is admitted with a new
postanesthesia; and other etiologies that could diagnosis
bring about fluid in the alveoli that impedes gas of gastrointestinal (GI) bleed. The physician
exchange. This situation can quickly escalate has ordered the client to receive 2 units of packed
into a medical emergency, so the nurse should red blood cells (PRBCs) for a hemoglobin (Hgb) of
assure oxygenation and implement measures to 6.8 g/dL. The nurse begins the infusion of the first
decrease pulmonary congestion. Quick assessment unit at 100 mL/hr. Fifteen minutes after the start of
and response are critical in preventing a the infusion, the client complains that she is feeling
medical emergency. chilled, short of breath, and is experiencing lumbar
(4) The client with the redness related to cellulitis is pain rated 8 on a 1–10 scale. Which of the following
not as critical. In fact, this is a common finding should be the nurse’s FIRST action?
with this procedure/diagnosis. Category: Unexpected response to therapies
22. The Answer is 1 (1) Vital signs should be obtained, and the physician
The nurse knows which of the following body systems notified after treatment is discontinued. The
is responsible for the production of erythro unit in question should not be restarted, and any
poietin? other units that were issued should not be
Category: Pathophysiology implemented.
(1) CORRECT: The urinary system is responsible (2) Just slowing the infusion will not resolve the
for the production of erythropoietin, which is issue of an allergic reaction to the treatment.
the primary hormone regulator that promotes (3) CORRECT: The symptoms of feeling chilled,
the development and differentiation of red blood being short of breath, and having back pain
cells in the bone marrow and initiates the production could indicate an acute hemolytic reaction. This
of hemoglobin. Ninety percent of erythropoietin medical emergency requires swift action on the
is produced by renal peritubular cells, part of the nurse, including immediately discontinuing
with the other 10% produced in the liver. The the infusion, flushing the IV site, and
cells that produce erythropoietin are sensitive to saving the unit of blood in question for testing.
levels of oxygen within the blood. If the level of (4) Treating the symptoms of the reaction will not
oxygen is low, the kidney cells release erythropoietin resolve the issue of an allergic reaction to the
to stimulate the bone marrow to produce treatment.
more red blood cells to increase the oxygen-carrying 25. The Answer is 1
capability of the blood. The nurse receives report on a client with a right
11: Physiological Integrity: Physiological total knee arthroplasty who developed
Adaptation methicillinresistant
279 Staphylococcus aureus (MRSA) in the surgical
(2) The cardiovascular system is not responsible for incision. The incision was cultured and showed
the production of erythropoietin. sensitivity to vancomycin. The client’s blood urea
(3) The lymphatic system is not responsible for the nitrogen (BUN) is 14 mg/dL and serum creatinine
production of erythropoietin. (Cr) is 0.9 mg/dL. Intake and output are balanced. A
(4) The endocrine system is not responsible for the peak and trough have been ordered. The third dose
production of erythropoietin. of vancomycin is to be given on the nurse’s shift. The
23. The Answer is 2 nurse should do which of the following?
The nurse is initiating cefazolin therapy following a Category: Unexpected response to therapies
physician’s order. The nurse notes that the client has (1) CORRECT: The nurse would expect to draw
an allergy to penicillin. The client states he becomes a trough 30 minutes prior to the third dose of
vancomycin, and draw a peak 60 minutes after 4. “It’s clear that this is an alien laboratory
infusion is complete. The physician orders this and I am in charge.”
set of labs to be drawn, or the order is part of a Directions: Each question or incomplete statement
hospital protocol for clients receiving vancomycin below is followed by four suggested answers or
intravenously. completions. In each case, HIGHLIGHT the statement
(2) Drawing a peak 30 minutes prior to dose and that best answers the question or completes
drawing a trough 60 minutes after infusion the statement. Allot 6 hours of uninterrupted time to
would yield an inaccurate result. take the practice test.
(3) The BUN and Cr levels that are given in this Content Review and Practice for the NCLEX-RN®
scenario Exam
are within normal limits. 284
(4) Giving the dose of vancomycin as ordered and The Practice Test
drawing the peak and trough with other evening 5. A nursing team consists of an RN, an LPN/
labs would yield an inaccurate result. LVN, and an NAP. The nurse should assign
which of the following clients to the LPN/LVN?
The Practice Test 1. A 72-year-old client with diabetes who
Part 3 requires a dressing change for a stasis
ulcer
283 2. A 42-year-old client with cancer of the
PrRACacTtICEice TeESsTt bone complaining of pain
1. The nurse is interviewing a client who is 3. A 55-year-old client with terminal cancer
being treated for obsessive-compulsive being transferred to hospice home care
disorder. Which of the following is the 4. A 23-year-old client with a fracture of the
MOST important question the nurse should right leg who asks to use the urinal
ask this client? 6. To determine the structural relationship of
1. “Do you find yourself forgetting simple one hospital department with another, the
things?” nurse should consult which of the following?
2. “Do you find it hard to stay on a task?” 1. Organizational chart
3. “Do you have trouble controlling 2. Job descriptions
upsetting thoughts?” 3. Personnel policies
4. “Do you experience feelings of panic in a 4. Policies and Procedures Manual
closed area?” 7. A client complains of pain in his right lower
2. Which of the following actions by the nurse extremity. The physician orders codeine 60
would be considered negligence? mg and aspirin grains X PO every 4 hours,
1. Obtaining a Guthrie blood test on a as needed for pain. Each codeine tablet
4-day-old infant contains 15 mg of codeine. Each aspirin
2. Massaging lotion on the abdomen of a tablet contains 325 mg of aspirin. Which of
3-year-old diagnosed with Wilms’ tumor the following should the nurse administer?
3. Instructing a 5-year-old asthmatic to blow 1. 2 codeine tablets and 4 aspirin tablets
on a pinwheel 2. 4 codeine tablets and 3 aspirin tablets
4. Playing kickball with a 10-year-old with 3. 4 codeine tablets and 2 aspirin tablets
juvenile arthritis (JA) 4. 3 codeine tablets and 3 aspirin tablets
3. The nurse on a postpartum unit is preparing 8. The nurse is leading an inservice about
4 clients for discharge. It would be MOST management issues. The nurse would
important for the nurse to refer which of the intervene if another nurse made which of the
following clients for home care? following statements?
1. A 15-year-old primipara who delivered a 1. “It is my responsibility to ensure that
7-lb. male 2 days ago the consent form has been signed and
2. An 18-year-old multipara who delivered attached to the client’s chart prior to
a 9-lb. female by cesarean section 2 days surgery.”
ago 2. “It is my responsibility to witness the
3. A 20-year-old multipara who delivered 1 signature of the client before surgery is
day ago and is complaining of cramping performed.”
4. A 22-year-old who delivered by cesarean 3. “It is my responsibility to provide a
section and is complaining of burning on detailed description of the surgery.”
urination 4. “It is my responsibility to answer questions
4. A client is telling the nurse about his that the client may have prior to surgery.”
perception of his thought patterns. Which of 9. A nurse in the outpatient clinic evaluates
the following statements by the client would the Mantoux test of a client whose history
validate the diagnosis of schizophrenia? indicates that she has been treated during
1. “I can’t get the same thoughts out of my the past year for an AIDS-related infection.
head.” The nurse should document that there
2. “I know I sometimes feel on top of the was a positive reaction if there is an area
world, then suddenly down.” of induration measuring which of the
3. “Sometimes I look up and wonder where I following?
am.” 1. 3 mm
2. 7 mm nurse assign to nursing assistive personnel?
3. 11 mm 1. Listening to the client’s breath sounds
4. 15 mm 2. Setting up the client’s lunch tray
10. The nurse in the newborn nursery has just 3. Obtaining a diet history
received report. Which of the following 4. Instructing the client on how to balance
infants should the nurse see FIRST? rest and activity
1. A 2-day-old infant who is lying quietly 16. The nurse is caring for clients on the
alert with a heart rate of 185 surgical floor and has just received report
2. A 1-day-old infant who is crying and has from the previous shift. Which of the
a bulging anterior fontanel following clients should the nurse see
3. A 12-hour-old infant who is being held, FIRST?
with respirations that are 45 breaths per 1. A 35-year-old admitted 3 hours ago with
minute and irregular a gunshot wound; 1.5 cm area of dark
4. A 5-hour-old infant who is sleeping and drainage noted on the dressing
whose hands and feet are blue bilaterally 2. A 43-year-old who had a mastectomy 2
11. While inserting a nasogastric tube, the days ago; 23 mL of serosanguinous fluid
nurse should use which of the following noted in the Jackson-Pratt drain
protective measures? 3. A 59-year-old with a collapsed lung due
1. Gloves, gown, goggles, and surgical cap to an accident; no drainage noted in the
2. Sterile gloves, mask, plastic bags, and previous 8 hours
gown 4. A 62-year-old who had an abdominalperineal
3. Gloves, gown, mask, and goggles resection 3 days ago; client
4. Double gloves, goggles, mask, and complains of chills
surgical cap 17. Which of the following actions by the nurse
PRAC TICE TES T would certainly be considered negligence?
285 1. Inserting a 16 Fr nasogastric tube and
Physiological Pra cItnitceeg Treistyt aspirating 15 mL of gastric contents
12. The nurse is caring for clients in the 2. Administering meperidine IM to a client
outpatient clinic. Which of the following prior to using the incentive spirometer
phone calls should the nurse return FIRST? 3. Turning and repositioning a client once
1. A client with hepatitis A who states, every 8 hours post-abdominal surgery
“My arms and legs are itching.” 4. Initially administering blood at 5 mL
2. A client with a cast on the right leg who per minute for 15 minutes
states, “I have a funny feeling in my Content Review and Practice for the NCLEX-RN®
right leg.” Exam
3. A client with osteomyelitis of the spine 286
who states, “I am so nauseous that I The Practice Test
can’t eat.” 18. A 1-day-old newborn diagnosed with
4. A client with rheumatoid arthritis who intrauterine growth retardation is observed
states, “I am having trouble sleeping.” by the nurse to be restless, irritable, and
13. The nursing team consists of 1 RN, 2 LPNs/ fist-sucking, and having a high-pitched,
LVNs, and 3 NAPs. The RN should care for shrill cry. Based on this data, which of the
which of the following clients? following actions should the nurse take
1. A client with a chest tube who is FIRST?
ambulating in the hall 1. Massage the infant’s back.
2. A client with a colostomy who requires 2. Tightly swaddle the infant in a flexed
assistance with a colostomy irrigation position.
3. A client with a right-sided 3. Schedule feeding times every 3 to 4
cerebrovascular accident (CVA) who hours.
requires assistance with bathing 4. Encourage eye contact with the infant
4. A client who is refusing medication to during feedings.
treat cancer of the colon 19. The nurse visits a neighbor who is at 20
14. The home care nurse is visiting a client weeks’ gestation. The neighbor complains
during the icteric phase of hepatitis of of nausea, headache, and blurred vision.
unknown etiology. The nurse would be The nurse notes that the neighbor appears
MOST concerned if the client made which nervous, is diaphoretic, and is experiencing
of the following statements? tremors. It would be MOST important for
1. “I must not share eating utensils with my the nurse to ask which of the following
family.” questions?
2. “I must use my own bath towel.” 1. “Are you having menstrual-like
3. “I’m glad that my husband and I can cramps?”
continue to have intimate relations.” 2. “When did you last eat or drink?”
4. “I must eat small, frequent feedings.” 3. “Have you been diagnosed with
15. A nurse plans for care of a client with diabetes?”
anemia who is complaining of weakness. 4. “Have you been lying on the couch?”
Which of the following tasks should the 20. The school nurse notes that a first-grade
child is scratching her head almost 4. “You will probably require the same
constantly. It would be MOST important dose of insulin that you are now taking.”
for the nurse to take which of the following 25. The nurse is caring for clients in a pediatric
actions? clinic. The mother of a 14-year-old male
1. Discuss basic hygiene with the parents. privately tells the nurse that she is worried
2. Instruct the child not to sleep with her about her son because she unexpectedly
dog. walked into his room and discovered him
3. Inform the parents that they must masturbating. Which of the following
contact an exterminator. responses by the nurse would be MOST
4. Observe the scalp for small white specks. appropriate?
21. A suicidal client who was admitted to 1. “Tell your son he could go blind doing
the psychiatric unit for treatment and that.”
observation a week ago suddenly appears 2. “Masturbation is a normal part of
cheerful and motivated. The nurse should sexual development.”
be aware of which of the following? 3. “He’s really too young to be
1. The client is likely sleeping well because masturbating.”
of the medication. 4. “Why don’t you give him more privacy?”
2. The client has made new friends and has 26. The nurse performs a home visit on a client
a support group. who delivered 2 days ago. The client states
3. The client may have finalized a suicide that she is bottle-feeding her infant. The
plan. nurse notes white, curdlike patches on the
4. The client is responding to treatment newborn’s oral mucous membranes. The
and is no longer depressed. nurse should take which of the following
22. The nurse is caring for clients in the actions?
GYN clinic. A client complains of an offwhite 1. Determine the newborn’s blood glucose
vaginal discharge with a curdlike level.
appearance. The nurse notes the discharge 2. Suggest that the newborn’s formula be
and vulvular erythema. It would be MOST changed.
important for the nurse to ask which of the 3. Remind the caregiver not to let the
following questions? infant sleep with the bottle.
1. “Do you douche?” 4. Explain that the newborn will need to
2. “Are you sexually active?” receive some medication.
3. “What kind of birth control do you 27. The nurse at the birthing facility is caring
use?” for a primipara woman in labor, who is 4
4. “Have you taken any cough medicine?” cm dilated and 25% effaced, and whose
23. The nurse is caring for a client in the fetal vertex is at +1. The physician informs
prenatal clinic. The nurse notes that the client that an amniotomy is to be
the client’s chart contains the following performed. The client states, “My friend’s
information: blood type AB, Rh-negative; baby died when the umbilical cord came
serology—negative; indirect Coombs test— out when her water broke. I don’t want you
negative; fetal paternity—unknown. The to do that to me!” Which of the following
nurse should anticipate taking which of the responses by the nurse is BEST?
following actions? 1. “If you are that concerned, you should
1. Administer Rho (D) immune globulin refuse the procedure.”
(RhoGAM). 2. “The procedure will help your labor go
2. Schedule an amniocentesis. faster.”
3. Obtain a direct Coombs test. 3. “That shouldn’t happen to you because
4. Assess maternal serum for alpha the baby’s head is engaged.”
fetoprotein level. 4. “We will monitor you carefully to
PRAC TICE TES T prevent cord prolapse.”
287 28. A primigravid woman comes to the clinic
Physiological Pra cItnitceeg Treistyt for her initial prenatal visit. She is at 32
24. The nurse is caring for a woman at 37 weeks’ gestation and says that she has
weeks’ gestation. The client was diagnosed just moved from out of state. The client
with insulin-dependent diabetes mellitis says that she has had periodic headaches
(IDDM) at age 7. The client states, “I am during her pregnancy, and that she is
so thrilled that I will be breastfeeding my continually bumping into things. The nurse
baby.” Which of the following responses by notes numerous bruises in various stages
the nurse is BEST? of healing around the client’s breasts and
1. “You will probably need less insulin abdomen. Vital signs are: BP 120/80, pulse
while you are breastfeeding.” 72, resp 18, and FHT 142. Which of the
2. “You will need to initially increase your following responses by the nurse is BEST?
insulin after the baby is born.” 1. “Are you battered by your partner?”
3. “You will be able to take an oral 2. “How do you feel about being
hypoglycemic instead of insulin after the pregnant?”
baby is born.” 3. “Tell me about your headaches.”
4. “You may be more clumsy due to your 2. Warm the dialysate solution.
size.” 3. Position the client on the left side.
Content Review and Practice for the NCLEX-RN® 4. Insert a Foley catheter.
Exam 34. The nurse teaches an elderly client with
288 right-sided weakness how to use a cane.
The Practice Test Which of the following behaviors by the
29. The nurse is teaching a class on natural client indicates that the teaching was
family planning. Which of the following effective?
statements by a client indicates that 1. The client holds the cane with his right
teaching has been successful? hand, moves the cane forward followed by
1. “When I ovulate, my basal body the right leg, and then moves the left leg.
temperature will be elevated for 2 days 2. The client holds the cane with his right
and then will decrease.” hand, moves the cane forward followed by
2. “My cervical mucus will be thick, his left leg, and then moves the right leg.
cloudy, and sticky when I ovulate.” 3. The client holds the cane with his left
3. “Because I am regular, I will be fertile hand, moves the cane forward followed by
about 14 days after the beginning of my the right leg, and then moves the left leg.
period.” 4. The client holds the cane with his left
4. “When I ovulate, my cervix will feel hand, moves the cane forward followed by
firm.” his left leg, and then moves the right leg.
30. The home care nurse plans care for a child PRAC TICE TES T
in a leg cast for treatment of a fractured 289
right ankle. The nurse enters the following Physiological Pra cItnitceeg Treistyt
nursing diagnosis on the care plan: skin 35. While caring for a client receiving total
integrity, risk for impaired. Which of the parenteral nutrition (TPN) through a
following actions by the nurse is BEST? central line, the nurse notices a small trickle
1. Teaching the child how to perform of opaque fluid leaking from around the
isometric exercises of the right leg central line dressing. It is MOST important
2. Teaching the mother to gently massage for the nurse to take which of the following
the child’s right foot with emollient actions?
cream 1. Prepare to change the central line
3. Instructing the mother to keep the leg dressing.
cast clean and dry 2. Verify that the client is on antibiotics.
4. Teaching the mother how to turn and 3. Place the client’s head lower than his
position the child feet.
31. The nurse is caring for a client who had a 4. Secure the Y-port where the lipids are
thyroidectomy 12 hours ago for treatment infusing.
of Graves’ disease. The nurse would be 36. A 46-year-old man is admitted to the
MOST concerned if which of the following hospital with a fractured right femur. He is
was observed? placed in balanced suspension traction with
1. The client’s blood pressure is 138/82, a Thomas splint and Pearson attachment.
pulse 84, respirations 16, oral temp 99° F During the first 48 hours, the nurse should
(37.2° C). assess the client for which of the following
2. The client supports his head and neck complications?
when turning his head to the right. 1. Pulmonary embolism
3. The client spontaneously flexes his wrist 2. Fat embolism
when the blood pressure is obtained. 3. Avascular necrosis
4. The client is drowsy and complains of a 4. Malunion
sore throat. 37. The nurse is helping an NAP provide
32. A client is admitted with complaints of a bed bath to a comatose client who is
severe pain in the right lower quadrant of incontinent. The nurse should intervene if
the abdomen. To assist with pain relief, the which of the following actions is noted?
nurse should take which of the following 1. The NAP answers the phone while
actions? wearing gloves.
1. Encourage the client to change positions 2. The NAP log-rolls the client to provide
frequently in bed. back care.
2. Massage the right lower quadrant of the 3. The NAP places an incontinence diaper
abdomen. under the client.
3. Apply warmth to the abdomen with a 4. The NAP positions the client on the left
heating pad. side, head elevated.
4. Use comfort measures and pillows to 38. A 70-year-old woman is brought to the
position the client. emergency room for treatment after being
33. The nurse prepares a client for peritoneal found on the floor by her daughter. X-rays
dialysis. Which of the following actions reveal a displaced subcapital fracture of the
should the nurse take FIRST? left hip and osteoarthritis. When comparing
1. Assess for a bruit and a thrill. the legs, the nurse would most likely make
which of the following observations? 1. Ask the woman’s family to provide
1. The client’s left leg is longer than the personal items such as photos or
right leg and externally rotated. mementos.
2. The client’s left leg is shorter than the 2. Select a room with a bed by the door so
right leg and internally rotated. the woman can look down the hall.
3. The client’s left leg is shorter than the 3. Suggest the woman eat her meals in the
right leg and adducted. room with her roommate.
4. The client’s left leg is longer than the 4. Encourage the woman to ambulate in
right leg and is abducted. the halls twice a day.
39. The nurse is caring for a client with a cast 44. The nurse teaches an elderly client how to use
on the left leg. The nurse would be MOST a standard aluminum walker. Which of the
concerned if which of the following is following behaviors by the client indicates
observed? that the nurse’s teaching was effective?
1. Capillary refill time is less than 3 1. The client slowly pushes the walker
seconds forward 12 inches, then takes small steps
2. Client complains of discomfort and forward while leaning on the walker.
itching 2. The client lifts the walker, moves it
3. Client complains of tightness and pain forward 10 inches, and then takes several
4. Client’s foot is elevated on a pillow small steps forward.
40. The nurse is discharging a client from an 3. The client supports his weight on the
inpatient alcohol treatment unit. Which walker while advancing it forward, then
of the following statements by the client’s takes small steps while balancing on the
wife indicates to the nurse that the family is walker.
coping adaptively? 4. The client slides the walker 18 inches
1. “My husband will do well as long as I forward, then takes small steps while
keep him engaged in activities that he holding onto the walker for balance.
likes.” 45. A nurse is supervising a group of elderly
2. “My focus is learning how to live my clients in a residential home setting. The
life.” nurse knows that the elderly are at greater
3. “I am so glad that our problems are risk of developing sensory deprivation for
behind us.” which of the following reasons?
4. “I’ll make sure that the children don’t 1. Increased sensitivity to the side effects of
give my husband any problems.” medications
Content Review and Practice for the NCLEX-RN® 2. Decreased visual, auditory, and
Exam gustatory abilities
290 3. Isolation from their families and
The Practice Test familiar surroundings
41. A nurse is caring for clients in the mental 4. Decreased musculoskeletal function and
health clinic. A woman comes to the clinic mobility
complaining of insomnia and anorexia. The 46. After receiving report, the nurse should see
client tearfully tells the nurse that she was which of the following clients FIRST?
laid off from a job that she had held for 15 1. A 14-year-old client in sickle-cell crisis
years. Which of the following responses by with an infiltrated IV
the nurse would be MOST appropriate? 2. A 59-year-old client with leukemia who
1. “Did your company give you a severance has received half of a packed red blood
package?” cell transfusion
2. “Focus on the fact that you have a 3. A 68-year-old client scheduled for a
healthy, happy family.” bronchoscopy
3. “Tell me what happened.” 4. A 74-year-old client complaining of a
4. “Losing a job is common nowadays.” leaky colostomy bag
42. A client with a history of alcoholism PRAC TICE TES T
is brought to the emergency room in 291
an agitated state. He is vomiting and Physiological Pra cItnitceeg Treistyt
diaphoretic. He says he had his last drink 47. The home care nurse is visiting a client
5 hours ago. The nurse would expect with a diagnosis of hepatitis of unknown
to administer which of the following etiology. The nurse knows that teaching has
medications? been successful if the client makes which of
1. Chlordiazepoxide hydrochloride the following statements?
2. Disulfiram 1. “I am so sad that I am not able to hold
3. Methadone hydrochloride my baby.”
4. Naloxone hydrochloride 2. “I will eat after my family eats.”
43. An elderly client is admitted to the nursing 3. “I will make sure that my children
home setting. The client is occasionally don’t use my eating utensils or drinking
confused and her gait is often unsteady. glasses.”
Which of the following actions by the nurse 4. “I’m glad that I don’t have to get help
would be MOST appropriate? taking care of my children.”
48. The nurse calculates the IV flow rate The Practice Test
for a postoperative client. The client is 54. The nurse is caring for a client with cervical
to receive 3,000 mL of Ringer’s lactate cancer. The nurse notes that the radium
solution IV to run over 24 hours. The IV implant has become dislodged. Which of
infusion set has a drop factor of 10 drops the following actions should the nurse take
per milliliter. The nurse should regulate FIRST?
the client’s IV to deliver how many drops 1. Grasp the implant with a sterile hemostat
per minute? and carefully reinsert it into the client.
1. 18 2. Wrap the implant in a blanket and place
2. 21 it behind a lead shield.
3. 35 3. Ensure the implant is picked up with
4. 40 long-handled forceps and placed in a
49. A client with emphysema becomes restless lead container.
and confused. Which of the following steps 4. Obtain a dosimeter reading on the client
should the nurse take next? and report it to the physician.
1. Encourage the client to perform pursedlip 55. The nurse in a primary care clinic is caring
breathing. for a 68-year-old man. History reveals that
2. Check the client’s temperature. the client has smoked 1 pack of cigarettes
3. Assess the client’s potassium level. per day for 45 years and drinks 2 beers per
4. Increase the client’s oxygen flow rate to day. He is complaining of a nonproductive
5 L/min. cough, chest discomfort, and dyspnea. The
50. The nurse is caring for a client 1 day after nurse hears isolated wheezing in the right
an abdominal-perineal resection for cancer middle lobe. It would be MOST important
of the rectum. The nurse should question for the nurse to complete which of the
which of the following orders? following orders?
1. Discontinue the nasogastric tube when 1. Pulmonary function tests
bowel sounds are heard. 2. Echocardiogram
2. Irrigate the colostomy. 3. Chest x-ray
3. Place petrolatum gauze over the stoma. 4. Sputum culture
4. Administer meperidine 50 mg IM for 56. The nurse is caring for a client with
pain. pernicious anemia. The nurse knows that
51. The nurse is caring for a client 4 hours after her teaching has been successful if the client
intracranial surgery. Which of the following makes which of the following statements?
actions should the nurse take immediately? 1. “In order to get better, I will take iron
1. Turn, cough, and deep-breathe the pills.”
client. 2. “I am going to attend smoking cessation
2. Place the client with the neck flexed and classes.”
head turned to the side. 3. “I will learn how to perform IM
3. Perform passive range-of-motion injections.”
exercises. 4. “I will increase my intake of
4. Move the client to the head of the bed carbohydrates.”
using a turning sheet. 57. The nurse is caring for clients in the
52. A 6-year-old child with a congenital heart Emergency Department of an acute care
disorder is admitted with congestive heart facility. Four clients have been admitted in
failure. Digoxin 0.12 mg is ordered for the last 20 minutes. Which of the following
the child. The bottle contains 0.05 mg of admissions should the nurse see FIRST?
digoxin in 1 mL of solution. Which of 1. A client complaining of chest pain that
the following amounts should the nurse is unrelieved by nitroglycerin
administer to the child? 2. A client with third-degree burns to the
1. 1.2 mL face
2. 2.4 mL 3. A client with a fractured left hip
3. 3.5 mL 4. A client complaining of epigastric pain
4. 4.2 mL 58. The nurse is caring for a client with a
53. The nurse is caring for a client with chest diagnosis of COPD, bronchitis-type, in
pain in the Emergency Department. Which the long-term care facility. The client is
of the following laboratory findings would wheezing, and his oxygen saturation is 85%.
MOST concern the nurse? Four hours ago, the oxygen saturation was
1. Erythrocyte sedimentation rate (ESR): 88%. It is MOST important for the nurse to
10 mm/hr take which of the following actions?
2. Hematocrit (Hct): 42% 1. Administer beclomethasone, 2 puffs per
3. Creatine phosphokinase-MB (CK-MB): metered-dose inhaler.
4 ng/mL 2. Listen to breath sounds.
4. Serum glucose: 100 mg/dL 3. Increase oxygen to 4 L per mask.
Content Review and Practice for the NCLEX-RN® 4. Administer albuterol, 2 puffs per
Exam metered-dose inhaler.
292 59. The nurse is caring for a client hospitalized
for observation after a fall. The client should place him in which of the following
states, “My friend fell last year, and no positions?
one thought anything was wrong. She 1. Side-lying
died 2 days later!” Which of the following 2. Supine
responses by the nurse is BEST? 3. High Fowler’s
1. “This happens to quite a few people.” 4. Semi-Fowler’s
2. “We are monitoring you, so you’ll be 65. A client is to receive 1,000 mL of 5%
okay.” dextrose in 0.45 NaCl intravenous solution
3. “Don’t you think I’m taking good care in an 8-hour period. The intravenous set
of you?” delivers 15 drops per milliliter. The nurse
4. “You’re concerned that it might happen should regulate the flow rate so it delivers
to you?” how many drops of fluid per minute?
PRAC TICE TES T 1. 15
293 2. 31
Physiological Pra cItnitceeg Treistyt 3. 45
60. The nurse is caring for clients on the 4. 60
pediatric unit. An 8-year-old client with 66. The nurse knows that the plan of care for
second- and third-degree burns on the a client with severe liver disease should
right thigh is being admitted. The nurse include which of the following actions?
should assign the new client to which of the 1. Administer Kayexelate enemas.
following roommates? 2. Offer a low-protein, high-carbohydrate
1. A 2-year-old with chickenpox diet.
2. A 4-year-old with asthma 3. Insert a Sengsteken-Blakemore tube.
3. A 9-year-old with acute diarrhea 4. Administer salt-poor albumin IV.
4. A 10-year-old with methicillin-resistant Content Review and Practice for the NCLEX-RN®
Staphylococcus aureus (MRSA) Exam
61. The nurse teaches a client about elastic 294
stockings. Which of the following The Practice Test
statements by the client indicates to the 67. A client with a diagnosis of delirium is
nurse that teaching was successful? admitted to the hospital. To evaluate the
1. “I will wear the stockings until the cause of a client’s delirium, blood is sent to
physician tells me to remove them.” the laboratory for analysis. The results are
2. “I should wear the stockings even when I as follows: Na+ 156, Cl– 100, K+ 4.0, HCO3
am asleep.” 21, BUN 86, glucose 100. Based on these
3. “Every 4 hours I should remove the laboratory results, the nurse should record
stockings for a half hour.” which of the following nursing diagnoses on
4. “I should put on the stockings before the client’s care plan?
getting out of bed in the morning.” 1. Alteration in patterns of urinary
62. The nurse is teaching a client who is elimination
scheduled for a paracentesis. Which of 2. Fluid volume deficit
the following statements by the client to 3. Nutritional deficit: less than body
the nurse indicates that teaching has been requirements
successful? 4. Self-care deficit: feeding
1. “I will be in surgery for less than an 68. A client is to receive 3,000 mL of 0.9%
hour.” NaCl IV in 24 hours. The intravenous set
2. “I must not void prior to the procedure.” delivers 15 drops per milliliter. The nurse
3. “The physician will remove 2 to 3 liters should regulate the flow rate so that the
of fluid.” client receives how many drops of fluid per
4. “I will lie on my back and breathe minute?
slowly.” 1. 21
63. The home care nurse is performing chest 2. 28
physiotherapy on an elderly client with 3. 31
chronic airflow limitations (CAL). Which of 4. 42
the following actions should the nurse take 69. The nurse is supervising the care of a client
FIRST? receiving TPN through a single-lumen
1. Perform chest physiotherapy prior to percutaneous central catheter. The nurse
meals. would be MOST concerned if which of the
2. Auscultate the chest prior to beginning following was observed?
the procedure. 1. The client receives insulin through the
3. Administer bronchiodilators after the single-lumen catheter.
procedure. 2. A mask is placed on the client when
4. Percuss each lobe prior to asking the changing the client’s dressing.
client to cough. 3. The client’s dressing is changed daily
64. A client is admitted to the hospital with a using sterile technique.
diagnosis of chronic bronchitis. He has a 4. The client is weighed 2 to 3 times per
10-year history of emphysema. The nurse week.
70. The nurse is caring for clients in the notes that there is minimal drainage from
outpatient clinic. A client tells the nurse the nasogastric (NG) tube. It is MOST
that he developed weakness and numbness important for the nurse to take which of the
in the legs the previous day and now his following actions?
body feels the same way. The client’s vital 1. Notify the physician.
signs are: BP 120/60, pulse 86, and resp 2. Monitor vital signs q 15 minutes.
20. The client denies any pain but appears 3. Check the tubing for kinks.
anxious to the nurse. It would be MOST 4. Replace the NG tube.
important for the nurse to ask which of the 76. When collecting a 24-hour urine specimen
following questions? for creatinine clearance, it is MOST
1. “Have you recently fallen or had some important for the nurse to do which of the
other type of physical injury?” following?
2. “Have you recently had a viral infection, 1. Obtain an order from the physician for
such as a cold?” insertion of a Foley catheter.
3. “Have you recently taken any over-thecounter 2. Obtain the client’s weight prior to
medication?” beginning the urine collection.
4. “Have you recently experienced 3. Discard the last voided specimen prior
headaches?” to ending the collection.
71. The nurse is admitting a client who is 4. Ask if a preservative is present in the
jaundiced due to pancreatic cancer. The container.
nurse should give the HIGHEST priority to 77. The nurse is planning discharge teaching
which of the following needs? for a client with Parkinson’s disease. To
1. Nutrition maintain safety, the nurse should make
2. Self-image which of the following suggestions to the
3. Skin integrity family?
4. Urinary elimination 1. Install a raised toilet seat.
72. Which of the following statements by a 2. Obtain a hospital bed.
client during a group therapy session would 3. Instruct the client to hold his arms in a
the nurse identify as reflecting a client’s dependent position when ambulating.
narcissistic personality disorder? 4. Perform an exercise program during the
1. “I’m sick of hearing about all your life late afternoon.
tragedies.” 78. The nurse is performing discharge teaching
2. “I know I’m interrupting others. So for a client with chronic pancreatitis. Which
what?” of the following statements by the client to
3. “I just can’t stop wanting to slash the nurse indicates that further teaching is
myself.” necessary?
4. “I just have no hope for the future.” 1. “I do not have to restrict my physical
PRAC TICE TES T activity.”
295 2. “I should take pancrelipase before
Physiological Pra cItnitceeg Treistyt meals.”
73. A teenage client is admitted to the hospital 3. “I will eat 3 meals per day.”
with anorexia nervosa. Which of the 4. “I am not allowed to drink any alcoholic
following statements by the client requires beverages.”
immediate follow-up by the nurse? Content Review and Practice for the NCLEX-RN®
1. “My gums were bleeding this morning.” Exam
2. “I’m getting fatter every day.” 296
3. “Nobody likes me because I’m so ugly.” The Practice Test
4. “I’m feeling dizzy and weak today.” 79. After a laparoscopic cholecystectomy, the
74. A client is admitted to the hospital client complains of abdominal pain and
for treatment of Pneumocystis jiroveci bloating. Which of the following responses
pneumonia and Kaposi’s sarcoma. The by the nurse is BEST?
client tells the nurse that he has been 1. “Increase your intake of fresh fruits and
considering organ donation when he dies. vegetables.”
Which of the following responses by the 2. “I’ll give you the prescribed pain
nurse is BEST? medication.”
1. “What does your family think about 3. “Why don’t you take a walk in the
your decision?” hallway.”
2. “You will help many people by donating 4. “You may need an indwelling catheter.”
your organs.” 80. The nurse in an outpatient clinic is
3. “Would you like to speak to the organ supervising student nurses administering
donor representative?” influenza vaccinations. The nurse should
4. “That is not possible based on your question the administration of the vaccine
illness.” to which of the following clients?
75. The nurse is caring for a client 5 hours 1. A 45-year-old male who is allergic to
after a pancreatectomy for cancer of shellfish
the pancreas. On assessment, the nurse 2. A 60-year-old female who says she has a
sore throat 86. The nurse is preparing discharge teaching
3. A 66-year-old female who lives in a for a client with a new colostomy. The nurse
group home knows teaching was successful when the
4. A 70-year-old female with congestive client chooses which of the following menu
heart failure options?
81. An arterial blood gas is ordered for a 1. Sausage, sauerkraut, baked potato, and
man after a myocardial infarction. After fresh fruit
obtaining the specimen, it would be MOST 2. Cheese omelet with bran muffin and
appropriate for the nurse to take which of fresh pineapple
the following actions? 3. Pork chop, mashed potatoes, turnips,
1. Obtain ice for the specimen. and salad
2. Apply direct pressure to the site. 4. Baked chicken, boiled potato, cooked
3. Apply a sterile dressing to the site. carrots, and yogurt
4. Observe the site for hematoma 87. A client is seen in the outpatient clinic
formation. to rule out acute renal failure. The nurse
82. The nurse is caring for a man who was would be MOST concerned if the client
involved in an auto accident the previous made which of the following statements?
day. The client has a double-lumen 1. “My urine is often pink-tinged.”
tracheostomy tube with a cuff. Which of the 2. “It is hard for me to start the flow of
following actions should the nurse perform? urine.”
1. Change the tracheostomy dressing every 3. “It is quite painful for me to urinate.”
8 hours and PRN. 4. “I urinate in the morning and again
2. Change the tracheostomy ties every 48 before dinner.”
hours. 88. The nurse is teaching a new mother how
3. Keep the inner cannula of the to breastfeed her newborn. The nurse
tracheostomy in place at all times. knows that teaching has been successful
4. Push the outer cannula back in if it if the client makes which of the following
accidentally “blows out.” statements?
83. The nurse performs discharge teaching with 1. “My baby’s weight should equal her
a client with emphysema. Which statement birthweight in 5 to 7 days.”
by the client indicates that teaching was 2. “My baby should have at least 6 to 8 wet
successful? diapers per day.”
1. “Cold weather will help my breathing 3. “My baby will sleep at least 6 hours
problems.” between feedings.”
2. “I should eat 3 balanced meals but limit 4. “My baby will feed for about 10 minutes
my fluid intake.” per feeding.”
3. “My outside activity should be limited 89. A man is admitted to the telemetry unit
when pollution levels are high.” for evaluation of complaints of chest
4. “An intensive exercise program is pain. Eight hours after admission, the
important in regaining my strength.” client goes into ventricular fibrillation.
84. The nurse assists the physician with The physician defibrillates the client.
the removal of a chest tube. Before the The nurse understands that the purpose
physician removes the chest tube, which of defibrillation is to do which of the
instruction should the nurse give to the following?
client? 1. Increase cardiac contractility and
1. “Exhale and bear down.” cardiac output.
2. “Hold your breath for 5 seconds.” 2. Cause asystole so the normal pacemaker
3. “Inhale and exhale rapidly.” can recapture.
4. “Cough as hard as you can.” 3. Reduce cardiac ischemia and acidosis.
85. A client comes into the emergency room 4. Provide energy for depleted myocardial
with complaints of sudden onset of severe cells.
right flank pain. While tests are being 90. A man is brought to the emergency room
performed, it is MOST important for complaining of chest pain. The nurse
the nurse to take which of the following performs an assessment of the client. Which
actions? of the following symptoms would be MOST
1. Make sure that he does not eat or drink characteristic of an acute myocardial
anything. infarction?
2. Strain all his urine through several 1. Colic-like epigastric pain
layers of gauze. 2. Sharp, well-localized, unilateral chest
PRAC TICE TES T pain
297 3. Severe substernal pain radiating down
Physiological Pra cItnitceeg Treistyt the left arm
3. Check his grip strength and pupil 4. Sharp, burning chest pain moving from
reactivity. place to place
4. Send blood and urine specimens to the 91. The nurse is caring for clients on the medical
lab for analysis. unit. A client is admitted with a diagnosis
of deep vein thrombosis (DVT). Admission 1. Place the client in a private room away
orders include heparin 2,000 units per hour from the nurses’ station.
in 5% dextrose in water. The nurse should 2. Ask the family to wait in the waiting
have which of the following available? room while the nurse admits the client.
1. Propranolol 3. Assign a different nurse daily to care for
2. Protamine zinc the client.
3. Protamine sulfate 4. Ask the client to state today’s date.
4. Vitamin K 98. A female client visits the clinic with
Content Review and Practice for the NCLEX-RN® complaints of right calf tenderness and
Exam pain. It would be MOST important for
298 the nurse to ask which of the following
The Practice Test questions?
92. A client returns to the clinic 2 weeks after 1. “Do you exercise excessively?”
discharge from the hospital. He is taking 2. “Have you had any fractures in the last
warfarin sodium 2 mg PO daily. Which of year?”
the following statements by the client to 3. “What type of birth control do you
the nurse indicates that further teaching is use?”
necessary? 4. “Are you under a lot of stress?”
1. “I have been taking an antihistamine PRAC TICE TES T
before bed.” 299
2. “I take aspirin when I have a headache.” Physiological Pra cItnitceeg Treistyt
3. “I use sunscreen when I go outside.” 99. A mother calls the well-baby clinic to
4. “I take Mylanta if my stomach gets report that her 4-month-old son has an
upset.” upper respiratory infection (URI) with a
93. To enhance the percutaneous absorption of temperature of 104° F (40° C). The infant
nitroglycerin ointment, it would be MOST is scheduled to receive his DPT and TOPV
important for the nurse to select a site that immunizations later that day. The mother
is which of the following? asks the nurse if she should bring him in for
1. Muscular his scheduled immunizations. Which of the
2. Near the heart following responses by the nurse would be
3. Non-hairy MOST appropriate?
4. Over a bony prominence 1. “Keep him at home. We’ll give him a
94. A client with chronic alcohol abuse has double dose next time.”
been admitted to a rehabilitation unit. 2. “Bring him in. His illness will not
The nurse knows that the client is denying interfere with his immunizations.”
alcoholism when he makes which of the 3. “Keep him at home until his
following statements? temperature and infection resolve.”
1. “My brother did this to me.” 4. “Bring him in. We’ll give some
2. “Drinking always calms my nerves.” antibiotics with the immunizations.”
3. “I can stop drinking anytime I feel like 100. The nurse in the postpartum unit cares for
it.” a client who delivered her first child the
4. “Let’s all plan to play cards tonight.” previous day. During her assessment of the
95. During the acute phase of a cerebrovascular client, the nurse notes multiple varicosities
accident (CVA), the nurse should maintain on the client’s lower extremities. Which
the client in which of the following of the following actions should the nurse
positions? perform?
1. Semi-prone with the head of the bed 1. Teach the client to rest in bed when the
elevated 60–90 degrees baby sleeps.
2. Lateral, with the head of the bed flat 2. Encourage early and frequent
3. Prone, with the head of the bed flat ambulation.
4. Supine, with the head of the bed elevated 3. Apply warm soaks for 20 minutes every
30–45 degrees 4 hours.
96. Which of the following statements by 4. Perform passive range-of-motion
a client during a group therapy session exercises 3 times daily.
requires immediate follow-up by the nurse? 101. A man fractures his left femur in a bicycle
1. “I know I’m a chronically compulsive accident. A cast is applied. The nurse
liar, but I can’t help it.” knows that which of the following exercises
2. “I don’t ever want to go home; I feel would be MOST beneficial for this client?
safer here.” 1. Passive exercise of the affected limb
3. “I don’t really care if I ever see my 2. Quadriceps setting of the affected limb
girlfriend again.” 3. Active range-of-motion exercises of the
4. “I’ll make sure that doctor is sorry for unaffected limb
what he said.” 4. Passive exercise of the upper
97. A client newly diagnosed with Alzheimer’s extremities
disease is admitted to the unit. Which of the 102. The nurse plans care for a client receiving
following actions by the nurse is BEST? electroconvulsive treatments (ECT).
Immediately after a treatment, the nonketotic coma
nurse should take which of the following 107. The nurse knows that it is MOST
actions? important for which of the following
1. Orient the client to time and place. clients to receive their scheduled
2. Talk about events prior to the client’s medication on time?
hospitalization. 1. A client diagnosed with myasthenia
3. Restrict fluid intake and encourage the gravis receiving pyridostigmine
client to ambulate. bromide
4. Initiate comfort measures to relieve 2. A client diagnosed with bipolar
vertigo. disorder receiving lithium carbonate
103. A client is to receive 35 mg/hr of 3. A client diagnosed with tuberculosis
intravenous aminophylline. The nurse receiving isonicotinic acid hydrazide
mixes 350 mg of aminophylline in 500 4. A client diagnosed with Parkinson’s
mL D5W. At which of the following rates disease receiving levodopa
should the nurse infuse this solution? 108. An 11-year-old boy is admitted to the
1. 20 mL/hr hospital for evaluation for a kidney
2. 35 mL/hr transplant. During the initial assessment,
3. 50 mL/hr the nurse learns that the client received
4. 70 mL/hr hemodialysis for 3 years due to renal
104. The nurse prepares an adult client for failure. The nurse knows that his illness
instillation of eardrops. The nurse should can interfere with this client’s achievement
use which of the following methods to of which of the following?
administer the eardrops? 1. Intimacy
1. Cool the solution for better adsorption. 2. Trust
Drop the medication directly into the 3. Industry
auditory canal. 4. Identity
2. Warm the solution. Flush the 109. The nurse assesses a client with a history
medication rapidly into the ear. of Addison’s disease who has received
3. Warm the solution. Drop the steroid therapy for several years. The nurse
medication along the side of the ear could expect the client to exhibit which of
canal. the following changes in appearance?
4. Warm the solution to 104° F (40° C). 1. Buffalo hump, girdle-obesity, gaunt
Drop the medication slowly into the facial appearance
ear canal. 2. Tanning of the skin, discoloration
Content Review and Practice for the NCLEX-RN® of the mucous membranes, alopecia,
Exam weight loss
300 3. Emaciation, nervousness, breast
The Practice Test engorgement, hirsutism
105. The nurse is inserting an IV catheter into 4. Truncal obesity, purple striations on
a client’s left arm. Suddenly the client the skin, moon face
exclaims, “It feels like an electric shock is 110. Haloperidol 5 mg tid is ordered for a
going all the way down my arm and into client with schizophrenia. Two days later,
my hand!” What is the FIRST action the the client complains of “tight jaws and a
nurse should take? stiff neck.” The nurse should recognize
1. Instruct the client to take slow, deep that these complaints are which of the
breaths. following?
2. Remove the needle from the client’s 1. Common side effects of antipsychotic
arm. medications that will diminish over
3. Tell the client this is a common time
response to IV insertion. 2. Early symptoms of extrapyramidal
4. Withdraw the needle slightly and then reactions to the medication
push it forward. PRAC TICE TES T
106. A client comes to the emergency room 301
with complaints of nausea, vomiting, and Physiological Pra cItnitceeg Treistyt
abdominal pain. He is a type 1 diabetic 3. Psychosomatic complaints resulting
(IDDM). Four days earlier, he reduced his from a delusional system
insulin dose when flu symptoms prevented 4. Permanent side effects of haloperidol
him from eating. The nurse performs an 111. The nurse is caring for a woman who
assessment of the client that reveals poor states she was beaten and sexually
skin turgor, dry mucous membranes, and assaulted by a male friend. Which of the
fruity breath odor. The nurse should be following should the nurse do FIRST?
alert for which of the following problems? 1. Encourage the client to call her family
1. Hypoglycemia lawyer.
2. Viral illness 2. Ask for a psychiatry consult.
3. Ketoacidosis 3. Stay with the client during the physical
4. Hyperglycemic hyperosmolar exam.
4. Wash and dress the client’s wounds morning surgery. The nurse obtains the
before the physical exam. client’s vital signs: temperature 97.4° F
112. The nurse cares for a client after surgery (36° C), radial pulse 84 strong and regular,
for removal of a cataract in her right eye. respirations 16 and unlabored, and blood
The client complains of severe eye pain pressure 132/74. Which of the following
in her right eye. The nurse knows this actions should the nurse take FIRST?
symptom is which of the following? 1. Notify the physician of the client’s vital
1. Expected; the nurse should administer signs.
analgesic to the client. 2. Obtain orthostatic blood pressures
2. Expected; the nurse should maintain lying and standing.
the client on bed rest. 3. Lower the side rails and place the bed
3. Unexpected and may signify a in its lowest position.
detached retina. 4. Record the data on the client’s
4. Unexpected and may signify preoperative checklist.
hemorrhage. 118. A woman is hospitalized with a diagnosis
113. A client returns to his room after a lower of bipolar disorder. While she is in the
GI series. When he is assessed by the client activities room on the psychiatric
nurse, he complains of weakness. Which unit, she flirts with male clients and
of the following nursing diagnoses should disrupts unit activities. Which of the
receive priority in planning his care? following approaches would be MOST
1. Alteration in sensation-perception, appropriate for the nurse to take at this
gustatory time?
2. Constipation, colonic 1. Set limits on the client’s behavior and
3. High risk for fluid-volume deficit remind her of the rules.
4. Nutrition: less than body requirements 2. Distract the client and escort her back
114. A client hospitalized with a gastric ulcer is to her room.
scheduled for discharge. The nurse teaches 3. Instruct the other clients to ignore this
the client about an anti-ulcer diet. Which client’s behavior.
of the following statements by the client 4. Tell the client that she is behaving
indicates to the nurse that dietary teaching inappropriately and send her to her
was successful? room.
1. “I must eat bland foods to help my 119. A client is brought to the emergency room
stomach heal.” bleeding profusely from a stab wound in
2. “I can eat most foods, as long as they the left chest area. The nurse’s assessment
don’t bother my stomach.” reveals a blood pressure of 80/50, pulse
3. “I cannot eat fruits and vegetables of 110, and respirations of 28. The nurse
because they cause too much gas.” should expect which of the following
4. “I should eat a low-fiber diet to delay potential problems?
gastric emptying.” 1. Hypovolemic shock
115. A 6-year-old boy is returned to his room 2. Cardiogenic shock
after a tonsillectomy. He remains sleepy 3. Neurogenic shock
from the anesthesia but is easily awakened. 4. Septic shock
The nurse should place the child in which 120. A client is admitted to the hospital for
of the following positions? surgical repair of a detached retina in
1. Sims’ the right eye. In planning care for this
2. Side-lying client postoperatively, the nurse should
3. Supine encourage the client to do which of the
4. Prone following?
116. A client is preparing to take her 1-day-old 1. Perform self-care activities.
infant home from the hospital. The nurse 2. Maintain patches over both eyes.
discusses the test for phenylketonuria 3. Limit movement of both eyes.
(PKU) with the mother. The nurse’s 4. Refrain from excessive talking.
teaching should be based on an 121. The nurse cares for a client receiving a
understanding that the test is MOST balanced complete food by tube feeding.
reliable after which of the following? The nurse knows that the MOST common
1. A source of protein has been ingested. complication of a tube feeding is which of
2. The meconium has been excreted. the following?
3. The danger of hyperbilirubinemia has 1. Edema
passed. 2. Diarrhea
4. The effects of delivery have subsided. 3. Hypokalemia
Content Review and Practice for the NCLEX-RN® 4. Vomiting
Exam 122. A 6-week-old infant is brought to the
302 hospital for treatment of pyloric stenosis.
The Practice Test The nurse enters the following nursing
117. The nurse is completing a client’s diagnosis on the infant’s care plan: “fluid
preoperative checklist prior to an early volume deficit related to vomiting.” Which
of the following assessments supports this 127. A client is admitted for treatment of
diagnosis? pulmonary edema. During the admission
1. The infant eagerly accepts feedings. interview, she states she has a 6-year
2. The infant vomited once since history of congestive heart failure (CHF).
admission. The nurse performs an initial assessment.
3. The infant’s skin is warm and moist. When the nurse auscultates the breath
4. The infant’s anterior fontanel is sounds, the nurse should expect to hear
depressed. which of the following?
PRAC TICE TES T 1. Crackling
303 2. Wheezing
Physiological Pra cItnitceeg Treistyt 3. Whistling
123. A client is diagnosed with 4. Absent breath sounds
thrombocytopenia due to acute 128. A man is diagnosed with cancer of the
lymphocytic leukemia. She is admitted larynx and comes to the hospital for a
to the hospital for treatment. To which of total laryngectomy. When admitting this
the following should the nurse assign the client, the nurse should assess laryngeal
client? nerve function by doing which of the
1. To a private room so she will not infect following?
other clients and health care workers 1. Assess the extent of neck edema.
2. To a private room so she will not be 2. Check his ability to swallow.
infected by other clients and health 3. Observe for excessive drooling.
care workers 4. Tap the side of his neck gently and
3. To a semiprivate room so she will have observe for facial twitching.
stimulation during her hospitalization Content Review and Practice for the NCLEX-RN®
4. To a semiprivate room so she will have Exam
the opportunity to express her feelings 304
about her illness The Practice Test
124. A woman comes to the clinic because 129. The nurse supervises care at an adult
she thinks she is pregnant. Tests day-care center. Four meal choices are
are performed and the pregnancy is available to the residents. The nurse
confirmed. The client’s last menstrual should ensure that a resident on a lowcholesterol
period began on September 8 and lasted diet receives which of the
for 6 days. The nurse calculates that her following meals?
expected date of confinement (EDC) is 1. Egg custard and boiled liver
which of the following? 2. Fried chicken and potatoes
1. May 15 3. Hamburger and french fries
2. June 15 4. Grilled flounder and green beans
3. June 21 130. The nurse cares for a client with a possible
4. July 8 bowel obstruction. A nasogastric (NG)
125. A 2-month-old infant is brought to the tube is to be inserted. Before inserting the
pediatrician’s office for a well-baby visit. tube, the nurse explains its purpose to the
During the examination, congenital client. Which of the following explanations
subluxation of the left hip is suspected. by the nurse is MOST accurate?
The nurse would expect to see which of the 1. “It empties the stomach of fluids and
following symptoms? gas.”
1. Lengthening of the limb on the 2. “It prevents spasms of the sphincter of
affected side Oddi.”
2. Deformities of the foot and ankle 3. “It prevents air from forming in the
3. Asymmetry of the gluteal and thigh small and large intestine.”
folds 4. “It removes bile from the gallbladder.”
4. Plantar flexion of the foot 131. The nurse cares for a client diagnosed
126. After 2 weeks of receiving lithium therapy, with cholecystitis. The client says to the
a client in the psychiatric unit becomes nurse, “I don’t understand why my right
depressed. Which of the following shoulder hurts, when the gallbladder is not
evaluations of the client’s behavior by the near my shoulder!” Which of the following
nurse would be MOST accurate? responses by the nurse is BEST?
1. The treatment plan is not effective; the 1. “Sometimes small pieces of the
client requires a larger dose of lithium. gallstones break off and travel to other
2. This is a normal response to lithium parts of the body.”
therapy; the client should continue with 2. “There is an invisible connection
the current treatment plan. between the gallbladder and the right
3. This is a normal response to lithium shoulder.”
therapy; the client should be monitored 3. “The gallbladder is on the right side of
for suicidal behavior. the body and so is that shoulder.”
4. The treatment plan is not effective; the 4. “Your shoulder became tense because
client requires an antidepressant. you were guarding against the
gallbladder pain.” how she can prevent her child from getting
132. The nurse teaches a primigravid woman ear infections so often. The nurse’s response
how to measure the frequency of uterine should be based on an understanding
contractions. The nurse should explain that the recurrence of otitis media can be
to the client that the frequency of uterine decreased by which of the following?
contractions is determined by which of the 1. Covering the child’s ears while bathing
following? 2. Treating upper respiratory infections
1. By timing from the beginning of one quickly
contraction to the end of the next 3. Administering nose drops at bedtime
contraction 4. Isolating her child from other children
2. By timing from the beginning of one 137. A client receives 10 units of NPH insulin
contraction to the end of the same every morning at 8 a.m. At 4 p.m., the nurse
contraction observes that the client is diaphoretic and
3. By the number of contractions that slightly confused. The nurse should take
occur within a given period of time which of the following actions FIRST?
4. By the strength of the contraction at its 1. Check vital signs.
peak 2. Check urine for glucose and ketones.
133. The nurse is teaching a woman who is 3. Give 6 oz. of skim milk.
receiving estrogen replacement therapy. 4. Call the physician.
Which of the following statements by the 138. Prior to the client undergoing a scheduled
nurse indicates that the nurse is aware intravenous pyelogram (IVP), the nurse
of the possible complications of estrogen reviews the client’s health history. It
therapy? would be MOST important for the nurse
1. “Take an analgesic before you take to obtain the answer to which of the
estrogen, because estrogen may cause following questions?
discomfort.” 1. Does the client have difficulty voiding?
2. “Make sure you keep your clinic 2. Does the client have any allergies to
appointments, especially your shellfish or iodine?
gynecologic checkup.” 3. Does the client have a history of
3. “Limit your fluid intake, because constipation?
estrogen promotes the retention of 4. Does the client have frequent
fluids.” headaches?
4. “Increase roughage in your diet to 139. A child with chickenpox (varicella) is
avoid constipation.” brought by her parents to the physician
134. Several days after being admitted for for evaluation. The nurse knows the
depression, a man is observed sitting alone rash characteristic of chickenpox can be
in the clients’ dining room. The nurse described as which of the following?
notes that the client has not finished his 1. Maculopapular
meal. Which of the following nursing 2. Small, irregular red spots with minute
measures would be MOST appropriate? bluish-white centers
1. Allow the client to eat in his room until 3. Round or oval erythematous scaling
he becomes more comfortable eating patches
with other clients. 4. Petechiae
PRAC TICE TES T 140. A primigravid woman at 28 weeks’ gestation
305 takes a 3-hour glucose tolerance test. The
Physiological Pra cItnitceeg Treistyt results indicate a fasting blood sugar of
2. Ask the client’s family to bring foods 100 mg/dL and a 2-hour post-load blood
that he likes to eat. sugar of 300 mg/dL. Which of the following
3. Order small, frequent meals and sit nursing diagnoses should be considered the
with the client while he eats in the HIGHEST priority at this time?
dining room. 1. Potential impaired family coping
4. Do not focus on eating behaviors related to diagnosis of gestational
because his appetite will improve over diabetes mellitus (GDM)
time. 2. Potential noncompliance related to
135. A client is being treated for injuries sustained lack of knowledge or lack of adequate
in an automobile accident. The client has support system
a central venous pressure (CVP) line in 3. Potential for altered parenting related
place. The nurse recognizes that CVP to disappointment
measurement reflects which of the following? 4. Ineffective family coping related to
1. Cardiac output anticipatory grieving
2. Pressure in the left ventricle Content Review and Practice for the NCLEX-RN®
3. Pressure in the right atrium Exam
4. Pressure in the pulmonary artery 306
136. A mother brings her 4-year-old daughter The Practice Test
to the pediatrician for treatment of chronic 141. The nurse cares for a client admitted for
otitis media. The mother asks the nurse a possible herniated intervertebral disk.
Ibuprofen, propoxyphene hydrochloride, complaining of nausea, vomiting, and
and cyclobenzaprine hydrochloride severe right upper quadrant pain. His
are ordered PRN. Several hours after temperature is 101.3° F (38.5° C) and
admission, the client complains of pain. an abdominal x-ray reveals an enlarged
Which of the following actions should the gallbladder. He is given a diagnosis of
nurse do FIRST? acute cholecystitis and is scheduled for
1. Administer ibuprofen. surgery. After administering an analgesic
2. Call the physician to determine which to the client, the nurse recognizes that
medication should be given. which of the following actions is the
3. Gather more information from the HIGHEST priority?
client about the complaint. 1. Assessing the client’s need for dietary
4. Allow the client some time to rest and teaching
see if the pain subsides. PRAC TICE TES T
142. When planning care for a client 307
hospitalized with depression, the nurse Physiological Pra cItnitceeg Treistyt
includes measures to increase his selfesteem. 2. Assessing the client’s fluid and
Which of the following actions electrolyte status
should the nurse take to meet this goal? 3. Examining the client’s health history
1. Encourage him to accept leadership for allergies to antibiotics
responsibilities in milieu activities. 4. Determining whether the client has
2. Set simple, realistic goals with him to signed consent for surgery
help him experience success. 147. A mother with 4 children calls the clinic
3. Help him to accept his illness and the for advice on how to care for her oldest
adjustments that are required. child, who has developed chickenpox.
4. Assure him that when he is discharged, Which of the following statements by
he will be able to resume his previous the mother indicates a need for further
activities. teaching?
143. The nurse finds a visitor unconscious 1. “I should keep my child home from
on the floor of a client’s room during school until the vesicles are crusted.”
visiting hours at the hospital. Which of the 2. “I can use calamine lotion if needed.”
following nursing assessments is consistent 3. “I should remove the crusts so the skin
with cardiopulmonary arrest? can heal.”
1. Absent pulse, fixed and dilated pupils 4. “I can use mittens if scratching
2. Absent respirations, fixed and dilated becomes a problem.”
pupils 148. The nurse is teaching a woman who
3. Absent pulse and respirations comes to the clinic at 32 weeks’ gestation
4. Thready pulse and pupillary changes with a diagnosis of pregnancy-induced
144. A client is transferred to an extended care hypertension (PIH). Which of the following
facility after a cerebrovascular accident statements by the client indicates to the
(CVA). The client has right-sided paralysis nurse that further teaching is required?
and has been experiencing dysphagia. 1. “Lying in bed on my left side is likely to
The nurse observes an aide preparing the increase my urinary output.”
client to eat lunch. Which of the following 2. “If the bed rest works, I may lose a
situations would require an intervention pound or two in the next few days.”
by the nurse? 3. “I should be sure to maintain a diet
1. The client is in bed in high Fowler’s that has a good amount of protein.”
position. 4. “I will have to keep my room darkened
2. The client’s head and neck are and not watch much television.”
positioned slightly forward. 149. The nurse evaluates the care provided to a
3. The aide puts the food in the back of client hospitalized for treatment of adrenal
the client’s mouth on the unaffected crisis. Which of the following changes
side. would indicate to the nurse that the client
4. The aide waters down the pudding to is responding favorably to medical and
help the client swallow. nursing treatment?
145. The home care nurse plans care for a 1. The client’s urinary output has
client with pernicious anemia. A monthly increased.
intramuscular injection is ordered for 2. The client’s blood pressure has
the client. The nurse knows that in an increased.
adult, the best muscle to administer an 3. The client has lost weight.
intramuscular injection is which of the 4. The client’s peripheral edema has
following? decreased.
1. Gluteus maximus 150. After completing an assessment, the nurse
2. Deltoid determines that a client is exhibiting early
3. Vastus lateralis symptoms of a dystonic reaction related
4. Dorsogluteal to the use of an antipsychotic medication.
146. A man comes to the emergency room Which of the following actions by the
nurse would be MOST appropriate? 2. A nursing student takes his blood
1. Reality-test with the client and assure pressure wearing a mask and gloves.
her that her physical symptoms are not 3. A technician wears gloves to perform a
real. veinipuncture.
2. Teach the client about common side 4. A nurse attendant allows visitors to
effects of antipsychotic medications. enter his room without masks.
3. Explain to the client that there is 156. A woman comes to the physician’s office
no treatment that will relieve these for a routine prenatal checkup at 34 weeks’
symptoms. gestation. Abdominal palpation reveals
4. Notify the physician and obtain the fetal position as right occipital anterior
an order for IM diphenhydramine (ROA). At which of the following sites
hydrochloride. would the nurse expect to find the fetal
151. The physician orders heparin for a client. heart tone?
In order to evaluate the effectiveness of the 1. Below the umbilicus, on the mother’s
client’s heparin therapy, the nurse should left side
monitor which of the following laboratory 2. Below the umbilicus, on the mother’s
values? right side
1. Platelet count 3. Above the umbilicus, on the mother’s
2. Clotting time left side
3. Bleeding time 4. Above the umbilicus, on the mother’s
4. Prothrombin time right side
Content Review and Practice for the NCLEX-RN® 157. A client is admitted to the hospital with
Exam complaints of seizures and a high fever. A
308 brain scan is ordered. Before the scan, the
The Practice Test client asks the nurse what position he will
152. A client comes to the clinic for evaluation be in while the procedure is being done.
of acute onset of seizures. A thorough Which of the following statements by the
history and physical examination is nurse is MOST accurate?
performed. The nurse would expect which 1. “You will be in a side-lying position
of the following diagnostic tests to be with the foot of the bed elevated.”
performed FIRST? 2. “You will be in a semi-upright sitting
1. Magnetic resonance imaging (MRI) position, with your knees flexed.”
2. Cerebral angiography 3. “You will be lying supine with a small
3. Electroencephalogram (EEG) pillow under your head.”
4. Electromyogram (EMG) 4. “You will be flat on your back, with
153. The nurse performs dietary teaching with your feet higher than your head.”
a client on a low-protein diet. The nurse PRAC TICE TES T
knows that teaching has been successful if 309
the client identifies which of the following Physiological Pra cItnitceeg Treistyt
meals as LOWEST in protein? 158. A man is admitted to the psychiatric
1. Cranberries and broiled chicken hospital with a diagnosis of obsessivecompulsive
2. Tomatoes and flounder disorder. He is unable to stay
3. Broccoli and veal employed because his ritualistic behavior
4. Spinach and tofu causes him to be late for work. Which of
154. A client has a vagotomy with antrectomy the following interpretations by the nurse
to treat a duodenal ulcer. Postoperatively, of the client’s behavior is MOST accurate?
the client develops dumping syndrome. 1. He is responding to auditory
Which of the following statements by the hallucinations and trying to gain
client indicates to the nurse that further control over his behavior.
dietary teaching is necessary? 2. He is fulfilling an unconscious desire
1. “I should eat bread with each meal.” to punish himself.
2. “I should eat smaller meals more 3. He is attempting to reduce anxiety by
frequently.” taking control of the environment.
3. “I should lie down after eating.” 4. He is malingering in order to avoid
4. “I should avoid drinking fluids with responsibilities at work.
my meals.” 159. A client diagnosed with chronic
155. A man is admitted to the hospital with a lymphocytic leukemia is admitted to
diagnosis of acquired immunodeficiency the hospital for treatment of hemolytic
syndrome (AIDS). He is being treated for anemia. Which of the following measures
Pneumocystis jiroveci pneumonia. The incorporated into the nursing care plan
nurse evaluates the care provided to this BEST addresses the client’s needs?
client by other members of the health care 1. Encourage activities with other clients
team. The nurse should intervene in which in the dayroom.
of the following situations? 2. Isolate the client from visitors and
1. A housekeeper cleans up spilled blood clients to avoid infection.
with a bleach solution. 3. Provide a diet high in vitamin C.
4. Provide a quiet environment to the formation of antibodies.”
promote adequate rest. 4. “RhoGAM is given to you to
160. The nurse plans morning care for a client encourage the production of
hospitalized after a cerebrovascular antibodies.”
accident (CVA) resulting in left-sided 165. The nurse performs client teaching for a
paralysis and homonymous hemianopia. woman with osteoarthritis. The client asks
During morning care, the nurse should do what she can do to effectively decrease
which of the following? pain and stiffness in her joints before
1. Provide care from the client’s right side. beginning her daily routine. The nurse
2. Speak loudly and distinctly when should instruct the client to do which of
talking with the client. the following?
3. Reduce the level of lighting in the 1. “Perform isometric exercises for 10
client’s room to prevent glare. minutes.”
4. Provide all of the client’s care to reduce 2. “Do range-of-motion exercises, then
his energy expenditure. apply ointment to your joints.”
161. The nurse prepares for the admission of 3. “Take a warm bath and rest for a few
a client with a perforated duodenal ulcer. minutes.”
Which of the following should the nurse 4. “Stretch all muscle groups.”
expect to observe as the primary initial 166. The nurse cares for a client receiving
symptom? paroxetine. It is MOST important for the
1. Fever nurse to report which of the following to
2. Pain the physician?
3. Dizziness 1. The client states there is no change in
4. Vomiting her appetite.
162. A 3-week-old boy is admitted with a 2. The client states she has started taking
diagnosis of pyloric stenosis. The mother digoxin.
tells the nurse that this is her first child 3. The client states she applies sunscreen
and asks if there is anything she can do to before going outside.
prevent this from happening to her next 4. The client states she drives her car to
child. Which of the following statements work.
by the nurse BEST addresses her concern? 167. A client returns to his room after a cardiac
1. “This type of thing generally happens catheterization. Which of the following
to first children.” assessments by the nurse would justify
2. “When you have your second child, at calling the physician?
least you’ll know what signs to look for.” 1. Pain at the site of the catheter insertion
3. “This is a structural problem; it is not a 2. Absence of a pulse distal to the
reflection of your parenting skills.” catheter insertion site
4. “This is an inherited condition; it is not 3. Drainage on the dressing covering the
your fault.” catheter insertion site
163. The nurse cares for a client diagnosed with 4. Redness at the catheter insertion site
bipolar disorder. The client paces endlessly 168. An 8-year-old boy is seen in a clinic
in the halls and makes hostile comments for treatment of attention-deficit/
to other clients. The client resists the hyperactivity disorder (ADHD).
nurse’s attempts to move him to a room in Medication has been prescribed for
the unit. Which of the following actions by the child along with family counseling.
the nurse is MOST important? The nurse teaches the parents about
1. Offer the client fluids every hour. the medication and discusses parenting
2. Inform the client about the unit rules. strategies. Which of the following
3. Administer haloperidol IM. statements by the parents indicates that
4. Encourage the client to rest. further teaching is necessary?
Content Review and Practice for the NCLEX-RN® 1. “We will give the medication at night
Exam so it doesn’t decrease his appetite.”
310 2. “We will provide a regular routine for
The Practice Test sleeping, eating, working, and playing.”
164. The nurse is caring for an Rh-negative 3. “We will establish firm but reasonable
mother who has delivered an Rh-positive limits on his behavior.”
child. The mother states, “The doctor 4. “We will reduce distractions
told me about RhoGAM, but I’m still a and external stimuli to help him
little confused.” Which of the following concentrate.”
responses by the nurse is MOST 169. A client has been taking aluminum
appropriate? hydroxide daily for 3 weeks. The nurse
1. “RhoGAM is given to your child to should be alert for which of the following
prevent the development of antibodies.” side effects?
2. “RhoGAM is given to your child to 1. Nausea
supply the necessary antibodies.” 2. Hypercalcemia
3. “RhoGAM is given to you to prevent 3. Constipation
4. Anorexia should the nurse ask FIRST?
PRAC TICE TES T 1. “What has happened to cause you to
311 want to end your life?”
Physiological Pra cItnitceeg Treistyt 2. “How have you planned to kill
170. A client recovering from a laparoscopic yourself?”
laser cholecystectomy says to the nurse, 3. “When did you start to feel as though
“I hate the thought of eating a low-fat you wanted to die?”
diet for the rest of my life.” Which of the 4. “Do you want me to prevent you from
following responses by the nurse is MOST killing yourself?”
appropriate? Content Review and Practice for the NCLEX-RN®
1. “I will ask the dietician to come talk to Exam
you.” 312
2. “What do you think is so bad about The Practice Test
following a low-fat diet?” 175. A man is admitted for treatment of heart
3. “It may not be necessary for you to failure. The physician orders an IV of 125
follow a low-fat diet for that long.” mL of normal saline per hour and central
4. “At least you will be alive and not venous pressure (CVP) readings every 4
suffering that pain.” hours. Sixteen hours after admission, the
171. A client returns to his room after a client’s CVP reading is 3 cm/H2O. Which
transurethral resection of the prostate of the following evaluations of the client’s
(TURP) for benign prostatic hypertrophy fluid status by the nurse would be MOST
(BPH). Which of the following would accurate?
cause the nurse to suspect postoperative 1. The client has received enough fluid.
hemorrhage? 2. The client’s fluid status remains
1. Decreased blood pressure, increased unaltered.
pulse, increased respirations 3. The client has received too much fluid.
2. Fluctuating blood pressure, decreased 4. The client needs more fluid.
pulse, rapid respirations 176. An agitated client throws a chair across
3. Increased blood pressure, bounding the dayroom on the psychiatry floor and
pulse, irregular respirations threatens the other clients with physical
4. Increased blood pressure, irregular harm. Which of the following should the
pulse, shallow respirations nurse do FIRST?
172. The home care nurse screens a group of 1. Tell the client that his wife will be
residents in a dependent living facility called to the hospital.
for risk factors to pneumonia. The nurse 2. Ask the client why he is so angry.
determines that which of the following 3. Remove the other clients from the
clients is MOST at risk to develop dayroom.
pneumonia? 4. Assemble staff and put the client in
1. A 72-year-old female who has left-sided preventive seclusion.
hemiparesis after a cerebrovascular 177. The nurse is caring for a depressed client
accident who spends most of the day sitting at
2. A 76-year-old male who has a history a window, and is about to implement a
of hypertension and type 2 diabetes physical activity plan for him. The nurse
3. An 80-year-old female who walks 1 knows that the purpose of this plan is to
mile every day and has a history of do which of the following?
depression 1. Help the client understand the
4. An 87-year-old male who smokes and problems creating the depression.
has a history of lung cancer 2. Reduce the client’s risk for obesity and
173. The nurse performs teaching with a client diabetes.
undergoing a paracentesis for treatment 3. Transform self-destructive impulses
of cirrhosis. The client asks what position into positive behaviors.
he will be in for the procedure. The nurse’s 4. Encourage socialization and improve
reply should be based on an understanding self-esteem.
that the MOST appropriate position for 178. The nurse is caring for a client with
the client is which of the following? bipolar disorder. Which of the following
1. Sitting with his lower extremities well behaviors by the client indicates to the
supported nurse that a manic episode is subsiding?
2. Side-lying with a pillow between his 1. The client tells several jokes at a group
knees meeting.
3. Prone with his head turned to the left 2. The client sits and talks with other
side clients at mealtimes.
4. Dorsal-recumbent with a pillow at the 3. The client begins to write a book about
back of his head his life.
174. A man calls the Suicide Prevention 4. The client initiates an effort to start a
Hotline and states that he is going to kill radio station on the unit.
himself. Which of the following questions 179. A client hospitalized for treatment of
delusions tells the nurse that he is really 184. The nurse is assessing a client newly
the head of the hospital system and diagnosed with initial-stage chronic
that his “cover” is being a client to get glomerulonephritis. Which of the
information on client abuse. Which of the following findings should the nurse expect
following statements by the nurse to the to see? Select all that apply.
client is BEST initially? 1. Hypotension
1. “Tell me what you mean about being 2. Proteinuria
head of the hospital system and getting 3. Severe anemia
client abuse information.” 4. Hematuria
2. “I think you should share this story 5. Azotemia
with the other clients at dinnertime and 6. Nausea
see what they say.” 185. A 56-year-old male client with a history
3. “You are not the head of the hospital of myocardial infarction is admitted for
system, you are an accountant under evaluation of chest pain. Several hours
treatment for a mental disorder.” later, the client goes into ventricular
4. “It worries me when you say these fibrillation and a code blue is called.
things; it means you are not responding The Emergency Department physician
to the medication.” defibrillates the client. The nurse knows
PRAC TICE TES T that the purpose of defibrillation is to do
313 which of the following?
Physiological Pra cItnitceeg Treistyt 1. Energize myocardial cells.
180. The nurse is caring for a client in labor. 2. Improve left ventricular function.
The nurse palpates a firm, round form 3. Increase cardiac output.
in the uterine fundus, small parts on the 4. Produce momentary asystole to allow
woman’s right side, and a long, smooth, the natural pacemaker to resume
curved section on the left side. Based on activity.
these findings, the nurse should anticipate Content Review and Practice for the NCLEX-RN®
auscultating the fetal heart in which of the Exam
following locations? 314
1. A The Practice Test
2. B 186. The physician orders 0.25 mg digoxin for
3. C a client diagnosed with heart failure. The
4. D client’s pulse is 86 prior to administration
181. A 69-year-old female client admitted with of the prescribed dose. The nurse should
pneumonia is receiving gentamicin. For this do which of the following?
client, which of the following laboratory 1. Give half of the prescribed dose (0.125
values would be MOST important for the mg).
nurse to monitor? 2. Delay the dose until the pulse is below
1. BUN and creatinine 60.
2. Hemoglobin and hematocrit 3. Omit the dose, and record the pulse
3. Sodium and potassium rate as the reason.
4. Platelet count and clotting time 4. Give the full dose as ordered.
182. The nurse is preparing a client newly 187. The nurse knows that atorvastatin
diagnosed with Addison’s disease for administered to a client is effective
discharge. Which of the following when there is a reduction in which of the
statements by the client indicates a need following?
for further instruction from the nurse? 1. Triglycerides
1. “I understand that I will need lifelong 2. Chest pain
cortisone replacement therapy.” 3. Blood pressure
2. “During times of stress, I will need to 4. PTT
decrease my medication.” 188. A 37-year-old female has been prescribed
3. “I must be careful not to injure myself.” sumatriptan for severe migraines. The
4. “I should always carry a medical nurse explains that the client should watch
identification card.” for which of the following adverse drug
183. The nurse suspects a client has meningitis. effects?
The nurse places the client in a dorsal 1. Constipation
recumbent position, puts her hands behind 2. Bradycardia
the client’s neck, and bends it forward. The 3. Somnolence
nurse knows that pain and resistance may 4. Sudden numbness or weakness
indicate neck injury or arthritis, but if the 189. The client is resuming a diet after
client also flexes the hips and knees, this undergoing a Billroth II procedure. To
positive response is which of the following? minimize complications from eating, the
1. Trousseau’s sign nurse instructs the client to do which of
2. Brudzinki’s sign the following?
3. Homans’ sign 1. Drink fluids with meals.
4. Chvostek’s sign 2. Increase intake of carbohydrates and
salt. client back to bed. The nurse notifies
3. Increase fat and protein. the physician and completes an incident
4. Eat 3 large meals a day. report. Which of the following is the
190. The nurse is caring for a client who is MOST appropriate nursing action?
having difficulty eating due to mouth sores 1. Document in the client’s chart that an
from chemotherapy treatments. Which incident report has been completed.
of the following interventions is MOST 2. Make a copy of the incident report for
appropriate to promote basic comfort and the nurse manager.
nutrition? 3. Document the incident in the client’s
1. Obtain an order for TPN. chart.
2. Keep the client NPO. 4. Place the incident report in the client’s
3. Administer a stool softener as ordered. chart.
4. Provide frequent oral hygiene. 196. The nurse is performing an initial postoperative
191. The nurse is caring for an adult male client assessment on a client who has
who has just undergone spinal fusion for a just returned from surgery with a chest
herniated intervertebral disk. To promote tube and water seal drainage system.
comfort and minimize complications, the The nurse should immediately intervene
nurse tells the client to avoid which of the if she makes which of the following
following? observations?
1. Bending the knees when lying on one 1. There are no dependent loops in the
side chest tube.
2. Sitting for longer than 20 minutes at a 2. The chest tube is not clamped.
time 3. The chest tube and drainage system is
3. Using an extra-firm mattress above the client’s chest.
4. Sitting in a hardback chair 4. The fluid level in the water seal is at 2 cm.
192. The nurse is preparing a client for surgery. 197. The nurse is present during an informed
When obtaining informed consent, the consent discussion between the client and
nurse should INITIALLY do which of the the physician regarding recommended
following? surgery. The physician discusses the risks,
1. Explain the risks, benefits, and benefits, and alternatives of the procedure
alternatives of the procedure. with the client. The nurse knows that the
2. Tell the client that obtaining the client’s decision whether or not to have the
signature is routine for all surgeries. surgery is based on which of the following
3. Witness the client’s signature. ethical principles?
4. Assess whether the client’s 1. Nonmaleficence
understanding of the procedure is 2. Beneficence
sufficient to give consent. 3. Autonomy
193. The nurse is preparing to administer 4. Capacity
heparin sodium to a client diagnosed with 198. The nurse is caring for a terminal cancer
thrombophlebitis. The nurse should ensure client at home. The nurse knows that
that which of the following is available if which of the following ethical principles
the client develops a significant bleeding BEST supports keeping client and
problem? family care consistent with the nurse’s
PRAC TICE TES T professional code of ethics?
315 1. Virtues
Physiological Pra cItnitceeg Treistyt 2. Fidelity
1. Phytonadione (vitamin K) 3. Beneficence
2. Fresh frozen plasma (FFP) 4. Justice
3. Protamine sulfate Content Review and Practice for the NCLEX-RN®
4. Reteplase Exam
194. A client is being admitted to the hospital 316
for elective surgery. During the admission The Practice Test
assessment, the nurse asks the client if he 199. The nurse is caring for a client
has an advance directive. The nurse knows receiving intravenous therapy through
that clients have the right to play an active a peripherally inserted central catheter
role in their care and treatment, and this is (PICC). Which of the following actions
guaranteed by which of the following? implemented by the nurse will decrease the
1. The Health Insurance Portability and risk of infection?
Accountability Act (HIPAA) 1. Assess vital signs every 4 hours.
2. The Client Self-Determination Act 2. Ask the physician for an order for
3. The Civil Rights Act antibiotics.
4. The Americans with Disabilities Act 3. Maintain sterile technique during all
195. The nurse enters a client’s hospital phases of PICC care.
room to find the client sitting on the 4. Administer acetaminophen (Tylenol)
bathroom floor. The nurse assesses the before dressing changes.
client, obtains assistance, and assists the 200. The nurse is caring for a client with
chronic obstructive pulmonary disease 1. Elevate the head of the bed 45 degrees.
(COPD) and is planning to obtain an 2. Keep the client’s arm immobilized for
arterial blood gas (ABG). Which of the the first 24 hours.
following should the nurse plan to do to 3. Keep the client’s leg immobilized for
prevent bleeding following the procedure? the first 12 hours.
1. Apply 2 × 2 gauze to the puncture site 4. Tell the client to lie on the procedural
and hold pressure for 5 minutes. side for 2 hours.
2. Have the client hold the puncture site 206. A 65-year-old woman with metastatic
in a dependent position for 5 minutes. breast cancer has been admitted to the
3. Apply a warm compress to the hospital with neutropenic fever. She
puncture site for 15 minutes. informs the nurse that she does not
4. Encourage the client to open and close want CPR or artificial ventilation to be
the hand rapidly for several minutes. performed under any circumstances.
201. The nurse is caring for a client diagnosed The nurse explains that this information
with acute myocardial infarction (MI) can be outlined in an advance directive.
and a history of severe uncontrolled The nurse understands that which of the
hypertension. The nurse should question following addresses the client’s right to
which of the following physician orders? identify treatment desires in advance?
1. Limit physical activity for the first 12 1. The Patient’s Bill of Rights
hours 2. The Patient Self-Determination Act
2. IV nitroglycerin 3. The Health Insurance Portability and
3. Thrombolytic therapy Accountability Act (HIPAA)
4. Oxygen therapy 4. The Americans with Disabilities Act
202. The nurse is preparing to administer 207. After receiving morning report on a
warfarin to a client diagnosed with atrial medical/surgical unit, which of the
fibrillation. The nurse knows that which following clients should the nurse address
of the following nursing diagnoses takes FIRST?
priority? 1. A 36-year-old man who underwent
1. Risk for imbalanced fluid volume surgery to repair multiple fractures in
2. Risk for injury his left leg after an automobile accident
3. Constipation reports coughing up blood.
4. Risk for unstable blood glucose 2. A 56-year-old woman newly diagnosed
203. Prior to administering a tuberculin with diabetes has a fasting blood sugar
(Mantoux) skin test, the nurse in an of 83 mg/dL.
outpatient clinic is educating a client 3. A 68-year-old man with head and
suspected of having tuberculosis (TB). neck cancer receiving a continuous
The nurse determines that the client 5-fluorouracil infusion reports feeling
understands the teaching when the client nauseated.
states which of the following? 4. A 28-year-old woman with sickle cell
1. “I know the test will tell me how long anemia reports a pain level of 6 on a
I’ve been infected with TB.” scale of 1–10.
2. “This test will tell me if I am 208. A 58-year-old Spanish-speaking woman
contagious.” is being discharged after having a central
3. “I will need to come back and have a venous access device placed. Which of the
nurse look at the site in a week.” following BEST describes the nurse’s role
4. “The test will tell us if I’ve ever been in advocating for her client?
infected with TB bacteria.” 1. The nurse uses a translator when she
204. The nurse is preparing to enter the private, provides the client with discharge
well-ventilated isolation room of a client instructions.
with active tuberculosis (TB). Which of 2. The nurse provides both written and
the following actions should the nurse take verbal discharge instructions.
before entering the room? 3. The nurse ensures the client has
1. Wash her hands and wear a gown and transportation home upon discharge.
gloves. 4. The nurse provides discharge
2. Wash her hands. instructions in a private room.
3. Wash her hands and place a particulate 209. A 26-year-old man being admitted for an
filter respirator over her nose and mouth. emergency appendectomy asks the nurse
4. Ask the client to don a mask. why she is asking about his medications
205. The nurse is preparing a female client for and history of previous illnesses. In
a cardiac catheterization with the femoral addition to explaining why it is relevant to
approach. The nurse should do which of the care of the client, the nurse knows this
the following when the client returns to client responsibility has been outlined in
her room after the procedure? which of the following?
PRAC TICE TES T 1. The Americans with Disabilities Act
317 2. The Patient’s Bill of Rights
Physiological Pra cItnitceeg Treistyt 3. Nursing Scope and Standards of
Practice underwent a cesarean section 3 days
4. The Health Insurance Portability and prior
Accountability Act (HIPAA) 4. A double room with a curtain divider
210. A nurse is working on the medical/surgical 215. A 38-year-old client with breast cancer
unit. The nurse knows that which of the will be self-administering filgrastim
following tasks should NOT be delegated subcutaneously. The nurse knows that
to nursing assistive personnel (NAP)? teaching should include which of the
1. Setting up a meal tray for a 75-year-old following?
client with Alzheimer’s disease 1. Dispose of needles in a punctureresistant
2. Assessing a newly postoperative client’s container.
pain level 2. Wear chemotherapy-resistant gloves.
3. Setting up a water basin for a 45-yearold 3. Recap the needles for reuse.
client who wishes to shave at the 4. Neupogen has been prescribed to boost
bedside platelets.
4. Transferring a 70-year-old client 216. A 76-year-old woman has been admitted
awaiting discharge from the bed to a to a rehabilitation center after a hip
wheelchair replacement. During an episode of
Content Review and Practice for the NCLEX-RN® confusion in which she became a danger
Exam to herself, the client was placed in a vest
318 restraint. The nurse knows that which of
The Practice Test the following are also considered types of
211. A nurse receives a phone call from a restraints? Select all that apply.
family member asking for health-related 1. Administering a haloperidol (Haldol)
information on a client being treated injection
for suspected myocardial infarction in 2. Raising 4 bed side rails
the Emergency Department. The nurse 3. Assigning a nurse’s aide to sit and
explains she cannot disclose personal observe the client
information about the client without the 4. Applying wrist cuffs and tying them to
client’s consent. The nurse knows this the bed.
represents which of the following ethical 5. Clipping a tray across the front of the
principles? client’s wheelchair
1. Accountability PRAC TICE TES T
2. Autonomy 319
3. Beneficence Physiological Pra cItnitceeg Treistyt
4. Confidentiality 217. A physician has written an order for
212. A nurse is caring for a 48-year-old man escitalopram oxalate 10 mg PO daily for a
with a new colostomy. Which of the 15-year-old client with depression. After
following activities BEST describes the performing an initial assessment, the nurse
nurse’s role as an advocate for the client? calls the physician to verify the order.
1. Ensuring the skin is dry before Which of the following BEST explains
re-adhering the pouch the nurse’s concern about the safety of the
2. Teaching the client how to change and order?
care for the ostomy pouch 1. The client reported a history of facial
3. Providing the client’s wife with a list of swelling and difficulty breathing while
foods to avoid on citalopram.
4. Explaining to the client that 2. The drug has not been approved for
psychological adjustment to an ostomy use in the client’s age group.
can take time 3. The ordered dose is higher than the
213. A client who has scabies has been suggested range.
admitted to the medical/surgical unit. The 4. The ordered dose is lower than the
nurse knows he should use which of the suggested range.
following precautions when caring for this 218. A 75-year-old client has an unsteady gait
client? and requires assistance with ambulation.
1. Droplet precautions The nurse decides to use a gait belt. The
2. Airborne precautions nurse knows she should do which of the
3. Contact precautions following when using a gait belt? Select all
4. Precautions are not necessary with this that apply.
client 1. Secure the gait belt loosely around the
214. A client who has a localized herpes client’s waist.
simplex virus (HSV) infection is admitted 2. Twist her upper body to position the
to the maternity unit. The nurse knows client.
the client should IDEALLY be placed in 3. Remove the gait belt after use.
which of the following? 4. Place the gait belt over the client’s
1. Any available room clothes with the clip in front.
2. A single, unoccupied room 5. Use the gait belt to help lift the client
3. A double room with a client who from a sitting into a standing position
219. After receiving report at the start of a night in preparation for intermittent enteral
shift, the nurse finds an elderly client lying feedings. The nurse knows to do which
on the floor with the bedrails down. When of the following when administering
documenting findings, which of the following medications via an NG tube?
BEST describes what the nurse should do? 1. Crush the enteric coated aspirin.
1. Complete an incident report at the end 2. Mix the medications with the client’s
of his shift, when he is less busy. feeding formula.
2. Complete an incident report using 3. Flush the tube using a 15-mL syringe.
clear, concise, and factual language. 4. Administer each medication separately.
3. Complete an incident report and place 225. A 3-year-old client with acute otitis
a copy of it in the client’s medical media has been prescribed ofloxacin ear
record. drops. The nurse knows that which of
4. Ask the evening shift nurse to complete the following statements by the father
an incident report because the fall demonstrates that he understands how to
occurred on her shift. properly administer the ear drops?
220. A nurse is caring for an 8-year-old girl 1. “I can stop giving the ear drops as soon
with urinary retention. The nurse is as my daughter’s fever is gone.”
preparing to insert a Foley catheter. 2. “I should give the drops directly on the
Which of the following catheter sizes is eardrum to help get rid of the infection
most appropriate for this client? quickly.”
1. Number 8 French 3. “I should warm the ear drops before
2. Number 16 French giving them by wrapping the bottle in
3. Number 20 French my hand.”
4. Number 22 French 4. “My daughter should lie flat while I
221. A 42-year-old male client weighs 196 lbs. give the drops.”
(89.1 kg) and is 65 inches (1.65 meters) tall. 226. A 68-year-old woman recently diagnosed
Based on the client’s body mass index with hypertension has started taking
(BMI), the nurse knows this client would furosemide 40 mg PO twice daily. During
fall into which of the following categories? a clinic appointment, she reports new
1. Underweight onset muscle weakness and abdominal
2. Normal weight cramping. Lab tests are performed. The
3. Overweight nurse knows which of the following results
4. Obese is the best explanation for the symptoms
222. A 10-year-old girl is being seen in the experienced by the client?
Pediatric Emergency Department 1. Potassium 3.0 mEq/L
following a motor vehicle accident. She has 2. Creatinine 1.5 mg/dL
been stabilized but reports a pain level of 8 3. Fasting glucose 145 mg/dL
on a scale of 1 to 10. The nurse is preparing 4. Total calcium 10.0 mg/dL
to transfer the client to x-ray. The nurse 227. A 64-year-old man with heart failure has
knows that which nonpharmacologic recently been told by his physician to
intervention should NOT be used to help increase his digoxin dose to 0.25 mg. He
reduce pain in this client? has 125 mcg tablets on hand. Which of the
1. Offer choices when possible. following statements by the client to the
2. Reassure the client that the procedure home health nurse indicates the client has
will not hurt. understood the teaching provided about
3. Provide complete explanations about the medication?
what is going to happen. 1. “I should take one tablet.”
4. Use distraction, relaxation, and 2. “I should notify my doctor if I
imagery. experience diarrhea.”
Content Review and Practice for the NCLEX-RN® 3. “I don’t need to follow up with my
Exam doctor unless I’m having a problem.”
320 4. “I can take over-the-counter
The Practice Test medications without the approval of
223. In preparation for doxorubicin my physician.”
administration, the nurse is assessing a 228. The nurse is preparing to administer a
client’s arm to determine where to attempt tuberculin skin test to a pregnant 26-yearold
venipuncture. The nurse knows which of client. The nurse knows which of the
the following veins is the BEST choice to following statements about tuberculin skin
start the IV? testing is TRUE?
1. The non-dominant antecubital fossa 1. The test should not be administered
2. The distal forearm during pregnancy.
3. The wrist 2. The test should be read between 24 and
4. A vein used for venipuncture within 48 hours after administration.
the previous 24 hours 3. The reaction is measured in millimeters
224. A 24-year-old client with anorexia has of the induration.
had a nasogastric (NG) tube placed PRAC TICE TES T
321 priority for care for this client?
Physiological Pra cItnitceeg Treistyt 1. Nutrition
4. The test should be administered into 2. Hygiene
the outer surface of the forearm. 3. Fall risk
229. The nurse takes report on a client who 4. Cardiac care
underwent a thyroidectomy 24 hours ago. 235. The nurse is planning the care of an
The nurse understands that the client is elderly male client with very poor oral
at risk for hypocalcemia. Which of the hygiene and gum disease. The nurse knows
following assessment findings indicate the that the teeth and gums can be which of
client may be hypocalcemic? Select all that the following in the chain of infection?
apply. 1. The method of transmission for bacteria
1. Positive Trousseau’s sign 2. A portal of entry for bacteria
2. Negative Chvostek’s sign 3. The pathogen
3. Numbness around the mouth. 4. A portal of exit
4. Positive Moro reflex test Content Review and Practice for the NCLEX-RN®
5. “Pins and needles” sensation in client’s Exam
feet 322
230. A 58-year-old client is receiving the The Practice Test
monoclonal antibody rituximab and 236. In the event of a fire, the nurse should do
develops an infusion reaction manifested which of the following FIRST?
by chest pain and dyspnea. The nurse 1. Leave the building.
should do which of the following FIRST? 2. Attempt to get clients out of immediate
1. Assess the client’s airway. danger.
2. Stop the infusion. 3. Work to contain the fire.
3. Slow down the rate of infusion. 4. Determine the order in which to
4. Administer epinephrine. evacuate clients.
231. The nurse is assigned as the team leader 237. The nurse knows that which of the
on a busy medical/surgical unit. Which of following BEST describes the role of a
the following BEST describes the “rights” nursing supervisor?
of delegation the nurse must consider 1. Chooses and implements interventions
when assigning tasks to other members of 2. Attends meetings to keep staff up to
the health care team? date
1. Right task, right timing, right client, 3. Does not require special skills to
right person, and right date oversee other professionals
2. Right task, right client, right direction, 4. Is friendly and can make contributions
right supervision, and right date to an employee evaluation
3. Right client, right direction, right day, 238. The nurse is conversing with a young
right medication, and right unit adult client regarding an ordered blood
4. Right task, right circumstance, right transfusion. It is clear to the nurse that
person, right direction, and right the client does not understand the risks
supervision involved with the procedure. Which of the
232. Which of the following pediatric clients following statements BEST describes the
should the nurse provide assessment and nurse’s role regarding informed consent
intervention for FIRST? for this procedure?
1. A 15-month-old who has developed hives 1. The nurse tells the client not to worry
2. A 2-year-old who is ventilated but because blood transfusions are very
stable common.
3. A 12-year-old recovering from surgical 2. The nurse informs the ordering
repair of a fractured femur who physician that the client does not
complains of some difficulty breathing understand the risks and will need
4. A 2-month-old whose apnea alarm is further explanation.
sounding with an oxygen saturation 3. The nurse has someone else witness the
reading of 82% signature on the consent.
233. The nurse knows that she would NOT be 4. The nurse describes alternative
required to use airborne precautions for treatments.
which of the following clients? 239. The nurse is caring for a famous basketball
1. A young adult with possible player who may have sustained a careerchanging
tuberculosis who is also HIV positive injury. When asked by coworkers
2. A middle-aged adult with herpes about the status of the client, she responds
simplex that she is not able to discuss her client.
3. A teenager with chickenpox and a sore Which of the following ethical principles
throat BEST supports her statement?
4. A college student with possible rubella 1. Justice
234. The nurse is caring for an elderly female 2. Beneficence
with dementia. The nurse knows that 3. Confidentiality
which of the following should be the 4. Accountability
240. The nurse is on duty on a busy cardiac dispense when needed.”
telemetry unit. Which of the following 4. “All medications and cleaning supplies
situations requires the nurse’s immediate must be locked in a child-proof cabinet
attention? on the pediatric unit at all times.”
1. The wife of a cardiac client states that 244. The nurse knows that which of the
his IV pump is alarming and he is not following is the MOST appropriate
receiving the pain medication dose due infection control method when caring for
to the pump malfunctioning. clients on a surgical unit?
2. The daughter of an elderly client states 1. Hand hygiene before charting or using
that her mother is uncomfortable and the keyboard
that her electrodes have come off. 2. Handwashing before and after contact
3. The new NAP reports that she cannot with each client
wake her elderly client to take his blood 3. Use of gloves
pressure because he is sleeping soundly 4. Use of gowns with each client
and snoring, but she obtained his pulse 245. The nurse on the adult medical unit
and it is 30. She wants you to come to assesses an elderly client with vertigo.
see if you can wake him. Which of the following interventions
4. The new admission from earlier today demonstrates that the nurse understands
is complaining that he has not been the symptoms of vertigo?
assessed in over an hour and he would 1. The nurse recognizes this client as at
like to order dinner. risk for falls and relays this to the other
241. The nurse knows that which of the team members.
following terms BEST defines the 2. The nurse allows the client to ambulate
multidisciplinary care planning for a alone.
young adult with breast cancer? 3. The nurse encourages the client to sit
1. Team work up quickly before standing.
2. Team building 4. The nurse makes no change in routine
3. Case management precautions because vertigo is an
4. Collaboration expected symptom for this age group.
PRAC TICE TES T 246. The nurse is preparing to change a sterile
323 surgical dressing. While repositioning
Physiological Pra cItnitceeg Treistyt herself, the client touches a sterile sponge.
242. The charge RN is preparing assignments Which of the following is the BEST
on a busy medical unit. For this shift, nursing intervention to promote and
there are several LPNs, several RNs, maintain surgical asepsis?
and one NAP. Which of the following 1. Reassure the client and continue with
assignments by the charge RN is the dressing change.
appropriate? Select all that apply. 2. Reassure the client but instruct her to
1. The NAP is assigned to give morning keep her hands free from the sterile
baths. field. Clear the contaminated area,
2. An LPN is assigned to perform an initial rewash hands, and assemble another
assessment on a newly admitted client. sterile field to start over.
3. An LPN is assigned to clients who are 3. Have the client sterilize her hands so
prescribed oral medications, and will the episode is not repeated.
do vital signs on those clients. 4. Continue with the dressing change,
4. The clients with IV medications are avoiding the items that came in contact
divided among the RNs. with the client’s hands.
5. AN LPN is assigned to insert a urinary Content Review and Practice for the NCLEX-RN®
catheter. Exam
243. The nurse is educating new nursing staff 324
members about safety on the pediatric unit. The Practice Test
Which of the following comments by one of 247. The nurse is caring for a client who has
the new staff members BEST demonstrates just undergone an open laparotomy with
that teaching has been successful? ileostomy. The nurse knows that client
1. “A toddler may be taken to the car in a education should include which of the
wheelchair when discharged and, after following topics?
that, the hospital is not responsible 1. Constipation management
for how the child is transported in the 2. Limited activity
family car.” 3. Stoma care and skin care
2. “School-aged children do not require 4. Urinary incontinence
booster seats if they are less than 80 248. The nurse is assessing a client who has
pounds, and they do not require bicycle multiple sclerosis and can no longer
helmets when they are more than 80 live alone due to immobility. Which of
pounds.” the following statements by the nurse
3. “Medications can be left at the bedside demonstrates her understanding of this
for pediatric clients, and the parent will client’s impaired physical mobility?
1. “Do you have any areas of pain, actions should the nurse do FIRST?
pressure, or open ulcers on your legs, 1. Protect the client’s airway.
ankles, or hips?” 2. Restrain the client.
2. “They do make motor wheelchairs. 3. Record the length of the seizure.
Maybe we can look into that.” 4. Report this to the physician.
3. “How often do you have episodes of 254. The nurse is educating a client with a history
diarrhea?” of hyponatremia on diet choices. Which of
4. “What kinds of meals would you like the following statements by the client BEST
prepared while in the hospital?” indicates the teaching was successful?
249. The nurse is educating a client who is 1. “I should maintain a low-sodium diet.”
scheduled for surgery in the near future 2. “I can drink as much beer as I want
about autologous blood donation. Which to.”
of the following statements by the client 3. “I should avoid caffeine.”
indicates the teaching has been successful? 4. “I should drink a lot of water.”
1. “I cannot donate blood for myself 255. The nurse is caring for an alert and
because of my age.” oriented teen with a head injury who
2. “I will not need a transfusion after complains of a slight headache. Which
major surgery.” of the following symptoms exhibited
3. “I can be an autologous blood donor by the client would require immediate
6 weeks before my surgery in the event intervention by the nurse?
that I may need a transfusion.” 1. The client complains of a continued
4. “I cannot get a transfusion reaction headache and becomes drowsy.
with my own blood.” 2. The headache becomes worse and the
250. The RN is providing education to the LPN client shows a decrease in the level of
about administrating oral medications. consciousness.
Which of the following statements 3. The client vomits one time and
demonstrates to the RN that the LPN continues to have a slight headache.
understands the teaching? 4. The client has no headache but has
1. “Giving oral medications is simple and little memory of the incident.
requires little training.” 256. The nurse is admitting a client to the
2. “If the client can’t swallow a timereleased neurology unit at the medical center. The
tablet, I will crush it.” nurse has arrived at the advance directive
3. “It is okay to crush the client’s section of the initial nursing assessment
extended-release tablet to put it in flowsheet. The nurse assisting with the
applesauce.” admission would intervene if the primary
4. “I can break this scored tablet for the admitting nurse made which of the
partial dose ordered for the client.” following statements to the client?
251. The nurse is reviewing the lab work of a 1. “Do you have someone who would be
pediatric client admitted for chemotherapy a surrogate decision maker for you if
treatment. For which of the following you were unable to make decisions for
laboratory values should the nurse call the yourself?”
physician? 2. “Are you familiar with what an
1. BUN 5, creatinine 0.7 advance directive is?”
2. WBC 0, hemoglobin 2 3. “I should find out if you want an
3. Hemoglobin 9.5, WBC 14 advance directive, but you seem tired
4. Magnesium 2 and confused so I will ask you later.”
252. The nurse is doing a follow-up telephone 4. “Let me tell you a little bit about what
call with a new mother regarding her an advance directive is so that you can
newborn. The mother states the baby’s decide if you want one set up.”
eyes look yellow. Which of the following 257. The nurse is caring for a client on the
is the MOST appropriate response by the medical/surgical unit who is receiving
nurse? an intravenous insulin drip due to severe
1. “How often are you nursing your uncontrollable episodic hyperglycemia.
baby?” After several hours of administering the
2. “Are you breastfeeding or bottle insulin and monitoring blood glucose
feeding?” levels regularly, the glucose levels are
3. “Do you know what your baby’s normalizing. The physician orders the
bilirubin level was before discharge?” nurse to maintain the IV insulin drip
4. “Has your baby been seen by the despite the nurse’s concerns. Which of
pediatrician?” the following actions by the nurse is the
PRAC TICE TES T MOST appropriate?
325 1. The nurse should explain the
Physiological Pra cItnitceeg Treistyt procedure of administering the insulin
253. The nurse is assessing a young adult client intravenously to the client.
who begins to have a grand mal seizure 2. The nurse should maintain the
for the first time. Which of the following intravenous medication according to
the physician’s orders. 3. A middle-aged client recovering from
3. The nurse should wait until the next abdominal surgery who is complaining
blood glucose level check is due and of wheezing and has a new oxygen
make a decision then about next steps. requirement
4. The nurse should contact the nursing 4. An elderly client 1-day post-op for a
supervisor and possibly the supervisor hip replacement whose blood pressure
of the physician who ordered the is elevated
medication. 261. On a medical/surgical unit, each nurse is
Content Review and Practice for the NCLEX-RN® paired with nursing assistive personnel
Exam (NAP) for the night shift. The nurse
326 should assign which of the following
The Practice Test clients to the NAP?
258. The home care nurse is caring for an 1. A middle-aged client receiving
elderly client who lives alone. The nurse chemotherapy and complaining of
notices that the client is beginning to nausea and vomiting
show signs of failure to thrive at home and 2. A middle-aged client who is an
has no family to assist him. The nurse is unstable diabetic requiring a blood
unsure how long this client will be able to glucose level check
remain in his home alone. Which of the 3. An elderly client complaining of pain
following is the next step the nurse should from restless leg syndrome
take based on this assessment? 4. A young adult client recovering from a
1. The nurse should consult with the case drug overdose requesting to leave the
manager employed by the home care unit against medical advice
agency. 262. The nurse is working on a state-of-the-art
2. The nurse should call 911 for an nursing unit with completely electronic
emergency response due to concerns medical records. The rooms are semiprivate,
for safety. with two clients to a room, and
3. The nurse should call the client’s equipped with a computer for each client.
community center for advice. Which of the following actions by the
4. The nurse should call the client’s nurse is the MOST appropriate?
neighbors to ask them to look in on the PRAC TICE TES T
client. 327
259. The new staff nurse working on the Physiological Pra cItnitceeg Treistyt
intensive care unit is concerned about her 1. After each use of the computer and
client’s status. The client has continued to upon leaving the client room, log off
decline throughout the shift. The client’s from the computer.
blood pressure, heart rate, and oxygen 2. After each use of the computer and
saturation have progressively dropped in a upon leaving the client room, face the
relatively short period of time. The nurse computer away from where visitors
inquires with the charge nurse assigned to would be able to see the screen.
that shift. The charge nurse says “Don’t 3. The nurse should not be concerned
worry, the client will be fine, he always about the security of the information
does that.” Which of the following actions because there is a single computer for
should the nurse take? each client and therefore no risk of the
1. The nurse should call the nursing information being seen.
supervisor on duty to assist. 4. The nurse should pull the curtain to
2. The nurse should wait and see how the cover the computer screen so that
client does. visitors cannot view it.
3. The nurse should agree with the charge 263. The nurse is working on a unit that is
nurse because that nurse has more equipped with electronic medication
experience. administration processes. This includes
4. The nurse should discuss this with a computer at the bedside that allows for
other nurses on the unit. scanning a bar code on the medication
260. The nurse on a busy surgical unit has just order, the medication label, and the
received report from the previous shift on client’s identification band. Which of the
the clients assigned to that shift. Which of following is the BEST method for the
the following clients should the nurse see nurse to practice regularly?
FIRST? 1. The nurse should rely solely on the barcoding
1. A young adult client who fractured scanner because it promotes safer
his arm while playing football, had medication administration practices.
surgical repair of the fracture, and is 2. The nurse should rely on a combination
awaiting discharge of nursing judgment and decisionmaking
2. A middle-aged client recovering from along with the computerized
a knee replacement who is currently on system.
the continuous passive motion machine 3. The nurse should never give a
with the physical therapist medication that a bar-coding system
scans as “incorrect medication.” If you are looking for additional preparation materials
4. The nurse should override any for the NCLEX-RN® exam, Kaplan has
medication that the machine scans classroombased
as “incorrect medication” and and online courses to prepare you for the NCLEX-RN®
administer it. exam. These courses are designed to
264. The nurse is administering medications develop both your knowledge of the nursing content
to a client on an inpatient psychiatric as well as your critical thinking skills. And Kaplan
unit. The client states “I don’t usually has courses specifically designed to fit your lifestyle
take a pink pill” when the nurse gives a and budget. Learn more at: kaplannursing.com or
cup holding 4 different pills to the client. call 1-800-527-8378 (outside the United States and
Which of the following is the MOST Canada call 1-212-997-5883).
appropriate response by the nurse? YOUR PRACTICE TEST SCORES
1. The nurse checks the medication
administration record, determines it is 331
correct, and tells the client to take the Answer Key
medication. 1. 3
2. The nurse discounts the client’s 2. 2
concern because he is a psychiatric 3. 4
client and doesn’t know any better. 4. 4
3. The nurse asks the client for a list of 5. 1
medications he routinely takes, and 6. 1
tells the client that she will review and 7. 3
confirm the order with the physician. 8. 3
4. The nurse tells the client that 9. 2
sometimes drugs come in different 10. 1
colors depending on what pharmacy 11. 3
they come from. 12. 2
265. The nurse is working at a skilled nursing 13. 4
facility. The nurse enters the client’s room 14. 3
and sees the client attempting to pull 15. 2
himself up from a sitting position on the 16. 4
floor. The nurse inquires with the client 17. 3
as to what happened. The client responds 18. 2
“I fell.” Which of the following should the 19. 2
nurse document in the incident report? 20. 4
1. The nurse should file an incident report 21. 3
stating “Client fell, no injury noted.” 22. 3
2. The nurse should file an incident report 23. 1
stating “Client fell on floor.” 24. 1
3. The nurse should document the event 25. 2
only in the client’s medical record and 26. 4
not in an incident report. 27. 3
4. The nurse should file an incident report 28. 1
stating “Client found on floor. Client 29. 3
stated ‘I fell.’ Assessment completed, 30. 1
no injury noted, physician notified.” 31. 3
32. 4
329 33. 2
The test included in this book is designed to provide 34. 3
practice answering exam-style questions along with 35. 3
a review of nursing content. Your results on this test 36. 2
indicate where you are now. It is not designed to 37. 1
predict your ability to pass the NCLEX-RN® exam. 38. 3
• If you scored 70 percent or better, you have a good 39. 3
understanding of essential nursing content, and 40. 2
you are able to utilize the critical thinking skills 41. 3
required to answer exam-style questions. 42. 1
• If you scored 60 to 69 percent, you have areas of 43. 1
essential nursing content that need further review, 44. 2
or you may need continued work to master the critical 45. 2
thinking skills needed to correctly answer 46. 1
exam-style questions. 47. 3
• If you scored 59 percent or less, you need 48. 2
concentrated study of nursing content and continued 49. 1
practice utilizing the critical thinking skills required to 50. 2
be successful on the NCLEX-RN® exam. 51. 4
52. 2 118. 2
53. 3 119. 1
54. 3 120. 3
55. 3 121. 2
56. 3 122. 4
57. 2 123. 2
58. 4 124. 2
59. 4 125. 3
60. 2 126. 3
61. 4 127. 1
62. 3 128. 2
63. 2 129. 4
64. 3 130. 1
65. 2 131. 2
66. 2 132. 3
67. 2 133. 2
68. 3 134. 3
69. 3 135. 3
70. 2 136. 2
71. 1 137. 3
72. 1 138. 2
73. 4 139. 1
74. 4 140. 2
75. 3 141. 3
76. 4 142. 2
77. 1 143. 3
78. 3 144. 4
79. 3 TChoen tPernatc tRiecvei eTwes atnd Practice
80. 2 for the NCLEX-RN ® Exam
81. 2 332
82. 1 The Practice Test
83. 3 145. 3
84. 1 146. 2
85. 2 147. 3
86. 4 148. 4
87. 4 149. 2
88. 2 150. 4
89. 2 151. 2
90. 3 152. 3
91. 3 153. 1
92. 2 154. 1
93. 3 155. 2
94. 3 156. 2
95. 4 157. 3
96. 4 158. 3
97. 4 159. 4
98. 3 160. 1
99. 3 161. 2
100. 2 162. 3
101. 2 163. 3
102. 1 164. 3
103. 3 165. 3
104. 3 166. 2
105. 2 167. 2
106. 3 168. 1
107. 1 169. 3
108. 3 170. 3
109. 4 171. 1
110. 2 172. 4
111. 3 173. 1
112. 4 174. 2
113. 3 175. 4
114. 2 176. 4
115. 2 177. 4
116. 1 178. 2
117. 4 179. 1
180. 1 246. 2
181. 1 247. 3
182. 2 248. 1
183. 2 249. 3
184. 2 and 4 250. 4
185. 4 251. 2
186. 4 252. 2
187. 1 253. 1
188. 4 254. 3
189. 3 255. 2
190. 4 256. 3
191. 2 257. 4
192. 4 258. 1
193. 3 259. 1
194. 2 260. 3
195. 3 261. 2
196. 3 262. 1
197. 3 263. 2
198. 2 264. 3
199. 3 265. 4
200. 1 333
201. 3 Prac tice Test
202. 2 1. The Answer is 3
203. 4 The nurse is interviewing a client who is being
204. 3 treated for obsessive-compulsive disorder. Which of
205. 3 the following is the MOST important question the
206. 2 nurse should ask this client?
207. 1 Reworded Question: What are the signs and
208. 1 symptoms
209. 2 of obsessive-compulsive disorder?
210. 2 Strategy: “MOST important” indicates there may be
211. 4 more than one correct response.
212. 2 Needed Info: Obsessive-compulsive disorder is
213. 3 characterized by a history of obsessions and
214. 2 compulsions.
215. 1 Obsessions are recurrent and persistent
216. 1, 2, 4, and 5 thoughts, ideas, impulses, or images that are
217. 1 experienced
218. 3 and 4 as intrusive and senseless. The client knows
219. 2 that the thoughts are ridiculous or morbid but cannot
220. 1 stop, forget, or control them. Compulsions are
221. 4 repetitive behaviors performed in a certain way to
222. 2 prevent discomfort and neutralize anxiety.
223. 2 Category: Assessment/Psychosocial Integrity
224. 4 (1) “Do you find yourself forgetting simple
225. 3 things?”—should be used to assess client with
226. 1 suspected cognitive disorder
227. 2 (2) “Do you find it hard to stay on a task?”—assesses
228. 3 for disorders that disrupt the ability to concentrate,
229. 1, 3, and 5 such as depression
230. 2 (3) “Do you have trouble controlling upsetting
231. 4 thoughts?”—CORRECT: one feature of obsessive-
232. 4 compulsive disorder is the client’s inability
233. 2 to control intrusive thoughts that repeat over
234. 3 and over
235. 2 (4) “Do you experience feelings of panic in a closed
236. 2 area?”—appropriate for client with suspected
237. 1 panic disorder related to closed spaces or
238. 2 claustrophobia
239. 3 2. The Answer is 2
240. 3 Which of the following actions by the nurse would
241. 4 be considered negligence?
242. 1, 3, 4, and 5 Reworded Question: What is an incorrect behavior?
243. 4 Strategy: Think about the consequence of each
244. 2 action.
245. 1 Needed Info: Negligence is the unintentional failure
of the nurse to perform an act that a reasonable Strategy: Consider each answer in turn. Which is
person would or would not perform in similar relevant to schizophrenia?
circumstances; Needed Info: Schizophrenia is generally characterized
can be an act of commission or omission. by delusions (grandiose, religious, paranoid,
Standards of care: the actions that other nurses nihilistic, or delusions of reference or influence),
would do in the same or similar circumstances that confusion, hallucinations, and illusions
provide for quality client care. Nurse practice acts: (misinterpretations
state laws that determine the scope of the practice of real external stimuli).
of nursing. Category: Assessment/Psychosocial Integrity
Category: Analysis/Safe and Effective Care (1) “I can’t get the same thoughts out of my head.”—
Environment/ recurrent, intrusive thoughts are characteristic
Management of Care of obsessive-compulsive disorder
(1) Obtaining a Guthrie blood test on a 4-day-old (2) “I know I sometimes feel on top of the world,
infant—obtain after ingestion of protein, no then suddenly down.”—rapid, changing moods
later than 7 days after delivery are characteristic of the manic phase of bipolar
(2) Massaging lotion on the abdomen of a 3-yearold disorder
diagnosed with Wilms’ tumor—CORRECT: (3) “Sometimes I look up and wonder where I
manipulation of mass may cause dissemination am.”—confused, disoriented thoughts are
of cancer cells characteristic
(3) Instructing a 5-year-old asthmatic to blow on a of cognitive disorders
pinwheel—exercise that will extend expiratory (4) “It’s clear that this is an alien laboratory and I
time and increase expiratory pressure am in charge.”—CORRECT: illogical, disorganized
(4) Playing kickball with a 10-year-old with juvenile thoughts are typical of schizophrenia
arthritis (JA)—excellent moving and stretching 5. The Answer is 1
exercise A nursing team consists of an RN, an LPN/LVN,
3. The Answer is 4 and an NAP. The nurse should assign which of the
The nurse on a postpartum unit is preparing 4 clients following clients to the LPN/LVN?
for discharge. It would be MOST important for the Reworded Question: Which client is an appropriate
nurse to refer which of the following clients for home assignment for the LPN/LVN?
care? Strategy: Think about the skill level involved in each
Prac tice Test client’s care.
ANSWERS AND Expla nations Needed Info: LPN/LVN: assists with implementation
TChoen tPernatc tRiecvei eTwes atnd Practice of care; performs procedures; differentiates normal
for the NCLEX-RN ® Exam from abnormal; cares for stable clients with
334 predictable
The Practice Test conditions; has knowledge of asepsis and dressing
Reworded Question: Who is the most unstable client? changes; administers medications (varies with
Strategy: Think ABCs. educational background and state nurse practice
Needed Info: Need to meet the client’s needs. Physical act).
stability is the nurse’s first concern. Most unstable Category: Planning/Safe and Effective Care
client should be seen first. Environment/
Category: Implementation/Safe and Effective Care Management of Care
Environment/Management of Care (1) A 72-year-old client with diabetes who requires a
(1) A 15-year-old primipara who delivered a 7-lb. dressing change for a stasis ulcer—CORRECT:
male 2 days ago—stable situation, no indication stable client with an expected outcome
of problems with mother or baby (2) A 42-year-old client with cancer of the bone
(2) An 18-year-old multipara who delivered a 9-lb. complaining
female by cesarean section 2 days ago—stable of pain—requires assessment; RN is the
situation, no indication of problems with mother appropriate caregiver
or baby (3) A 55-year-old client with terminal cancer being
(3) A 20-year-old multipara who delivered 1 day ago transferred to hospice home care—requires
and is complaining of cramping—stable client, nursing judgment; RN is the appropriate caregiver
cramping due to uterine contraction (4) A 23-year-old client with a fracture of the right
(4) A 22-year-old who delivered by cesarean section leg who asks to use the urinal—standard,
and is complaining of burning on urination— unchanging procedure; assign to the NAP
CORRECT: unstable client, indicates urinary 6. The Answer is 1
tract infection, requires follow-up To determine the structural relationship of one
4. The Answer is 4 hospital
A client is telling the nurse about his perception of department with another, the nurse should consult
his thought patterns. Which of the following which of the following?
statements Practice Test Explanations
by the client would validate the diagnosis of 335
schizophrenia? Practice Test Answers Taensdt Reworded
Reworded Question: What behaviors or thought Question: How does the nurse determine
patterns the relationship of one hospital department to
characterize schizophrenia? another?
Strategy: Think about each answer. (3) “It is my responsibility to provide a detailed
Needed Info: The lateral lines on an organizational description of the surgery.”—CORRECT: physician
chart define the division and specializations of should provide explanation
labor; the vertical lines explain the lines of authority (4) “It is my responsibility to answer questions that
and responsibility. the client may have prior to surgery.”—describes
Category: Implementation/Safe and Effective Care the nurse’s responsibility
Environment/Management of Care 9. The Answer is 2
(1) Organizational chart—CORRECT: delineates A nurse in the outpatient clinic evaluates the Mantoux
the overall organization structure, showing test of a client whose history indicates that she
which departments exist and their relationships has been treated during the past year for an
with one another both laterally and vertically AIDSrelated
(2) Job descriptions—focus is not on departmental infection. The nurse should document that
relationships there was a positive reaction if there is an area of
(3) Personnel policies—define policies for the induration measuring which of the following?
organization’s Reworded Question: What is a positive reaction for a
employees client who is immunocompromised?
(4) Policies and Procedures Manual—defines Strategy: Think about each answer choice.
standards Needed Info: Given intradermally in the forearm;
of care for an institution read in 48–72 hours. 10 mm induration (hard area
7. The Answer is 3 TChoen tPernatc tRiecvei eTwes atnd Practice
A client complains of pain in his right lower extremity. for the NCLEX-RN ® Exam
The physician orders codeine 60 mg and aspirin 336
grains X PO every 4 hours, as needed for pain. Each The Practice Test
codeine tablet contains 15 mg of codeine. Each aspirin under skin) = significant (positive) reaction. Greater
tablet contains 325 mg of aspirin. Which of the than 5 mm for clients with AIDS = positive reaction.
following should the nurse administer? Does not mean active disease is present but
Reworded Question: What amount of medication indicates exposure to TB or the presence of inactive
should you give? (dormant) disease. Multiple puncture test done for
Strategy: Remember how to calculate dosages. routine screening.
Needed Info: 60 mg = 1 grain. Category: Analysis/Safe and Effective Care
Category: Implementation/Physiological Integrity/ Environment/
Pharmacological and Parenteral Therapies Safety and Infection Control
(1) 2 codeine tablets and 4 aspirin tablets—inaccurate (1) 3 mm—nonsignificant reaction
(2) 4 codeine tablets and 3 aspirin tablets—inaccurate (2) 7 mm—CORRECT: greater than 5-mm area
(3) 4 codeine tablets and 2 aspirin tablets—CORRECT: positive for client with HIV-infection history
60/x = 15/1, x = 4; 10 grains = 600 mg; (3) 11 mm—area of 10 mm or more indicates positive
325/1 = 600/x, x = 1.8 (round to 2) reaction for client without an HIV infection
(4) 3 codeine tablets and 3 aspirin tablets—inaccurate (4) 15 mm—area of 10 mm or more indicates positive
8. The Answer is 3 reaction for client without an HIV infection
The nurse is leading an inservice about management 10. The Answer is 1
issues. The nurse would intervene if another nurse The nurse in the newborn nursery has just received
made which of the following statements? report. Which of the following infants should the
Reworded Question: What are the nurse’s nurse see FIRST?
responsibilities Reworded Question: Which infant is most unstable?
regarding obtaining consent? Strategy: Remember ABCs (airway, breathing,
Strategy: Think about each answer. Does it describe circulation).
the nurse’s responsibility for consent? Needed Info: Need to meet client’s needs. Physical
Needed Info: Requirements: capacity-age (adult), stability of client is nurse’s first concern. Most
competent, voluntary; info must be given in unstable client should be seen first.
understandable Category: Evaluation/Safe and Effective Care
form. Legal responsibility: physician’s Environment/
responsibility to get consent form signed; when Management of Care
nurse witnesses a signature it means there’s reason (1) A 2-day-old infant lying quietly alert with a heart
to rate of 185—CORRECT: infant has tachycardia;
believe client is informed about upcoming treatment. normal resting rate is 120–160; requires further
Category: Evaluation/Safe and Effective Care investigation
Environment/ (2) A 1-day-old infant crying, with a bulging anterior
Management of Care fontanel—crying causes increased intracranial
(1) “It is my responsibility to ensure that the consent pressure, which causes fontanel to bulge
form has been signed and attached to the client’s (3) A 12-hour-old infant being held; the respirations
chart prior to surgery.”—describes the nurse’s are 45 breaths per minute and irregular—normal
responsibility respiratory rate is 30–60 breaths per minute with
(2) “It is my responsibility to witness the signature apneic episodes
of the client before surgery is performed.”—signature (4) A 5-hour-old infant sleeping with the hands and
indicates that the nurse saw the client sign feet blue bilaterally—acrocyanosis is normal for
the form 2–6 hours postdelivery due to poor peripheral
circulation Needed Info: Determine nursing care required to
11. The Answer is 3 meet clients’ needs; take into account time required,
While inserting a nasogastric tube, the nurse should complexity of activities, acuity of client, infection
use which of the following protective measures? control issues. Consider knowledge and abilities of
Reworded Question: What is the correct universal staff members and decide which staff person is best
precaution? able to provide care. Give assignments to staff
Strategy: Think about each answer choice. How is members
each measure protecting the nurse? (assign responsibility for total client care; avoid
Needed Info: Mask, eye protection, face shield protect assigning only procedures). Provide additional help
mucous membrane exposure; used if activities as needed.
are likely to generate splash or sprays. Gowns used Category: Planning/Safe and Effective Care
if activities are likely to generate splashes or sprays. Environment/
Category: Planning/Safe and Effective Care Management of Care
Environment/ (1) A client with a chest tube who is ambulating in
Safety and Infection Control the hall—LPN/LVN can care for client
(1) Gloves, gown, goggles, and surgical cap—surgical (2) A client with a colostomy who requires assistance
caps offer protection to hair but aren’t with a colostomy irrigation—assign to the
required LPN/LVN
(2) Sterile gloves, mask, plastic bags, and gown— (3) A client with a right-sided cerebral vascular
plastic bags provide no direct protection and accident
aren’t part of universal precautions (CVA) who requires assistance with bathing—
(3) Gloves, gown, mask, and goggles—CORRECT: assign to an NAP
must use universal precautions on all clients; prevent (4) A client who is refusing medication to treat
skin and mucous membrane exposure when cancer of the colon—CORRECT: requires the
contact with blood or other body fluids is anticipated assessment skills of the RN
(4) Double gloves, goggles, mask, and surgical 14. The Answer is 3
cap—surgical cap not required; unnecessary to The home care nurse is visiting a client during the
double-glove icteric phase of hepatitis of unknown etiology. The
12. The Answer is 2 nurse would be MOST concerned if the client made
The nurse is caring for clients in the outpatient clinic. which of the following statements?
Which of the following phone calls should the nurse Reworded Question: What is an incorrect statement
return FIRST? about caring for a client with hepatitis?
Reworded Question: Which client should the nurse Strategy: “MOST concerned” indicates you are
call back first? looking for an incorrect statement.
Strategy: Think ABCs. Needed Info: Hepatitis A (HAV): high risk groups
Needed Info: Need to meet client’s needs. Physical include young children, institutions for custodial
stability is nurse’s first concern. Most unstable client care, international travelers; transmission by fecal/
should be contacted first. oral, poor sanitation; nursing considerations include
Category: Analysis/Safe and Effective Care prevention, improved sanitation, treat with gamma
Environment/ globulin early post-exposure, no preparation of food.
Management of Care Hepatitis B (HBV): high risk groups include drug
(1) A client with hepatitis A who states, “My arms addicts, fetuses from infected mothers, homosexually
and legs are itching.”—caused by accumulation active men, transfusions, health care workers;
Practice Test Explanations transmission by parenteral, sexual contact, blood/
337 body fluids; nursing considerations include vaccine
Practice Test Answers Taensdt of bile salts under (Heptavax-B, Recombivax HB), immune globulin
the skin; treat with calamine (HBIG) post-exposure, chronic carriers (potential
lotion and antihistamines for chronicity 5–10%). Hepatitis C (HVC): high risk
(2) A client with a cast on the right leg who states, groups include transfusions, international travelers;
“I have a funny feeling in my right leg.”—CORRECT: transmission by blood/body fluids; nursing
may indicate neurovascular compromise; considerations
requires immediate assessment include great potential for chronicity. Delta
(3) A client with osteomyelitis of the spine who hepatitis: high risk groups same as for HBV;
states, “I am so nauseous that I can’t eat.”— transmission
requires follow-up, but not highest priority coinfects with HBV, close personal contact.
(4) A client with rheumatoid arthritis who states, “I Category: Evaluation/Safe and Effective Care
am having trouble sleeping.”—requires assessment, Environment/
but not a priority Safety and Infection Control
13. The Answer is 4 (1) “I must not share eating utensils with my
The nursing team consists of 1 RN, 2 LPNs/LVNs, family.”—
and 3 NAPs. The RN should care for which of the prevents transmission; handwashing
following clients? before eating and after toileting very important
Reworded Question: Which client is an appropriate (2) “I must use my own bath towel.”—prevents
assignment for the RN? transmission; don’t share bed linens
Strategy: Think about the skill level involved in each (3) “I’m glad that my husband and I can continue
client’s care. to have intimate relations.”—CORRECT: avoid
sexual contact until serologic indicators return (4) A 62-year-old who had an abdominal-perineal
to normal resection 3 days ago; client complains of chills—
(4) “I must eat small, frequent feedings.”—easier to CORRECT: at risk for peritonitis; should be
tolerate than 3 standard meals; diet should be assessed for further symptoms of infection
high in carbohydrates and calories 17. The Answer is 3
TChoen tPernatc tRiecvei eTwes atnd Practice Which of the following actions by the nurse would
for the NCLEX-RN ® Exam certainly be considered negligence?
338 Reworded Question: What is negligent behavior?
The Practice Test Strategy: Think about the consequences of each
15. The Answer is 2 action.
A nurse plans for care of a client with anemia who Needed Info: Negligence: unintentional failure of
is complaining of weakness. Which of the following nurse to perform an act that a reasonable person
tasks should the nurse assign to nursing assistive would or would not perform in similar circumstances;
personnel can be an act of commission or omission.
(NAP)? Standards of care: the actions that other nurses
Reworded Question: What is an appropriate would do in same or similar circumstances that
assignment provide for quality client care. Nurse practice acts:
for an NAP? state laws that determine the scope of the practice
Strategy: Think about the skill level involved in each of nursing.
task. Category: Evaluation/Safe and Effective Care
Needed Info: Nursing assistive personnel (NAPs): Environment/
assist with direct client care activities (bathing, Management of Care
transferring, ambulating, feeding, toileting, obtaining (1) Inserting a 16 Fr nasogastric tube and aspirating
vital signs/height/weight/intake/output, 15 mL of gastric contents—correct procedure;
housekeeping, verify placement by checking the pH
transporting, stocking supplies); includes (2) Administering meperidine (Demerol) IM to a
nurse aides, assistants, technicians, orderlies, nurse client prior to using the incentive spirometer—
extenders; scope of nursing practice is limited. reducing the client’s pain enables the client to
Category: Planning/Safe and Effective Care take a deep breath
Environment/ (3) Turning and repositioning a client every shift
Management of Care after post-abdominal surgery—CORRECT:
(1) Listen to the client’s breath sounds—requires Postoperative clients should be turned and
assessment; should be performed by RN repositioned
(2) Set up the client’s lunch tray—CORRECT: every 2 hours after surgery to promote
standard, circulation and reduce the risk of skin breakdown
unchanging procedure; decreases cardiac (except if contraindicated, such as in neurologic
workload or musculoskeletal surgery demanding
(3) Obtain a diet history—involves assessment; immobilization)
should be performed by RN Practice Test Explanations
(4) Instruct the client on how to balance rest and 339
activity—assessment and teaching required; Practice Test Answers Taensdt (4) Initially
should be performed by RN administering blood at 5 mL per minute
16. The Answer is 4 for 15 minutes—correct procedure; start blood
The nurse is caring for clients on the surgical floor with normal saline and 19-gauge needle
and has just received report from the previous shift. 18. The Answer is 2
Which of the following clients should the nurse see A 1-day-old newborn diagnosed with intrauterine
FIRST? growth retardation is observed by the nurse to be
Reworded Question: Which client is the least stable? restless, irritable, and fist-sucking, and having a
Strategy: Think ABCs. high-pitched, shrill cry. Based on this data, which of
Needed Info: Need to meet the client’s needs. Physical the following actions should the nurse take FIRST?
stability is the nurse’s first concern. Most unstable Reworded Question: What do you do for a newborn
client should be seen first. experiencing withdrawal?
Category: Analysis/Safe and Effective Care Strategy: Determine the outcome of each answer.
Environment/ Needed Info: Drug withdrawal may manifest from as
Management of Care early as 12 hrs after birth up to 10 days after delivery.
(1) A 35-year-old admitted 3 hours ago with a Symptoms: high-pitched cry, hyperreflexia,
gunshot decreased sleep, diaphoresis, tachypnea, excessive
wound; 1.5-cm area of dark drainage noted mucus, vomiting, uncoordinated sucking. Nursing
on the dressing—does not indicate acute bleeding; care: assess muscle tone, irritability, vital signs;
small amount of blood administer phenobarbital as ordered; report
(2) A 43-year-old who had a mastectomy 2 days ago; symptoms
23 mL of serosanguinous fluid noted in the Jackson- of respiratory distress; reduce stimulation; provide
Pratt drain—expected outcome adequate nutrition/fluids; monitor mother/child
(3) A 59-year-old with a collapsed lung due to an interactions.
accident; no drainage noted in the previous 8 Category: Implementation/Health Promotion and
hours—indicates resolution Maintenance
(1) Massage the infant’s back—may result in TChoen tPernatc tRiecvei eTwes atnd Practice
overstimulation for the NCLEX-RN ® Exam
of the infant 340
(2) Tightly swaddle the infant in a flexed position— The Practice Test
CORRECT: promotes infant’s comfort and (3) Inform the parents that they must contact an
security exterminator—not enough information to make
(3) Schedule feeding times every 3–4 hours—small, this determination
frequent feedings are preferable (4) Observe the scalp for small white specks—
(4) Encourage eye contact with the infant during CORRECT:
feedings—may result in overstimulation of nits (eggs) appear as small, white, oval
infant flakes attached to hair shaft
19. The Answer is 2 21. The Answer is 3
The nurse visits a neighbor who is at 20 weeks’ A suicidal client who was admitted to the psychiatric
gestation. unit for treatment and observation a week ago
The neighbor complains of nausea, headache, suddenly appears cheerful and motivated. The nurse
and blurred vision. The nurse notes that the neighbor should be aware of which of the following?
appears nervous, is diaphoretic, and is experiencing Reworded Question: What is the significance of
tremors. It would be MOST important for the nurse sudden
to ask which of the following questions? mood changes in a depressed client?
Reworded Question: What is the priority assessment Strategy: Know the signs of impending suicide.
question? Needed Info: Assessment for suicidal ideation, suicidal
Strategy: “MOST important” indicates there may be gestures, suicidal threats, and actual suicidal
more than one correct response. attempt. Clients who have developed a suicide plan
Needed Info: Assessment: irritability, confusion, are more serious about following through, and are at
tremors, blurring of vision, coma, seizures, grave risk. Clients emerging from severe depression
hypotension, have more energy with which to formulate and carry
tachycardia, skin cool and clammy, diaphoresis. out a suicide plan (for which they had no energy
Plan/Implementation: liquids containing sugar before treatment). The nurse should determine risk
if conscious, skim milk is ideal if tolerated; dextrose for suicide; suspect suicidal ideation in depressed
50% IV if unconscious, glucagon; follow with client;
additional ask the client if he is thinking about suicide; ask
carbohydrate in 15 minutes; determine and the client about the advantages and disadvantages
treat cause; client education; exercise regimen. of suicide to determine how the client sees his
Category: Assessment/Health Promotion and situation;
Maintenance evaluate client’s access to a method of suicide;
(1) “Are you having menstrual-like cramps?”— develop a formal “no suicide” contract with client;
symptoms of preterm labor and support the client’s reason to live.
(2) “When did you last eat or drink?”—CORRECT: Category: Analysis/Psychosocial Integrity
classic symptoms of hypoglycemia; offer (1) The client is likely sleeping well because of the
carbohydrate medication—improved sleep patterns would not
(3) “Have you been diagnosed with diabetes?”— explain the client’s sudden mood change
need to determine if she is hypoglycemic (2) The client has made new friends and has a support
(4) “Have you been lying on the couch?”—not relevant group—support on the nursing unit would
to hypoglycemia not explain the mood change
20. The Answer is 4 (3) The client may have finalized a suicide plan—
The school nurse notes that a first-grade child is CORRECT: as depressed clients improve, their
scratching her head almost constantly. It would be risk for suicide is greater because they are able to
MOST important for the nurse to take which of the mobilize more energy to plan and execute suicide
following actions? (4) The client is responding to treatment and is no
Reworded Question: What is the best assessment? longer depressed—sudden cheerful and energetic
Strategy: Determine if assessment or implementation mood does not indicate resolution of depression
is appropriate. 22. The Answer is 3
Needed Info: Pediculosis (lice). Assessment: scalp— The nurse is caring for clients in the GYN clinic. A
white eggs (nits) on hair shafts, itchy; body—macules client complains of an off-white vaginal discharge
and papules; pubis—red macules. Nursing with a curdlike appearance. The nurse notes the
consideration: discharge
OTC pyrethrin (RID, A-200), permethrin 1% and vulvular erythema. It would be MOST
(Nix); kills both lice and nits with 1 application; may important for the nurse to ask which of the following
suggest repeating in 7 days if necessary. questions?
Category: Assessment/Health Promotion and Reworded Question: What is a predisposing factor to
Maintenance developing candidiasis?
(1) Discuss basic hygiene with parents—makes an Strategy: “MOST important” indicates there may be
assumption; must assess first more than one correct response.
(2) Instruct the child not to sleep with her dog— Needed Info: Candida albicans. Symptoms: odorless,
must first assess to determine the problem cheesy white discharge; itching, inflames vagina and
perineum. Treatment: topical clotrimazole (Gyne-
Lotrimin), nystatin (Mycostatin). of the breastfeeding diabetic?
Category: Assessment/Health Promotion and Strategy: Determine the outcome of each answer
Maintenance choice.
(1) “Do you douche?”—not a factor in the Needed Info: Nursing care of diabetic during
development pregnancy: reinforce need for careful monitoring
of candidiasis throughout pregnancy; evaluate understanding of
(2) “Are you sexually active?”—candidiasis not usually modifications in diet/insulin coverage. Teach client
sexually transmitted; predisposing factors and significant other: diet (eat prescribed amount of
include glycosuria, pregnancy, and oral contraceptives food daily at same times); home glucose monitoring;
(3) “What kind of birth control do you use?”— insulin (purpose, dosage, administration, action,
CORRECT: oral contraceptives predispose individuals side effects, potential change in amount needed
to candidiasis during
(4) “Have you taken any cough medicine?”—no pregnancy as fetus grows and immediately after
relationship between cough medicine and candidiasis delivery); no oral hypoglycemics (teratogenic). Assist
23. The Answer is 1 with stress reduction; fetal surveillance.
The nurse is caring for a client in the prenatal clinic. Category: Planning/Health Promotion and
The nurse notes that the client’s chart contains the Maintenance
following information: blood type AB, Rh-negative; (1) “You will probably need less insulin while you
serology—negative; indirect Coombs test—negative; are breastfeeding.”—CORRECT: breastfeeding
fetal paternity—unknown. The nurse should anticipate has an antidiabetogenic effect; less insulin is
taking which of the following actions? needed
Reworded Question: What should the nurse do in this (2) “You will need to initially increase your insulin
situation? after the baby is born.”—insulin needs will
Strategy: Determine if it is appropriate to assess or decrease due to antidiabetogenic effect of
implement. breastfeeding
Needed Info: RhoGAM: given to unsensitized and physiological changes during immediate
Rhnegative postpartum period
mother after delivery or abortion of an (3) “You will be able to take an oral hypoglycemic
Practice Test Explanations instead of insulin after the baby is born.”—client
341 has IDDM: insulin required
Practice Test Answers Taensdt Rh-positive infant (4) “You will probably require the same dose of
or fetus to prevent development insulin that you are now taking.”—during third
of sensitization. Direct Coombs test done on cord trimester, insulin requirements increase due to
blood after delivery; if both are negative and neonate increased insulin resistance
is Rh-positive, mother is given RhoGAM. RhoGAM 25. The Answer is 2
is usually given to unsensitized mothers within 72 The nurse is caring for clients in a pediatric clinic.
hours of delivery, but may be effective up to 3–4 The mother of a 14-year-old male privately tells the
weeks after delivery. Administration of RhoGAM nurse that she is worried about her son because she
at 26–28 weeks’ gestation also recommended. Rho- unexpectedly walked into his room and discovered
GAM is ineffective against Rh-positive antibodies him masturbating. Which of the following responses
already present in the maternal circulation. by the nurse would be MOST appropriate?
Category: Implementation/Health Promotion and Reworded Question: What is the most therapeutic
Maintenance response?
(1) Administer Rho (D) immune globulin (Rho- Strategy: Remember therapeutic communication.
GAM)—CORRECT: no indication of sensitization; Needed Info: Male changes in puberty: increase in
RhoGAM will prevent possibility that genital size; breast swelling; pubic, facial, axillary,
she’ll become sensitized; given at 28 weeks’ gestation and chest hair; deepening voice; production of
if Coombs test is negative functional
(2) Schedule an amniocentesis—amniotic fluid sperm; nocturnal emissions. Psychosexual
aspirated by needle through abdominal and development: masturbation as expression of sexual
uterine walls to detect a genetic disorder tension; sexual fantasies; experimental sexual
(3) Obtain a direct Coombs test—obtained from intercourse.
newborns, not from pregnant woman Category: Implementation/Health Promotion and
(4) Assess maternal serum for alpha fetoprotein Maintenance
level—predicts neural tubal defects, done (1) “Tell your son he could go blind doing that.”—
between 16 and 18 weeks false information
24. The Answer is 1 TChoen tPernatc tRiecvei eTwes atnd Practice
The nurse is caring for a woman at 37 weeks’ for the NCLEX-RN ® Exam
gestation. 342
The client was diagnosed with insulin-dependent The Practice Test
diabetes mellitis (IDDM) at age 7. The client (2) “Masturbation is a normal part of sexual
states, “I am so thrilled that I will be breastfeeding development.”—
my baby.” Which of the following responses by the CORRECT: true statement provides
nurse is BEST? opportunity for sexual self-exploration
Reworded Question: What are the insulin (3) “He’s really too young to be masturbating.”—
requirements boys typically begin masturbating in early adolescence
(4) “Why don’t you give him more privacy?”— (4) “We will monitor you carefully to prevent cord
judgmental; prolapse.”—monitoring will not prevent prolapsed
doesn’t take advantage of opportunity to cord
teach 28. The Answer is 1
26. The Answer is 4 A primigravid woman comes to the clinic for her initial
The nurse performs a home visit on a client who prenatal visit. She is at 32 weeks’ gestation and
delivered 2 days ago. The client states that she is says that she has just moved from out of state. The
bottle-feeding her infant. The nurse notes white, client says that she has had periodic headaches during
curdlike her pregnancy, and that she is continually bumping
patches on the newborn’s oral mucous into things. The nurse notes numerous bruises in
membranes. The nurse should take which of the various stages of healing around the client’s breasts
following and abdomen. Vital signs are: BP 120/80, pulse
actions? 72, resp 18, and FHT 142. Which of the following
Reworded Question: What is the treatment for responses by the nurse is BEST?
thrush? Reworded Question: What is the best assessment?
Strategy: Determine the outcome of each answer Strategy: Determine if it is appropriate to assess or
choice. implement.
Needed Info: Thrush (oral candidiasis): white plaque Needed Info: Symptoms of domestic abuse: frequent
on oral mucous membranes, gums, or tongue; visits to physician’s office or emergency room for
treatment includes good handwashing, nystatin Practice Test Explanations
(Mycostatin). 343
Category: Implementation/Health Promotion and Practice Test Answers Taensdt unexplained
Maintenance trauma; client being cued, silenced, or
(1) Determine the newborn’s blood glucose level— threatened by an accompanying family member;
thrush in newborns is caused by poor handwashing evidence
or exposure to an infected vagina during of multiple old injuries, scars, healed fractures
birth seen on x-ray; fearful, evasive, or inconsistent replies,
(2) Suggest that the newborn’s formula be changed— or nonverbal behaviors such as flinching when
not related to thrush approached or touched. Nursing care: provide privacy
(3) Remind the caregiver not to let the infant sleep during initial interview to ensure perpetrator of
with the bottle—not related to thrush violence does not remain with client; carefully
(4) Explain that the newborn will need to receive document
some medication—CORRECT: thrush most all injuries (with consent); determine safety of
often treated with nystatin (Mycostatin) client by asking specific questions about weapons,
27. The Answer is 3 substance abuse, extreme jealousy; develop with
The nurse at the birthing facility is caring for a client
primipara a safety or escape plan; refer client to community
woman in labor, who is 4 cm dilated and resources.
25% effaced, and whose fetal vertex is at +1. The Category: Assessment/Health Promotion and
physician Maintenance
informs the client that an amniotomy is to be (1) “Are you battered by your partner?”—CORRECT:
performed. The client states, “My friend’s baby died evidence of injury should be investigated;
when the umbilical cord came out when her water assess head, neck, chest, abdomen, breasts,
broke. I don’t want you to do that to me!” Which of upper extremities
the following responses by the nurse is BEST? (2) “How do you feel about being pregnant?”—injuries
Reworded Question: What is the most therapeutic take priority
response? (3) “Tell me about your headaches.”—injuries take
Strategy: “BEST” indicates that there may be more priority
than one correct response. (4) “You may be more clumsy due to your size.”—
Needed Info: Amniotomy: artificial rupture of assumption; need to assess
membranes. 29. The Answer is 3
Presenting part should be engaged to prevent The nurse is teaching a class on natural family
cord prolapse. Obtain FHR before and after procedure. planning.
Assess color, odor, consistency of amniotic fluid. Which of the following statements by a client
Check maternal temperature q 2 hrs; notify head care indicates that teaching has been successful?
provider if temp is 100.4° F (38° C) or higher. Reworded Question: What is a true statement about
Category: Implementation/Health Promotion and natural family planning?
Maintenance Strategy: Think about each statement. Is it true
(1) “If you are that concerned, you should refuse the about natural family planning?
procedure.”—giving advice, nontherapeutic Needed Info: Natural family planning—Action:
(2) “The procedure will help your labor go faster.”— periodic abstinence from intercourse during fertile
doesn’t respond to client’s concerns period; based on regularity of ovulation; variable
(3) “That shouldn’t happen to you because the effectiveness. Teaching: fertile period may be
baby’s head is engaged.”—CORRECT: umbilical determined
prolapse usually occurs when the presenting by a drop in basal body temp before and a
part isn’t engaged slight rise after ovulation, and/or a change in cervical
mucus from thick, cloudy, and sticky during nonfertile be included in teaching of cast care; improving
period to more abundant, clear, thin, stretchy, circulation is best way to prevent impaired skin
and slippery during ovulation. integrity under cast
Category: Evaluation/Health Promotion and (4) Teaching the mother how to turn and position
Maintenance the child—no info provided about mobility of
(1) “When I ovulate, my basal body temperature will child; will prevent hazards of immobility
be elevated for 2 days and then will decrease.”— 31. The Answer is 3
basal body temp decreases prior to ovulation; The nurse is caring for a client who had a
after ovulation, temp increases thyroidectomy
(2) “My cervical mucus will be thick, cloudy, and 12 hours ago for treatment of Graves’ disease.
sticky when I ovulate.”—fertile mucus appears The nurse would be MOST concerned if which of the
clear, thin, watery, and stretchy following was observed?
(3) “Because I am regular, I will be fertile about 14 Reworded Question: What is a complication after a
days after the beginning of my period.”—CORRECT: thyroidectomy?
ovulation occurs approx. 14 days after Strategy: “MOST concerned” indicates a
start of menstrual period complication.
(4) “When I ovulate, my cervix will feel firm.”—cervix Needed Info: Nursing care for Graves’ disease/
softens slightly during ovulation hyperthyroidism: limit activities and provide frequent
30. The Answer is 1 rest periods; advise light, cool clothing; avoid
The home care nurse plans care for a child in a leg stimulants; use calm, unhurried approach; administer
cast for treatment of a fractured right ankle. The antithyroid medication, irradiation with I131 PO.
nurse enters the following nursing diagnosis on the Post-thyroidectomy care: low or semi-Fowler’s
care plan: skin integrity, risk for impaired. Which of position;
the following actions by the nurse is BEST? support head, neck, shoulders to prevent flexion
Reworded Question: What is the priority action to or hyperextension of suture line; tracheostomy
prevent skin breakdown? set at bedside; observe for complications—laryngeal
Strategy: Determine the outcome of each answer nerve injury, thyroid storm, hemorrhage, respiratory
choice. obstruction, tetany (decreased calcium from
Needed Info: Immediate nursing care for plaster cast: parathyroid
don’t cover cast until dry (48 hours), handle with involvement), check Chvostek’s and Trousseau’s
palms not fingertips; don’t rest on hard surfaces; signs.
elevate Category: Assessment/Physiological Integrity/
affected limb above heart on soft surface until Reduction of Risk Potential
dry; don’t use head lamp; check for blueness or (1) The client’s blood pressure is 138/82, pulse 84,
paleness, respirations 16, oral temp 99° F (37.2° C)—vital
pain, numbness, tingling (if present, elevate signs within normal limits
area; if it persists, contact physician); child should (2) The client supports his head and neck when
remain inactive while cast is drying. Intermediate turning his head to the right—prevents stress on
nursing care: mobilize client, isometric exercises; the incision
check for break in cast or foul odor; tell client not (3) The client spontaneously flexes his wrist when
to scratch skin under cast and not to put anything the blood pressure is obtained—CORRECT:
underneath cast; if fiberglass cast gets wet, dry with carpal spasms indicate hypocalcemia
hair dryer on cool setting. After-cast nursing care: (4) The client is drowsy and complains of a sore
wash skin gently, apply baby powder/cornstarch/ throat—expected outcome after surgery
baby oil; have client gradually adjust to movement 32. The Answer is 4
without support of cast; swelling is common, elevate A client is admitted with complaints of severe pain
limb and apply elastic bandage. in the lower right quadrant of the abdomen. To assist
Category: Implementation/Physiological Integrity/ with pain relief, the nurse should take which of the
Reduction of Risk Potential following actions?
(1) Teaching the child how to perform isometric Reworded Question: What is an appropriate
exercises nonpharmacological
of the right leg—CORRECT: contraction of method for pain relief?
TChoen tPernatc tRiecvei eTwes atnd Practice Strategy: Determine the outcome of each answer
for the NCLEX-RN ® Exam choice.
344 Needed Info: Establish a 24-hour pain profile. Teach
The Practice Test client about pain and its relief: explain quality and
muscle without moving joint; promotes venous location of impending pain; slow, rhythmic breathing
return and circulation, prevents thrombi; quadriceps to promote relaxation; effects of analgesics and
setting (push back knees into bed) and benefits of preventative approach; splinting
gluteal setting (push heels into bed) techniques
(2) Teaching the mother to gently massage the to reduce pain. Reduce anxiety and fears.
child’s right foot with emollient cream—will help Provide comfort measures: proper positioning; cool,
prevent dryness of foot but does not address skin well ventilated, quiet room; back rub; allow for rest.
under cast Category: Implementation/Physiological Integrity/
(3) Instructing the mother to keep the leg cast clean Basic Care and Comfort
and dry—because client is a young child, should
(1) Encourage the client to change positions (shock absorber for stability). Flex elbow 30 degrees
frequently and hold handle up; tip of cane should be 15 cm
in bed—unnecessary movement will lateral
increase pain, should be avoided to base of the 5th toe. Hold cane in hand opposite
(2) Massage the lower right quadrant of the affected extremity; advance cane and affected
abdomen— leg; lean on cane when moving good leg. To manage
if appendicitis is suspected, massage or stairs, step up on good leg, place the cane and
palpation should never be performed as these affected leg on step; reverse when going down (“up
actions may cause the appendix to rupture with the good, down with the bad”); same sequence
(3) Apply warmth to the abdomen with a heating used with crutches.
pad—if pain is caused by appendicitis, increased Category: Evaluation/Physiological Integrity/Basic
circulation from heat may cause appendix to Care and Comfort
rupture (1) The client holds the cane with his right hand,
(4) Use comfort measures and pillows to position moves the cane forward followed by the right leg,
the client—CORRECT: non-pharmacological and then moves the left leg—should hold cane
methods of pain relief with the stronger (left) hand
Practice Test Explanations (2) The client holds the cane with his right hand,
345 moves the cane forward followed by his left leg,
Practice Test Answers Taensdt 33. The Answer and then moves the right leg—should hold cane
is 2 with the stronger (left) hand
The nurse prepares a client for peritoneal dialysis. (3) The client holds the cane with his left hand,
Which of the following actions should the nurse take moves the cane forward followed by the right
FIRST? leg, and then moves the left leg—CORRECT: the
Reworded Question: What is the priority action for a cane acts as a support and aids in weight-bearing
client undergoing peritoneal dialysis? for the weaker right leg
Strategy: Determine if it is appropriate to assess or (4) The client holds the cane with his left hand,
implement. moves the cane forward followed by his left leg,
Needed Info: Peritoneal dialysis: takes place within and then moves the right leg—cane needs to be a
peritoneal cavity to remove excess fluids and waste support and aid in weight-bearing for the weaker
products usually removed by the kidneys. Procedure: right leg
rubber catheter surgically inserted into abdominal 35. The Answer is 3
cavity; 1–2 liters of fluid infused into peritoneal While caring for a client receiving total parenteral
space by gravity; fluid stays in cavity for approx. 20 nutrition (TPN) through a central line, the nurse
minutes; fluid drained by gravity. Complications: notices a small trickle of opaque fluid leaking from
peritonitis, abdominal pain, insufficient return of around the central line dressing. It is MOST important
fluid. Nursing care before procedure: obtain baseline for the nurse to take which of the following
vitals, breath sounds, weight, glucose, and electrolyte actions?
levels. During procedure: take vital signs, Reworded Question: What is the best action if the
ongoing assessment for respiratory distress, pain, nurse suspects a break in the central line?
discomfort; use aseptic technique; check abdominal Strategy: “MOST important” indicates there may be
dressing around catheter for wetness. more than one correct response.
Category: Implementation/Physiological Integrity/ TChoen tPernatc tRiecvei eTwes atnd Practice
Reduction of Risk Potential for the NCLEX-RN ® Exam
(1) Assess for a bruit and a thrill—used with 346
hemodialysis The Practice Test
through an AV fistula, graft, or shunt Needed Info: TPN: method of supplying nutrients to
(2) Warm the dialysate solution—CORRECT: solution the body by the IV route. Nursing care: site of
should be warmed to body temp in warmer catheter
or with heating pad; don’t use microwave oven; changed every 4 weeks, change IV tubing and
cold dialysate increases discomfort filters every 24 hours, dressing changed 2–3 times
(3) Position the client on the left side—client should a week and PRN; initial rate of infusion 50 mL/hr
be in supine or low Fowler’s position wearing a and gradually increased (100–125 mL/hr) as client’s
mask fluid and electrolyte tolerance permits; increased
(4) Insert a Foley catheter—unnecessary, client can rate of infusion causes hyperosmolar state (headache,
void without a catheter nausea, fever, chills, malaise); slowed rate of
34. The Answer is 3 infusion results in “rebound” hypoglycemia caused
The nurse teaches an elderly client with right-sided by delayed pancreatic reaction to change in insulin
weakness how to use a cane. Which of the following requirements.
behaviors by the client indicates that the teaching Category: Implementation/Physiological Integrity/
was effective? Pharmalogical and Parenteral Therapies
Reworded Question: What is the appropriate (1) Prepare to change the central line dressing—
technique dressing might be removed later to further assess
used to ambulate with a cane? the situation, but this is not the most important
Strategy: Determine the outcome of each answer action
choice. (2) Verify that the client is on antibiotics—no evidence
Needed Info: Cane tip should have concentric rings of line infection
(3) Place the client’s head lower than his feet— Needed Info: Standard precautions (barrier) used
CORRECT: with all clients: primary strategy for nosocomial
indicates a break in the line, which places infection control. Most important way to reduce
client at risk for air embolism; turn client on left Practice Test Explanations
side and place head lower than feet; notify physician 347
(4) Secure the Y-port where the lipids are infusing— Practice Test Answers Taensdt transmission of
leakage is occurring at the IV site, not the Y site pathogens. Gloves: use clean, nonsterile
36. The Answer is 2 when touching blood, body fluids, secretions,
A 46-year-old man is admitted to the hospital with excretions, contaminated articles; remove promptly
a fractured right femur. He is placed in balanced after use, before touching items and environmental
suspension traction with a Thomas splint and Pearson surfaces.
attachment. During the first 48 hours, the nurse Category: Evaluation/Safe and Effective Care
should assess the client for which of the following Environment/
complications? Safety and Infection Control
Reworded Question: What complication of a fracture (1) The NAP answers the phone while wearing
is seen in the first 48 hours? gloves—CORRECT: contaminated gloves
Strategy: Be careful! They are asking for the should be removed before answering the phone
complication (2) The NAP log-rolls the client to provide back
that occurs during the first 48 hours. Later care—correct way to roll a client to maintain
complications may be included as answer choices. proper alignment
Needed Info: Complications of fractures: (1) (3) The NAP places an incontinence diaper under
compartment the client—appropriate to use incontinence diapers
syndrome (increased pressure externally for this client
[casts, dressings] or internally [bleeding, edema] (4) The NAP positions the client on the left side,
resulting in compromised circulation); head elevated—appropriate position to prevent
Signs/Symptoms aspiration and protect the airway
(S/S): pallor, weak pulse, numbness, pain; (2) 38. The Answer is 3
shock: bone is vascular; (3) fat embolism; (4) deep A 70-year-old woman is brought to the emergency
vein thrombosis; (5) infection, avascular necrosis; (6) room for treatment after being found on the floor
delayed union, nonunion, malunion. by her daughter. X-rays reveal a displaced subcapital
Category: Assessment/Physiological fracture of the left hip and osteoarthritis. When
Integrity/Physiological comparing the legs, the nurse would most likely
Adaptation make which of the following observations?
(1) Pulmonary embolism—obstruction of pulmonary Reworded Question: What is a symptom of a hip
system by thrombus from venous system or fracture?
right side of heart; seen 2–3 days to several weeks Strategy: Think about each answer choice.
after fracture Needed Info: Symptoms of fracture: swelling, pallor,
(2) Fat embolism—CORRECT: fat moves into ecchymosis; loss of sensation to other body parts;
bloodstream from fracture; formed by alteration deformity; pain/acute tenderness; muscle spasms;
in lipids in blood; fat combines with platelets loss of function, abnormal mobility; crepitus (grating
to form emboli; S/S: abnormal behavior due sound on movement); shortening of affected
to cerebral anoxia (confusion, agitation, delirium, limb; decreased or absent pulses distal to injury;
coma), abnormal ABGs (pO2 below 60 mm affected extremity colder than contralateral part.
Hg), increased resp; chest pain, dyspnea, pallor, Emergency nursing care: immobilize joint above
hypertension, petechiae on chest, upper arms, and below fracture by use of splints before client is
abdomen; treatment: high Fowler’s, high moved; in open fracture, cover the wound with sterile
concentration dressings or cleanest material available, control
O2, ventilation with PEEP (positive bleeding by direct pressure; check temp, color,
end expiratory pressure) to decrease pulmonary sensation,
edema, IVs, steroids, Dextran to prevent shock capillary refill distal to fracture; in emergency
(3) Avascular necrosis—(seen later than 48 hrs) bone room, give narcotic adequate to relieve pain (except
loses blood supply and dies; seen with chronic in presence of head injury).
renal disease or prolonged steroid use; treatment: Category: Assessment/Physiological
bone graft, joint fusion, prosthetic replacement Integrity/Physiological
(4) Malunion—bone fragments heal in deformed Adaptation
position as a result of inadequate reduction and (1) The client’s left leg is longer than the right leg and
immobilization; treatment: surgical or manual externally rotated—leg is shorter due to contraction
manipulation to realign of muscles attached above and below fracture
37. The Answer is 1 site
The nurse is helping an NAP provide a bed bath (2) The client’s left leg is shorter than the right leg
to a comatose client who is incontinent. The nurse and internally rotated—leg is usually externally
should intervene if which of the following actions rotated
is noted? (3) The client’s left leg is shorter than the right leg
Reworded Question: What is an incorrect action? and adducted—CORRECT: extremity shortens
Strategy: “Should intervene” indicates that you are due to contraction of muscles attached above
looking for something wrong. and below fracture site, fragments overlap by
1–2 inches problems related to drinking in family relationships,
(4) The client’s left leg is longer than the right leg and work, etc.; help client to see/admit problem; confront
is abducted—extremity shortens and externally denial with slow persistence; maintain relationship
rotates with client; establish control of problem drinking;
39. The Answer is 3 provide support; Alcoholics Anonymous; disulfiram
The nurse is caring for a client with a cast on the left (Antabuse): drug used to maintain sobriety, based
leg. The nurse would be MOST concerned if which on behavioral therapy.
of the following is observed? Category: Analysis/Psychosocial Integrity
Reworded Question: What is a complication of a cast? (1) “My husband will do well as long as I keep him
Strategy: “MOST concerned” indicates a complication. engaged in activities that he likes.”—wife is
Needed Info: Immediate nursing care for plaster cast: accepting responsibility, codependent behavior
don’t cover cast until dry (48 hours), handle with (2) “My focus is learning how to live my life.”—
palms CORRECT: wife is working to change codependent
not fingertips; don’t rest on hard surfaces; elevate patterns
affected limb above heart on soft surface until dry; (3) “I am so glad that our problems are behind
don’t use head lamp; check for blueness or paleness, us.”—unrealistic; discharge is not the final step
pain, numbness, tingling (if present, elevate area; if of treatment
it (4) “I’ll make sure that the children don’t give my
persists, contact physician); client should remain husband any problems.”—wife is accepting
inactive responsibility, codependent behavior
while cast is drying. Intermediate nursing care: 41. The Answer is 3
mobilize client, isometric exercises; check for break in A nurse is caring for clients in the mental health
cast or foul odor; tell client not to scratch skin under clinic. A woman comes to the clinic complaining of
cast and not to put anything underneath cast; if insomnia and anorexia. The client tearfully tells the
fiberglass nurse that she was laid off from a job that she had
cast gets wet, dry with hair dryer on cool setting. held for 15 years. Which of the following responses
After-cast nursing care: wash skin gently, apply baby by the nurse would be MOST appropriate?
powder/cornstarch/baby oil; have client gradually Reworded Question: What is the most therapeutic
adjust to movement without support of cast; swelling response?
is common, elevate limb and apply elastic bandage. Strategy: Remember therapeutic communication.
TChoen tPernatc tRiecvei eTwes atnd Practice Needed Info: Nursing considerations, explore client’s
for the NCLEX-RN ® Exam understanding of the problem: focus on the present;
348 emphasize client’s strengths; avoid blaming;
The Practice Test determine
Category: Analysis/Physiological how client handled similar situations; provide
Integrity/Physiological support; mobilize client’s coping strategies.
Adaptation Category: Implementation/Psychosocial Integrity
(1) Capillary refill time is less than 3 seconds— (1) “Did your company give you a severance
capillary package?”—
refill time is within normal limits yes/no question, nontherapeutic
(2) Client complains of discomfort and itching—a (2) “Focus on the fact that you have a healthy, happy
casted extremity may itch or feel uncomfortable family.”—gives advice, false assurance
due to prolonged immobility (3) “Tell me what happened.”—CORRECT:
(3) Client complains of tightness and pain—CORRECT: explores situation; allows client to verbalize
client with a pressure ulcer usually (4) “Losing a job is common nowadays.”—dismisses
reports pain and tightness in the area; infection the client’s concern
or necrosis will result in feeling of warmth and a 42. The Answer is 1
foul odor A client with a history of alcoholism is brought to
(4) Client’s foot is elevated on a pillow—newly the emergency room in an agitated state. He is
casted extremity may be slightly elevated to help vomiting
relieve edema; should remain in correct anatomical and diaphoretic. He says he had his last drink
position and below heart level to allow 5 hours ago. The nurse would expect to administer
sufficient arterial perfusion which of the following medications?
40. The Answer is 2 Practice Test Explanations
The nurse is discharging a client from an inpatient 349
alcohol treatment unit. Which of the following Practice Test Answers Taensdt Reworded
statements Question: What is the best medication to
by the client’s wife indicates to the nurse that treat acute alcohol withdrawal?
the family is coping adaptively? Strategy: Think about the action of each drug.
Reworded Question: What indicates that the client’s Needed Info: Alcohol sedates the central nervous
family is coping with the client’s alcoholism? system; rebound during withdrawal. Early symptoms
Strategy: Think about what each statement means. occur 4–6 hours after last drink. Symptoms:
Needed Info: Nursing care for chronic alcohol tremors; easily startled; insomnia; anxiety; anorexia;
dependence: safety; monitor for withdrawal; reality alcoholic hallucinosis (48 hours after last drink).
orientation; increase self-esteem and coping skills; Nursing care: administer sedation as needed, usually
balanced diet; abstinence from alcohol; identify benzodiazepines; monitor vital signs, particularly
pulse; take seizure precautions; provide quiet, Needed Info: Elbows flexed at 20–30-degree angle
well-lit environment; orient client frequently; don’t when standing with hands on grips. Lift and move
leave hallucinating, confused client alone; administer walker forward 8–10 inches. With partial or
anticonvulsants as needed, thiamine IV or IM, nonweight-
and IV glucose. bearing, put weight on wrists and arms and
Category: Planning/Psychosocial Integrity step forward with affected leg, supporting self on
(1) Chlordiazepoxide hydrochloride (Librium)— arms, and follow with good leg. Nurse should stand
CORRECT: antianxiety; used to treat symptoms behind client, hold onto gait belt at waist as needed
of acute alcohol withdrawal; Side effects (S/E): for balance. Sit down by grasping armrest on affected
lethargy, hangover, agranulocytosis side, shift weight to good leg and hand, lower self
(2) Disulfiram (Antabuse)—used as a deterrent to into chair. Client should wear sturdy shoes.
compulsive drinking; contraindicated if client TChoen tPernatc tRiecvei eTwes atnd Practice
drank alcohol in previous 12 hours for the NCLEX-RN ® Exam
(3) Methadone hydrochloride (Dolophine)—opioid 350
analgesic; used to treat narcotic withdrawal; The Practice Test
syndrome, Category: Evaluation/Physiological Integrity/Basic
S/E: seizures, respiratory depression Care and Comfort
(4) Naloxone hydrochloride (Narcan)—narcotic (1) The client slowly pushes the walker forward 12
antagonist used to reverse narcotic-induced inches, then takes small steps forward while
respiratory depression; S/E: ventricular fibrillation, leaning on the walker—should not push the
seizures, pulmonary edema walker
43. The Answer is 1 (2) The client lifts the walker, moves it forward 10
An elderly client is admitted to the nursing home inches, and then takes several small steps forward—
setting. CORRECT: walker needs to be picked
The client is occasionally confused and her gait up, placed down on all legs
is often unsteady. Which of the following actions by (3) The client supports his weight on the walker
the nurse would be MOST appropriate? while advancing it forward, then takes small
Reworded Question: What are visual cues for a client steps while balancing on the walker—should not
who is confused? support weight on walker while trying to move it
Strategy: Determine the outcome of each answer (4) The client slides the walker 18 inches forward,
choice. then takes small steps while holding onto the
Needed Info: Nursing care for Alzheimer’s disease: walker for balance—walker should be picked
provide calm, predictable environment with regular up, not slid forward
routine; give clear and simple explanations; display 45. The Answer is 2
clock and calendar; color-code objects and areas; A nurse is supervising a group of elderly clients in
monitor medications and food intake; secure doors a residential home setting. The nurse knows that
leading from house/unit; gently distract and redirect the elderly are at greater risk of developing sensory
during wandering behavior; avoid restraints deprivation for which of the following reasons?
(increases combativeness); organize daily activities Reworded Question: Why do the elderly have sensory
into short, achievable steps; discourage long naps deprivation?
during the day. If client experiences catastrophic Strategy: Think about each answer choice.
reaction, remain calm and stay with client; provide Needed Info: Plan/Implementation: assist clients
distraction such as music, rocking, stroking. with adjusting to lifestyle changes; allow clients to
Category: Implementation/Psychosocial Integrity verbalize concerns; prevent isolation; provide
(1) Ask the woman’s family to provide personal assistance
items such as photos or mementos—CORRECT: as required.
provides visual stimulation to reduce sensory Category: Analysis/Psychosocial Integrity
deprivation (1) Increased sensitivity to the side effects of
(2) Select a room with a bed by the door so the medications—
woman can look down the hall—provides only many medications alter GI functioning
occasional stimulation but do not cause decreased vision, hearing, or
(3) Suggest the woman eat her meals in the room taste
with her roommate—needs to eat in the dining (2) Decreased visual, auditory, and gustatory
hall with others for stimulation abilities—
(4) Encourage the woman to ambulate in the halls CORRECT: gradual loss of sight, hearing,
twice a day—unsafe due to unsteady gait and and taste interferes with normal functioning
confusion (3) Isolation from their families and familiar
44. The Answer is 2 surroundings—
The nurse teaches an elderly client how to use a clients are in contact with other
standard aluminum walker. Which of the following residents and staff who provide stimulation
behaviors by the client indicates that the nurse’s (4) Decreased musculoskeletal function and
teaching was effective? mobility—
Reworded Question: What is the correct technique clients can be placed in wheelchairs and moved
when ambulating with a walker? 46. The Answer is 1
Strategy: Determine the outcome of each answer After receiving report, the nurse should see which of
choice. the following clients FIRST?
Reworded Question: Who is the priority client? young children will need help
Strategy: Think ABCs. 48. The Answer is 2
Needed Info: Consider the following factors: chief The nurse calculates the IV flow rate for a
complaint; age of client; medical history; potential postoperative
for life-threatening event client. The client is to receive 3,000 mL of
Category: Analysis/Safe and Effective Care Ringer’s lactate solution IV to run over 24 hours.
Environment/ The IV infusion set has a drop factor of 10 drops per
Management of Care milliliter. The nurse should regulate the client’s IV to
(1) A 14-year-old client in sickle-cell crisis with an deliver how many drops per minute?
infiltrated IV—CORRECT: IV fluids are critical Reworded Question: What is the IV flow rate?
to reduce clotting and pain Strategy: Remember the formula to calculate IV
(2) A 59-year-old client with leukemia who has flow rate: Total volume × drop factor divided by the
received half of a packed red blood cell transfusion— time in minutes.
no indication that client is unstable Needed Info: Ringer’s lactate: electrolyte solution
(3) A 68-year-old client scheduled for a used to expand extracellular fluid volume, and
bronchoscopy— reduce blood viscosity.
stable client Category: Implementation/Physiological Integrity/
(4) A 74-year-old client complaining of a leaky Pharmacological and Parenteral Therapies
colostomy bag—stable client (1) 18—incorrect
47. The Answer is 3 (2) 21—CORRECT: (3,000 × 10) divided by (24 ×
The home care nurse is visiting a client with a 60) = 30,000 divided by 1,440 = 20.8 = 21
diagnosis (3) 35—incorrect
of hepatitis of unknown etiology. The nurse (4) 40—incorrect
knows that teaching has been successful if the client 49. The Answer is 1
makes which of the following statements? A client with emphysema becomes restless and
Reworded Question: What is a correct statement confused.
about hepatitis? Which of the following actions should the
Strategy: Determine the outcome of each statement. nurse take next?
Needed Info: Hepatitis A (HAV): high risk groups Reworded Question: What should the nurse do to
include young children, institutions for custodial raise the oxygen levels of a client with emphysema?
care, international travelers; transmission by fecal/ Strategy: Determine the outcome of each answer
oral, poor sanitation; nursing considerations include choice.
prevention, improved sanitation, treat with gamma Needed Info: Emphysema: overinflation of alveoli
globulin early postexposure, no preparation of food. resulting in destruction of alveoli walls; predisposing
Hepatitis B (HBV): high risk groups include drug factors include smoking, chronic infections,
addicts, fetuses from infected mothers, homosexually environmental
active men, transfusions, health care workers; pollution. Teaching includes breathing exercises;
transmission by parenteral, sexual contact, blood/ stop smoking; avoid hot/cold air or allergens;
Practice Test Explanations instructions regarding medications; avoid crowds
351 or close contact with persons who have colds or flu;
Practice Test Answers Taensdt body fluids; adequate rest and nutrition; oral hygiene;
nursing considerations include vaccine prophylactic
(Heptavax-B, Recombivax HB), immune globulin flu vaccines; observe sputum for indications
(HBLg) postexposure, chronic carriers (potential of infection.
for chronicity 5–10%). Hepatitis C (HVC): high risk Category: Implementation/Physiological Integrity/
groups include transfusions, international travelers; Reduction of Risk Potential
transmission by blood/body fluids; nursing (1) Encourage the client to perform pursed-lip
considerations breathing—CORRECT: prevents collapse of
include great potential for chronicity. Delta lung unit and helps client control rate and depth
hepatitis: high risk groups same as for HBV; of breathing
transmission (2) Check the client’s temperature—confusion is
coinfects with HBV, close personal contact. probably due to decreased oxygenation
Category: Evaluation/Physiological Integrity/ (3) Assess the client’s potassium level—confusion
Reduction of Risk Potential is probably due to decreased oxygenation, not
(1) “I am so sad that I am not able to hold my electrolyte imbalance
baby.”—hepatitis not spread by casual contact (4) Increase the client’s oxygen flow rate to 5
(2) “I will eat after my family eats.”—can eat with L/min—should receive low flow oxygen to prevent
family; cannot share eating utensils carbon dioxide narcosis
(3) “I will make sure that my children don’t use my 50. The Answer is 2
eating utensils or drinking glasses.”—CORRECT: The nurse is caring for a client 1 day after an
to prevent transmission, families should abdominal-
not share eating utensils or drinking glasses; perineal resection for cancer of the rectum. The
wash hands before eating and after using toilet nurse should question which of the following orders?
(4) “I’m glad that I don’t have to get help taking Reworded Question: What is an incorrect behavior?
care of my children.”—need to alternate rest and Strategy: Determine the outcome of each answer
activity to promote hepatic healing; mothers of choice.
TChoen tPernatc tRiecvei eTwes atnd Practice 0.12 mg is ordered for the child. The bottle contains
for the NCLEX-RN ® Exam 0.05 mg of digoxin in 1 mL of solution. Which of
352 the following amounts should the nurse administer
The Practice Test to the child?
Needed Info: Skin care for stoma: effect on skin Reworded Question: How much of the medication
depends on composition, quality, consistency of should you give?
drainage, medication, location of stoma, frequency Strategy: Remember how to calculate dosages. Be
of removal of appliance adhesive. Principles of skin careful and don’t make math errors.
protection: use skin sealant under all tapes; use skin Needed Info: Formula: dose on hand over 1 mL =
barrier to protect skin around stoma; cleanse skin dose desired.
gently and pat dry, not rub; change appliance Category: Implementation/Physiological Integrity/
immediately Pharmacological and Parenteral Therapies
when seal breaks. (1) 1.2 mL—inaccurate
Category: Analysis/Physiological Integrity/Basic (2) 2.4 mL—CORRECT: 0.05 mg/1 mL = 0.12 mg/x
Care and Comfort mL, 0.05x = 0.12, x = 2.4 mL
(1) Discontinue the nasogastric tube when bowel (3) 3.5 mL—inaccurate
sounds are heard—this usually means peristalsis (4) 4.2 mL—inaccurate
has occurred and the physician will also order a 53. The Answer is 3
clear liquid diet, then advance as tolerated The nurse is caring for a client with chest pain in the
(2) Irrigate the colostomy—CORRECT: colostomy Emergency Department. Which of the following
begins to function 3–6 days after surgery laboratory
(3) Place petrolatum gauze over the stoma—done if findings would MOST concern the nurse?
no pouch in place; keeps stoma moist; cover with Reworded Question: What is the most significant lab
dry, sterile dressing value for an MI?
(4) Administer meperidine (Demerol) 50 mg IM for Strategy: “MOST concerned” indicates that you are
pain—prevents post-op pain looking for a problem.
51. The Answer is 4 Needed Info: Special tests are ordered for clients
The nurse is caring for a client 4 hours after suspected of having an myocardial infarction (MI).
intracranial Creatine phosphokinase (CK-MB) is usually ordered
surgery. Which of the following actions should Practice Test Explanations
the nurse take immediately? 353
Reworded Question: What is a priority after Practice Test Answers Taensdt along with total CK
intracranial in a client with chest pain to
surgery? determine whether the pain is due to a heart attack.
Strategy: Determine the outcome of each answer It may also be ordered in a client with a high total CK
choice. to determine whether damage is to the heart or other
Needed Info: Monitor vital signs hourly. Elevate muscles. Increased CK-MB can usually be detected
head 15–30 degrees to promote venous drainage in heart attack patients about 3–4 hours after onset
from brain. Avoid neck flexion and head rotation of chest pain. The concentration of CK-MB peaks
(support in cervical collar or neck rolls). Reduce in 18–24 hours and then returns to normal within
environmental stimuli. Prevent the Valsalva 72 hours.
maneuver; Category: Analysis/Physiological Integrity/Reduction
teach client to exhale while turning or moving of Risk Potential
in bed. Administer stool softeners. Restrict fluids (1) Erythrocyte sedimentation rate (ESR): 10 mm/
to 1,200–1,500 mL/day. Administer medications: hr—rate at which RBCs settle out of unclotted
osmotic diuretics, corticosteroid therapy, blood in 1 hour; indicates inflammation/neurosis;
anticonvulsant normal: men 1–15 mm/hr, women 1–20 mm/
meds. hr
Category: Implementation/Physiological Integrity/ (2) Hematocrit (Hct): 42%—relative volume of
Reduction of Risk Potential plasma to RBC; increased with dehydration;
(1) Turn, cough, and deep-breathe the client— decreased with fluid volume excess; normal: men
coughing is discouraged, can increase intracranial 40–45%, women 37–45%
pressure (3) Creatine phosphokinase (CK-MB): 4 ng/mL—
(2) Place the client with the neck flexed and head CORRECT: enzyme specific to myocardium;
turned to the side—will increase ICP; keep head indicates tissue necrosis or injury to heart muscle;
in a neutral position normal: 0–3 ng/mL
(3) Perform passive range-of-motion exercises— (4) Serum glucose: 100 mg/dL—indicates insulin
changes in client’s position can increase intracranial production; normal: 60–110 mg/dL
pressure 54. The Answer is 3
(4) Move the client to the head of the bed using a The nurse is caring for a client with cervical cancer.
turning sheet—CORRECT: client’s body should The nurse notes that the radium implant has become
be moved as a unit to prevent increased ICP; prevent dislodged. Which of the following actions should the
disruption of the ICP monitoring system nurse take FIRST?
52. The Answer is 2 Reworded Question: What is the best action when a
A 6-year-old child with a congenital heart disorder radium implant becomes dislodged?
is admitted with congestive heart failure. Digoxin Strategy: Think about the outcome of each answer
choice. smoking history, and age are suspicious of lung
Needed Info: Limit radioactive exposure: assign client cancer; wheezing caused by constrictive airways
to private room; place “Caution: Radioactive (4) Sputum culture—determines if infection is
Material” sign on door; wear dosimeter film badge present;
at all times when interacting with client (measures crackles present with infections
amount of exposure); do not assign pregnant nurse 56. The Answer is 3
to client; rotate staff caring for client; organize tasks The nurse is caring for a client with pernicious anemia.
so limited time is spent in client’s room; limit visitors; The nurse knows that her teaching has been
encourage client to do own care; provide shield successful if the client makes which of the following
in room. Client care: use antiemetics for nausea; statements?
consider Reworded Question: What is true about pernicious
body image; provide comfort measures; provide anemia?
good nutrition. Strategy: Determine the outcome of each answer
Category: Implementation/Safe and Effective Care choice.
Environment/Safety and Infection Control Needed Info: Pernicious anemia is caused by failure
(1) Grasp the implant with a sterile hemostat and to absorb vitamin B12 because of a deficiency of
carefully reinsert it into the client—the implant intrinsic factor from the gastric mucosa. Symptoms:
should be picked up with long-handled forceps, pallor, slight jaundice, glossitis, fatigue, weight loss,
not a hemostat, and deposited into a lead container paresthesias of hands and feet, disturbances of
in the room, not reinserted into the client balance
(2) Wrap the implant in a blanket and place it behind and gait. Treatment: vitamin B12 IM monthly.
a lead shield—the implant should be picked up Category: Evaluation/Physiological
with long-handled forceps and put into a lead Integrity/Physiological
container in the room Adaptation
(3) Ensure the implant is picked up with long-handled (1) “In order to get better, I will take iron pills.”—
forceps and placed in a lead container— pernicious anemia is due to vitamin B deficiency
CORRECT: the priority is to secure the implant (2) “I am going to attend smoking cessation
to prevent unwanted and dangerous radiation classes.”—no direct link to smoking
exposure; the implant should be picked up with (3) “I will learn how to perform IM injections.”—
long-handled forceps and then placed in a lead CORRECT: many clients are instructed how to
container; this equipment should be kept in the give monthly IM B12 injection
room of any client receiving this therapy so that (4) “I will increase my intake of carbohydrates.”—
it is readily available; institutional guidelines and pernicious anemia is caused by faulty absorption
procedures for managing dislodgement should of vitamin B12
be followed; radiology is usually involved as 57. The Answer is 2
soon as dislodgement occurs The nurse is caring for clients in the Emergency
(4) Obtain a dosimeter reading on the client and Department of an acute care facility. Four clients
report it to the physician—the priority is to have been admitted in the last 20 minutes. Which
secure the implant and place it into a lead container of the following admissions should the nurse see
55. The Answer is 3 FIRST?
The nurse in a primary care clinic is caring for a Reworded Question: Who is the priority client?
68-year-old man. History reveals that the client has Strategy: Think ABCs.
smoked 1 pack of cigarettes per day for 45 years and Needed Info: Factors to consider: chief complaint;
drinks 2 beers per day. He is complaining of a age of client; medical history; potential for
nonproductive lifethreatening
cough, chest discomfort, and dyspnea. event.
The nurse hears isolated wheezing in the right middle Category: Analysis/Physiological Integrity/Reduction
lobe. It would be MOST important for the nurse of Risk Potential
to complete which of the following orders? (1) A client complaining of chest pain that is
Reworded Question: Which order should the nurse unrelieved
complete first? by nitroglycerin—airway issue takes priority
Strategy: “MOST important” indicates the possibility (2) A client with third-degree burns to the face—
of more than one good answer. CORRECT: face, neck, chest, or abdominal
TChoen tPernatc tRiecvei eTwes atnd Practice burns result in severe edema, causing airway
for the NCLEX-RN ® Exam restriction
354 (3) A client with a fractured left hip—airway issue
The Practice Test takes priority
Needed Info: Symptoms to look for: cough; change in (4) A client complaining of epigastric pain—airway
a chronic cough; wheezing; recurring fever. issue takes priority
Category: Analysis/Physiological Integrity/Reduction 58. The Answer is 4
of Risk Potential The nurse is caring for a client with a diagnosis of
(1) Pulmonary function tests—evaluates lung COPD, bronchitis-type, in the long-term care facility.
capacity; done for constrictive disease such as The client is wheezing, and his oxygen saturation
asthma is 85%. Four hours ago, the oxygen saturation
(2) Echocardiogram—determines cardiac structure was 88%. It is MOST important for the nurse to take
(3) Chest x-ray—CORRECT: client’s symptoms, which of the following actions?
Reworded Question: What is the best action for a should assign the new client to which of the following
client roommates?
with COPD? Reworded Question: Who is the appropriate
Strategy: Determine the outcome of each answer roommate
choice. for a client with burns?
Needed Info: Emphysema: overinflation of alveoli Strategy: Think about the transmission of diseases.
resulting in destruction of alveoli walls; predisposing Needed Info: Droplet precautions: used with
factors include smoking, chronic infections, pathogens
environmental pollution. Teaching includes breathing transmitted by infectious droplets; involves contact
exercises; stop smoking; avoid hot/cold air or of conjunctivae or mucous membranes of nose
allergens; instructions regarding medications; avoid or mouth, or during coughing, sneezing, talking, or
crowds or close contact with persons who have colds procedures such as suctioning or bronchoscopy;
or flu; adequate rest and nutrition; oral hygiene; private
Practice Test Explanations room or with client with same infection; spatial
355 separation of 3 feet between infected client and
Practice Test Answers Taensdt prophylactic flu visitors
vaccines; observe sputum for indications or other clients; door may remain open; place
of infection. mask on client during transportation.
Category: Implementation/Physiological Integrity/ Category: Implementation/Safe and Effective Care
Pharmacological and Parenteral Therapies Environment/Safety and Infection Control
(1) Administer beclomethasone, 2 puffs per (1) A 2-year-old with chickenpox—infectious disease
metered-dose inhaler—administer brochodilator (2) A 4-year-old with asthma—CORRECT: client
first to open passageways not infectious; lowest risk of cross-contamination
(2) Listen to breath sounds—situation does not (3) A 9-year-old with acute diarrhea—requires contact
require further assessment precautions
(3) Increase oxygen to 4 L per mask—increased (4) A 10-year-old with methicillin-resistant
oxygen Staphylococcus
levels in client’s blood may lead to respiratory aureus (MRSA)—requires contact
depression isolation
(4) Administer albuterol, 2 puffs per metered-dose 61. The Answer is 4
inhaler—CORRECT: brochodilator, relaxes The nurse teaches a client about elastic stockings.
bronchial smooth muscles Which of the following statements by the client
59. The Answer is 4 indicates
The nurse is caring for a client hospitalized for to the nurse that teaching was successful?
observation Reworded Question: What is a correct statement
after a fall. The client states, “My friend fell about elastic stockings?
last year, and no one thought anything was wrong. Strategy: Determine the outcome of each answer
She died 2 days later!” Which of the following choice.
responses by the nurse is BEST? Needed Info: Maintain pressure on muscles of the
Reworded Question: What is the most therapeutic lower extremities. Don’t use if there are any skin
response? lesions or gangrenous areas. Remove and reapply at
Strategy: Remember therapeutic communication. least twice per day. Stockings should be clean and
Needed Info: Therapeutic communication: using dry.
silence allows client time to think and reflect; conveys TChoen tPernatc tRiecvei eTwes atnd Practice
acceptance; allows client to take lead in conversation; for the NCLEX-RN ® Exam
using general leads or broad openings 356
(encourages client to talk, indicates interest in client); The Practice Test
clarification (encourages description of feelings Category: Evaluation/Physiological Integrity/Basic
and details of particular experience; makes sure Care and Comfort
nurse understands client); reflecting (paraphrases (1) “I will wear the stockings until the physician tells
what client says; reflects what client says, especially me to remove them.”—remove daily for bathing
feelings conveyed). and inspection of the extremities
Category: Implementation/Psychosocial Integrity (2) “I should wear the stockings even when I am
(1) “This happens to quite a few people.”— asleep.”—elastic stockings promote venous
nontherapeutic; return; not necessary during prolonged periods
doesn’t address client’s concerns of sleep
(2) ‘We are monitoring you, so you’ll be okay.”— (3) “Every 4 hours I should remove the stockings for
nontherapeutic; “don’t worry” response a half-hour.”—stockings should be worn when
(3) “Don’t you think I’m taking good care of you?”— client is up to promote venous return
nontherapeutic; focus is on the nurse (4) “I should put on the stockings before getting out
(4) “You’re concerned that it might happen to of bed in the morning.”—CORRECT: promote
you?”—CORRECT: reflects client’s feelings venous return by applying external pressure on
60. The Answer is 2 veins
The nurse is caring for clients on the pediatric unit. 62. The Answer is 3
An 8-year-old client with second- and third-degree The nurse is teaching a client who is scheduled for
burns on the right thigh is being admitted. The nurse a paracentesis. Which of the following statements
by the client to the nurse indicates that teaching has cough—may cause fractures of the ribs; percussion
been successful? helps loosen thick secretions
Reworded Question: What is a correct statement 64. The Answer is 3
about paracentesis? A client is admitted to the hospital with a diagnosis
Strategy: Determine the outcome of each answer of chronic bronchitis. He has a 10-year history of
choice. emphysema. The nurse should place him in which
Needed Info: Paracentesis: removal of fluid from the of the following positions?
peritoneal cavity; 2–3 liters may be removed. Prep: Practice Test Explanations
informed consent; void, take vital signs; measure 357
abdominal girth; weigh client. During procedure: Practice Test Answers Taensdt Reworded
take vital signs q 15 minutes. After procedure: Question: What is the best position for a
document client with a respiratory problem?
amount, color, characteristics of drainage Strategy: Picture the client as described.
obtained; assess pressure dressing for drainage; Needed Info: Fowler’s position: 45–60 degrees. High
position Fowler’s position: 80–90 degrees. Used to promote
in bed until vital signs are stable. cardiac and respiratory function. Chronic bronchitis
Category: Evaluation/Physiological Integrity/ S/S: productive cough, wheezing, shortness of breath
Reduction of Risk Potential (SOB), exercise intolerance. Treatment:
(1) “I will be in surgery for less than an hour.”—not bronchodilators,
a surgical procedure antihistamines, steroids, antibiotics, expectorants.
(2) “I must not void prior to the procedure.”—bladder Theophylline: bronchodilator. Side effects:
is emptied prior to the procedure to prevent restlessness, dizziness, palpitations, tachycardia,
puncture anorexia. Emphysema S/S: marked dyspnea on
(3) “The physician will remove 2 to 3 liters of exertion
fluid.”—CORRECT: fluid removed slowly to that proceed to dyspnea at rest, use of accessory
decrease ascites; can remove up to 4–6 liters in muscles for breathing, barrel chest, “pink puffer”
severe cases (normal O2 level and dyspnea). Treatment: low-flow
(4) “I will lie on my back and breathe slowly.”— O2 (1–3 L/min), CO2 resp stimulus obliterated.
positioned Category: Implementation/Physiological Integrity/
in an upright position with feet supported Basic Care and Comfort
63. The Answer is 2 (1) Side-lying—diaphragm against abdominal
The home care nurse is performing chest organs
physiotherapy (2) Supine—can’t breathe
on an elderly client with chronic airflow limitations (3) High Fowler’s—CORRECT: head of bed elevated
(CAL). Which of the following actions should 60–90 degrees; gravity displaces abdominal
the nurse take FIRST? organs
Reworded Question: What should the nurse do prior (4) Semi-Fowler’s—head of bed elevated 30–45
to beginning chest physiotherapy? degrees
Strategy: Determine whether to assess or implement. 65. The Answer is 2
Needed Info: Postural drainage: uses gravity to A client is to receive 1,000 mL of 5% dextrose in 0.45
facilitate NaCl intravenous solution in an 8-hour period. The
removal of bronchial secretions; client is placed intravenous set delivers 15 drops per milliliter. The
in a variety of positions to facilitate drainage into nurse should regulate the flow rate so it delivers how
larger airways; secretions may be removed by many drops of fluid per minute?
coughing Reworded Question: What is the correct IV flow rate?
or suctioning. Percussion and vibration: usually Strategy: Use the correct formula and be careful not
performed during postural drainage to augment to make math errors.
the effect of gravity drainage; percussion: rhythmic Needed Info: Formula: total volume × drip factor
striking of chest wall with cupped hands over areas divided by the total time in minutes.
where secretions are retained; vibration: hand and Category: Planning/Physiological
arm muscles of person doing vibration are tensed, Integrity/Pharmacological
and a vibrating pressure is applied to chest as client and Parenteral Therapies
exhales. (1) 15—incorrect
Category: Assessment/Physiological Integrity/ (2) 31—CORRECT: (1,000 × 15) divided by (8 × 60)
Reduction of Risk Potential (3) 45—incorrect
(1) Perform chest physiotherapy prior to meals— (4) 60—incorrect
prevents nausea, vomiting, aspiration 66. The Answer is 2
(2) Auscultate the chest prior to beginning the The nurse knows the plan of care for a client with
procedure— severe liver disease should include which of the
CORRECT: identify areas of the lung following
that require drainage; auscultate chest at end of actions?
procedure to determine effectiveness Reworded Question: What is included in the plan of
(3) Administer bronchiodilators after the procedure— care?
given before chest physiotherapy to dilate Strategy: Determine the outcome of each answer
the bronchioles and to liquify secretions choice.
(4) Percuss each lobe prior to asking the client to Needed Info: Nutrition—Early stages: high-protein,
high-carb diet; advanced stages: fiber, protein, fat, milliliter. The nurse should regulate the flow rate so
and sodium restrictions; small, frequent feedings; that the client receives how many drops of fluid per
fluid restriction; avoid alcohol. Administer blood minute?
products; observe vital signs for shock; monitor Reworded Question: How should you regulate the IV
abdominal girth; maintain skin integrity; assess flow rate?
degree of jaundice; promote rest; promote adequate Strategy: Use the formula and avoid making math
respiratory function; reduce exposure to infection; errors.
reduce ascites (sodium and fluid restrictions, Needed Info: Formula: total volume × the drop factor
diuretics). divided by the total time in minutes
Category: Implementation/Physiological Integrity/ Category: Planning/Physiological
Basic Care and Comfort Integrity/Pharmacological
(1) Administer Kayexelate enemas—decreases and Parenteral Therapies
serum potassium levels (1) 21—inaccurate
(2) Offer a low-protein, high-carbohydrate diet— (2) 28—inaccurate
CORRECT: hepatic coma caused by increased (3) 31—CORRECT: (3,000 × 15) divided by (24 ×
levels of ammonia from breakdown of protein 60)
(3) Insert a Sengsteken-Blakemore tube—applies (4) 42—inaccurate
pressure against bleeding esophageal varices 69. The Answer is 3
(4) Administer salt-poor albumin IV—balances The nurse is supervising care of a client receiving
osmotic pressure TPN through a single-lumen percutaneous central
67. The Answer is 2 catheter. The nurse would be MOST concerned if
A client with a diagnosis of delirium is admitted to which of the following was observed?
the hospital. To evaluate the cause of a client’s Reworded Question: What is an incorrect action?
delirium, Strategy: “MOST concerned” indicates that you are
blood is sent to the laboratory for analysis. The looking for an incorrect intervention.
results are as follows: NA+ 156, Cl− 100, K+ 4.0, Needed Info: TPN: method of supplying nutrients to
HCO3 the body by the IV route. Nursing care: site of
21, BUN 86, glucose 100. Based on these laboratory catheter
results, the nurse should record which of the following changed every 4 weeks, change IV tubing and
nursing diagnoses on the client’s care plan? filters every 24 hours, dressing changed 2–3 times
Reworded Question: What nursing diagnosis is a week and PRN; initial rate of infusion 50 mL/hr
appropriate? and gradually increased (100–125 mL/hr) as client’s
Strategy: Determine if each lab value is normal or fluid and electrolyte tolerance permits; increased
abnormal. Decide what the abnormal lab values rate of infusion causes hyperosmolar state (headache,
indicate about the client and how it would influence nausea, fever, chills, malaise); slowed rate of
TChoen tPernatc tRiecvei eTwes atnd Practice infusion results in “rebound” hypoglycemia caused
for the NCLEX-RN ® Exam by delayed pancreatic reaction to change in insulin
358 requirements.
The Practice Test Category: Evaluation/Physiological
your development of appropriate nursing diagnoses Integrity/Pharmacological
for that client. and Parenteral Therapies
Needed Info: Normal Na+: 135–145 mEq/L. (1) The client receives insulin through the
Hypernatremia: singlelumen—
dehydration and insufficient water intake. insulin compatible with TPN solution
Normal Cl: 95–105 mEq/L. Normal K: 3.5–5.0 (2) A mask is placed on the client when changing the
mEq/L. Normal HCO3: 22–26 mEq/L. Decreased client’s dressing—decreases chance of airborne
levels seen with starvation, renal failure, diarrhea. contamination; nurse also wears mask when
Normal BUN (blood, urea, nitrogen): 6–20 mg/100 changing dressing
mL. Elevated levels indicate rapid protein catabolism, (3) The client’s dressing is changed daily using sterile
kidney dysfunction, dehydration. Normal glucose: technique—CORRECT: dressing changed
70–100 mg/dL. 1–2 times per week and PRN
Category: Analysis/Physiological Integrity/Reduction (4) The client is weighed 2–3 times per week—assess
of Risk Potential fluid balance
(1) Alteration in patterns of urinary elimination— 70. The Answer is 2
would have altered K+ The nurse is caring for clients in the outpatient clinic.
(2) Fluid volume deficit—CORRECT: elevated Na+, A client tells the nurse that he developed weakness
decreased CO2, elevated BUN, other values are and numbness in the legs the previous day and now
normal; elevated Na+ and BUN seen with dehydration his body feels the same way. The client’s vital signs
(3) Nutritional deficit: less than body requirements— are: BP 120/60, pulse 86, and resp 20. The client
seen with decreased CO2, but would denies any pain but appears anxious to the nurse.
have altered K+ It would be MOST important for the nurse to ask
(4) Self-care deficit: feeding—no information to which of the following questions?
support this Practice Test Explanations
68. The Answer is 3 359
A client is to receive 3,000 mL of 0.9% NaCl IV in Practice Test Answers Taensdt Reworded
24 hours. The intravenous set delivers 15 drops per Question: What is a possible cause of Guillain-
Barré syndrome (GBS)? blame others, make excuses, and provide alibis for
Strategy: Determine the relationship between the self-focused behaviors.
answers and Guillain-Barré syndrome (GBS). Category: Analysis/Psychosocial Integrity
Needed Info: GBS Plan/Implementation: intervention (1) “I’m sick of hearing about all your life tragedies.”—
is symptomatic; steroids in acute phase; CORRECT: lack of empathy is the main
plasmapheresis; characteristic of a narcissistic personality disorder
aggressive respiratory care; prevent (2) “I know I’m interrupting others. So what?”—
hazards of immobility; maintain adequate nutrition; prominent behavior in an antisocial personality
physical therapy; pain-reducing measures; eye care; disorder
prevention of complications (UTI, aspiration); (3) “I just can’t stop wanting to slash myself.”—
psychosocial characteristic of a borderline personality disorder
support. (4) “I just have no hope for the future.”—
Category: Assessment/Physiological characteristic
Integrity/Physiological of depression
Adaptation 73. The Answer is 4
(1) “Have you recently fallen or had some other type A teenage client is admitted to the hospital with
of physical injury?”—symptoms consistent with anorexia nervosa. Which of the following statements
GBS; not related to injury by the client requires immediate follow-up by
(2) “Have you recently had a viral infection, such as the nurse?
a cold?”—CORRECT: GBS often preceded by a Reworded Question: Which problem has the highest
viral infection as well as immunizations/vaccinations priority for this client?
(3) “Have you recently taken any over-the-counter TChoen tPernatc tRiecvei eTwes atnd Practice
medication?”—no association with symptoms; for the NCLEX-RN ® Exam
appropriate question for health history 360
(4) “Have you recently experienced headaches?”— The Practice Test
GBS affects peripheral, not central nervous Strategy: Remember Maslow’s hierarchy of needs.
system Needed Info: Anorexia nervosa: a disorder
71. The Answer is 1 characterized
The nurse is admitting a client who is jaundiced by restrictive eating resulting in emaciation,
due to pancreatic cancer. The nurse should give the disturbance in body image, and an intense fear of
HIGHEST priority to which of the following needs? being obese. Physical needs must be met first in
Reworded Question: What is the highest priority for order to keep the client in stable condition. A difficult
a client with pancreatic cancer? area to maintain is that of appropriate hydration
Strategy: Remember Maslow. and fluid and electrolyte balance.
Needed Info: Medical treatment: high-calorie, bland, Category: Planning/Psychosocial Integrity
low-fat diet; small, frequent feedings; avoid alcohol; (1) “My gums were bleeding this morning.”—vitamin
anticholinergics; antineoplastic chemotherapy deficiencies occur in anorectic clients, but
Category: Planning/Physiological Integrity/Reduction not the highest priority
of Risk Potential (2) “I’m getting fatter every day.”—body image
(1) Nutrition—CORRECT: profound weight loss disturbance is a perceptual problem with anorectics,
and anorexia occur with pancreatic cancer but not the highest priority; this is a psychosocial
(2) Self-image—jaundiced clients are concerned need
about how they look, but physiological needs (3) “Nobody likes me because I’m so ugly.”—chronic
take priority low self-esteem is a psychodynamic factor, but
(3) Skin integrity—jaundice causes dry skin and not the highest priority; this is a psychosocial
pruritis; scratching can lead to skin breakdown need
(4) Urinary elimination—urine is dark due to (4) “I’m feeling dizzy and weak today.”—CORRECT:
obstructive process; kidney function is not fluid volume deficit is client’s highest
affected priority; dehydration is common and could lead
72. The Answer is 1 to irreversible renal damage and vital sign alterations
Which of the following statements by a client during 74. The Answer is 4
a group therapy session would the nurse identify as A client is admitted to the hospital for treatment of
reflecting a client’s narcissistic personality disorder? Pneumocystis jiroveci pneumonia and Kaposi’s
Reworded Question: Which statement would a client sarcoma.
with narcissistic personality disorder be most likely The client tells the nurse that he has been considering
to make? organ donation when he dies. Which of the
Strategy: Think about each answer choice. following responses by the nurse is BEST?
Needed Info: Clients with narcissistic personality Reworded Question: Can this client be an organ
disorder display grandiosity about their self- donor?
importance Strategy: Think about each answer choice.
and achievements. These clients overvalue Needed Info: Criteria for organ/tissue donation: no
themselves and are indifferent to others’ criticism; history of significant disease process in organ/tissue
the feelings of others are not understood or to be donated; no untreated sepsis; brain death of
considered. donor; no history of extracranial malignancy; relative
They have a sense of entitlement and expect hemodynamic stability; blood group compatibility;
special treatment, and also use rationalization to newborn donors must be full-term (more than
200 g); only absolute restriction to organ donation is Needed Info: Hydrate client before test. Encourage
documented case of HIV infection. Family members hourly intake of water during test. Have client void
can give consent. Nurse can discuss organ donation and discard urine; note the time; save all urine for
with other death-related topics (funeral home to be specified time. Do not contaminate specimen with
used, autopsy request). feces or toilet paper.
Category: Implementation/Physiological Integrity/ Category: Implementation/Physiological Integrity/
Physiological Adaptation Reduction of Risk Potential
(1) “What does your family think about your (1) Obtain an order from the physician for insertion
decision?”— of a Foley catheter—not necessary unless client
client has the right to make the decision is incontinent
(2) “You will help many people by donating your (2) Obtain the client’s weight prior to beginning the
organs.”—clients with documented HIV are prohibited urine collection—not necessary
from donating organs (3) Discard the last voided specimen prior to ending
(3) “Would you like to speak to the organ donor the collection—collect all urine voided during
representative?”— the time period
passing the responsibility (4) Ask if a preservative is present in the container—
(4) “That is not possible based on your illness.”— CORRECT: save all urine in a container with no
CORRECT: clients with documented HIV are preservatives; refrigerate or keep on ice
prohibited from donating organs 77. The Answer is 1
75. The Answer is 3 The nurse is planning discharge teaching for a client
The nurse is caring for a client 5 hours after a with Parkinson’s disease. To maintain safety, the
pancreatectomy nurse should make which of the following suggestions
for cancer of the pancreas. On assessment, to the family?
the nurse notes that there is minimal drainage from Reworded Question: What is a correct client teaching
the nasogastric (NG) tube. It is MOST important for Parkinson’s disease?
for the nurse to take which of the following actions? Strategy: Determine the outcome of each answer
Reworded Question: What is the best action when an choice.
NG tube is not draining? Needed Info: Symptoms: tremors, akinesia, rigidity,
Strategy: Determine whether it is appropriate to weakness, “motorized propulsive gait, slurred
assess or implement. monotonous speech, dysphagia, drooling, mask-like
Needed Info: Insertion of Levin/Salem sump: expression. Nursing care: Encourage finger exercises.
measure Administer Artane, Cogentin, L-dopa, Parlodel,
distance from tip of nose to earlobe, plus distance Sinemet, Symmetrel. Teach client ambulation
from earlobe to bottom of xyphoid process. modification. Promote family understanding of the
Mark distance on tube with tape and lubricate end disease (intellect/sight/hearing not impaired, disease
of tube. Insert tube through nose to stomach. Offer progressive but slow, doesn’t lead to paralysis). Refer
sips of water and advance tube gently; bend head for speech therapy, potential stereotactic surgery.
forward. Category: Implementation/Physiological Integrity/
Observe for respiratory distress. Secure with Basic Care and Comfort
hypoallergenic tape. Verify tube position initially (1) Install a raised toilet seat—CORRECT: helps
and before feeding. Aspirate for gastric contents and client to be independent; slightly elevate the back
check pH. Inject approx. 15 mL of air into stomach leg of chairs
while listening over epigastric area (not always (2) Obtain a hospital bed—no indications that this
accurate). is needed
Category: Assessment/Physiological Integrity/Basic (3) Instruct the client to hold his arms in a dependent
Care and Comfort position when ambulating—should swing
(1) Notify the physician—should assess first arms to assist in balance when walking
(2) Monitor vital signs q 15 minutes—does not (4) Perform an exercise program during the late
address lack of drainage afternoon—activities should be scheduled for
Practice Test Explanations late morning when energy level is highest and
361 client won’t be rushed
Practice Test Answers Taensdt (3) Check the 78. The Answer is 3
tubing for kinks—CORRECT: assess The nurse is performing discharge teaching for a client
prior to implementing; maintain tubing in a with chronic pancreatitis. Which of the following
dependent position statements by the client to the nurse indicates that
(4) Replace the NG tube—assess before further teaching is necessary?
implementing Reworded Question: What is an incorrect statement
76. The Answer is 4 about pancreatitis?
When collecting a 24-hour urine specimen for Strategy: This is a negative question; you are looking
creatinine for incorrect information.
clearance, it is MOST important for the nurse Needed Info: Plan/Implementation: NPO, gastric
to do which of the following? decompression. Meds: antacids, analgesics,
Reworded Question: What is the correct procedure antibiotics,
for a 24-hour urine analysis? anticholinergics. Maintain fluid/electrolyte
Strategy: “MOST important” indicates there may be balance. Monitor for signs of infection. Cough and
more than one response that appears correct. deep breathe; semi-Fowler’s position. Monitor for
shock and hyperglycemia. TPN. Treatment of exocrine vaccine to which of the following clients?
insufficiency: meds containing amylase, lipase, Reworded Question: What is a contraindication to
trypsin to aid digestion. Long-term: avoid alcohol; receiving flu vaccine?
low-fat, bland diet; small, frequent meals. Monitor Strategy: Think about what each answer choice
S/S of diabetes mellitus. means.
TChoen tPernatc tRiecvei eTwes atnd Practice Needed Info: Influenza vaccine: given yearly,
for the NCLEX-RN ® Exam preferably
362 Oct.–Nov.; recommended for people age 65
The Practice Test or older; people under 65 with heart disease, lung
Category: Evaluation/Physiological Integrity/ disease, diabetes, immunosuppression; chronic care
Reduction of Risk Potential facility residents
(1) “I do not have to restrict my physical activity.”— Category: Assessment/Health Promotion and
no specific restrictions on activity Maintenance
(2) “I should take pancrelipase before meals.”— (1) A 45-year-old male who is allergic to shellfish—
pancreatic enzyme replacement; take before or allergy to eggs is a contraindication
with meals (2) A 60-year-old female who says she has a sore
(3) “I will eat 3 meals per day.”—CORRECT: small, throat—CORRECT: vaccine deferred in presence
frequent feedings are most beneficial of acute respiratory disease
(4) “I am not allowed to drink any alcoholic (3) A 66-year-old female who lives in a group
beverages.”— home—vaccine deferred only if client has an
complete abstinence from alcohol active immunization
required (4) A 70-year-old female with congestive heart
79. The Answer is 3 failure—
After a laparoscopic cholecystectomy, the client no contraindication
complains of abdominal pain and bloating. Which 81. The Answer is 2
of the following responses by the nurse is BEST? An arterial blood gas is ordered for a man after a
Reworded Question: What is the best intervention for myocardial infarction. After obtaining the specimen,
a client complaining of free air pain? it would be MOST appropriate for the nurse to
Strategy: “BEST” indicates there may be more than take which of the following actions?
one response that appears correct. Reworded Question: What is the priority action after
Needed Info: Cholecystectomy: removal of an ABG?
gallbladder. Strategy: Take care of the client first.
T-tube inserted to ensure drainage of bile Needed Info: ABG: measurement of partial pressure
from common bile duct until edema diminishes. of oxygen, CO2, and pH of blood; assessment of
Check amount of drainage (usually 500–1,000 mL/ Practice Test Explanations
day, decreases as fluid begins to drain into 363
duodenum). Practice Test Answers Taensdt acid-base status of
Protect skin around incision from bile body. Use a heparinized syringe.
drainage irritation (use zinc oxide or water-soluble Needle inserted 45–60 degrees to skin surface and
lubricant). Keep drainage bag at same level as advanced into radial artery. Apply pressure after
gallbladder. needle is removed. Put specimen on ice.
Maintain client in semi-Fowler’s position Category: Implementation/Physiological Integrity/
after T-tube is removed; observe dressing for bile; Reduction of Risk Potential
notify physician if there is significant drainage. (1) Obtain ice for the specimen—should be done,
Evaluate but not the most important
pain to check for other problems. Monitor for (2) Apply direct pressure to the site—CORRECT:
S/S of K+ and Na+ loss; flattened or inverted T waves prevents bleeding, hematoma; maintain for 5
on EKG; muscle weakness; abdominal distension; minutes, 15 minutes if on anticoagulant
headache; apathy; nausea or vomiting; jaundice. (3) Apply a sterile dressing to the site—Band-Aid is
Category: Implementation/Physiological Integrity/ applied
Physiological Adaptation (4) Observe the site for hematoma formation—more
(1) “Increase your intake of fresh fruits and important to prevent hematoma
vegetables.”— 82. The Answer is 1
no indication of constipation The nurse is caring for a man who was involved in
(2) “I’ll give you the prescribed pain medication.”— an auto accident the previous day. The client has a
less pain medication needed with laparoscopic double-lumen tracheostomy tube with a cuff. Which
procedure of the following actions should the nurse perform?
(3) “Why don’t you take a walk in the hallway.”— Reworded Question: What is a correct action when
CORRECT: “free air” pain caused by CO2; caring for a tracheostomy?
ambulation will increase absorption Strategy: Determine the outcome of each answer
(4) “You may need an indwelling catheter.”—pain choice.
due to retention of CO2 Needed Info: Cuffed tracheostomy tube permits
80. The Answer is 2 mechanical ventilation and seals off lower airways.
The nurse in an outpatient clinic is supervising student Inject air with a syringe into one-way valve in pilot
nurses administering influenza vaccinations. line. Nursing responsibilities: change client’s position
The nurse should question the administration of the frequently, provide humidification and hydration,
suction as necessary. Needed Info: Pneumothorax: air in pleural space
Category: Implementation/Physiological Integrity/ causes collapse of lung. Chest tubes: used with
Basic Care and Comfort Pleurevac
(1) Change the tracheostomy dressing every 8 hours 3-chamber system to restore negative pressure
and PRN—CORRECT: prevents infection; use in pleural space. Removal: chest tube is clamped,
pre-cut gauze pads client
(2) Change the tracheostomy ties every 48 hours— does Valsalva maneuver; apply petroleum gauze
change PRN; keep old ties on until new ties are dressing sealed with tape.
in place; 1 finger space between tie and neck Category: Implementation/Physiological Integrity/
(3) Keep the inner cannula of the tracheostomy in Reduction of Risk Potential
place at all times—remove and clean q 8 hours (1) “Exhale and bear down.”—CORRECT: Valsalva
and PRN using H2O maneuver; increased intrathoracic pressure;
(4) Push the outer cannula back in if it accidentally occlusive dressing applied
“blows out”—maintain open airway and contact (2) “Hold your breath for 5 seconds.”—unnecessary
physician (3) “Inhale and exhale rapidly.”—unsafe
83. The Answer is 3 (4) “Cough as hard as you can.”—unnecessary
The nurse performs discharge teaching with a client 85. The Answer is 2
with emphysema. Which statement by the client A client comes into the emergency room with
indicates that teaching was successful? complaints
Reworded Question: What is true about emphysema? of sudden onset of severe right flank pain.
Strategy: Determine the outcome of each answer While tests are being performed, it is MOST important
choice. for the nurse to take which of the following
Needed Info: Emphysema: chronic progressive actions?
respiratory Reworded Question: What is the priority action for a
disease caused by destruction of alveolar client with suspected renal calculi?
walls. Complications: acute respiratory infections, Strategy: “MOST important” indicates a priority
cardiac failure or cor pulmonale, cardiac question.
dysrhythmias. Needed Info: Symptoms of renal calculi: pain
Symptoms: cough, dyspnea, wheezing, barrel (depends on location of stone), diaphoresis, nausea
chest, use of accessory muscles to breathe. and vomiting, fever and chills, hematuria. Nursing
Treatment: care: monitor intake and outake (I & O) and temp;
bronchodilators, corticosteroids, cromolyn force fluids; strain urine and check pH of urine;
sodium, oxygen, diaphragmatic and pursed-lip administer analgesics. Diet for prevention of stones
breathing maneuvers, energy conservation, diet (most stones contain calcium, phosphorus, and/or
therapy. oxalate): consume foods low in calcium, sodium, and
Category: Evaluation/Physiological oxalates; avoid vitamin D–enriched foods; decrease
Integrity/Physiological purine sources; to make urine alkaline, restrict citrus
Adaptation fruits, milk, potatoes; to acidify urine, increase
(1) “Cold weather will help my breathing problems.”— consumption of eggs, fish, cranberries.
will exacerbate breathing problems by Category: Implementation/Physiological Integrity/
causing bronchospasms Physiological Adaptation
(2) “I should eat 3 balanced meals but limit my (1) Make sure that he does not eat or drink anything—
fluid intake.”—need small, frequent feedings to not most important
increase caloric intake, limit SOB caused by eating; (2) Strain all his urine through several layers of
hydration will liquify secretions gauze—CORRECT: symptoms suggestive of
(3) “My outside activity should be limited when urinary calculi, should strain urine for passage
pollution of stone
levels are high.”—CORRECT: pollution (3) Check his grip strength and pupil reactivity—
will act as irritant by causing bronchospasms symptoms suggestive of urinary calculi, not
(4) “An intensive exercise program is important neurological
in regaining my strength.”—unable to tolerate (4) Send blood and urine specimens to the lab for
intensive exercise; conditioning program to conserve analysis—not most important if urinary calculi
and increase pulmonary ventilation is suspected
84. The Answer is 1 86. The Answer is 4
The nurse assists the physician with the removal of The nurse is preparing discharge teaching for a client
a chest tube. Before the physician removes the chest with a new colostomy. The nurse knows teaching
tube, which instruction should the nurse give to the was successful when the client chooses which of the
client? following menu options?
TChoen tPernatc tRiecvei eTwes atnd Practice Reworded Question: What is the appropriate diet for
for the NCLEX-RN ® Exam a client with a colostomy?
364 Strategy: Recall the type of diet required and then
The Practice Test select the menu that is appropriate.
Reworded Question: What should the client do when Needed Info: Diet: a low-residue diet for 4–6 weeks
a chest tube is removed? post-op, avoiding gas-forming, odor-producing, or
Strategy: Determine the outcome of each answer excessively laxative/constipating foods.
choice. Category: Evaluation/Physiological Integrity/Basic
Care and Comfort the first 6–12 months of life; human milk is considered
(1) Sausage, sauerkraut, baked potato, and fresh ideal food. Colostrum is secreted at first; clear
fruit—sausage and sauerkraut are gas-producing and colorless; contains protective antibodies; high
and should be avoided with a new colostomy in protein and minerals. Milk is secreted after 2–4
(2) Cheese omelet with bran muffin and fresh days; milky white appearance; contains more fat and
pineapple— lactose than colostrum.
bran muffin and fresh fruit are highfiber Category: Evaluation/Health Promotion and
(residue) Maintenance
(3) Pork chop, mashed potatoes, turnips, and (1) “My baby’s weight should equal her birthweight
salad—turnips are odor-causing and salad is in 5 to 7 days.”—breastfeeding infants should
high-residue surpass birthweight in 10–14 days
(4) Baked chicken, boiled potato, cooked carrots, (2) “My baby should have at least 6 to 8 wet diapers
and yogurt—CORRECT: provides balanced per day.”—CORRECT: indicates newborn
nutrition, high-protein, low-residue, low-fat, and is ingesting an adequate amount of nutrition;
non-irritating foods should have at least 2 bowel movements per day
Practice Test Explanations (3) “My baby will sleep at least 6 hours between
365 feedings.”—newborns feed approximately every
Practice Test Answers Taensdt 87. The Answer 2–3 hours during the day and every 4 hours at
is 4 night
A client is seen in the outpatient clinic to rule out (4) “My baby will feed for about 10 minutes per
acute renal failure. The nurse would be MOST feeding.”—should feed for approx. 15–20 minutes
concerned per breast
if the client made which of the following 89. The Answer is 2
statements? A man is admitted to the telemetry unit for evaluation
Reworded Question: What is a symptom of acute of complaints of chest pain. Eight hours after
renal failure? admission, the client goes into ventricular fibrillation.
Strategy: “MOST concerned” indicates you are The physician defibrillates the client. The nurse
looking for a symptom of acute renal failure. understands that the purpose of defibrillation is to
Needed Info: Symptoms of oliguric phase of acute do which of the following?
renal failure: urinary output less than 400 mL/day; Reworded Question: Why is a client defibrillated?
irritability, drowsiness, confusion, coma; restlessness, Strategy: Think about each answer choice.
twitching, seizures; increased serum K+, BUN, Needed Info: Defibrillation: produces asystole of
creatinine, Ca+, Na+, pH; anemia; pulmonary edema, heart to provide opportunity for natural pacemaker
CHF, hypertension. Symptoms of diuretic or recovery (SA node) to resume as pacer of heart activity
phase: urinary output of 4–5 L/day; increased Category: Analysis/Physiological
serum BUN; Na+ and K+ loss in urine; increased Integrity/Physiological
mental and physical activity. Adaptation
Category: Assessment/Physiological (1) Increase cardiac contractility and cardiac output—
Integrity/Physiological inaccurate
Adaptation (2) Cause asystole so the normal pacemaker can
(1) “My urine is often pink-tinged.”—seen with urinary recapture—CORRECT: allows SA node to
tract infections or trauma; hematuria not resume as pacer of heart activity
usually a symptom of acute renal failure TChoen tPernatc tRiecvei eTwes atnd Practice
(2) “It is hard for me to start the flow of urine.”— for the NCLEX-RN ® Exam
urinary hesitancy not usually seen with acute 366
renal failure The Practice Test
(3) “It is quite painful for me to urinate.”—dysuria (3) Reduce cardiac ischemia and acidosis—inaccurate
seen with urinary tract infections, not with acute (4) Provide energy for depleted myocardial cells—
renal failure inaccurate
(4) “I urinate in the morning and again before 90. The Answer is 3
dinner.”— A man is brought to the emergency room complaining
CORRECT: symptoms of acute renal of chest pain. The nurse performs an assessment
failure include decreased urinary output (anuria of the client. Which of the following symptoms
or ologuria), hypotension, tachycardia, lethargy; would be MOST characteristic of an acute myocardial
normal output 1,200–1,500 mL/day or 50–63 infarction?
mL/hr, normal voiding pattern 5–6 times/day Reworded Question: What type of pain is
and once at night characteristic
88. The Answer is 2 in an MI?
The nurse is teaching a new mother how to breastfeed Strategy: Think about the cause of each type of pain.
her newborn. The nurse knows that teaching Needed Info: MI signs and symptoms: chest pain
has been successful if the client makes which of the radiating to neck, jaw, shoulder, back, or left arm;
following statements? unrelieved by nitroglycerin. Also fever, apprehension,
Reworded Question: What indicates that a newborn dizziness, diaphoresis, palpitations, shortness
is receiving adequate nutrition when breastfeeding? of breath.
Strategy: Think about each statement. Is it true? Category: Assessment/Physiological Integrity
Needed Info: Breastfeeding is recommended for /Physiological
Adaptation Practice Test Answers Taensdt 93. The Answer
(1) Colic-like epigastric pain—indicates GI disorder is 3
(2) Sharp, well-localized, unilateral chest pain— To enhance the percutaneous absorption of
symptom of pneumothorax nitroglycerine
(3) Severe substernal pain radiating down the left ointment, it would be MOST important
arm—CORRECT: crushing; may radiate; unrelated for the nurse to select a site that is which of the
to emotion or exercise following?
(4) Sharp, burning chest pain moving from place to Reworded Question: What is the best site for
place—anxiety state nitroglycerin
91. The Answer is 3 ointment?
The nurse is caring for clients on the medical unit. Strategy: Think about each site.
A client is admitted with a diagnosis of deep vein Needed Info: Nitroglycerin: used in treatment of
thrombosis (DVT). Admission orders include heparin angina pectoris to reduce ischemia and relieve
2,000 units per hour in 5% dextrose in water. The pain by decreasing myocardial oxygen consumption;
nurse should have which of the following available? dilates veins and arteries. Side effects: throbbing
Reworded Question: What is an antidote for heparin? headache, flushing, hypotension, tachycardia.
Strategy: Think about the action of each medication. Nursing responsibilities: teach appropriate
Needed Info: Heparin: anticoagulant. Side effects: administration,
hemorrhage, thrombocytopenia. Antidote: protamine storage, expected pain relief, side effects.
sulfate. Ointment applied to skin; sites rotated to avoid skin
Category: Planning/Physiological irritation. Prolonged effect up to 24 hours.
Integrity/Pharmacological Category: Implementation/Physiological Integrity/
and Parenteral Therapies Pharmacological and Parenteral Therapies
(1) Propranolol—beta-blocker; reduces myocardial (1) Muscular—not most important
oxygen consumption (2) Near the heart—not most important
(2) Protamine zinc—long-acting insulin (3) Non-hairy—CORRECT: skin site free of hair
(3) Protamine sulfate—CORRECT: antidote, acts as will increase absorption; avoid distal part of
base to neutralize heparin; give IV over 3 minutes extremities due to less than maximal absorption
(4) Vitamin K—antidote to warfarin (4) Over a bony prominence—most important is
92. The Answer is 2 that the site be non-hairy
A client returns to the clinic 2 weeks after discharge 94. The Answer is 3
from the hospital. He is taking warfarin sodium 2 mg A client with chronic alcohol abuse has been admitted
PO daily. Which of the following statements by the to a rehabilitation unit. The nurse knows that the
client to the nurse indicates that further teaching is client is denying alcoholism when he makes which of
necessary? the following statements?
Reworded Question: What is contraindicated for Reworded Question: Which statement signifies
warfarin? denial?
Strategy: Think about what each statement means Strategy: What are the defense mechanisms and how
and how it relates to warfarin. are they manifested?
Needed Info: Warfarin sodium: anticoagulant. Side Needed Info: Defense mechanisms include:
effects: hemorrhage, fever, rash. Prothrombin time repression,
(PT) used to monitor effectiveness; PT usually denial, suppression, rationalization,
maintained intellectualization,
at 1.5–2 times normal. Antidote: vitamin K identification, introjection, compensation,
(aquamephyton). Nursing responsibilities: check reaction formation, sublimation, displacement,
for bleeding gums, bruises, nosebleeds, petechiae, projection, conversion, undoing, dissociation,
melena, tarry stools, hematuria. Use electric razor, regression.
soft toothbrush; green leafy vegetables (contain Category: Analysis/Psychosocial Integrity
vitamin (1) “My brother did this to me.”—projection: blaming
K). someone else for one’s difficulties
Category: Evaluation/Physiological (2) “Drinking always calms my nerves.”—
Integrity/Pharmacological rationalization:
and Parenteral Therapies the attempt to prove that one’s feelings
(1) “I have been taking an antihistamine before or behavior are justifiable
bed.”—no contraindication (3) “I can stop drinking anytime I feel like it.”—
(2) “I take aspirin when I have a headache.”— CORRECT: denial is the unconscious refusal to
CORRECT: admit an unacceptable idea or behavior
inhibits platelet aggregation; effect lasts (4) “Let’s all plan to play cards tonight.”—
3–8 days suppression:
(3) “I use sunscreen when I go outside.”—correct the voluntary exclusion from awareness of
behavior feelings, ideas, or situations that produce anxiety
(4) “I take Mylanta if my stomach gets upset.”— 95. The Answer is 4
correct During the acute phase of a cerebrovascular accident
information (CVA), the nurse should maintain the client in
Practice Test Explanations which of the following positions?
367 Reworded Question: What is the best way to position
a client during the acute phase of a CVA? Strategy: Determine whether to assess or implement.
Strategy: Remember the positioning strategy. Needed Info: Alzheimer’s disease (senile dementia):
Needed Info: Nursing responsibilities during acute chronic, progressive, degenerative, resulting in
phase: maintain client airway; monitor vital signs; cerebral
neurological assessment (Glasgow coma scale); atrophy. S/S: changes in memory, confusion,
passive disorientation, change in personality; most common
ROM exercises; NPO for 24–48 hours; tube after age 65. Nursing responsibilities: reorient as
feedings. needed; speak slowly; place clocks and calendars in
Category: Implementation/Physiological Integrity/ room; place bed in low position with side rails up.
Physiological Adaptation Category: Assessment/Psychosocial Integrity
(1) Semi-prone with the head of the bed elevated (1) Place the client in a private room away from the
60–90 degrees—on left side with legs flexed on nurses’ station—should be in a semiprivate room
abdomen, hip flexion increases intrathoracic near nurses’ station; needs frequent assessment
pressure (2) Ask the family to wait in the waiting room while
(2) Lateral, with the head of the bed flat—helps with the nurse admits the client—familiar people
drainage of secretions, but not the best decrease confusion of unfamiliar environment
(3) Prone, with the head of the bed flat—interferes (3) Assign a different nurse daily to care for the
with respiration client—
(4) Supine, with the head of the bed elevated 30–45 consistency is important
degrees—CORRECT: facilitates venous drainage (4) Ask the client to state today’s date—CORRECT:
from brain; reduces intracranial pressure; assessment is the first step in planning care
keeps head in midline 98. The Answer is 3
TChoen tPernatc tRiecvei eTwes atnd Practice A female client visits the clinic with complaints of
for the NCLEX-RN ® Exam right calf tenderness and pain. It would be MOST
368 important for the nurse to ask which of the following
The Practice Test questions?
96. The Answer is 4 Reworded Question: What is a predisposing factor to
Which of the following statements by a client during developing venous thrombosis (VT)?
a group therapy session requires immediate followup Strategy: Determine why you would ask each
by the nurse? question.
Reworded Question: Which statement indicates the Needed Info: Thrombophlebitis (phlebitis,
possibility of impending danger? phlebothrombosis,
Strategy: Think about which statement would make or deep vein thrombosis [DVT]): clot
you question the client’s intentions. formation in a vein secondary to inflammation of
Needed Info: In Tarasoff v. The Regents of the vein or partial vein obstruction. Risk factors: history
University of varicose veins, hypercoagulation, cardiovascular
of California (1976), the court established a disease, pregnancy, oral contraceptives, immobility,
duty to warn of threats of harm to others. Failure to recent surgery or injury.
warn, coupled with subsequent injury to the Category: Assessment/Physiological
threatened Integrity/Pharmacological
person, exposes the mental health professional and Parenteral Therapies
to civil damages for malpractice. Based on this and (1) “Do you exercise excessively?”—could cause
other rulings in many states, the mental health shin splints
caregiver (2) “Have you had any fractures in the last year?”—
must take responsibility to warn society of not relevant to client’s complaints
potential danger. (3) “What type of birth control do you use?”—
Category: Implementation/Psychosocial Integrity CORRECT: increased risk of DVT with oral
(1) “I know I’m a chronically compulsive liar, but I contraceptives
can’t help it.”—this statement is revealing, but (4) “Are you under a lot of stress?”—should be
does not indicate impending threat concerned
(2) “I don’t ever want to go home; I feel safer here.”— about possibility of DVT
this statement is a response to anxiety or fear, Practice Test Explanations
but does not indicate immediate danger 369
(3) “I don’t really care if I ever see my girlfriend Practice Test Answers Taensdt 99. The Answer
again.”—this statement does not imply a threat is 3
or impending violence A mother calls the well-baby clinic to report that her
(4) “I’ll make sure that doctor is sorry for what he 4-month-old son has an upper respiratory infection
said.”—CORRECT: under the Tarasoff Act, a (URI) with a temperature of 104° F (40° C). The
threatened person, including health professionals, infant is scheduled to receive his DPT and TOPV
must be warned about threats or potential immunizations later that day. The mother asks the
threats to personal safety nurse if she should bring him in for his scheduled
97. The Answer is 4 immunizations. Which of the following responses by
A client newly diagnosed with Alzheimer’s disease is the nurse would be MOST appropriate?
admitted to the unit. Which of the following actions Reworded Question: Is a URI and elevated temp
by the nurse is BEST? contraindication
Reworded Question: What is the best assessment? for routine immunization?
Strategy: Picture the client as described. following exercises would be MOST beneficial for
Needed Info: URI: acute rhinitis (cold), pharyngitis, this client?
tonsillitis. Nursing responsibilities: liquid to soft Reworded Question: What exercise is best for a client
diet, cool mist vaporizer, analgesics, antipyretics, in a cast?
antibiotics. Contraindications for immunizations: Strategy: Picture the client as described. Imagine
moderate to severe febrile illness, severe anaphylactic client performing each type of exercise. Also think
reaction from previous immunization, congenital about the key words “MOST beneficial.”
disorders of immune system, immunosuppressive Needed Info: Fracture: break in continuity of bone.
therapy, anaphylactic egg hypersensitivity for MMR Complications: hemorrhage (bone vascular), shock,
and OPV. fat embolism (long bones), sepsis, peripheral nerve
Category: Implementation/Safe and Effective Care TChoen tPernatc tRiecvei eTwes atnd Practice
Environment/Safety and Infection Control for the NCLEX-RN ® Exam
(1) “Keep him at home. We’ll give him a double dose 370
next time.”—immunization not given, schedule The Practice Test
resumed when infant well damage, delayed union, nonunion. Treatment:
(2) “Bring him in. His illness will not interfere with reduction (closed or open), immobilization (cast,
his immunizations.”—febrile illness contraindication traction, splints, internal and external fixation). Cast
for all immunizations allows early mobility. Nursing responsibilities: teach
(3) “Keep him at home until his temperature and isometric exercises.
infection resolve.”—CORRECT: immunization Category: Planning/Physiological Integrity/Reduction
contraindicated during infectious or inflammatory of Risk Potential
state, pre-existing symptoms could mask (1) Passive exercise of the affected limb—nurse
adverse or allergic reaction moves extremity; unable to do
(4) “Bring him in. We’ll give some antibiotics with (2) Quadriceps setting of the affected limb—
the immunizations.”—involves giving immunization CORRECT:
with febrile illness isometric exercise: contraction of muscle
100. The Answer is 2 without movement of joint; maintains strength
The nurse in the postpartum unit cares for a client (3) Active ROM exercises of the unaffected limb—
who delivered her first child the previous day. During not best
her assessment of the client, the nurse notes multiple (4) Passive exercise of the upper extremities—need
varicosities on the client’s lower extremities. Which strengthening, not passive exercises
of the following actions should the nurse perform? 102. The Answer is 1
Reworded Question: What is the best way to prevent The nurse plans care for a client receiving
thrombophlebitis? electroconvulsive
Strategy: Think about what causes thrombophlebitis. treatments (ECT). Immediately after a
Needed Info: High risk of developing thrombophlebitis treatment, the nurse should take which of the
during pregnancy and immediate postpartum following
period. Thrombophlebitis: inflammation of vein actions?
associated with formation of a thrombus or blood Reworded Question: What should you do right after
clot. Other risk factors: prolonged immobility, use a client has ECT?
of oral contraceptives, sepsis, smoking, dehydration, Strategy: Picture the client as described in the
and CHF. S/S: pain in the calf, localized edema of question.
one extremity, positive Homan’s sign (pain in calf Needed Info: ECT: stimulation of convulsions similar
when foot is dorsiflexed). Treatment: bed rest and to grand mal seizures as treatment for depression.
elevation of extremity, anticoagulant (heparin). Requires 6–12 treatments. Preparation: NPO 4 hrs,
Category: Planning/Health Promotion and informed consent, void, remove jewelry, atropine
Maintenance 30 min before to reduce secretions. During:
(1) Teach the client to rest in bed when the baby shortacting
sleeps—not preventive; bed rest can cause IV anesthesia and muscle relaxant, O2, suction
thrombophlebitis available. After: confusion and memory loss for
(2) Encourage early and frequent ambulation— recent events, stay with client and orient, check vital
CORRECT: facilitates emptying of blood vessels signs.
in lower extremities Category: Implementation/Psychosocial Integrity
(3) Apply warm soaks for 20 minutes every 4 (1) Orient the client to time and place—CORRECT:
hours—not a preventive measure but an intervention short-term memory loss common side effect
used to treat; must be ordered by physician; (2) Talk about events prior to the client’s
can be intermittent or continuous hospitalization—
(4) Perform passive range-of-motion (ROM) exercises long-term memory not affected
3 times daily—early ambulation more (3) Restrict fluid intake and encourage the client to
effective; passive ROM retains joint function, ambulate—should encourage fluids, rest
maintains circulation; passive exercises: no (4) Initiate comfort measures to relieve vertigo—
assistance dizziness not common side effect
from client 103. The Answer is 3
101. The Answer is 2 A client is to receive 35 mg/hr of intravenous
A man fractures his left femur in a bicycle accident. aminophylline.
A cast is applied. The nurse knows that which of the The nurse mixes 350 mg of aminophylline
in 500 mL D5W. At which of the following rates to prevent permanent nerve injury; can
should the nurse infuse this solution? also occur with blood draws
Reworded Question: What is the IV flow rate? (3) Tell the client this is a common response to IV
Strategy: Set up a ratio and solve for x. If you miss insertion—it does sometimes happen; important
this, review your nursing math. to remove needle immediately
Needed Info: Formula: med on hand over volume on (4) Withdraw the needle slightly and then push it
hand = desired med over x. Solve for x. forward—action can damage the nerve
Category: Implementation/Physiological Integrity/ 106. The Answer is 3
Pharmacological and Parenteral Therapies A client comes to the emergency room with
(1) 20 mL/hr—incorrect complaints
(2) 35 mL/hr—incorrect of nausea, vomiting, and abdominal pain.
(3) 50 mL/hr—CORRECT: 350 mg/500 mL = 35 He is a type 1 diabetic (IDDM). Four days earlier,
mg/x, 350x = 17,500, x = 50 he reduced his insulin dose when flu symptoms
(4) 70 mL/hr—incorrect prevented him from eating. The nurse performs an
104. The Answer is 3 assessment of the client that reveals poor skin turgor,
The nurse prepares an adult client for instillation of dry mucous membranes, and fruity breath odor.
eardrops. The nurse should use which of the following The nurse should be alert for which of the following
methods to administer the eardrops? problems?
Reworded Question: How are eardrops given? Reworded Question: What do these symptoms
Strategy: Picture yourself doing the procedure. indicate?
Picture Strategy: Think about each answer.
an arrow pointing upward for a “tall” adult to Needed Info: Diabetes mellitus: disorder of
straighten ear canal. carbohydrate
Needed Info: Pull earlobe up and back for adult. Pull metabolism: insufficient insulin to meet metabolic
earlobe down and back for child. needs. Type 1 (juvenile): insulin dependent,
Category: Implementation/Safe and Effective Care prone to ketoacidosis. Type 2 (adult onset): controlled
Environment/Safety and Infection Control by diet and oral agents, non–ketosis prone.
(1) Cool the solution for better absorption. Drop the In ketoacidosis, the body becomes dehydrated from
medication directly into the auditory canal— osmotic diuresis. The fruity breath odor develops
stimulates acoustic nerve reflex, may cause nausea from acetone, a component of ketone bodies. Rate
and vomiting and depth of resp increase (Kussmaul) in attempt to
(2) Warm the solution. Flush the medication rapidly blow off excess carbonic acid. Difference between
into the ear—causes pressure against tympanic ketoacidosis and hyperglycemic hyperosmolar
membrane, possible rupture nonketonic
(3) Warm the solution. Drop the medication along syndrome (HHNKS)—lack of ketonuria.
the side of the ear canal—CORRECT: prevents Category: Planning/Physiological Integrity/Reduction
acoustic nerve reflex and dizziness; will not damage of Risk Potential
tympanic membrane (1) Hypoglycemia—cause: too much insulin; blood
Practice Test Explanations sugar below 60 mg; S/S: tachycardia, perspiration,
371 confusion, lethargy, numb lips, anxiety,
Practice Test Answers Taensdt (4) Warm the hunger
solution to 104° F (40° C). Drop the (2) Viral illness—not best answer
medication slowly into the ear canal—too hot, (3) Ketoacidosis—CORRECT: cause: insufficient
95–98.6° F (35–37° C) insulin; S/S: polyuria, polydipsia, nausea,
105. The Answer is 2 vomiting, dry mucous membranes, weight loss,
The nurse is inserting an IV catheter into a client’s abdominal pain, hypotension, shock, coma
left arm. Suddenly the client exclaims, “It feels like (4) Hyperglycemic hyperosmolar nonketotic
an electric shock is going all the way down my arm coma—(HHNK) extreme hyperglycemia (800–
and into my hand!” What is the FIRST action the 2,000 mg/dL) with absence of acidosis; some
nurse should take? insulin production so don’t mobilize fats for
Reworded Question: What should the nurse do if a energy or form ketones; usually seen in type 2;
nerve is struck when inserting an IV? cause: infections, stress, meds (steroids, thiazide
Strategy: Determine the outcome of each answer. Is diuretics), TPN; S/S: polyphagia, polyuria,
it appropriate? polydipsia, glycosuria, dehydration, abdominal
Needed Info: When choosing location to insert discomfort, hyperpyrexia, changes in level of
peripheral IV, consider the condition of vein, type consciousness (LOC), hypotension, shock; treatment:
of fluid/med to be infused, duration of therapy, and fluid replacement (2 L 0.45% NaCl over 2
client’s age, size, and status. hrs), K+, Na+, Cl−, phosphates, insulin given IV
Category: Implementation/Physiological Integrity/ 107. The Answer is 1
Reduction of Risk Potential The nurse knows that it is MOST important for
(1) Instruct the client to take slow, deep breaths— which of the following clients to receive their
incorrect action; will cause harm to the client scheduled
(2) Remove the needle from the client’s arm— medication on time?
CORRECT: Reworded Question: Which medication, if given late,
electric shock sensation indicates needle might cause harm to the client?
point is touching a nerve; remove needle immediately Strategy: Think about each answer.
TChoen tPernatc tRiecvei eTwes atnd Practice sensitive about school expectations; may be
for the NCLEX-RN ® Exam impaired due to absences from school, growth
372 retardation, and emotional difficulties
The Practice Test (4) Identity—adolescence; peer groups important;
Needed Info: Myasthenia gravis is deficiency of used to define identity, establish body image,
acetylcholine form new relationships; alternative: role diffusion
at myoneural junction; symptoms include 109. The Answer is 4
muscular weakness produced by repeated movements The nurse assesses a client with a history of Addison’s
that soon disappears following rest, diplopia, disease who has received steroid therapy for several
ptosis, impaired speech, and dysphagia years. The nurse could expect the client to exhibit
Category: Analysis/Physiological which of the following changes in appearance?
Integrity/Pharmacological Reworded Question: What changes are seen in a client
and Parenteral Therapies after taking steroids long-term?
(1) A client diagnosed with myasthenia gravis Strategy: All the options in an answer choice must
receiving be correct for the option to be right.
pyridostigmine bromide—CORRECT: Pyridostigmine Needed Info: Meds: cortisone and hydrocortisone
bromide is a cholinesterase inhibitor usually given in divided doses: 2/3rds in morning and
which increases acetylcholine concentration at 1/3rd in late afternoon with food to decrease GI
the neuromuscular junction; early administration irritation.
can precipitate a cholinergic crisis; late Teach to report S/S of excessive drug therapy
administration can precipitate myasthenic crisis (rapid weight gain, round face, fluid retention).
(2) A client diagnosed with bipolar disorder receiving Category: Assessment/Physiological
lithium carbonate—Lithium carbonate is Integrity/Physiological
a mood stabilizer; targeted blood level = 1–1.5 Adaptation
mEq/L (1) Buffalo hump, girdle-obesity, gaunt facial
(3) A client diagnosed with tuberculosis receiving appearance—hump and girdle-obesity true;
isonicotinic acid hydrazide (INH)—INH is gaunt face seen with lack of steroids
given in a single daily dose; side effects include (2) Tanning of the skin, discoloration of the mucous
hepatitis, peripheral neuritis, rash, and fever membranes, alopecia, weight loss—tanning and
(4) A client diagnosed with Parkinson’s disease weight loss seen with lack of steroids; rest not
receiving levodopa—Levodopa is thought to seen
restore dopamine levels in extrapyramidal centers; Practice Test Explanations
sudden withdrawal can cause parkinsonian 373
crisis; priority is to administer pyridostigmine Practice Test Answers Taensdt (3) Emaciation,
bromide nervousness, breast engorgement,
108. The Answer is 3 hirsutism—nothing to do with steroids; hirsutism:
An 11-year-old boy is admitted to the hospital for excessive growth of hair
evaluation for a kidney transplant. During the initial (4) Truncal obesity, purple striations on the skin,
assessment, the nurse learns that the client received moon face—CORRECT: due to excess glucocorticoids
hemodialysis for 3 years due to renal failure. The 110. The Answer is 2
nurse knows that his illness can interfere with this Haloperidol 5 mg tid is ordered for a client with
client’s achievement of which of the following? schizophrenia. Two days later, the client complains
Reworded Question: What developmental stage is of “tight jaws and a stiff neck.” The nurse should
altered in a client due to this chronic disease? recognize that these complaints are which of the
Strategy: Picture the person described in the following?
question. Reworded Question: Why does the client taking
Think about his activities and interests. This haloperidol
helps eliminate incorrect answer choices. An 11- have these symptoms?
yearold Strategy: Think about each answer and how it relates
is usually in grade school thinking about homework, to haloperidol.
doing chores at home. Needed Info: Haloperidol (Haldol) is a medication
Needed Info: Eric Erikson developed a theory of the used in the treatment of psychotic disorders. High
stages of personality development that progressed in incidence of extrapyramidal reactions:
predictable stages from birth to death. Other stages: pseudoparkinsonism
autonomy versus shame and doubt (task of 1–3 yrs); (rigidity and tremors), akathisia (motor
initiative versus guilt (task of 3–6 yrs). restlessness), dystonia (involuntary jerking,
Category: Analysis/Health Promotion and uncoordinated
Maintenance body movements), tardive dyskinesia
(1) Intimacy—young adult: 20–40 yrs; achieving (abnormal movements of lips, jaws, tongue).
sexual and loving relationship with another; Schizophrenia:
alternative: isolation retreat from reality, flat affect, suspiciousness,
(2) Trust—infancy; results from consistent care by a hallucinations, delusions, loose associations,
loving caretaker; teaches that basic needs will be psychomotor retardation or hyperactivity, regression.
met; alternative: mistrust Nursing responsibilities: maintain safety, meet
(3) Industry—CORRECT: 6–12 yrs; aspires to be physical needs, decrease sensory stimuli. Treatment:
the best; learns social skills, how to finish tasks; antipsychotic meds, individual therapy.
Category: Analysis/Psychosocial Integrity contact lenses after surgery. Complications:
(1) Common side effects of antipsychotic medications glaucoma,
that will diminish over time—gets worse; infection, bleeding, retinal detachment.
untreated, life-threatening Category: Analysis/Physiological
(2) Early symptoms of extrapyramidal reactions to Integrity/Physiological
the medication—CORRECT: dystonic reaction, Adaptation
airway may become obstructed (1) Expected; the nurse should administer analgesic
(3) Psychosomatic complaints resulting from a to the client—severe pain not expected; mild
delusional discomfort
system—not accurate treated with analgesics
(4) Permanent side effects of haloperidol—reversible (2) Expected; the nurse should maintain the client
when treated with IV diphenhydramine on bed rest—activity restrictions: no coughing,
hydrochloride (Benadryl) bending at waist, vomiting, sneezing, lifting
111. The Answer is 3 more than 15 lb., squeezing eyelid, straining at
The nurse is caring for a woman who states she was stool, lying on affected side; these increase
beaten and sexually assaulted by a male friend. intraocular
Which of the following should the nurse do FIRST? pressure; however client need not remain
Reworded Question: What is the MOST important on bed rest
initial nursing action to take with a sexual assault (3) Unexpected and may signify a detached retina—
victim? lens was removed during surgery
Strategy: Discriminate between what is appropriate (4) Unexpected and may signify hemorrhage—
and inappropriate nursing behavior. CORRECT: ruptured blood vessel or suture
Needed Info: Nursing care for crime victims must causing hemorrhage or increased intraocular
address both physical and emotional needs. The pressure; notify physician if restless, increased
nurse must be cautious not to disturb or eliminate pulse, drainage on dressing
any evidence until the victim has been examined by 113. The Answer is 3
a physician. The nurse must document all evidence A client returns to his room after a lower GI series.
found during the nursing assessment. After the client When he is assessed by the nurse, he complains of
has been examined and a course of action weakness. Which of the following nursing diagnoses
determined, should receive priority in planning his care?
the nurse can begin to address the expressed Reworded Question: What is most important for a
needs of the client, such as contacting legal counsel client after a GI series?
or the chaplain. Strategy: Establish priorities.
Category: Implementation/Psychosocial Integrity Needed Info: Upper GI series (barium swallow):
(1) Encourage the client to call her family lawyer— radiologic visualization of esophagus, stomach,
not the first action the nurse should take with duodenum, jejunum. Lower GI series (barium
this client enema): visualize colon. Prep for test: NPO 6–8 hrs,
(2) Ask for a psychiatry consult—nurse should not enemas, laxatives, fluid restriction. Post-test:
initiate a psychiatry consult laxatives
(3) Stay with the client during the physical exam— to remove barium. Nursing responsibilities
CORRECT: provide consistent emotional and after test: check abdomen for distention, encourage
physical support for the client fluids. Initially stool is white from barium. Should
(4) Wash and dress the client’s wounds before the return to normal color in 72 hrs.
physical exam—contraindicated; eradicates Category: Analysis/Physiological
potential evidence Integrity/Physiological
112. The Answer is 4 Adaptation
The nurse cares for a client after surgery for removal (1) Alteration in sensation-perception, gustatory—
of a cataract in her right eye. The client complains of not highest priority; gustatory: pertaining to
severe eye pain in her right eye. The nurse knows this sense of taste
symptom is which of the following? (2) Constipation, colonic—not highest priority
Reworded Question: Is pain after cataract surgery (3) High risk for fluid-volume deficit—CORRECT:
normal? prep for test: low-residue or clear liquid diet 2
Strategy: Think about each answer and how it relates days, NPO midnight, enemas, laxatives; posttest:
to cataract surgery. laxatives to remove barium
Needed Info: Cataract: change in the transparency (4) Nutrition, less than body requirements—not
of crystalline lens of eye. Causes: aging, trauma, highest priority
congenital, systemic disease. S/S: blurred vision, 114. The Answer is 2
TChoen tPernatc tRiecvei eTwes atnd Practice A client hospitalized with a gastric ulcer is scheduled
for the NCLEX-RN ® Exam for discharge. The nurse teaches the client about an
374 anti-ulcer diet. Which of the following statements by
The Practice Test the client indicates to the nurse that dietary teaching
decrease in color perception, photophobia. Treated was successful?
by removal of lens under local anesthesia with Reworded Question: What statement is true about an
sedation. anti-ulcer diet?
Intraocular lens implantation, eyeglasses, or Strategy: “Dietary teaching was successful” means
you are looking for correct information.
Needed Info: Gastric ulcers form within 1 inch of the the blood. If not recognized, resultant high levels of
pylorus of the stomach usually caused by break in phenylketone in the brain cause mental retardation.
mucosa. Onset 45–54 years old, men twice as often Guthrie test: screening for PKU. Treatment: dietary
as women, pain increased by food, relieved by restriction of foods containing phenylalanine. Blood
vomiting, levels of phenylalanine monitored to evaluate the
hematemesis common. Treatment: antacids effectiveness of the dietary restrictions.
(Amphojel), H2 receptor antagonists (Tagamet), Category: Analysis/Health Promotion and
anticholinergics (Bentyl), mucosal barrier fortifiers Maintenance
(Carafate). (1) A source of protein has been ingested—CORRECT:
Category: Evaluation/Physiological Integrity/Basic recommended to be performed before
Care and Comfort newborns leave hospital; if initial blood sample
(1) “I must eat bland foods to help my stomach is obtained within first 24 hrs recommended to
heal.”—transitional diet used for severe inflammation; be repeated at 3 weeks
not speed healing (2) The meconium has been excreted—no
(2) “I can eat most foods, as long as they don’t relationship;
bother my stomach.”—CORRECT: not severely dark-green, tarry stool passed within first
restricted; small, frequent feedings; avoid foods 48 hrs of birth
known to increase gastric acidity: coffee, alcohol, (3) The danger of hyperbilirubinemia has passed—
seasonings, milk no relationship; excessive accumulation of bilirubin
(3) “I cannot eat fruits and vegetables because they in blood; S/S: jaundice (yellow discoloration
cause too much gas.”—restricted only if they of skin); common finding in newborn; not cause
bother stomach for concern
(4) “I should eat a low-fiber diet to delay gastric (4) The effects of delivery have subsided—no
emptying.”—used with acute diverticulitis, relationship
Practice Test Explanations 117. The Answer is 4
375 The nurse is completing a client’s preoperative
Practice Test Answers Taensdt ulcerative colitis; checklist prior to an early morning surgery. The
foods with fiber: cereals, whole nurse obtains the client’s vital signs: temperature
grains products, fruits, vegetables 97.4° F (36° C), radial pulse 84 strong and regular,
115. The Answer is 2 respirations 16 and unlabored, and blood pressure
A 6-year-old boy is returned to his room after a 132/74. Which of the following actions should the
tonsillectomy. nurse take FIRST?
He remains sleepy from the anesthesia Reworded Question: What should you do for a client
but is easily awakened. The nurse should place the with normal vital signs?
child in which of the following positions? Strategy: Identify normal serum electrolyte values.
Reworded Question: What is the best position after Needed Info: Normal electrolyte values (range may
tonsillectomy to help with drainage of oral secretions? vary by laboratory): Na+ (sodium) 135–145 mEq/L,
Strategy: Picture the client as described. K+ (potassium) 3.6–5.4, Cl− (chloride) 96–106
Category: Implementation/Safe and Effective Care mEq/L,
Environment/Safety and Infection Control mEq/L = milliequivalents per liter.
(1) Sims’—on side with top knee flexed, thigh drawn Category: Assessment/Physiological Integrity/
up to chest, and lower knee less sharply flexed: Reduction of Risk Potential
used for vaginal or rectal examination (1) Notify the physician of the client’s vital signs—
(2) Side-lying—CORRECT: most effective to facilitate most physicians do not want to be notified about
drainage of secretions from the mouth and normal values
pharynx; reduces possibility of airway obstruction TChoen tPernatc tRiecvei eTwes atnd Practice
(3) Supine—increased risk for aspiration, would not for the NCLEX-RN ® Exam
facilitate drainage of oral secretions 376
(4) Prone—risk for airway obstruction and aspiration, The Practice Test
unable to observe the child for signs of (2) Obtain orthostatic blood pressures lying and
bleeding such as increased swallowing standing—there is no information to support
116. The Answer is 1 this action
A client is preparing to take her 1-day-old infant (3) Lower the side rails and place the bed in its lowest
home from the hospital. The nurse discusses the test position—bed side rails should be raised, not
for phenylketonuria (PKU) with the mother. The lowered
nurse’s teaching should be based on an understanding (4) Record the data on the client’s preoperative
that the test is MOST reliable after which of the checklist—CORRECT: the vital signs are normal
following? and should be recorded in the client’s medical
Reworded Question: When is the PKU test most record
reliable? 118. The Answer is 2
Strategy: Focus on the key words in the question. A woman is hospitalized with a diagnosis of bipolar
Think about what you know about the PKU test. disorder. While she is in the client activities room
Needed Info: PKU: genetic disorder caused by a on the psychiatric unit, she flirts with male clients
deficiency in liver enzyme phenylalanine hydroxylase. and disrupts unit activities. Which of the following
Body can’t metabolize essential amino acid approaches would be MOST appropriate for the
phenylalanine, allows phenyl acids to accumulate in nurse to take at this time?
Reworded Question: How should you deal with a client post-op: check eye patch for drainage, position
with bipolar disorder who is disruptive? with detached area dependent; no rapid eye
Strategy: Determine the outcome of each answer. Is movement (reading, sewing); no coughing, vomiting,
it desirable? sneezing.
Needed Info: Nursing responsibilities: accompany Category: Planning/Physiological Integrity/Reduction
client to room when hyperactivity escalates, set limits, of Risk Potential
remain nonjudgmental. (1) Perform self-care activities—activity restrictions
Category: Planning/Psychosocial Integrity depend on location and size of tear
(1) Set limits on the client’s behavior and remind her Practice Test Explanations
of the rules—too confrontational 377
(2) Distract the client and escort her back to her Practice Test Answers Taensdt (2) Maintain
room—CORRECT: clients are easily distracted, patches over both eyes—only affected
nonthreatening action eye covered
(3) Instruct the other clients to ignore this client’s (3) Limit movement of both eyes—CORRECT: bed
behavior—does not ensure safety rest with eye patch or shield
(4) Tell the client that she is behaving inappropriately (4) Refrain from excessive talking—no restriction
and send her to her room—too confrontational, 121. The Answer is 2
may agitate The nurse cares for a client receiving a balanced
119. The Answer is 1 complete food by tube feeding. The nurse knows that
A client is brought to the emergency room bleeding the MOST common complication of a tube feeding
profusely from a stab wound in the left chest area. is which of the following?
The nurse’s assessment reveals a blood pressure Reworded Question: What is a common complication
of 80/50, pulse of 110, and respirations of 28. The of a tube feeding?
nurse should expect which of the following potential Strategy: Focus on the words “MOST common”
problems? which means there may be more than one answer.
Reworded Question: What type of shock is described? And in this situation there is—(
Strategy: Form a mental image of the person 4) is a complication
described. —but not common.
Needed Info: Symptoms of hypovolemic shock: Needed Info: Tube feedings are used with clients
tachycardia, reduced output, irritability. Treatment: unable to tolerate the oral route but who have a
O2, IV fluids to restore volume, Adrenaline, functioning GI tract. May be given by intermittent
Apresoline. or continuous infusion. Elevate head of bed 30–45
Nursing responsibilities: check airway, vital degrees. Give at room temp. Check for placement
signs, insert IV, check blood gases, CVP and residual before feeding or every 4–8 hrs (should
measurements, be less than 50% of previous hour’s intake). Replace
insert catheter, hourly I & O, position flat residual to prevent fluid and electrolyte imbalances
with legs elevated, keep warm. unless it appears abnormal (coffee ground–like
Category: Planning/Physiological material).
Integrity/Physiological Don’t hang solution more than 6 hrs. Flush tubing
Adaptation with 20–30 mL water every 4 hrs. Change feeding
(1) Hypovolemic shock—CORRECT: loss of circulating set every 24 hrs. Balanced compete food product/
volume supplement containing intact protein.
(2) Cardiogenic shock—decrease in cardiac output; Category: Evaluation/Physiological Integrity/Basic
cause: cardiac dysfunction, MI, CHF Care and Comfort
(3) Neurogenic shock—increase in vascular bed; (1) Edema—not frequently seen; if present, physician
cause: spinal anesthesia, spinal cord injury may change formula to contain less Na+
(4) Septic shock—decreased cardiac output, (2) Diarrhea—CORRECT: intolerance to solution,
hypotension; rate; give slowly; other symptoms of intolerance:
cause: gram+ or gram– bacteria nausea, vomiting, aspiration, glycosuria, diaphoresis
120. The Answer is 3 (3) Hypokalemia—normal potassium 3.5–5.0
A client is admitted to the hospital for surgical repair mEq/L; not commonly seen; common causes:
of a detached retina in the right eye. In planning diuretics, diarrhea, GI drainage
care for this client postoperatively, the nurse should (4) Vomiting—can happen with rapid increase in
encourage the client to do which of the following? rate; give feeding slowly
Reworded Question: What should you do after surgery 122. The Answer is 4
for detached retina? A 6-week-old infant is brought to the hospital for
Strategy: Picture the client as described. treatment of pyloric stenosis. The nurse enters the
Needed Info: Detached retina: separation of retina following nursing diagnosis on the infant’s care plan:
from pigmented epithelium. S/S: curtain falling “fluid volume deficit related to vomiting.” Which of
across field of vision, black spots, flashes of light, the following assessments supports this diagnosis?
sudden onset. Treatment: surgical repair Reworded Question: What would indicate volume
(photocoagulation, deficit?
electrodiathermy, cryosurgery, scleral Strategy: Think about each answer and how it relates
buckling). Complications: infection, redetachment, to fluid volume deficit.
increased intraocular pressure. Nursing Needed Info: Pyloric stenosis: obstruction of the
responsibilities sphincter between stomach and duodenum. Onset:
within 2 months of birth. S/S: vomiting that becomes nurse calculates that her expected date of
projectile. Treatment: surgery. Nursing confinement
responsibilities: (EDC) is which of the following?
small frequent feedings with glucose water or Reworded Question: How do you calculate the EDC?
electrolyte solutions 4–6 hrs post-op. Small frequent Strategy: Perform the calculation required and
feedings with formula 24 hrs post-op. check for math errors!
Category: Analysis/Physiological Needed Info: EDC or estimated date of delivery
Integrity/Physiological (EDD): calculated according to Nägele’s rule (first
Adaptation day of the last normal menstrual period minus 3
(1) The infant eagerly accepts feedings—may vomit months plus 7 days and 1 year). Assumes that every
after eating woman has a 28-day cycle and pregnancy occurred
(2) The infant vomited once since admission—don’t on 14th day. Most women deliver within a period
assume will continue to vomit extending from 7 days before to 7 days after the
(3) The infant’s skin is warm and moist—normal; EDC.
would be cool and dry with fluid volume deficit Category: Implementation/Health Promotion and
(4) The infant’s anterior fontanelle is depressed— Maintenance
CORRECT: indicates dehydration (1) May 15—too early
123. The Answer is 2 (2) June 15—CORRECT: September 8 minus 3
A client is diagnosed with thrombocytopenia due months = June 8 + 7 days plus 1 year = June 15
to acute lymphocytic leukemia. She is admitted to of next year
the hospital for treatment. To which of the following (3) June 21—EDC is calculated from first, not last,
should the nurse assign the client? day of last normal menstrual period
Reworded Question: What are the needs of a client (4) July 8—not accurate
with acute lymphocytic leukemia and 125. The Answer is 3
thrombocytopenia? A 2-month-old infant is brought to the pediatrician’s
Strategy: What is the highest priority for this client? office for a well-baby visit. During the examination,
Needed Info: Lymphocytic leukemia, disease congenital subluxation of the left hip is suspected.
characterized The nurse would expect to see which of the following
by proliferation of immature WBCs. Immature symptoms?
cells unable to fight infection as competently Reworded Question: What will you see with congenital
as mature white cells. Treatment: chemotherapy, hip dislocation?
antibiotics, blood transfusions, bone marrow Strategy: Form a mental image of the deformity.
transplantation. Needed Info: Subluxation: most common type of
Nursing responsibilities: private room, congenital hip dislocation. Head of femur remains
TChoen tPernatc tRiecvei eTwes atnd Practice in contact with acetabulum but is partially displaced.
for the NCLEX-RN ® Exam Diagnosed in infant less than 4 weeks old.
378 S/S: unlevel gluteal folds, limited abduction of hip,
The Practice Test shortened femur affected side, Ortolani’s sign (click).
no raw fruits or vegetables, small frequent meals, O2, Treatment: abduction splint, hip spica cast, Bryant’s
good skin care. traction, open reduction.
Category: Planning/Safe and Effective Care Category: Assessment/Health Promotion and
Environment/ Maintenance
Management of Care (1) Lengthening of the limb on the affected side—
(1) To a private room so she will not infect other inaccurate
clients and health care workers—poses little or (2) Deformities of the foot and ankle—inaccurate
no threat (3) Asymmetry of the gluteal and thigh folds—
(2) To a private room so she will not be infected by CORRECT: restricted movement on affected
other clients and health care workers—CORRECT: side
protects client from exogenous bacteria, (4) Plantar flexion of the foot—seen with clubfoot
risk for developing infection from others due 126. The Answer is 3
to depressed WBC count, alters ability to fight After 2 weeks of receiving lithium therapy, a client
infection in the psychiatric unit becomes depressed. Which of
(3) To a semiprivate room so she will have stimulation the following evaluations of the client’s behavior by
during her hospitalization—should be the nurse would be MOST accurate?
placed in a room alone Reworded Question: Is the depression normal, or
(4) To a semiprivate room so she will have the something to be concerned about?
opportunity Strategy: Think about each answer and how it relates
to express her feelings about her illness— to lithium therapy.
ensure that client is provided with opportunities Practice Test Explanations
to express feelings about illness 379
124. The Answer is 2 Practice Test Answers Taensdt Needed Info:
A woman comes to the clinic because she thinks she Lithium is used to control manic episodes
is pregnant. Tests are performed and the pregnancy of bipolar psychosis; nursing care includes
is confirmed. The client’s last menstrual period monitor blood levels 2–3 times a week when started
began on September 8 and lasted for 6 days. The and monthly while on maintenance. Need fluid
intake of 2,500–3,000 mL/day and adequate salt
intake. Side effects include dizziness, hand tremors, Hemovac container. Teach use of artificial larynx or
impaired vision. esophageal speech.
Category: Evaluation/Psychosocial Integrity Category: Assessment/Physiological Integrity/
(1) The treatment plan is not effective; the client Reduction of Risk Potential
requires a larger dose of lithium—not accurate (1) Assess the extent of neck edema—not accurate
(2) This is a normal response to lithium therapy; the (2) Check his ability to swallow—CORRECT
client should continue with the current treatment (3) Observe for excessive drooling—seen with facial
plan—does not address safety needs paralysis, Bell’s palsy
(3) This is a normal response to lithium therapy; the (4) Tap the side of his neck gently and observe for
client should be monitored for suicidal behavior— facial twitching—Chvostek’s sign: test for
CORRECT: delay of 1–3 wks before med hypocalcemia
benefits seen and tetany, tap over facial nerve on side
(4) The treatment plan is not effective; the client of face, if mouth twitches, indicates tetany
requires an antidepressant—normal response 129. The Answer is 4
127. The Answer is 1 The nurse supervises care at an adult day-care center.
A client is admitted for treatment of pulmonary Four meal choices are available to the residents.
edema. During the admission interview, she states The nurse should ensure that a resident on a low-
she has a 6-year history of congestive heart failure cholesterol
(CHF). The nurse performs an initial assessment. diet receives which of the following meals?
When the nurse auscultates the breath sounds, the TChoen tPernatc tRiecvei eTwes atnd Practice
nurse should expect to hear which of the following? for the NCLEX-RN ® Exam
Reworded Question: What will you hear when listening 380
to the breath sounds for a client with CHF? The Practice Test
Strategy: Picture the situation as described. Reworded Question: What should a client on a
Needed Info: Use diaphragm of stethoscope to listen lowcholesterol
to breath sounds. Normal breath sounds: (1) diet eat?
vesicular: Strategy: Think about each answer.
low-pitched, swishing sounds heard at bases Needed Info: Low-cholesterol diet should reduce
of lungs, (2) bronchial: loud, high-pitched, hollow total fat to 20–25% of total calories and reduce the
sounds heard over large tracheal airways during ingestion of saturated fat. Carbohydrates (especially
expiration, (3) bronchovesicular: breeze sound heard complex carbohydrates) should be 55–60% of
over central large airways. calories.
Category: Assessment/Physiological High-cholesterol foods: eggs, dairy products,
Integrity/Physiological meat, fish, shellfish, poultry.
Adaptation Category: Implementation/Physiological Integrity/
(1) Crackling—CORRECT: rales; air passes over Basic Care and Comfort
fluid; heard during inspiration; found with pulmonary (1) Egg custard and boiled liver—high amounts of
edema, pneumonia cholesterol
(2) Wheezing—passage of air through narrowed (2) Fried chicken and potatoes—avoid fried foods
airway; heard during inspiration and expiration; (3) Hamburger and french fries—avoid fried foods
found with asthma (4) Grilled flounder and green beans—CORRECT:
(3) Whistling—noisy respirations; air through fish instead of meat, increase vegetables
obstructed larynx 130. The Answer is 1
(4) Absent breath sounds—pneumothorax; collapse The nurse cares for a client with a possible bowel
of lung obstruction. A nasogastric (NG) tube is to be
128. The Answer is 2 inserted. Before inserting the tube, the nurse
A man is diagnosed with cancer of the larynx and explains its purpose to the client. Which of the
comes to the hospital for a total laryngectomy. When following
admitting this client, the nurse should assess explanations by the nurse is MOST accurate?
laryngeal Reworded Question: What is the purpose of an NG
nerve function by doing which of the following? tube?
Reworded Question: How do you assess normal Strategy: Think about how an NG tube works.
functioning Needed Info: Gastric tubes can also be used for tube
of the laryngeal nerve? What functions are feedings. Decompression relieves pressure caused by
controlled by the laryngeal nerve? GI contents and gases that remain in stomach due to
Strategy: Think about each answer and how it relates obstruction.
to laryngeal nerve function. Category: Implementation/Physiological Integrity/
Needed Info: Risk factors: smoking, chronic Basic Care and Comfort
bronchitis, (1) “It empties the stomach of fluids and gas.”—
polluted air, alcohol abuse. S/S: chronic hoarseness, CORRECT: used for decompression, gavage,
lump in neck, difficulty swallowing, persistent lavage, gastric analysis
sore throat. Treatment: radiation therapy, surgical (2) “It prevents spasms of the sphincter of Oddi.”—
removal. Total laryngectomy: loss of voice. Nursing controls release of pancreatic juices and bile into
responsibilities post-op: position semi-Fowler’s duodenum
to high Fowler’s, care for cuffed tracheostomy tube, (3) “It prevents air from forming in the small and
chest physiotherapy, assess drainage on dressing or large intestine.”—goes only to duodenum
(4) “It removes bile from the gallbladder.”—action is aware of the possible complications of estrogen
of T-tube therapy?
131. The Answer is 2 Reworded Question: What complications are seen
The nurse cares for a client diagnosed with with the use of estrogen therapy?
cholecystitis. Strategy: Think about each answer and the effects
The client says to the nurse, “I don’t understand of estrogen therapy.
why my right shoulder hurts, when the gallbladder Needed Info: Estrogen therapy predisposes to cancer
is not near my shoulder!” Which of the following of reproductive organs. Other side effects are
responses by the nurse is BEST? nausea, skin rashes, pruritis, breast secretion, and
Reworded Question: Why does the client’s shoulder thromboembolic disorders. Used cautiously with
hurt? family history of breast or genital tract cancer.
Strategy: “BEST” indicates discrimination is Category: Implementation/Health Promotion and
required to answer the question. Maintenance
Needed Info: Cholecystitis is inflammation of the (1) “Take an analgesic before you take estrogen,
gallbladder; indications include intolerance to fatty because estrogen may cause discomfort.”—not
foods, indigestion, severe pain in upper right quadrant accurate; may cause nausea, weight gain, lethargy
of abdomen radiating to back and right shoulder; (2) “Make sure you keep your clinic appointments,
leukocytosis, and diaphoresis. especially your gynecologic checkup.”—CORRECT:
Category: Implementation/Physiological Integrity/ have checkup at 6 months
Physiological Adaptation (3) “Limit your fluid intake, because estrogen
(1) “Sometimes small pieces of the gallstones break promotes
off and travel to other parts of the body.”—gallstones the retention of fluids.”—causes fluid
do not become emboli retention and edema; monitor weight; restrict
(2) “There is an invisible connection between the Na+ intake; don’t limit fluids
gallbladder and the right shoulder.”—CORRECT: (4) “Increase roughage in your diet to avoid
describes referred pain; when visceral constipation.”—
branch of a pain receptor fiber is stimulated, not a complication of estrogen therapy
vasodilation and pain may occur in a distant 134. The Answer is 3
body area; right shoulder or scapula is the Several days after being admitted for depression, a
referred-pain site for gallbladder man is observed sitting alone in the clients’ dining
(3) “The gallbladder is on the right side of the body room. The nurse notes that the client has not finished
and so is that shoulder.”—anatomically correct his meal. Which of the following nursing measures
but is not the best explanation would be MOST appropriate?
(4) “Your shoulder became tense because you were Reworded Question: How would you meet this client’s
guarding against the gallbladder pain.”—possible; needs?
not the best explanation Strategy: Determine the outcome of each answer. Is
132. The Answer is 3 it desired?
The nurse teaches a primigravid woman how to Needed Info: Symptoms of depression: withdrawn,
measure the frequency of uterine contractions. The regressive behavior, psychomotor retardation.
nurse should explain to the client that the frequency Category: Planning/Psychosocial Integrity
of uterine contractions is determined by which of the (1) Allow the client to eat in his room until he
following? becomes more comfortable eating with other
Reworded Question: How do you determine the clients—social isolation, reinforces depression
frequency (2) Ask the client’s family to bring foods that he likes
of uterine contractions? to eat—does not address problem
Strategy: Think about each answer. (3) Order small, frequent meals and sit with the
Practice Test Explanations client while he eats in the dining room—CORRECT:
381 diminished appetite, prevents social isolation
Practice Test Answers Taensdt Needed Info: There (4) Do not focus on eating behaviors because his
must be at least 3 contractions to appetite will improve over time—does not meet
establish frequency. nutritional needs
Category: Implementation/Health Promotion and 135. The Answer is 3
Maintenance A client is being treated for injuries sustained in an
(1) By timing from the beginning of one contraction automobile accident. The client has a central venous
to the end of the next contraction—not accurate pressure (CVP) line in place. The nurse recognizes
(2) By timing from the beginning of one contraction that CVP measurement reflects which of the following?
to the end of the same contraction—defines Reworded Question: What does CVP measure?
duration Strategy: Think about CVP and cardiac function.
(3) By the number of contractions that occur within Needed Info: CVP: central venous line placed in
a given period of time—CORRECT superior vena cava. To obtain a reading: client
(4) By the strength of the contraction at its peak— placed supine, 0 on manometer placed at level of
describes intensity right atrium (midaxillary line at 4th intercostal
133. The Answer is 2 space), turn stopcock to allow manometer to fill with
The nurse is teaching a woman who is receiving fluid, turn to allow fluid to go into client. Fluid will
estrogen replacement therapy. Which of the following fluctuate with resp. When stabilizes take reading at
statements by the nurse indicates that the nurse highest level of fluctuation. Normal: 4–10 cm/H2O.
Elevated: hypervolemia, CHF, pericarditis. Low: if unconscious; client education.
hypovolemia. Category: Planning/Physiological
TChoen tPernatc tRiecvei eTwes atnd Practice Integrity/Pharmacological
for the NCLEX-RN ® Exam and Parenteral Therapies
382 (1) Check vital signs—not first action; should
The Practice Test recognize
Category: Analysis/Physiological Integrity/Reduction S/S hypoglycemia
of Risk Potential (2) Check urine for glucose and ketones—indicates
(1) Cardiac output—Swan-Ganz line only hyperglycemia, no information about
(2) Pressure in the left ventricle—Swan-Ganz line hypoglycemia;
(3) Pressure in the right atrium—CORRECT: should recognize S/S hypoglycemia
determined (3) Give 6 oz. of skim milk—CORRECT: S/S of
by blood volume, vascular tone, action of hypoglycemia; give fast-acting sugar and protein;
right side of heart recheck blood sugar in 15 min
(4) Pressure in the pulmonary artery—Swan-Ganz (4) Call the physician—not necessary; unless
line: 4-lumen, balloon-tipped, flow-directed hypoglycemia
catheter is not corrected
136. The Answer is 2 138. The Answer is 2
A mother brings her 4-year-old daughter to the Prior to the client undergoing a scheduled intravenous
pediatrician pyelogram (IVP), the nurse reviews the client’s
for treatment of chronic otitis media. The health history. It would be MOST important for the
mother asks the nurse how she can prevent her child nurse to obtain the answer to which of the following
from getting ear infections so often. The nurse’s questions?
response should be based on an understanding that Reworded Question: What do you need to know
the recurrence of otitis media can be decreased by before an IVP?
which of the following? Strategy: Think about each answer and how it
Reworded Question: What will prevent the relates to an IVP.
development Needed Info: IVP: radiopaque dye that contains
of otitis media? What causes otitis media? iodine injected into the body and is filtered through
Strategy: Think about the causes of otitis media. the kidneys and excreted by the urinary tract.
Needed Info: Otitis media: frequently follows Visualizes
respiratory kidneys, ureters, and bladder. Preparation: NPO
infection. Reduce occurrences: holding child midnight, cathartics evening before test. Injection
upright for feedings, encourage gentle nose-blowing, Practice Test Explanations
teach modified Valsava maneuver (pinch nose, close 383
lips, and force air up through eustachian tubes), Practice Test Answers Taensdt of dye causes
blow up balloons or chew gum, eliminate tobacco flushing of face, nausea, salty taste in
smoke or known allergens. mouth.
Category: Analysis/Health Promotion and Category: Assessment/Physiological Integrity/
Maintenance Reduction in Risk Potential
(1) Covering the child’s ears while bathing—not (1) Does the client have difficulty voiding?—not
preventive most important
(2) Treating upper respiratory infections quickly— (2) Does the client have any allergies to shellfish
CORRECT: respiratory fluids are a medium for or iodine?—CORRECT: anaphylactic reaction;
bacteria; antihistamines used itching, hives, wheezing; treatment: antihistamines,
(3) Administering nose drops at bedtime—not O2, CPR, epinephrine, vasopressor
preventive (3) Does the client have a history of constipation?
(4) Isolating her child from other children—too —not essential info
extreme a measure (4) Does the client have frequent headaches?—not
137. The Answer is 3 most important
A client receives 10 units of NPH insulin every morning 139. The Answer is 1
at 8 a.m. At 4 p.m., the nurse observes that the client A child with chickenpox (varicella) is brought by her
is diaphoretic and slightly confused. The nurse parents to the physician for evaluation. The nurse
should take which of the following actions FIRST? knows the rash characteristic of chickenpox can be
Reworded Question: What is the cause of these described as which of the following?
symptoms? Reworded Question: What does the rash from
What is the first thing you should do? chickenpox
Strategy: “FIRST” indicates that this is a priority look like?
question. Strategy: Form a mental image of client with
Needed Info: NPH insulin: intermediate-acting characteristic
preparation: rash.
onset 1–4 hrs, peak 2–15 hrs, duration 12–28 Needed Info: Chickenpox transmission: direct
hrs. S/S hypoglycemia: confusion, tremors, contact,
hypotension, droplet. Incubation period: 13–17 days. Treatment:
cool clammy skin, diaphoresis. Treatment: if Acyclovir, diphenhydramine hydrochloride,
conscious, liquids containing sugar; dextrose 50% IV and/or calamine lotion for itching, good skin care
to prevent secondary infection, bathe daily, change Category: Analysis/Physiological
clothes and linens, strict isolation in hospital, at Integrity/Physiological
home isolate until vesicles have dried (usually 1 week Adaptation
after onset), short fingernails, avoid use of aspirin TChoen tPernatc tRiecvei eTwes atnd Practice
due to Reye’s syndrome. Measles transmission: for the NCLEX-RN ® Exam
direct contact, droplets. Incubation period: 10–20 384
days. Symptoms: fever, cough, conjunctivitis, The Practice Test
erythematous (1) Potential impaired family coping related to
maculopapular rash on face. Treatment: diagnosis
bed rest, antipyretics, antibiotics to prevent of GDM—not highest priority; psychosocial
secondary need
infection. Isolate until 5th day of rash, cool mist (2) Potential noncompliance related to lack of
vaporizer, good skin care, dim lights. knowledge or lack of adequate support system—
Category: Assessment/Health Promotion and CORRECT: client may not be able to meet physical
Maintenance needs because of lack of knowledge
(1) Maculopapular—CORRECT: prodromal stage: (3) Potential for altered parenting related to
slight fever, malaise and anorexia, maculopapular disappointment—
rash, becomes vesicular: fluid-filled vesicles psychosocial need; not highest
form crusts, or scabs, communicable from 1 day priority
before eruption of lesions (during prodromal (4) Ineffective family coping related to anticipatory
stage) up to 6 days after first crop of vesicles grieving—psychosocial need
appear and crusts form 141. The Answer is 3
(2) Small, irregular red spots with minute bluishwhite The nurse cares for a client admitted for a possible
centers—Koplik spots: prodromal stage herniated intervertebral disk. Ibuprofen,
of measles, first seen on buccal mucosa 2 days propoxyphene
before rash hydrochloride, and cyclobenzaprine hydrochloride
(3) Round or oval erythematous scaling patches— are ordered PRN. Several hours after
psoriasis: treatment: exposure to sunlight/ultraviolet admission, the client complains of pain. Which of
light, topical corticosteroids, coal-tar the following actions should the nurse do FIRST?
derivates Reworded Question: What should you do first?
(4) Petechiae—pinpoint, nonraised, perfectly round Strategy: Set priorities. Compare the answers to the
purplish red spots caused by intradermal or steps in the nursing process.
submucosal Needed Info: Herniated disk: knifelike pain
hemorrhage, seen in severe sepsis with aggravated
disseminated intravascular coagulation (DIC), by sneezing, coughing, straining.
Rocky Mountain spotted fever, and subacute Category: Planning/Physiological
bacterial endocarditis (SBE) Integrity/Pharmacological
140. The Answer is 2 and Parenteral Therapies
A primigravid woman at 28 weeks’ gestation takes a (1) Administer ibuprofen—implementation; not first
3-hour glucose tolerance test. The results indicate a step
fasting blood sugar of 100 mg/dL and a 2-hour (2) Call the physician to determine which medication
postload should be given—assess before implementing
blood sugar of 300 mg/dL. Which of the following (3) Gather more information from the client about
nursing diagnoses should be considered the the complaint—CORRECT: assess; first step in
HIGHEST priority at this time? nursing process
Reworded Question: What is most important for a (4) Allow the client some time to rest and see if the
newly diagnosed client with gestational diabetes pain subsides—implementation; not first step
mellitus (GDM)? 142. The Answer is 2
Strategy: Use Maslow’s hierarchy of needs to establish When planning care for a client hospitalized with
priorities. Remember to first meet physical depression, the nurse includes measures to increase
needs before addressing other concerns. his self-esteem. Which of the following actions
Needed Info: GDM: carbohydrate intolerance that should the nurse take to meet this goal?
occurs during pregnancy in women with no prior Reworded Question: How do you increase the
history selfesteem
of diabetes. May exhibit the classic symptoms of of a depressed client?
diabetes: polyuria (excessive urination), polydipsia Strategy: Think about each answer in relation to
(excessive thirst), and polyphagia (hunger). Half the depression.
women are asymptomatic. Diagnosed: 3-hr glucose Needed Info: Increase self-esteem: warm, supportive
tolerance test (GTT) (administer a high glucose load environment, consistent daily care.
to fasting pt; blood glucose levels are measured Category: Implementation/Psychosocial Integrity
fasting, (1) Encourage him to accept leadership
and at 1-hr intervals for 3 hrs; test is positive if responsibilities
2 or more of the blood sugars are elevated). Normal: in milieu activities—too demanding
fasting, 60–110 mg/dL; 1 hr—190; 2 hrs—165; (2) Set simple, realistic goals with him to help
3 hrs—145. him experience success—CORRECT: sense of
accomplishment
(3) Help him to accept his illness and the adjustments (3) The aide puts the food in the back of the client’s
that are required—does not help feelings mouth on the unaffected side—helps client handle
of hopelessness food
(4) Assure him that when he is discharged, he will (4) The aide waters down the pudding to help the
be able to resume his previous activities—false client swallow—CORRECT: requires intervention,
reassurance usually able to better handle soft or semisoft
143. The Answer is 3 foods; difficulty with liquids
The nurse finds a visitor unconscious on the floor of 145. The Answer is 3
a client’s room during visiting hours at the hospital. The home care nurse plans care for a client with
Which of the following nursing assessments is pernicious
consistent anemia. A monthly intramuscular injection
with cardiopulmonary arrest? is ordered for the client. The nurse knows that in an
Reworded Question: What are the signs of adult, the best muscle to administer an intramuscular
cardiopulmonary injection is which of the following?
arrest? Reworded Question: Where should you give an IM
Strategy: Think about the steps you would take to injection in an adult?
evaluate an unconscious client. Strategy: Think about each answer in relation to
Needed Info: Cardiopulmonary arrest: heart, anatomy and physiology.
circulation Needed Info: Pernicious anemia: lack of intrinsic
and respirations cease. CPR: (1) determine factor
unresponsiveness, (2) open airway (head tip–chin from stomach leading to decreased absorption
lift maneuver or jaw thrust), (3) determine of vitamin B12. S/S: low hemoglobin and hematocrit.
breathlessness Diagnosed: Schilling test (measures absorption of
(look, listen, feel), (4) perform rescue breathing orally administered radioactive B12 by amount of
(2 slow breaths, chest rise 1–2 in.), (5) determine radioactive B12 excreted in urine in 24 hrs).
pulselessness (check carotid pulse 5–10 sec), (6) Treatment:
provide lifelong B12 injections, iron supplements. Factors
circulation (chest compressions 1.5–2 in.). to consider when selecting site for injection:
Category: Assessment/Physiological amount of muscle mass and condition, amount and
Integrity/Physiological character of med, type of med, frequency of
Adaptation injections.
(1) Absent pulse, fixed dilated pupils—not accurate Category: Implementation/Physiological Integrity/
(2) Absent respirations, fixed and dilated pupils— Pharmacological and Parenteral Therapies
not accurate (1) Gluteus maximus—possible injury to sciatic
(3) Absent pulse and respirations—CORRECT: no nerve
palpable pulse; no breath sounds; ashen color (2) Deltoid—not well developed in some adults,
(4) Thready pulse and pupillary changes—not especially elderly; possible injury to brachial
accurate artery; can only use for small amount of med
Practice Test Explanations (3) Vastus lateralis—CORRECT: no major nerves
385 or blood vessels; to locate, palpate greater trochanter
Practice Test Answers Taensdt 144. The Answer and knee joint; divide distance between
is 4 them into quadrants; inject into middle of upper
A client is transferred to an extended care facility quadrant
after a cerebrovascular accident (CVA). The client (4) Dorsogluteal—possible injury to sciatic nerve
has right-sided paralysis and has been experiencing 146. The Answer is 2
dysphagia. The nurse observes an aide preparing the A man comes to the emergency room complaining
client to eat lunch. Which of the following situations of nausea, vomiting, and severe right upper quadrant
would require an intervention by the nurse? pain. His temperature is 101.3° F (38.5° C) and
Reworded Question: Which option is wrong? an abdominal x-ray reveals an enlarged gallbladder.
Strategy: This is a negative question. Make sure you He is given a diagnosis of acute cholecystitis and is
know if you are looking for a correct situation or a scheduled for surgery. After administering an
problematic situation. analgesic
Needed Info: Dysphagia: difficulty swallowing. to the client, the nurse recognizes that which
Provide of the following actions is the HIGHEST priority?
support if necessary for the head, have the client Reworded Question: What should you do after giving
upright, feed the client slowly in small amounts, an analgesic to the client?
place food on unaffected side of mouth. Maintain Strategy: Establish priorities. Remember Maslow’s
upright position for 30–45 minutes after eating. hierarchy of needs. Meet physical needs first.
Good oral care after eating. Needed Info: S/S: pain in upper midline area radiating
Category: Evaluation/Physiological Integrity/ around to back, jaundice, nausea, vomiting,
Reduction of Risk Potential flatulence, bloating, belching, intolerance to fatty
(1) The client is in bed in high Fowler’s position— foods. Treatment: cholecystectomy (removal of
correct positioning, or may sit in chair gallbladder).
(2) The client’s head and neck are positioned slightly Post-op: T-tube inserted for drainage from
forward—correct positioning; helps client chew bile duct. Complications: hemorrhage, pneumonia,
and swallow thrombophlebitis, urinary retention, ileus. Preop
nursing responsibilities: Meperidine for pain Reworded Question: What is not accurate about the
(morphine care of a woman with PIH?
contraindicated; causes spasms for sphincter Strategy: This is a negative question. It can be
TChoen tPernatc tRiecvei eTwes atnd Practice reworded to say, “All of the following are true
for the NCLEX-RN ® Exam EXCEPT.”
386 Needed Info: PIH, preeclampsia, toxemia:
The Practice Test development
of Oddi), nitroglycerin to relax smooth muscle, of hypertension (increase 30 mmHg systolic
NG tube for decompression, IVs. Post-op nursing or 15 mmHg diastolic) with proteinuria and/or
responsibilities: change position every 2 hrs, check edema (dependent or facial) after 20 weeks’
breath sounds and vital signs every 4 hrs, I & O, gestation.
antiembolitic Risk factors: parity (first-time mothers), age
stockings. (younger than 20 or older than 35), geographic
Category: Planning/Physiological location
Integrity/Physiological (southern or western U.S.), multifetal gestation,
Adaptation hydatidiform mole, hypertension, and diabetes.
(1) Assessing the client’s need for dietary teaching— Prevention: early prenatal care, identify high risk
not highest priority clients, recognize S/S early; bed rest lying on left
(2) Assessing the client’s fluid and electrolyte status— side, daily weights. Treatment: urine checks for
CORRECT: hypokalemia and hypomagnesemia proteinuria;
common diet (increased protein and decreased Na+).
(3) Examining the client’s health history for allergies Can develop into eclampsia (convulsions or coma).
to antibiotics—not highest priority Category: Evaluation/Health Promotion and
(4) Determining whether the client has signed consent Maintenance
for surgery—not highest priority (1) “Lying in bed on my left side is likely to increase
147. The Answer is 3 my urinary output.”—true; bed rest promotes
A mother with 4 children calls the clinic for advice good perfusion of blood to uterus; decreases BP
on how to care for her oldest child, who has developed and promotes diuresis
chickenpox. Which of the following statements (2) “If the bed rest works, I may lose a pound or
by the mother indicates a need for further teaching? two in the next few days.”—true; causes diuresis;
Reworded Question: What teaching is necessary for results in reduction of retained fluids; instruct
parent of child with chickenpox? to monitor weight daily and notify physician if
Strategy: Be careful! This is a negative question. You notices abrupt increase even after resting in bed
are looking for incorrect info. for 12 hrs
Needed Info: Teaching: calamine lotion for itching, (3) “I should be sure to maintain a diet that has a
good skin care to prevent secondary infection, bathe good amount of protein.”—true; replaces protein
daily, change clothes and linens, isolate until vesicles lost in urine; increases plasma colloid osmotic
have dried (usually 1 week after onset), short Practice Test Explanations
fingernails, 387
avoid use of aspirin due to Reye’s syndrome. Practice Test Answers Taensdt pressure; avoid
Category: Evaluation/Safe and Effective Care salty foods; avoid alcohol; drink
Environment/ 8 glasses of water daily; eat foods high in roughage
Safety and Infection Control (4) “I will have to keep my room darkened and not
(1) “I should keep my child home from school until watch much television.”—CORRECT: incorrect
the vesicles are crusted.”—correct information; info, not necessary; diversional activities helpful
chickenpox transmitted by direct contact 149. The Answer is 2
with droplets of infected person; communicable The nurse evaluates the care provided to a client
period: 2 days before rash until vesicles crusted hospitalized
(scabbed), then child may interact with siblings for treatment of adrenal crisis. Which of
and others the following changes would indicate to the nurse
(2) “I can use calamine lotion if needed.”—correct that the client is responding favorably to medical and
information, used to treat itching nursing treatment?
(3) “I should remove the crusts so the skin can Reworded Question: What shows a positive response
heal.”—CORRECT: indicates need for further to treatment for adrenal crisis?
teaching; good skin care important; crusts usually Strategy: Think about each answer.
not removed, can cause scarring Needed Info: In adrenal crisis the required adrenal
(4) “I can use mittens if scratching becomes a hormones exceed the supply available. Usually
problem.”— precipitated
rash itches; mittens used to prevent by stress, surgery, trauma, or infection. S/S:
scratching hypotension, cool pale skin, increased urinary output,
148. The Answer is 4 dehydration.
The nurse is teaching a woman who comes to the Category: Evaluation/Physiological
clinic at 32 weeks’ gestation with a diagnosis of Integrity/Physiological
pregnancy-induced hypertension (PIH). Which of Adaptation
the following statements by the client indicates to to (1) The client’s urinary output has increased—
the nurse that further teaching is required? indicates
continuing lack of hormones; will decrease (2) Clotting time—CORRECT: or partial
with treatment thromboplastin
(2) The client’s blood pressure has increased— time (PTT); 1.5–2 times control, clotting
CORRECT: time 2–3 times control
hypotension S/S of adrenal insufficiency; TChoen tPernatc tRiecvei eTwes atnd Practice
without treatment Na+ level falls, resulting in volume for the NCLEX-RN ® Exam
depletion and hypotension; K+ rises, resulting 388
in cardiac dysrhythmias The Practice Test
(3) The client has lost weight—indicates continuing (3) Bleeding time—duration of bleeding after small
loss of water and continuing lack of hormones puncture wound; detects platelet and vascular
(4) The client’s peripheral edema has decreased— problems; not altered
edema not seen with adrenal crises (4) Prothrombin time—PT used to monitor warfarin
150. The Answer is 4 therapy
After completing an assessment, the nurse 152. The Answer is 3
determines A client comes to the clinic for evaluation of acute
that a client is exhibiting early symptoms of onset of seizures. A thorough history and physical
a dystonic reaction related to the use of an examination is performed. The nurse would expect
antipsychotic which of the following diagnostic tests to be performed
medication. Which of the following actions by FIRST?
the nurse would be MOST appropriate? Reworded Question: What test is used to diagnose
Reworded Question: What is the first thing you do for seizure disorders?
a client with a dystonic reaction? Strategy: Consider the purpose of each test.
Strategy: Set priorities. Remember Maslow’s Needed Info: EEG: recording of electrical activity
hierarchy of brain. Electrodes attached to scalp, waveforms
of needs. recorded. Checked relaxing, hyperventilating,
Needed Info: Dystonic reaction: muscle tightness in sleeping, with lights flickering. Prep: kept awake
throat, neck, tongue, mouth, eyes, neck, and back; night before, shampoo hair. Stimulants (tea, coffee,
difficulty talking and swallowing. Treatment: IM or alcohol, cola, cigarettes), antidepressants,
IV diphenhydramine hydrochloride (Benadryl) or tranquilizers,
benztropine mesylate (Cogentin). anticonvulsants withheld 24–48 hours before
Category: Implementation/Psychosocial Integrity test. After test, seizure precautions and wash hair.
(1) Reality-test with the client and assure her that Seizure: uncontrolled discharge of electrical activity
her physical symptoms are not real—real symptoms, from brain.
not delusions Category: Planning/Physiological Integrity/Reduction
(2) Teach the client about common side effects of of Risk Potential
antipsychotic medications—physical needs are (1) Magnetic resonance imaging (MRI)—uses
highest priority magnetic
(3) Explain to the client that there is no treatment that fields to get detailed pictures; prep: remove
will relieve these symptoms—diphenhydramine jewelry, metal objects, lie still, may feel
hydrochloride used IM or IV claustrophobic;
(4) Notify the physician and obtain an order for IM not first choice for diagnosing seizure
diphenhydramine hydrochloride—CORRECT: disorders
emergency situation, can occlude airway (2) Cerebral angiography—dye injected into catheter
151. The Answer is 2 in femoral artery, x-rays taken; prep: check
The physician orders heparin for a client. In order sensitivity to dye; post-test: pressure on insertion
to evaluate the effectiveness of the client’s heparin site; not first choice for diagnosing seizure disorders
therapy, the nurse should monitor which of the (3) Electroencephalogram (EEG)—CORRECT
following (4) Electromyogram (EMG)—evaluates activity
laboratory values? of muscles; electrodes placed in nerves, small
Reworded Question: What blood work is done to amount of electricity applied
monitor heparin therapy? 153. The Answer is 1
Strategy: Think about each answer. The nurse performs dietary teaching with a client on
Needed Info: Heparin: anticoagulant. Side effects: a low-protein diet. The nurse knows that teaching
hemorrhage, thrombocytopenia. Antidote: Protamine has been successful if the client identifies which of
sulfate. When given subcutaneously, inject the following meals as lowest in protein?
slowly; leave needle in place 10 seconds, then Reworded Question: Which foods are the LOWEST
withdraw; in protein?
don’t massage site; rotate sites. Nursing Strategy: Consider each meal and eliminate those
responsibilities: check for bleeding gums, bruises, with high-protein components.
nosebleeds, petechiae, melena, tarry stools, Needed Info: Avoid high-protein foods: eggs, milk
hematuria; products, meat, beans, nuts, cereals.
use electric razor and soft toothbrush. Category: Evaluation/Physiological Integrity/Basic
Category: Assessment/Physiological Integrity/ Care and Comfort
Reduction of Risk Potential (1) Cranberries and broiled chicken—CORRECT:
(1) Platelet count—evaluates platelet production; cranberries, no protein; chicken, 7 g/oz.
not altered
(2) Tomatoes and flounder—tomato, 2 g/oz.; inappropriate practice; mask and gloves necessary
flounder, only when possibility of contact with blood
8 g/oz. and body fluids; when taking a BP, very low risk
(3) Broccoli and veal—broccoli, 2 g/oz.; veal, 7 g/oz. for contact with blood and body fluids; behavior
(4) Spinach and tofu—spinach, 2 g/serving; tofu, 7 insensitive to client’s feelings, does not promote
g/oz. trust
154. The Answer is 1 (3) A technician wears gloves to perform a
A client has a vagotomy with antrectomy to treat a venipuncture—
duodenal ulcer. Postoperatively, the client develops safe practice, barrier precaution used
dumping syndrome. Which of the following to prevent skin and mucous-membrane exposure
statements if contact with blood or other body fluids of any
by the client indicates to the nurse that further client anticipated
dietary teaching is necessary? (4) A nurse attendant allows visitors to enter his
Reworded Question: What is contraindicated for the room without masks—appropriate activity:
client with dumping syndrome? visitors do not need masks, PCP parasite found
Strategy: Be careful! You are looking for incorrect in lungs of healthy people, thought to cause
information. subclinical
Needed Info: Antrectomy: surgery to reduce pulmonary infection worldwide, only
acidsecreting dangerous to immunosuppressed clients; sick
portions of stomach. Delays or eliminates people not permitted to visit client
gastric phase of digestion. Dumping syndrome 156. The Answer is 2
occurs in clients after a gastric resection. It occurs A woman comes to the physician’s office for a routine
after eating and is related to the reduced capacity prenatal checkup at 34 weeks’ gestation. Abdominal
of the stomach. Undigested food is dumped into palpation reveals the fetal position as right occipital
the jejunum, resulting in distention, cramping, anterior (ROA). At which of the following sites
pain, diarrhea 15–30 min after eating. Subsides in would the nurse expect to find the fetal heart tone?
6–12 months. S/S 5–30 min after eating: vertigo, Reworded Question: The fetus is ROA. Where should
tachycardia, syncope, diarrhea, nausea. Treatment: the nurse listen for the FHT?
sedatives, antispasmodics; high-protein, high-fat, Strategy: Picture the situation described. It may be
low-carbohydrate, dry diet. Eat in semirecumbent helpful for you to draw this out so that you can
position, lying down after eating. imagine
Practice Test Explanations where the heartbeat would be found.
389 Needed Info: Describing fetal position: practice of
Practice Test Answers Taensdt Category: defining position of baby relative to mother’s pelvis.
Evaluation/Physiological Integrity/Basic The point of maximum intensity (PMI) of the
Care and Comfort fetus: point on mother’s abdomen where FHT is the
(1) “I should eat bread with each meal.”—CORRECT: loudest, usually over the fetal back. Divide mother’s
incorrect info; should decrease intake of pelvis into 4 parts or quadrants: right and left
carbohydrates anterior (front), and right and left posterior (back).
(2) “I should eat smaller meals more frequently.”— Abbreviated: R and L for right and left, and A and P
true; 5–6 small meals per day for anterior and posterior. The head, particularly the
(3) “I should lie down after eating.”—true; delays occiput, is the most common presenting part, and is
gastric emptying time abbreviated O. LOA is most common fetal
(4) “I should avoid drinking fluids with my presentation
meals.”—true; no fluids 1 hr before, with, or 2 and FHT heard on left side. In a vertex presentation,
hrs after meal FHT is heard below the umbilicus. In a breech
155. The Answer is 2 presentation, FHT is heard above umbilicus.
A man is admitted to the hospital with a diagnosis of Category: Assessment/Health Promotion and
acquired immunodeficiency syndrome (AIDS). He is Maintenance
being treated for Pneumocystis jiroveci pneumonia. (1) Below the umbilicus, on the mother’s left side—
The nurse evaluates the care provided to this client found on right, not left, side
by other members of the health care team. The nurse (2) Below the umbilicus, on the mother’s right
should intervene in which of the following situations? side—CORRECT: occiput and back are pressing
Reworded Question: Which situation describes an against right side of mother’s abdomen; FHT
unsafe or inappropriate practice? would be heard below umbilicus on right side
Strategy: Picture each situation as described in the (3) Above the umbilicus, on the mother’s left side—
question. found in breech presentation
Category: Evaluation/Safe and Effective Care (4) Above the umbilicus, on the mother’s right
Environment/ side—found in breech presentation
Safety and Infection Control TChoen tPernatc tRiecvei eTwes atnd Practice
(1) A housekeeper cleans up spilled blood with a for the NCLEX-RN ® Exam
bleach solution—appropriate activity, solution 390
of 1:10 sodium hypochlorite, or bleach with The Practice Test
water, kills AIDS virus 157. The Answer is 3
(2) A nursing student takes the client’s blood pressure A client is admitted to the hospital with complaints
wearing a mask and gloves—CORRECT: of seizures and a high fever. A brain scan is ordered.
Before the scan, the client asks the nurse what is admitted to the hospital for treatment of
position hemolytic anemia. Which of the following measures
he will be in while the procedure is being done. incorporated into the nursing care plan BEST
Which of the following statements by the nurse is addresses the client’s needs?
MOST accurate? Reworded Question: What should you do for a client
Reworded Question: What is the proper position for with anemia?
a brain scan? Strategy: Although the client has leukemia, he is
Strategy: Think about each answer. admitted with anemia. You must focus on the anemia.
Needed Info: Brain scan: measures amount of uptake Needed Info: Lymphocytic leukemia: characterized
by the brain of radioactive isotopes. Damaged tissue by proliferation of lymphocytes. S/S: fatigue,
absorbs more than normal tissue. Nursing care weakness, headache, easy bruising, bleeding gums,
before: withhold medications (antihypertensives, epistaxis, fever, generalized pain. Diagnostic tests:
vasoconstrictors, vasodilators for 24 hrs). During CBC, bone marrow aspiration, lumbar puncture,
the test, client will need to change position while x-rays, lymph node biopsy. Treatment: total body
pictures irradiation or radiation to spleen, chemotherapy.
of the brain are taken. Test is painless. After Nursing responsibilities: low-bacteria diet (no raw
test, force fluids to promote excretion of isotopes. fruits or vegetables), institute bleeding precautions
Urine doesn’t need special handling. (soft toothbrush, don’t floss, no injections, no aspirin,
Category: Implementation/Physiological Integrity/ pad bed rails, use air mattress, use paper tape),
Reduction of Risk Potential antiemetics, comfort measures. Hemolytic anemia
(1) “You will be in a side-lying position with the foot Practice Test Explanations
of the bed elevated.”—incorrect 391
(2) “You will be in a semi-upright sitting position Practice Test Answers Taensdt S/S: jaundice,
with your knees flexed.”—incorrect splenomegaly, hepatomegaly, fatigue,
(3) “You will be lying supine with a small pillow weakness. Treatment: O2, blood transfusions,
under your head.”—CORRECT corticosteroids.
(4) “You will be flat on your back, with your feet Category: Planning/Physiological Integrity/Basic
higher than your head.”—incorrect Care and Comfort
158. The Answer is 3 (1) Encourage activities with other clients in the
A man is admitted to the psychiatric hospital with dayroom—does not meet need for rest
a diagnosis of obsessive-compulsive disorder. He (2) Isolate the client from visitors and clients to avoid
is unable to stay employed because his ritualistic infection—no info given about WBC or reverse
behavior causes him to be late for work. Which of isolation; on reverse isolation if neutrophil count
the following interpretations by the nurse of the is less than 500/mm3
client’s (3) Provide a diet high in vitamin C—needed for
behavior is MOST accurate? wound healing and resistance to infection; not
Reworded Question: Why does the client perform best choice
ritualistic behavior? (4) Provide a quiet environment to promote adequate
Strategy: Think about each answer in relation to rest—CORRECT: primary problem activity
compulsive activity. intolerance due to fatigue
Needed Info: Obsession: recurrent or persistent 160. The Answer is 1
thought, image, or impulse. Compulsion: repetitive, The nurse plans morning care for a client hospitalized
purposeful, or intentional behavior performed after a cerebrovascular accident (CVA) resulting
in a stereotypical manner. Nursing responsibilities: in left-sided paralysis and homonymous hemianopia.
accept ritualistic behavior, structure environment, During morning care, the nurse should do which
meet physical needs, minimize choices. Anafranil: of the following?
tricyclic antidepressant. Side effects: dizziness, Reworded Question: What should you do for morning
libido change, nervousness, dry mouth, sweating, care for this client?
urine retention, constipation, photosensitivity. Strategy: Think about the outcome of each answer
Category: Analysis/Psychosocial Integrity choice.
(1) He is responding to auditory hallucinations and Needed Info: Homonymous hemianopia: blindness
trying to gain control over his behavior— in half of each visual field caused by damage
hallucinations: to brain. Client cannot see past midline toward the
false sensory perceptions in the absence side opposite the lesion without turning the head
of external stimuli, associated with schizophrenia toward that side. Approach client from side that is
(2) He is fulfilling an unconscious desire to punish not visually impaired. Reduce noise and complexity
himself—not accurate of decision-making.
(3) He is attempting to reduce anxiety by taking Category: Implementation/Physiological Integrity/
control of the environment—CORRECT: Physiological Adaptation
unconscious attempt to reduce anxiety (1) Provide care from the client’s right side—
(4) He is malingering in order to avoid responsibilities CORRECT:
at work—conscious feigning of illness approach from side with intact vision
to promote secondary gain, conscious effort to (2) Speak loudly and distinctly when talking with
manipulate the client—no hearing loss
159. The Answer is 4 (3) Reduce the level of lighting in the client’s room to
A client diagnosed with chronic lymphocytic leukemia prevent glare—increase light to assist with vision
(4) Provide all of the client’s care to reduce his 163. The Answer is 3
energy expenditure—encourage independence The nurse cares for a client diagnosed with bipolar
161. The Answer is 2 disorder. The client paces endlessly in the halls and
The nurse prepares for the admission of a client with makes hostile comments to other clients. The client
a perforated duodenal ulcer. Which of the following resists the nurse’s attempts to move him to a room in
should the nurse expect to observe as the primary the unit. Which of the following actions by the nurse
initial symptom? is MOST important?
Reworded Question: What symptom is seen first with Reworded Question: What is priority for the client
a perforated abdominal ulcer? who is experiencing mania?
Strategy: Discrimination is required to answer the Strategy: “MOST important” indicates priority.
question. Needed Info: Bipolar disorder is a chronic mood
Needed Info: Perforation of ulcer: medical emergency. syndrome
Gastroduodenal contents empty into peritoneal that causes mania, hypomania, and depression;
cavity resulting in peritonitis, paralytic ileus, during mania, client is hyperactive, anxious,
septicemia, and shock. S/S: sudden, sharp pain; and unable to meet physical needs; also see flight of
abdomen becomes tender, rigid. Treatment: fluids, ideas, inappropriate dress, and a lack of inhibitions.
electrolytes, antibiotics, NG suction, vagotomy, Category: Planning/Psychosocial Integrity
hemigastrectomy. S/S of duodenal ulcer: 25–30 years (1) Offer the client fluids every hour—appropriate
old, male-female 4:1, blood type O, pain 2–3 hrs action; at risk for cardiac collapse due to dehydration;
after meal and hs, food intake relieves pain. first give medication to decrease hyperactivity
Treatment: (2) Inform the client about the unit rules—
small frequent feedings; avoid coffee, alcohol, inappropriate;
seasonings; antacids (Maalox) 1 hr before or after administer medication, reduce environmental
meals; anticholinergics (Probanthine), take 30 stimuli
minutes (3) Administer haloperidol IM—CORRECT:
before meals; histamine receptor site antagonists decrease hyperactive behavior so client can take
(Tagamet), take with meals. fluids and food
Category: Assessment/Physiological (4) Encourage the client to rest—important; first
Integrity/Physiological decrease hyperactive behavior
Adaptation 164. The Answer is 3
(1) Fever—later with peritonitis (S/S: pain, nausea, The nurse is caring for an Rh-negative mother who
vomiting, rigid abdomen, low-grade fever, absent has delivered an Rh-positive child. The mother
bowel sounds, shallow respirations) states, “The doctor told me about RhoGAM, but
(2) Pain—CORRECT: sudden, sharp, begins I’m still a little confused.” Which of the following
midepigastric; responses by the nurse is MOST appropriate?
boardlike abdomen Reworded Question: What is RhoGAM and why is
(3) Dizziness—later with shock (S/S: hypotension, it used?
tachycardia, tachypnea, decreased urinary output, Strategy: Remember what you know about Rho-
decreased LOC) GAM.
(4) Vomiting—seen with peritonitis Needed Info: RhoGAM: given to unsensitized
162. The Answer is 3 Rhnegative
A 3-week-old boy is admitted with a diagnosis of (Rh–) mother after delivery or abortion
pyloric stenosis. The mother tells the nurse that this of an Rh-positive (Rh+) infant or fetus to prevent
is her first child and asks if there is anything she can development of sensitization. Rh– mother produces
do to prevent this from happening to her next child. antibodies in response to the Rh+ RBCs of fetus. If
TChoen tPernatc tRiecvei eTwes atnd Practice occurs during pregnancy, fetus is affected. If occurs
for the NCLEX-RN ® Exam during delivery, later pregnancies may be affected.
392 An indirect Coombs test is performed on the mother
The Practice Test during pregnancy, and a direct Coombs test is done
Which of the following statements by the nurse on cord blood after delivery. If both are negative and
BEST addresses her concern? the neonate is Rh+, the mother is given RhoGAM to
Reworded Question: What should you say to the prevent sensitization. RhoGAM is usually given to
mother about the possibility of this happening in unsensitized mothers within 72 hrs of delivery, but
the future? may be effective when given 3–4 weeks after delivery.
Strategy: Remember your therapeutic communication To be effective, RhoGAM must be given after
techniques. the first delivery and repeated after each subsequent
Category: Implementation/Psychosocial Integrity delivery. RhoGAM is ineffective against Rh+
(1) “This type of thing generally happens to first antibodies
children”—inaccurate that are already present in the maternal circulation.
(2) “When you have your second child, at least you’ll The administration of RhoGAM at 26–28
know what signs to look for”—invalidates concerns weeks’ gestation is also recommended.
(3) “This is a structural problem; it is not a reflection Category: Implementation/Health Promotion and
of your parenting skills”—CORRECT: provides Maintenance
acknowledgment; contains facts (1) “RhoGAM is given to your child to prevent the
(4) “This is an inherited condition; it is not your development of antibodies.”—not given to neonate
fault”—does not acknowledge feelings (2) “RhoGAM is given to your child to supply the
necessary antibodies.”—not given to neonate A client returns to his room after a cardiac
Practice Test Explanations catheterization.
393 Which of the following assessments by the
Practice Test Answers Taensdt (3) “RhoGAM is nurse would justify calling the physician?
given to you to prevent the formation Reworded Question: What is the most serious
of antibodies.”—CORRECT: prevents complication
maternal circulation from developing antibodies that can occur after a cardiac catheterization?
(4) “RhoGAM is given to you to encourage the How would you know it occurred?
production Strategy: Think about each answer. Recognize and
of antibodies.”—not accurate; given to eliminate expected outcomes.
discourage antibody production Needed Info: Cardiac catheterization prep: may feel
165. The Answer is 3 palpitations as catheter is passed and feelings of heat
The nurse performs client teaching for a woman and desire to cough as dye is injected (check allergies
with osteoarthritis. The client asks what she can do to iodine and shellfish). Obtain consent. No solid
to effectively decrease pain and stiffness in her joints food for 6–8 hrs or liquids 4 hrs before test. Mark
before beginning her daily routine. The nurse should peripheral pulses. Post-test: check vital signs every
instruct the client to do which of the following? 30 min for 2 hrs. Keep extremity of insertion site
Reworded Question: What should the client with straight 4–6 hrs. If femoral artery used, bed rest 6–
osteoarthritis do first thing in the morning? 12
Strategy: Which answer would reduce an hrs with bed flat. Check pressure dressing for
osteoarthritic drainage.
client’s pain? Check pulses, color, warmth, sensation every
Category: Implementation/Physiological Integrity/ 30 min. Monitor cardiac rhythm. Encourage fluids.
Basic Care and Comfort Category: Evaluation/Physiological Integrity/
(1) “Perform isometric exercises for 10 minutes.”— Reduction of Risk Potential
done to preserve muscle strength; tighten muscle, (1) Pain at the site of catheter insertion—expected;
hold for few seconds, then relax without moving pain med given
joint (2) Absence of a pulse distal to the catheter insertion
(2) “Do range-of-motion exercises then apply site—CORRECT: decrease in blood supply;
ointment report change in sensation, color, pulses to physician
to your joints.”—done after ointment immediately
applied; ROM does not reduce pain TChoen tPernatc tRiecvei eTwes atnd Practice
(3) “Take a warm bath and rest for a few minutes.”— for the NCLEX-RN ® Exam
CORRECT: heat reduces pain, spasms, stiffness 394
in joints The Practice Test
(4) “Stretch all muscle groups.”—would be painful (3) Drainage on the dressing covering the catheter
166. The Answer is 2 insertion site—some expected; pressure dressing
The nurse cares for a client receiving paroxetine. It applied; may have sandbag applied 4–6 hrs
is MOST important for the nurse to report which of (4) Redness at the catheter insertion site—some
the following to the physician? expected
Reworded Question: What is a potential drug 168. The Answer is 1
interaction? An 8-year-old boy is seen in a clinic for treatment
Strategy: “MOST important” indicates priority. of attention-deficit/hyperactivity disorder (ADHD).
Needed Info: Paroxetine (Paxil) is a selective serotinin Medication has been prescribed for the child along
reuptake inhibitor (SSRI) used to treat depression, with family counseling. The nurse teaches the parents
panic disorder, obsessive-compulsive disorder; about the medication and discusses parenting
side effects include palpitations, bradycardia, nausea strategies. Which of the following statements by the
and vomiting, and decreased appetite. parents indicates that further teaching is necessary?
Category: Evaluation/Physiological Reworded Question: What information is wrong for
Integrity/Pharmacological child with ADHD?
and Parenteral Therapies Strategy: Be careful! You are looking for incorrect
(1) The client states there is no change in her info.
appetite— Needed Info: ADHD: developmentally inappropriate
causes anorexia; monitor weight and nutritional inattention, impulsivity, hyperactivity. Treatment:
intake; report continued weight loss medication (methylphenidate hydrochloride),
(2) The client states she has started taking digoxin— family counseling, remedial education, environmental
CORRECT: may decrease effectiveness of manipulation (decrease external stimuli),
digoxin psychotherapy.
(3) The client states she applies sunscreen before Category: Evaluation/Psychosocial Integrity
going outside—appropriate action; prevents (1) “We will give the medication at night so it doesn’t
photosensitivity reactions decrease his appetite.”—CORRECT: incorrect
(4) The client states she drives her car to work— info; stimulants (methylphenidate hydrochloride)
driving is acceptable after determining client’s used; side effects: insomnia, palpitations, growth
response to drugs suppression, nervousness, decreased appetite;
167. The Answer is 2 give 6 hrs before bedtime
(2) “We will provide a regular routine for sleeping,
eating, working, and playing.”—true hypertrophy (BPH). Which of the following
(3) “We will establish firm but reasonable limits on would cause the nurse to suspect postoperative
his behavior.”—true hemorrhage?
(4) “We will reduce distractions and external stimuli Reworded Question: What are the signs of post-op
to help him concentrate.”—true hemorrhage?
169. The Answer is 3 Strategy: The entire answer choice must be correct
A client has been taking aluminum hydroxide daily for the answer to be correct. Read each one carefully.
for 3 weeks. The nurse should be alert for which of Needed Info: Symptoms of hemorrhage: restlessness,
the following side effects? dizziness, pallor, cool and clammy skin, dyspnea,
Reworded Question: What is a side effect of rapid thready pulse, fall in BP, decrease in level of
Amphojel? consciousness. Treatment: elevate legs 45 degrees,
Strategy: Think about each answer. knees straight, trunk flat, head slightly elevated,
Needed Info: Aluminum hydroxide: antacid that IV fluids (Ringer’s lactate, normal saline, D5W,
reduces the total amount of acid in the GI tract and dextran), packed cells, vasoactive meds (Levophed,
elevates the gastric pH level. May cause Nipride).
hypophosphatemia. Category: Assessment/Physiological
Shake suspension well and give with milk Integrity/Physiological
or water. Adaptation
Category: Assessment/Physiological (1) Decreased blood pressure, increased pulse,
Integrity/Pharmacological increased respirations—CORRECT: caused by
and Parenteral Therapies decreased blood volume, as intravascular volume
(1) Nausea—not common decreases and BP falls, heart rate increases
(2) Hypercalcemia—seen with calcium-containing in attempt to maintain cardiac output, respirations
antacids (e.g., Tums); normal Ca 8.5–10.5 mg/dL increase in attempt to increase oxygenation
(3) Constipation—CORRECT: may need laxatives (2) Fluctuating blood pressure, decreased pulse,
or stool softeners rapid respirations—pulse rate will increase, not
(4) Anorexia—not common decrease
170. The Answer is 3 (3) Increased blood pressure, bounding pulse,
A client recovering from a laparoscopic laser irregular
cholecystectomy respirations—BP drops, pulse increases to
says to the nurse, “I hate the thought of compensate for decreased cardiac output
eating a low-fat diet for the rest of my life.” Which (4) Increased blood pressure, irregular pulse, shallow
of the following responses by the nurse is MOST respirations—BP drops, heart rate increases
appropriate? to maintain cardiac output
Reworded Question: Is a low-fat diet required 172. The Answer is 4
indefinitely? The home care nurse screens a group of residents in
Strategy: “MOST appropriate” indicates discrimination a
may be required to answer the question. dependent living facility for risk factors to pneumonia.
Needed Info: Laparoscopic laser cholecystectomy is The nurse determines that which of the following
removal of the gallbladder by laser through a clients is MOST at risk to develop pneumonia?
laparoscope; Reworded Question: Who is most likely to develop
monitor T-tube if present; observe for jaundice; pneumonia?
monitor intake and output; monitor for pain Strategy: Think about each answer.
and encourage early ambulation to rid the body of Needed Info: Pneumonia is an inflammatory process
carbon dioxide. that results in edema of lung tissues and extravasion
Category: Implementation/Physiological Integrity/ of fluid into alveoli, causing hypoxia; symptoms
Physiological Adaptation include fever, leukocytosis, productive cough,
(1) “I will ask the dietician to come talk to you.”— dyspnea,
passing the responsibility; nurse should respond and pleuritic pain.
to the client Category: Evaluation/Health Promotion and
(2) “What do you think is so bad about following a Maintenance
low-fat diet?”—does not respond directly to the (1) A 72-year-old female who has left-sided
client’s statement hemiparesis
(3) “It may not be necessary for you to follow a lowfat after a cerebrovascular accident—advanced
diet for that long.”—CORRECT: fat restriction age is a risk factor
is usually lifted as the client tolerates fat; (2) A 76-year-old male who has a history of
biliary ducts dilate sufficiently to accommodate hypertension
bile volume that was held by the gallbladder and type 2 diabetes—age is a risk factor
Practice Test Explanations (3) An 80-year-old female who walks 1 mile every
395 day and has a history of depression—age is a risk
Practice Test Answers Taensdt (4) “At least you factor
will be alive and not suffering that (4) An 87-year-old male who smokes and has a
pain.”—nontherapeutic and judgmental history
171. The Answer is 1 of lung cancer—CORRECT: advanced age,
A client returns to his room after a transurethral smoking, underlying lung disease, malnutrition,
resection of the prostate (TURP) for benign prostatic and bedridden status are risk factors for development
of pneumonia factors: male over age 50, age 15–19, poor social
173. The Answer is 1 attachments, client with previous attempts, client
The nurse performs teaching with a client undergoing with auditory hallucinations, overwhelming
a paracentesis for treatment of cirrhosis. The client precipitating
asks what position he will be in for the procedure. events (terminal disease, death or loss of loved
The nurse’s reply should be based on an one, failure at school, job).
understanding Category: Assessment/Psychosocial Integrity
that the MOST appropriate position for the client (1) “What has happened to cause you to want to end
is which of the following? your life?”—does not determine immediate need
Reworded Question: What is the correct position for for safety
a paracentesis? (2) “How have you planned to kill yourself?”—
Strategy: Visualize the procedure. CORRECT: lets you prioritize interventions to
Needed Info: Paracentesis: removal of fluid from assure safety
abdominal or peritoneal cavity. Can be used for (3) “When did you start to feel as though you wanted
diagnostic purposes, to remove ascitic fluid, to to die?”—does not determine immediate need
prepare for safety
for peritoneal dialysis. Preparation: have client (4) “Do you want me to prevent you from killing
void, take vital signs, weigh client, measure yourself?”—yes/no question, closed
abdominal 175. The Answer is 4
girth. During procedure: check vital signs every A man is admitted for treatment of heart failure. The
15 min. Measure and document amount of drainage physician orders an IV of 125 mL of normal saline
(2–3 L can be removed), characteristics. After per hour and central venous pressure (CVP) readings
procedure: every 4 hours. Sixteen hours after admission,
apply pressure dressing, check for leakage. Bed the client’s CVP reading is 3 cm/H2O. Which of the
rest until vital signs stable. Complications: following evaluations of the client’s fluid status by
hypovolemia the nurse would be MOST accurate?
and shock. Cirrhosis: degenerative liver disease; Reworded Question: What does this CVP reading
TChoen tPernatc tRiecvei eTwes atnd Practice indicate?
for the NCLEX-RN ® Exam Strategy: Consider each answer and remember
396 normal
The Practice Test CVP reading values.
tissue is replaced by scar tissue. Causes: alcoholism, Needed Info: CVP: central venous line placed in
hepatic inflammation or necrosis, chronic bilary superior vena cava. To obtain a reading: client
obstruction. S/S: ascites, lower-leg edema, jaundice, placed supine, 0 on manometer placed at level of
esophageal varices, hemorrhoids, bleeding right atrium (midaxillary line at 4th intercostal
tendencies, pruritis, dark urine, clay-colored stools. space), turn stopcock to allow manometer to fill with
Nursing responsibilities: high-protein, high- fluid, turn to allow fluid to go into client. Fluid will
carbohydrate, fluctuate with respirations. When stabilized, take
low-Na+ diet, good skin care, promote rest, reading at highest level of fluctuation. Normal: 4–10
reduce exposure to infection. cm/H2O. Elevated: hypervolemia, CHF, pericarditis.
Category: Analysis/Physiological Integrity /Reduction Low: hypovolemia.
of Risk Potential Category: Evaluation/Physiological Integrity/
(1) Sitting with his lower extremities well supported— Reduction of Risk Potential
CORRECT: Fowler’s position or sitting (1) The client has received enough fluid—inaccurate
on side of bed with feet on stool; easy access (2) The client’s fluid status remains unaltered—
to abdominal area; allows intestines to float to nothing to compare to
prevent laceration (3) The client has received too much fluid—inaccurate
(2) Side-lying with a pillow between his knees—not (4) The client needs more fluid—CORRECT: normal
accurate 4–10 cm/H2O; indicates hypovolemia
(3) Prone with his head turned to the left side—not Practice Test Explanations
accurate 397
(4) Dorsal-recumbent with a pillow at the back of Practice Test Answers Taensdt 176. The Answer
his head—not accurate is 4
174. The Answer is 2 An agitated client throws a chair across the dayroom
A man calls the Suicide Prevention Hotline and on the psychiatry floor and threatens the other clients
states that he is going to kill himself. Which of the with physical harm. Which of the following
following questions should the nurse ask FIRST? should the nurse do FIRST?
Reworded Question: What is most important to know Reworded Question: What is the nurse’s first action?
about a client who has threatened to kill himself? Strategy: Use Maslow. Safety first—the client must
Strategy: “FIRST” indicates priority. be removed from the situation.
Needed Info: Signs of suicide: symptoms of Needed Info: The nurse can initiate seclusion
depression, procedures
client gives away possessions, gets finances in in an escalating situation per hospital policy.
order, has a means, makes direct or indirect The principle of seclusion is containment, to avoid
statements, injury, and to prevent anticipated violence. Violence
leaves notes, has increased energy. Predisposing
must be prevented to ensure the safety of other Manic clients often try to take a leadership position
clients in an environment, and try to engage others.
and staff. Category: Assessment/Psychosocial Integrity
Category: Implementation/Psychosocial Integrity (1) The client tells several jokes at a group meeting—
(1) Tell the client that his wife will be called to the reflects an elated mood and no real participation
hospital—calling wife will not solve the immediate in the meeting; manic clients may tease, talk, and
problem and may complicate the situation joke excessively
(2) Ask the client why he is so angry—asking client (2) The client sits and talks with other clients at
for causes of anger is inappropriate when the client’s mealtimes—CORRECT: manic clients have difficulty
behavior is escalating socializing because of flight of ideas and
(3) Remove the other clients from the dayroom— TChoen tPernatc tRiecvei eTwes atnd Practice
allowing client to determine disposition of other for the NCLEX-RN ® Exam
clients gives client control over staff and other 398
clients The Practice Test
(4) Assemble staff and put the client in preventive intrusiveness; usually cannot sit to eat and will
seclusion—CORRECT: seclusion may be used carry fluids and food around
alone or in conjunction with medication to deescalate (3) The client begins to write a book about his life—
a potentially dangerous situation; nurse manic clients often write voluminously; may help
can initiate and terminate client seclusion based to express feelings, but does not reflect improvement,
on established protocols especially if thoughts are grandiose
177. The Answer is 4 (4) The client initiates an effort to start a radio station
The nurse is caring for a depressed client who spends on the unit—manic clients often try to take
most of the day sitting at a window, and is about a leadership position in an environment and try
to implement a physical activity plan for him. The to recruit others
nurse knows that the purpose of this plan is to do 179. The Answer is 1
which of the following? A client hospitalized for treatment of delusions tells
Reworded Question: What is the purpose of the the nurse that he is really the head of the hospital
different system and that his “cover” is being a client to get
plans typically implemented for clients with information on client abuse. Which of the following
mental illness? statements by the nurse is BEST initially?
Strategy: Consider the rationales for plans typically Reworded Question: How would you handle a client
used for clients with mental illness. with delusions?
Needed Info: Physical and mental health are linked. Strategy: Know when further assessment is needed
A level of fitness enhances a sense of mental and what the appropriate communication techniques
wellbeing. are to use with delusional clients.
Withdrawn clients have decreased motivation Needed Info: The initial approach to delusions is to
to exercise. Physical exercise can also distract clients clarify meanings. After clarification, the delusions
from stressful thoughts, and helps clients focus on should not be discussed as this could reinforce them.
things other than themselves. Arguing with a client about delusions may also
Category: Planning/Psychosocial Integrity reinforce
(1) Help the client understand the problems creating them. Delusions that entail injury or death
the depression—purpose of physical activity is should be addressed immediately, and client
not to provide insight into one’s problems protections
(2) Reduce the client’s risk for obesity and diabetes— put into place.
this would not necessarily reduce the risk Category: Assessment/Psychosocial Integrity
for obesity and diabetes (1) “Tell me what you mean about being head of the
(3) Transform self-destructive impulses into positive hospital system and getting client abuse
behaviors—physical activity may channel information.”—
energy differently, but does not guarantee to CORRECT: initial approach is to
change self-destructiveness further assess by clarifying the meaning of the
(4) Encourage socialization and improve selfesteem— delusion to the client
CORRECT: purpose of physical activity (2) “I think you should share this story with the
is to promote focused socialization with other clients at dinnertime and see what they
clients and staff and to increase a sense of selfesteem say.”—could cause disruption among other clients
178. The Answer is 2 and embarrass the client
The nurse is caring for a client with bipolar disorder. (3) “You are not the head of the hospital system, you
Which of the following behaviors by the client are an accountant under treatment for a mental
indicates disorder.”—arguing with client about delusion
to the nurse that a manic episode is subsiding? is ineffective and inappropriate and may
Reworded Question: What indicates normalizing strengthen the client’s belief in it
behavior? (4) “It worries me when you say these things; it
Strategy: Think about the behaviors that indicate means you are not responding to the medication.”—
mania. nurse is communicating disappointment
Needed Info: Manic clients may tease, talk, and joke to the client and treating the delusion as though
excessively. They usually cannot sit to eat and may it were a behavior under the client’s control
need to carry fluids and food around in order to eat. 180. The Answer is 1
The nurse is caring for a client in labor. The nurse (4) Platelet count and clotting time—these do not
palpates a firm, round form in the uterine fundus, usually change
small parts on the woman’s right side, and a long, 182. The Answer is 2
smooth, curved section on the left side. Based on The nurse is preparing a client newly diagnosed with
these findings, the nurse should anticipate Addison’s disease for discharge. Which of the following
auscultating statements by the client indicates a need for
the fetal heart in which of the following locations? further instruction from the nurse?
Reworded Question: If a fetus is LOA, where should Reworded Question: Which instruction about
the nurse listen for the fetal heart tone? Addison’s
Strategy: Examine the diagram carefully. Know the disease does the client not understand?
woman’s right from left. Strategy: Use the process of elimination, noting the
Needed Info: Fetal reference point: vertex key words “need for further instruction.” Be careful:
presentation— you are looking for an answer choice that contains
dependent upon degree of flexion of fetal incorrect information. If you had trouble with this
head on chest; full flexion/occiput (O), full extension question, review client teaching for Addison’s disease.
chin (M), moderate extension (military) brow (B). Needed Info: Addison’s disease is the most common
Breech presentation—sacrum (S). Shoulder form of adrenal hypofunction. It occurs when more
presentation— than 90 percent of the adrenal gland is destroyed.
scapula (SC). Maternal pelvis is designated Early diagnosis and adequate hydrocortisone
per her right/left and anterior/posterior. Position replacement therapy indicate a good prognosis.
= relationship of fetal reference point to mother’s Acute adrenal insufficiency, or adrenal crisis, is a
pelvis; expressed as standard 3-letter abbreviation: medical emergency requiring immediate treatment.
LOA (left occiput anterior) (most common), LOP Corticosteroid replacement is the primary lifelong
(left occiput posterior), ROA (right occiput anterior), treatment for clients with primary or secondary
ROP (right occiput posterior), LOT (left occiput adrenal hypofunction. An adrenal crisis usually
transverse), ROT (right occiput transverse). subsides
Category: Planning/Health Promotion and quickly with proper treatment, and subsequent
Maintenance oral maintenance doses of hydrocortisone preserve
(1) A—CORRECT: point of maximum intensity for stability.
fetal heart with fetus in LOA position Category: Analysis/Physiological Integrity/Reduction
Practice Test Explanations of Risk Potential
399 (1) “I understand that I will need lifelong cortisone
Practice Test Answers Taensdt (2) B—PMI location replacement therapy.”—indicates the client
for fetus in LOP position understands the discharge teaching; clients
(3) C—PMI location for fetus in ROA position with Addison’s disease require lifelong cortisone
(4) D—PMI location for fetus in ROP position replacement therapy
181. The Answer is 1 (2) “During times of stress, I will need to decrease
A 69-year-old female client admitted with pneumonia my medication.”—CORRECT: indicates the
is receiving gentamicin. For this client, which client does not understand discharge teaching
of the following laboratory values would be MOST and requires further instructions; during times
important for the nurse to monitor? of stress, clients with Addison’s disease need to
Reworded Question: What are the adverse effects of increase the medication dosage, not decrease it
Garamycin? (3) “I must be careful not to injure myself.”—indicates
Strategy: Use the process of elimination, noting the the client understands the discharge teaching;
key words “MOST important,” which indicates that clients with Addison’s disease should be
more than one answer choice may be correct. warned that infection, injury, or profuse sweating
Needed Info: Gentamicin (Garamycin) is a in hot weather may precipitate a crisis
broadspectrum (4) “I should always carry a medical identification
antibiotic used to treat bacterial infections, card.”—indicates the client understands the discharge
particularly those caused by gram-negative bacteria. teaching; clients with Addison’s disease
Side effects: neuromuscular blockage, ototoxic to the should always carry a medical identification
eighth cranial nerve (tinnitus, vertigo, hearing loss), card and wear a bracelet stating the name and
nephrotoxicity; less commonly, may cause anemia dosage of the steroid the client takes
and hypokalemia. Blood should be drawn for peak TChoen tPernatc tRiecvei eTwes atnd Practice
levels 1 hour after IM and 30 minutes; 1 hour after for the NCLEX-RN ® Exam
IV infusion and for trough just before next dose. 400
Category: Implementation/ Physiological Integrity/ The Practice Test
Reduction of Risk Potential 183. The Answer is 2
(1) BUN and creatinine—CORRECT: Gentamicin The nurse suspects a client has meningitis. The nurse
is nephrotoxic; proteinuria, oliguria, hematuria, places the client in a dorsal recumbent position, puts
thirst, increased BUN, decreased creatinine her hands behind the client’s neck, and bends it
clearance forward.
(2) Hemoglobin and hematocrit—Gentamicin can The nurse knows that pain and resistance may
cause anemia but is less common indicate neck injury or arthritis, but if the client also
(3) Sodium and potassium—hypokalemia is an flexes the hips and knees, this positive response is
infrequent problem which of the following?
Reworded Question: A positive response to which finding
sign helps establish a diagnosis of meningitis? (3) Severe anemia—this is a late-stage finding
Strategy: Knowledge regarding each of these tests is (4) Hematuria—CORRECT: this is an initial-stage
needed to answer this question. Review these tests to finding
determine which positive response is a sign of (5) Azotemia—this is a late-stage finding
meningitis (6) Nausea—this is a late-stage finding
if you had difficulty with this question. 185. The Answer is 4
Needed Info: Meningitis is an infection that causes A 56-year-old male client with a history of myocardial
inflammation of the brain and spinal meninges that infarction is admitted for evaluation of chest
can involve the meningeal membranes. When a client pain. Several hours later, the client goes into
is placed in a dorsal recumbent position and the ventricular
nurse puts her hands behind the client’s neck and fibrillation and a code blue is called. The
bends it forward, pain and resistance may indicate Emergency Department physician defibrillates the
neck injury or arthritis. However, if the client also client. The nurse knows that the purpose of
flexes the hips and knees, chances are that he has defibrillation
meningeal irritation and inflammation, which is a is to do which of the following?
sign of meningitis. Reworded Question: Why is a client defibrillated?
Category: Assessment/Physiological Integrity/ Practice Test Explanations
Reduction of Risk Potential 401
(1) Trousseau’s sign—this is for clients who have Practice Test Answers Taensdt Strategy: Think
suspected about each answer choice. What
hypocalcemia. The nurse places a blood does defibrillation do?
pressure cuff on the client’s arm and inflates it Needed Info: The goal of defibrillation is to
above the client’s systolic pressure. If it is positive, temporarily
the client will exhibit carpal spasm (ventral depolarize the irregularly beating heart
contraction of the thumb and digits) within 3 and allow more coordinated contractile activity to
minutes resume. Defibrillation depolarizes the myocardium
(2) Brudzinki’s sign—CORRECT: a positive and produces temporary asystole to provide the
response to this test is a sign of meningitis opportunity for the natural pacemaker of the heart
(3) Homans’ sign—discomfort behind the knee on to resume normal activity.
forced dorsiflexion of the foot, due to thrombosis Category: Implementation/Physiological Integrity/
in calf veins Physiological Adaptation
(4) Chvostek’s sign—spasm of the facial muscles (1) Energize myocardial cells—this is inaccurate
elicited by tapping the facial nerve in the region (2) Improve left ventricular function—this is
of the parotid gland; seen in tetany and is a sign inaccurate
of hypocalcemia (3) Increase cardiac output—this is inaccurate
184. The Answer is 2 and 4 (4) Produce momentary asystole to allow the natural
The nurse is assessing a client newly diagnosed with pacemaker to resume activity—CORRECT:
initial-stage chronic glomerulonephritis. Which of defibrillation produces momentary asystole
the following findings should the nurse expect to see? 186. The Answer is 4
Select all that apply. The physician orders 0.25 mg digoxin (Lanoxin) for a
Reworded Question: What are the signs/symptoms of client diagnosed with heart failure. The client’s pulse
initial-stage chronic glomerulonephritis? is 86 prior to administration of the prescribed dose.
Strategy: Focus on the key words “Select all that The nurse should do which of the following?
apply.” This indicates there is more than one correct Reworded Question: What are the side effects of
answer. Think about the signs and symptoms of Lanoxin?
initial-stage chronic glomerulonephritis as opposed Strategy: Think about each answer choice and the
to late-stage findings. action of digoxin.
Needed Info: Chronic glomerulonephritis is a slowly Needed Info: Digoxin (Lanoxin) is an antiarrhythmic
progressive, noninfectious disease characterized by prescribed for atrial fibrillation and heart failure.
inflammation of the renal glomeruli. By the time it The apical pulse should be monitored for 1 full minute
produces symptoms, it is usually irreversible and before administering, and the dose withheld and
eventually results in renal failure. Symptoms of the the physician notified if the pulse rate is less than 60
initial stage are nephrotic syndrome, hypertension, per minute in an adult.
proteinuria, and hematuria. Late-stage findings Category: Analysis/Physiological
include azotemia, nausea, vomiting, pruritus, Integrity/Pharmacological
dyspnea, and Parenteral Therapies
malaise, fatigability, mild to severe anemia, (1) Give half of the prescribed dose (0.125)—you
and severe hypertension. would need a physician order to change the dose
Category: Assessment/Physiological if warranted (only a physician can make that
Integrity/Physiological determination)
Adaptation (2) Delay the dose until the pulse is below 60—
(1) Hypotension—initial-stage findings include inaccurate;
hypertension; late-stage symptoms include the dose would be held with a pulse below
severe hypertension 60
(2) Proteinuria—CORRECT: this is an initial-stage (3) Omit the dose, and record the pulse rate as the
reason—inaccurate adverse effects of sumatriptan include sudden
(4) Give the full dose as ordered—CORRECT: the numbness or weakness, especially on one side of
dose should be given as prescribed the body
187. The Answer is 1 189. The Answer is 3
The nurse knows that the atorvastatin administered The client is resuming a diet after undergoing a
to a client is effective when there is a reduction in Billroth
which of the following? II procedure. To minimize complications from
Reworded Question: What are the indications for use eating, the nurse instructs the client to do which of
of atorvastatin? the following?
Strategy: Think about each answer choice and what Reworded Question: How does a client avoid dumping
atorvastatin (Lipitor) is prescribed for. syndrome, a complication of this surgical procedure?
Needed Info: Atorvastatin belongs to a group of Strategy: Focus on the surgical procedure and recall
drugs called “statins.” Therapeutic effects of that dumping syndrome is a complication of this
atorvastatin surgery.
include decreased LDLs, triglycerides, and If you had difficulty with this question, review
cholesterol, when used along with a proper diet. the prevention and management of complications of
Category: Evaluation/Physiological gastric surgery.
Integrity/Pharmacological Needed Info: The client who has had a Billroth II
and Parenteral Therapies procedure is at risk for dumping syndrome. The client
(1) Triglycerides—CORRECT: Atorvastatin lowers should eat small, frequent meals evenly spaced
LDLs, cholesterol, and triglycerides throughout the day; chew food thoroughly and drink
(2) Chest pain—Atorvastatin is not prescribed to fluids between meals rather than with them; decrease
reduce chest pain intake of carbohydrates and salt while increasing fat
(3) Blood pressure—Atorvastatin is not prescribed and protein. After meals, the client should lie down
for high blood pressure for 20–30 minutes.
(4) PTT—Atorvastatin is not prescribed to reduce Category: Implementation/Physiological Integrity/
PTT; a PTT is ordered when a client has unexplained Basic Care and Comfort
bleeding or clotting, and also ordered (1) Drink fluids with meals—client should drink
at intervals to monitor unfractionated heparin fluids between meals, not with meals
anticoagulant therapy (2) Increase intake of carbohydrates and salt—client
188. The Answer is 4 should decrease intake of carbohydrates and
A 37-year-old female has been prescribed salt
sumatriptan (3) Increase fat and protein—CORRECT: client
for severe migraines. The nurse explains that the should increase fat and protein
client (4) Eat 3 large meals a day—client should eat small,
should watch for which of the following adverse frequent meals evenly spaced throughout the day
drug effects? 190. The Answer is 4
Reworded Question: What are the adverse effects of The nurse is caring for a client who is having difficulty
Imitrex? eating due to mouth sores from chemotherapy
Strategy: Use the process of elimination. treatments. Which of the following interventions is
Needed Info: Sumatriptan (Imitrex) works by MOST appropriate to promote basic comfort and
narrowing nutrition?
the blood vessels around the brain. It can cause Reworded Question: What are the side effects of
serious side effects on the heart, including heart chemotherapy?
attack Strategy: Focus on the issue: promoting basic care
or stroke. Adverse side effects include chest pain; pain and nutrition for a client having pain due to mouth
TChoen tPernatc tRiecvei eTwes atnd Practice sores. If you had difficulty with this question, review
for the NCLEX-RN ® Exam the side effects of chemotherapy and appropriate
402 nursing interventions.
The Practice Test Needed Info: Mouth sores are a side effect of
spreading to arm or shoulder; sudden numbness or chemotherapy
weakness; confusion; problems with vision, speech, and can cause discomfort when eating and
or balance; nausea and bloody diarrhea; convulsions; swallowing. Mouth sores occur because chemotherapy
numbness or tingling in fingers or toes; and sudden destroys cells in the mouth and esophagus, as
numbness or weakness, especially on one side of the well as cancer cells.
body. Category: Implementation/Physiological Integrity/
Category: Planning/Physiological Basic Care and Comfort
Integrity/Pharmacological (1) Obtain an order for TPN—only used when oral
and Parenteral Therapies intake is not possible
(1) Constipation—adverse effects of sumatriptan (2) Keep the client NPO—would not promote nutrition
include bloody diarrhea, not constipation but increase nutritional risk
(2) Bradycardia—adverse effects of sumatriptan (3) Administer a stool softener as ordered—is used
include tachycardia, not bradycardia when constipation is present
(3) Somnolence—adverse effects of sumatriptan (4) Provide frequent oral hygiene—CORRECT:
include agitation, not somnolence aids in providing temporary relief of pain from
(4) Sudden numbness or weakness—CORRECT: mouth sores so that a client can eat
191. The Answer is 2 the nurse
The nurse is caring for an adult male client who has (2) Tell the client that obtaining the signature is
just undergone spinal fusion for a herniated routine for all surgeries—although this is true,
intervertebral it does not determine the client’s ability to give
disk. To promote comfort and minimize complications, informed consent
the nurse tells the client to avoid which of (3) Witness the client’s signature—this is true;
the following? however,
Practice Test Explanations the nurse should first assess the client’s
403 knowledge of the procedure
Practice Test Answers Taensdt Reworded (4) Assess whether the client’s understanding of the
Question: What should the client avoid procedure is sufficient to give consent—CORRECT:
after having undergone spinal fusion? informed consent means the client must
Strategy: Note the key word “avoid” in the question understand and comprehend the risks, benefits,
stem. Focus on the surgical procedure and recall that and alternatives of the procedure to be performed
activity and positioning should not cause discomfort 193. The Answer is 3
or unnecessary strain on the back. The nurse is preparing to administer heparin sodium
Needed Info: To avoid complications and promote to a client diagnosed with thrombophlebitis. The
comfort after spinal fusion, the client should be nurse should ensure that which of the following is
advised to bend at the knees when lifting (never at available if the client develops a significant bleeding
the waist); lie down when tired and sleep on his side problem?
with a pillow between his knees. Lying on the stomach Reworded Question: What is the antidote for heparin
causes strain on the back; use a firm mattress sodium?
to reduce tension on the spine. Choose a firm, Strategy: Focus on the name of the medication
hardback administered and remember that the antidote for
chair for sitting, no longer than 20 minutes at heparin sodium is protamine sulfate. Review antidotes
a time. for commonly administered medications.
Category: Implementation/Physiological Integrity/ Needed Info: Heparin sodium is indicated for
Basic Care and Comfort prophylaxis
(1) Bending the knees when lying on one side—will and treatment of venous thrombosis,
decrease the strain on the shoulders, neck, and pulmonary embolism, and atrial fibrillation with
arms embolization. Heparin prevents formation of clots.
(2) Sitting for longer than 20 minutes at a time— Its antidote is protamine sulfate.
CORRECT: puts strain on the back; is better to Category: Planning/Safe and Effective Care
walk around or lie down to rest Environment/
(3) Using an extra-firm mattress—reduces tension Management of Care
on the spine (1) Phytonadione (vitamin K)—is the antidote for
(4) Sitting in a hardback chair—provides support warfarin (Coumadin)
for the back TChoen tPernatc tRiecvei eTwes atnd Practice
192. The Answer is 4 for the NCLEX-RN ® Exam
The nurse is preparing a client for surgery. When 404
obtaining informed consent, the nurse should The Practice Test
INITIALLY (2) Fresh frozen plasma (FFP)—can also be used for
do which of the following? bleeding associated with warfarin therapy
Reworded Question: What are the elements of (3) Protamine sulfate—CORRECT: is the antidote
informed consent and what are the nurse’s for heparin sodium
responsibilities? (4) Reteplase—a thrombolytic that breaks up blood
Strategy: Think about the elements of informed clots, does not prevent formation of clots, and is
consent and the responsibilities of the physician and not an antidote
nurse. “INITIALLY” indicates there may be more 194. The Answer is 2
than one correct response. A client is being admitted to the hospital for elective
Needed Info: Informed consent is more than simply surgery. During the admission assessment, the
getting a client to sign a written consent form. It is nurse asks the client if he has an advance directive.
a process of communication between a client and The nurse knows that clients have the right to play
physician that results in the client’s authorization an active role in their care and treatment, and this is
or agreement to undergo a specific medical guaranteed by which of the following?
intervention. Reworded Question: Which act guarantees clients
The client should have an opportunity to the right to make their own care and treatment
ask questions to elicit a better understanding of the decisions?
treatment/procedure and make an informed decision Strategy: Recall the provisions of each federal act.
to proceed or refuse. Use the process of elimination. Review the different
Category: Assessment/Safe and Effective Care federal acts if you had difficulty with this question.
Environment/ Needed Info: The Client Self-Determination Act
Management of Care requires that, upon admission to hospitals, longterm
(1) Explain the risks, benefits, and alternatives of care facilities, and home health agencies, clients
the procedure—this is the responsibility of the be informed that they have the right to accept
practitioner performing the procedure, not of
or refuse medical care, and the right to refuse Practice Test Answers Taensdt Strategy: Use the
treatment. process of elimination. Three of
Category: Analysis/Safe and Effective Care the answer choices need no intervention.
Environment/ Needed Info: A chest tube with water seal drainage
Management of Care allows air and fluid to be removed from the
(1) The Health Insurance Portability and intrapleural
Accountability space. The underwater seal drainage to
Act (HIPAA)—provides for the rights which the chest tube is connected prevents backflow
and protections for participants and beneficiaries into the pleural space. The chest tube is attached to
in group health plans; protects personally a valve mechanism designed to allow air or fluid to
identifying information, and information about drain out of, but not into, the chest cavity. There
diagnosis and treatment should be no dependent loops or kinks in the tube,
(2) The Client Self-Determination Act—CORRECT: and the tube should be unclamped, including during
guarantees clients the right to make their transport and ambulation, unless ordered by the
own care and treatment decisions physician. The fluid level in the water seal should be
(3) The Civil Rights Act—prohibits employment at 2 cm.
discrimination on the basis of race, color, religion, Category: Assessment/Safe and Effective Care
sex, or national origin Environment/
(4) The Americans with Disabilities Act—recognizes Management of Care
and protects the civil rights of people with (1) There are no dependent loops in the chest tube—
disabilities no need to intervene because this is correct
195. The Answer is 3 (2) The chest tube is not clamped—no need to
The nurse enters a client’s hospital room to find intervene;
the client sitting on the bathroom floor. The nurse chest tube should be unclamped
assesses the client, obtains assistance, and assists the (3) The chest tube and drainage system is above
client back to bed. The nurse notifies the physician the client’s chest—CORRECT: chest tube and
and completes an incident report. Which of the drainage system should be below the client’s chest
following (4) The fluid level in the water seal is at 2 cm—no
is the MOST appropriate nursing action? need to intervene; this is the correct fluid level
Reworded Question: What is the nurse’s responsibility 197. The Answer is 3
related to incident reporting? The nurse is present during an informed consent
Strategy: Use the process of elimination. discussion
Needed Info: Incident reports are confidential and between the client and the physician regarding
privileged information. They should not be placed recommended surgery. The physician discusses
in a client’s chart, copied, or referenced in the chart. the risks, benefits, and alternatives of the procedure
The person who witnesses the incident should write with the client. The nurse knows that the client’s
the report. An objective entry of the incident should decision whether or not to have the surgery is based
be documented in the client’s chart. on which of the following ethical principles?
Category: Implementation/Safe and Effective Care Reworded Question: Which principle defines the
Environment/Management of Care right of individuals to make their own decisions?
(1) Document in the client’s chart that an incident Strategy: Use the process of elimination. If you had
report has been completed—reference to incident difficulty with this question, review ethical principles
reports should not be documented in a client’s of clinical practice.
chart Needed Info: Ethics help the nurse determine whether
(2) Make a copy of the incident report for the nurse an action is right or wrong and guide professional
manager—incident reports should never be copied practice. Nonmaleficence is the nurse’s duty to do no
(3) Document the incident in the client’s chart— harm. Beneficence is a nurse’s duty to do what is in
CORRECT: objective details of the incident the best interest of the client. Autonomy is the right
should be documented in the client’s chart of individuals to make their own decisions regarding
(4) Place the incident report in the client’s chart— care and treatment. Capacity is a client’s ability to
incident reports should not be placed in a client’s make medical decisions.
chart but routed to the hospital’s risk management Category: Analysis/Safe and Effective Care
or legal department according to hospital Environment/
policy Management of Care
196. The Answer is 3 (1) Nonmaleficence—is the nurse’s duty to do no
The nurse is performing an initial postoperative harm
assessment on a client who has just returned from (2) Beneficence—is the nurse’s duty to do what is in
surgery with a chest tube and water seal drainage the best interest of the client
system. The nurse should immediately intervene if (3) Autonomy—CORRECT: is the right of individuals
she makes which of the following observations? to make their own medical decisions
Reworded Question: Which observation of chest tube (4) Capacity—is the client’s ability to make medical
and water seal drainage system requires immediate decisions
attention? 198. The Answer is 2
Practice Test Explanations The nurse is caring for a terminal cancer client at
405 home. The nurse knows that which of the following
ethical principles BEST supports keeping client and
family care consistent with the nurse’s professional Reworded Question: What do you do to prevent
code of ethics? bleeding after an arterial puncture?
Reworded Question: Keeping client and family care Strategy: Use the process of elimination and focus
consistent with the nurse’s professional code of ethics on the issue, which is preventing bleeding.
is the definition of which principle? Needed Info: After a blood sample is obtained for an
Strategy: Think about the definition of each principle. ABG, pressure should be applied to the puncture site
Review ethical principles if you had trouble for 5 minutes and then a gauze pad should be taped
with this question. in place. Monitor the site for bleeding and check
Needed Info: A nurse should be able to identify the arm for signs of complications (swelling, pain,
ethical issues that affect client care. If ethical issues numbness, tingling).
arise, the nurse must be able to handle them Category: Planning/Safe and Effective Care
according Environment/
to ethical principles. Safety and Infection Control
Category: Analysis/Safe and Effective Care (1) Apply 2 × 2 gauze to the puncture site and hold
Environment/ pressure for 5 minutes—CORRECT: pressure
Management of Care should be applied to the arterial puncture site
(1) Virtues—refers to compassion, trustworthiness, for 5 minutes to prevent bleeding
integrity, and veracity (2) Have the client hold the puncture site in a
(2) Fidelity—CORRECT: refers to keeping faithful dependent
to ethical principles and the American Nurses position for 5 minutes—promotes bleeding
Association Code of Ethics for Nurses (3) Apply a warm compress to the puncture site for
(3) Beneficence—refers to a nurse’s duty to do what 15 minutes—promotes bleeding
is in the best interest of the client (4) Encourage the client to open and close the hand
(4) Justice—refers to a fair, equitable, and appropriate rapidly for several minutes—promotes bleeding
treatment 201. The Answer is 3
TChoen tPernatc tRiecvei eTwes atnd Practice The nurse is caring for a client diagnosed with acute
for the NCLEX-RN ® Exam myocardial infarction (MI) and a history of severe
406 uncontrolled hypertension. The nurse should question
The Practice Test which of the following physician orders?
199. The Answer is 3 Reworded Question: What is not an appropriate
The nurse is caring for a client receiving intravenous order for this client?
therapy through a peripherally inserted central Strategy: Think about the combination of MI and
catheter (PICC). Which of the following actions severe uncontrolled hypertension. Use the process
implemented by the nurse will decrease the risk of of elimination.
infection? Needed Info: Limiting physical activity, IV
Reworded Question: What should the nurse know nitroglycerin,
about preventing infection with intravenous therapy? thrombolytic therapy, and oxygen therapy are
Strategy: Note the key words “decrease the risk of all appropriate treatments for a client with an MI.
infection.” Use the process of elimination. However, severe uncontrolled hypertension is one of
Needed Info: Although PICC-related complications the many contraindications of thrombolytic therapy.
are low, those caring for PICC lines must know how Category: Planning/Safe and Effective Care
to prevent and manage them. After the PICC line is Environment/
inserted using sterile technique, the goal is to monitor Safety and Infection Control
for signs of infection and maintain sterile technique (1) Limit physical activity for the first 12 hours—
throughout care. this is an appropriate order
Category: Planning/Safe and Effective Care (2) IV nitroglycerin—this is an appropriate order
Environment/ (3) Thrombolytic therapy—CORRECT: thrombolytic
Safety and Infection Control therapy is contraindicated in severe uncontrolled
(1) Assess vital signs every 4 hours—will detect hypertension
signs of infection but is not associated with prevention (4) Oxygen therapy—this is an appropriate order
(2) Ask the physician for an order for antibiotics— Practice Test Explanations
are administered to treat, not prevent, infection 407
(3) Maintain sterile technique during all phases of Practice Test Answers Taensdt 202. The Answer
PICC care—CORRECT: during all phases of is 2
PICC care, there should be meticulous attention The nurse is preparing to administer warfarin
to cleanliness and aseptic technique (Coumadin)
(4) Administer acetaminophen (Tylenol) before to a client diagnosed with atrial fibrillation.
dressing changes—is not related to decreasing The nurse knows that which of the following nursing
the risk of infection diagnoses takes priority?
200. The Answer is 1 Reworded Question: Which nursing diagnosis is
The nurse is caring for a client with chronic obstructive appropriate for anticoagulant therapy?
pulmonary disease (COPD) and is planning to Strategy: Think safety, and use the process of
obtain an arterial blood gas (ABG). Which of the elimination.
following Needed Info: Anticoagulants inhibit clot formation
should the nurse plan to do to prevent bleeding by blocking the action of clotting factors or platelets.
following the procedure? Coumadin is prescribed for the prophylaxis and/or
treatment of complications associated with atrial on the environment. Persons who breathe in the
fibrillation. Bleeding is a major risk. air containing these TB germs can become infected.
Category: Planning/Safe and Effective Care Persons entering areas of high risk exposure should
Environment/ use respiratory protective equipment.
Safety and Infection Control Category: Implementation/Safe and Effective Care
(1) Risk for imbalanced fluid volume—is not Environment/Safety and Infection Control
associated (1) Wash her hands and wear gown and gloves—this
with anticoagulant therapy action is incomplete and incorrect
(2) Risk for injury—CORRECT: clients on (2) Wash her hands—this action is incomplete
anticoagulant (3) Wash her hands and place a particulate filter
therapy are at risk for injury respirator
(3) Constipation—is not associated with anticoagulant over her nose and mouth—CORRECT:
therapy a particulate filter respirator should be placed
(4) Risk for unstable blood glucose—is not associated over the nose and mouth in areas of high risk
with anticoagulant therapy exposure
203. The Answer is 4 TChoen tPernatc tRiecvei eTwes atnd Practice
Prior to administering a tuberculin (Mantoux) skin for the NCLEX-RN ® Exam
test, the nurse in an outpatient clinic is educating a 408
client The Practice Test
suspected of having tuberculosis (TB). The nurse (4) Ask the client to don a mask—this action is
determines that the client understands the teaching incorrect
when the client states which of the following? 205. The Answer is 3
Reworded Question: What is the purpose of the The nurse is preparing a female client for a cardiac
Mantoux catheterization with the femoral approach. The
skin test? nurse should do which of the following when the client
Strategy: Use the process of elimination. returns to her room after the procedure?
Needed Info: A Mantoux skin test is performed to Reworded Question: After cardiac catheterization,
see if a client has ever had TB. It is done by putting how should the client be positioned?
a small amount of TB protein under the top layer of Strategy: Visualize the approach used during the
skin on the inner forearm. If a client has ever been procedure. Use the process of elimination.
exposed to the bacteria, the skin will react to the Needed Info: During cardiac catheterization with
antigens by developing a firm red bump at the site the femoral approach, access to the coronary arteries
within 2 days. A TB skin test cannot tell how long a is gained through the femoral artery in the right
person has been infected with TB or if the infection or left groin. Upon completion of the procedure,
is inactive or active and can be passed to others. the sheath is removed and pressure is held over the
Category: Assessment/Safe and Effective Care site for approximately 20 minutes until hemostasis
Environment/ is achieved. A pressure dressing is applied and the
Safety and Infection Control client is instructed to lie on her back and keep the
(1) “I know the test will tell me how long I’ve been procedural leg straight for 12 hours to avoid bleeding
infected with TB.”—tuberculin test cannot tell and possible hematoma.
how long a client has been infected with TB Category: Implementation/Safe and Effective Care
(2) “This test will tell me if I am contagious.”— Environment/Safety and Infection Control
tuberculin test cannot tell if a client is contagious (1) Elevate the head of the bed 45 degrees—should
(3) “I will need to come back and have a nurse look be elevated no more than 30 degrees
at the site in a week.”—site of the skin test must (2) Keep the client’s arm immobilized for the first
be read within 48–72 hours 24 hours—arm is immobilized if the brachial
(4) “The test will tell us if I’ve ever been infected approach is used
with TB bacteria.”—CORRECT: tuberculin (3) Keep the client’s leg immobilized for the first 12
skin test is done to see if a client has ever had TB hours—CORRECT: affected leg is immobilized
204. The Answer is 3 for the first 12 hours
The nurse is preparing to enter the private, well- (4) Tell the client to lie on the procedural side for 2
ventilated hours—client is instructed to lie on her back for
isolation room of a client with active tuberculosis the first 12 hours
(TB). Which of the following actions should the 206. The Answer is 2
nurse take before entering the room? A 65-year-old woman with metastatic breast cancer
Reworded Question: What precautions should the has been admitted to the hospital with neutropenic
nurse take before entering the room of a client with fever. She informs the nurse that she does not want
active TB? CPR or artificial ventilation to be performed under
Strategy: Use the process of elimination. Note the any circumstances. The nurse explains that this
client’s diagnosis and the need for respiratory information can be outlined in an advance directive.
precautions. The nurse understands that which of the following
Needed Info: TB is caused by germs that are spread addresses the client’s right to identify treatment
from person to person through the air. The germs are desires in advance?
put into the air when a person with TB disease of the Reworded Question: What ensures that clients are
lungs or throat coughs, sneezes, speaks, or sings. The informed of their right to create advance directives?
germs can stay in the air for several hours depending Strategy: Knowledge regarding each of these choices
is required to answer this question. If you had is frequently associated with sickle cell anemia;
difficulty although the nurse should assess and intervene
with this question, review the Patient’s Bill to alleviate client’s pain, not the first priority
of Rights, the Patient Self-Determination Act, the 208. The Answer is 1
Health Insurance Portability and Accountability A 58-year-old Spanish-speaking woman is being
Act (HIPAA), and the Americans with Disabilities discharged
Act to be sure you understand the objective of each. after having a central venous access device
Needed Info: The Patient Self-Determination Act placed. Which of the following BEST describes the
outlines the requirements for informing clients they nurse’s role in advocating for her client?
have right to refuse medical treatment and to specify Reworded Question: Which answer best characterizes
their wishes for treatment through advance directives. advocacy?
Category: Analysis/Safe and Effective Care Strategy: Think about the definition of nurse
Environment/ advocacy.
Management of Care Focus on the key word “BEST,” which may
(1) The Patient’s Bill of Rights—does not address indicate there is more than one correct response.
advance directives Consider each answer and select the action that is
(2) The Patient Self-Determination Act—CORRECT: most representative of a nurse advocating for her
ensures that patients are informed of client.
their right to refuse medical treatment and (in Needed Info: Advocacy: the act of promoting a client’s
advance directives) to specify their wishes for rights and interests.
treatment Category: Implementation/Safe and Effective Care
(3) The Health Insurance Portability and Environment/Management of Care
Accountability (1) The nurse uses a translator when she provides the
Act (HIPAA)—does not address advance client with discharge instructions—CORRECT:
directives using a translator is the best example of an act
(4) The Americans with Disabilities Act— does not of advocacy that promotes the client’s rights and
address advance directives interests
207. The Answer is 1 (2) The nurse provides both written and verbal
After receiving morning report on a medical/surgical discharge
unit, which of the following clients should the instructions—providing written and verbal
nurse address FIRST? instructions may benefit the client, but is not
Reworded Question: Which client requires immediate the best example of advocacy
attention? (3) The nurse ensures the client has transportation
Strategy: Focus on the key word “FIRST.” This home upon discharge—asking the client if she
indicates the need to prioritize, or triage, the clients has a ride home is appropriate, but not the best
based on the need for nursing intervention. example of advocacy
Needed Info: Pulmonary embolism (PE) occurs (4) The nurse provides discharge instructions in a
when the pulmonary artery becomes occluded by a private room—providing discharge instructions
blood clot. PE can lead to death if not diagnosed in way that maintains privacy is important, but
and treated promptly. Although symptoms may be not the best example of advocacy
Practice Test Explanations 209. The Answer is 2
409 A 26-year-old man being admitted for an emergency
Practice Test Answers Taensdt nonspecific, appendectomy asks the nurse why she is asking about
hemoptysis (blood in sputum) in the his medications and history of previous illnesses. In
postoperative client is suspicious for PE. addition to explaining why it is relevant to the care of
Category: Analysis/Safe and Effective Care the client, the nurse knows this client responsibility
Environment/ has been outlined in which of the following?
Management of Care Reworded Question: Which of the choices defines
(1) A 36-year-old man who underwent surgery providing information about medications and past
to repair multiple fractures in his left leg after illness as a client’s responsibility?
an automobile accident reports coughing up Strategy: Be familiar with the Americans with
blood—CORRECT: hemoptysis in a postoperative Disabilities
trauma client is suspicious for PE, requires Act, the Patient’s Bill of Rights, Nursing
immediate intervention Scope and Standards of Practice, and the Health
(2) A 56-year-old woman newly diagnosed with Insurance Portability and Accountability Act
diabetes (HIPAA).
has a fasting blood sugar of 83 mg/dL— Needed Info: The Patient’s Bill of Rights was
fasting blood sugar of 83 mg/dL is considered adopted to boost confidence in the health care
within normal limits; does not require intervention system,
(3) A 68-year-old man with head and neck cancer emphasize the need for strong patient/provider
receiving a continuous 5-fluorouracil infusion relationship, and to define rights and responsibilities
reports feeling nauseated—nausea is a side effect for patients and providers. Included was the need for
of 5-fluorouracil treatment; although it should patients to take responsibility for their health care
be addressed, not the first priority by, among other things, providing information on
(4) A 28-year-old woman with sickle cell anemia medications and past health history to providers.
reports a pain level of 6 on a scale of 1–10—pain
Category: Analysis/Safe and Effective Care interests of the client. Confidentiality: maintaining
Environment/ privacy by not disclosing personal information.
Management of Care Category: Analysis/Safe and Effective Care
(1) The Americans with Disabilities Act—does not Environment/
define these responsibilities Management of Care
(2) The Patient’s Bill of Rights—CORRECT: (1) Accountability—does not relate to client privacy
includes a provision that the client is responsible (2) Autonomy—does not relate to client privacy
TChoen tPernatc tRiecvei eTwes atnd Practice (3) Beneficence—does not relate to client privacy
for the NCLEX-RN ® Exam (4) Confidentiality—CORRECT: not disclosing
410 personal information represents confidentiality
The Practice Test 212. The Answer is 2
for providing information about medications A nurse is caring for a 48-year-old man with a new
and past illnesses colostomy. Which of the following activities BEST
(3) Nursing Scope and Standards of Practice—does describes the nurse’s role as an advocate for the
not define these responsibilities client?
(4) The Health Insurance Portability and Reworded Question: Which task best advocates for
Accountability the client?
Act (HIPAA)—does not define these Strategy: Focus on the key word “BEST.” This
responsibilities indicates
210. The Answer is 2 there may be more than one correct response.
A nurse is working on the medical/surgical unit. Consider each option and select the action that best
The nurse knows that which of the following tasks advocates for the client.
should NOT be delegated to nursing assistive Needed Info: Nurse advocacy is based on the premise
personnel that a nurse acts to protect the best interests of the
(NAP)? client as well as to protect a client’s right to make
Reworded Question: Which task is outside the scope decisions. Nurse advocacy includes empowering a
of NAPs and should not be delegated? client through education.
Strategy: Focus on the key words “should NOT.” Category: Implementation/Safe and Effective Care
Think about the skill level required in each scenario, Environment/Management of Care
and use the process of elimination to select which (1) Ensuring the skin is dry before re-adhering the
task is not appropriate for an NAP. pouch—proper procedure, but does not provide
Needed Info: Nurses should not delegate tasks to client with education needed to care for his own
NAPs related to assessment, diagnosis, or ostomy
intervention (2) Teaching the client how to change and care for
that requires professional knowledge and skill. the ostomy pouch—CORRECT: best represents
Category: Implementation/Safe and Effective Care Practice Test Explanations
Environment/Management of Care 411
(1) Setting up a meal tray for a 75-year-old client Practice Test Answers Taensdt nurse’s role as
with Alzheimer’s disease— appropriate to delegate client advocate; educating client
(2) Assessing a newly postoperative client’s pain about how to care for himself empowers him to
level—CORRECT: best example of task that self-advocate
should not be delegated (3) Providing the client’s wife with a list of foods to
(3) Setting up a water basin for a 45-year-old client avoid—may be important to client in terms of
who wishes to shave at the bedside—appropriate dietary choices, but does not teach client directly
to delegate how to manage his ostomy
(4) Transferring a 70-year-old client awaiting (4) Explaining to the client that psychological
discharge adjustment to an ostomy can take time—likely
from the bed to a wheelchair—appropriate useful to client, but does not teach client directly
to delegate how to manage ostomy
211. The Answer is 4 213. The Answer is 3
A nurse receives a phone call from a family member A client who has scabies has been admitted to the
asking for health-related information on a client medical/surgical unit. The nurse knows he should
being treated for suspected myocardial infarction use which of the following precautions when caring
in the Emergency Department. The nurse explains for this client?
she cannot disclose personal information about the Reworded Question: What type of precautions should
client without the client’s consent. The nurse knows be initiated for a client with scabies?
this represents which of the following ethical Strategy: Consider the method by which scabies is
principles? transmitted and correlate that with the type of
Reworded Question: Which ethical principle relates precaution
to maintaining a client’s privacy? required to prevention its transmission.
Strategy: Use the process of elimination. Review Needed Info: Be familiar with the way in which scabies
ethical principles of clinical practice. is transmitted from person to person (contact)
Needed Info: Accountability: responsibility for one’s and identify the corresponding type of precaution
actions. Autonomy: right to make one’s own the nurse should take (contact).
decisions. Category: Implementation/Safe and Effective Care
Beneficence: duty to do what is in the best Environment/Safety and Infection Control
(1) Droplet precautions—scabies not transferred via injections requires the client to understand that
droplets, so droplet precautions not required needles
(2) Airborne precautions—scabies not transferred should not be recapped or reused, and that they
via the air, so airborne precautions not required should be placed in a puncture-resistant container.
(3) Contact precautions—CORRECT: mites from Category: Implementation/Safe and Effective Care
scabies can be transferred via contact; contact Environment/Safety and Infection Control
precautions should be initiated (1) Dispose of needles in a puncture-resistant
(4) Precautions are not necessary with this client— container—
at a minimum, standard precautions should be CORRECT: needles should not be
used with all clients, regardless of health status recapped or bent, and should always be disposed
214. The Answer is 2 of in a puncture-resistant container
A client who has a localized herpes simplex virus (2) Wear chemotherapy-resistant gloves—not
(HSV) infection is admitted to the maternity unit. required with filgrastim, as it is not classified as
The nurse knows the client should IDEALLY be a hazardous agent
placed in which of the following? (3) Recap the needles for reuse—needles should
Reworded Question: What type of room should a client never be recapped or reused
with HSV be assigned? (4) Neupogen has been prescribed to boost platelets—
Strategy: Consider how localized HSV infections is prescribed to increase white blood cell
are transmitted. Focus on the key word “IDEALLY.” count
This may mean more than one choice is acceptable. 216. The Answer is 1, 2, 4, and 5
Use the process of elimination to determine the best A 76-year-old woman has been admitted to a
choice. rehabilitation
Needed Info: A nurse caring for a client with a center after a hip replacement. During an
localized episode of confusion in which she became a danger
HSV infection should use contact precautions. to herself, the client was placed in a vest restraint.
To minimize the likelihood of infecting another client The nurse knows that which of the following are also
with HSV, the ideal room placement for this client considered types of restraints? Select all that apply.
is a single, unoccupied room. Reworded Question: What are examples of restraints?
Category: Implementation/Safe and Effective Care Strategy: Consider each response.
Environment/Safety and Infection Control Needed Info: Know the definition of medical
(1) Any available room—contact precautions restraints and be able to provide examples.
should be initiated for a client with a localized Category: Analysis/Safe and Effective Care
HSV infection; consideration should be given to Environment/
ideal room placement Safety and Infection Control
(2) A single, unoccupied room—CORRECT: best (1) Administering a haloperidol (Haldol) injection—
choice for this client, who, based on her localized CORRECT: a chemical restraint
HSV infection, should be placed on contact (2) Raising 4 bed side rails—CORRECT: a mechanical
precautions restraint
(3) A double room with a client who underwent a (3) Assigning a nurse’s aide to sit and observer the
cesarean section 3 days prior—client with an client—non-restraint effort to protect the client
active HSV infection should not share a room (4) Applying wrist cuffs and tying them to the bed—
with a client who underwent a cesarean section CORRECT: a mechanical restraint
3 days prior (5) Clipping a tray across the front of the client’s
(4) A double room with a curtain divider—not the wheelchair—CORRECT: trays that clip across
ideal room for a client requiring contact precautions the front of a wheelchair so that the client can’t
215. The Answer is 1 fall out easily are a form of mechanical restraint
A 38-year-old client with breast cancer will be 217. The Answer is 1
selfadministering A physician has written an order for escitalopram
filgrastim subcutaneously. The nurse oxalate 10 mg PO daily for a 15-year-old client with
knows that teaching should include which of the depression. After performing an initial assessment,
following? the nurse calls the physician to verify the order.
Reworded Question: What instructions should the Which of the following BEST explains the nurse’s
nurse give the client related to self-injections of concern about the safety of the order?
Neupogen? Reworded Question: What are contraindications to
Strategy: Consider each option and, using the process escitalopram oxalate therapy?
of elimination, select elements of teaching the Strategy: Using the process of elimination, determine
nurse should include. when escitalopram oxalate may not be appropriate
Needed Info: Filgrastim (Neupogen) is a drug used to to administer.
boost white blood cell counts in individuals undergoing Needed Info: Know indications/contraindications
chemotherapy treatment. It’s not classified as and recommended dose ranges for escitalopram
TChoen tPernatc tRiecvei eTwes atnd Practice oxalate (Lexapro).
for the NCLEX-RN ® Exam Category: Assessment/Safe and Effective Care
412 Environment/
The Practice Test Safety and Infection Control
a hazardous medication, so chemotherapy-resistant (1) The client reported a history of facial swelling
gloves are not required. Safe administration of and difficulty breathing while on citalopram—
CORRECT: facial swelling, difficulty breathing Safety and Infection Control
characteristic of allergic reaction; individuals (1) Complete an incident report at the end of his
who have experienced allergic reactions to citalopram shift, when he is less busy—the incident report
(Celexa) should not take escitalopram should be completed as soon as possible
oxalate (2) Complete an incident report using clear, concise,
(2) The drug has not been approved for use in the and factual language—CORRECT: the incident
client’s report should be completed using clear, concise,
age group—Escitalopram oxalate approved and factual language
for acute and maintenance treatment for major (3) Complete an incident report and place a copy of
depressive disorder in adults and in adolescents it in the client’s medical record—incident reports
aged 12–17 years should not be filed in the client’s medical record
(3) The ordered dose is higher than the suggested (4) Ask the evening shift nurse to complete an
range—typical starting dose for escitalopram incident
oxalate is 10 mg PO daily report because the fall occurred on her
(4) The ordered dose is lower than the suggested shift—an incident report should be completed
range—typical starting dose for escitalopram by the individual who identified the problem, in
oxalate; is 10 mg PO daily this case, the night shift nurse
218. The Answer is 3 and 4 220. The Answer is 1
A 75-year-old client has an unsteady gait and requires A nurse is caring for an 8-year-old girl with urinary
assistance with ambulation. The nurse decides to use retention. The nurse is preparing to insert a Foley
a gait belt. The nurse knows she should do which of catheter. Which of the following catheter sizes is
the following when using a gait belt? Select all that most appropriate for this client?
apply. Reworded Question: What size Foley catheter should
Practice Test Explanations be used for pediatric clients?
413 Strategy: Consider how Foley catheters are sized
Practice Test Answers Taensdt Reworded and select the best choice for a pediatric client.
Question: Which of the following are correct Needed Info: Foley catheters are sized according to
when using a gait belt? the French scale, abbreviated Fr. Each unit
Strategy: Focus on the key words “Select all that corresponds
apply.” More than one response may be correct. with a diameter of approximately 0.33 mm.
Think about elements of proper gait belt use, and So, an 18 Fr catheter equals a diameter of about 6
use the process of elimination to rule out the incorrect mm. The larger the number, the larger the catheter
responses. size. For a smaller pediatric client, the smaller size
Needed Info: Know proper technique for gait belt use. is the best choice.
Category: Implementation/Safe and Effective Care Category: Implementation/Physiological Integrity/
Environment/Safety and Infection Control Basic Care and Comfort
(1) Secure the gait belt loosely around the client’s (1) Number 8 French—CORRECT: the number 8
waist—the gait belt should fit snugly, with room French catheter is the smallest of the choices,
for the nurse’s fingers between the belt and client and is the right answer; typically, number 8 Fr
(2) Twist her upper body to position the client—the or number 10 Fr catheters are used in children
nurse should keep her back straight to practice (2) Number 16 French—number 16 Fr is usually
proper body mechanics and prevent injury used for adult women
(3) Remove the gait belt after use—CORRECT: gait (3) Number 20 French—number 20 Fr is usually
belts should be removed after use used for adult men
(4) Place the gait belt over the client’s clothes with (4) Number 22 French—number 22 Fr is usually
the clip in front—CORRECT: allows for easier used for adult men
adjustment TChoen tPernatc tRiecvei eTwes atnd Practice
(5) Use the gait belt to help lift the client from a sitting for the NCLEX-RN ® Exam
into a standing position—should never be 414
used to lift a client. The Practice Test
219. The Answer is 2 221. The Answer is 4
After receiving report at the start of a night shift, A 42-year-old male client weighs 196 lbs. (89.1 kg)
the nurse finds an elderly client lying on the floor and is 65 inches (1.65 meters) tall. Based on the
with the bedrails down. When documenting findings, client’s body mass index (BMI), the nurse knows
which of the following BEST describes what this client would fall into which of the following
the nurse should do? categories?
Reworded Question: What is the proper way to Reworded Question: What is this client’s BMI and
complete weight status?
incident reports? Strategy: Calculate the BMI and determine the
Strategy: Consider each response. Focus on the key corresponding
word “BEST.” weight status.
Needed Info: Know proper procedure for completing Needed Info: The formula for calculating a BMI is
incident reports, including who should fill them out [wt × 703 / (ht)2], or [196 × 703 / (65)2] = 32.6.
and when they should be filled out. BMI
Category: Analysis/Safe and Effective Care interpretation is the same for male and female adults
Environment/ over the age of 20. BMI below 18.5, underweight;
18.5–24.9, normal; 25.0–29.9, overweight; 30.0 and used in the treatment of cancer. It is a vesicant, which
above, obese means that it can cause blistering if it extravasates
Category: Assessment/Physiological Integrity during administration. Proper vein selection is critical
(1) Underweight—a BMI below 18.5 indicates a in preventing this complication. The antecubital
weight status of underweight fossa and wrist are not the ideal sites for vesicant
(2) Normal weight—a BMI of 18.5–24.9 indicates a administration because movement may increase the
normal weight status likelihood of dislodging the IV and causing
(3) Overweight—a BMI of 25.0–29.9 indicates a extravasation.
weight status of overweight In addition, extravasations are more difficult
(4) Obese—CORRECT: a BMI of 30.0 or more indicates to identify in the antecubital fossa. Sites distal to
a weight status of obese previous
222. The Answer is 2 venipunctures should not be used.
A 10-year-old girl is being seen in the Pediatric Practice Test Explanations
Emergency Department following a motor vehicle 415
accident. She has been stabilized but reports a pain Practice Test Answers Taensdt Category:
level of 8 on a scale of 1 to 10. The nurse is preparing Implementation/Physiological Integrity/
to transfer the client to x-ray. The nurse knows that Pharmacological and Parenteral Therapies
which nonpharmacologic intervention should NOT (1) The non-dominant antecubital fossa—not the
be used to help reduce pain in this client? best choice
Reworded Question: Which nonpharmacologic (2) The distal forearm—CORRECT: the distal forearm
intervention is the best venipuncture site because the soft
is not appropriate? tissue present in that location limits the potential
Strategy: Consider each response, and using the for harm to nerves and tendons should extravasation
process of elimination, select the intervention that occur
should not be used. (3) The wrist—not the best choice
Needed Info: Pain is individualized and can be (4) A vein used for venipuncture within the previous
affected by coping skills, emotional state, 24 hours—not the best choice
temperament, 224. The Answer is 4
and past experiences. Nonpharmacologic A 24-year-old client with anorexia has had a
interventions can be effective in managing pain nasogastric
and should be implemented in an age-appropriate (NG) tube placed in preparation for intermittent
manner. For school-aged children, the following enteral feedings. The nurse knows to do which
examples of nonpharmacologic interventions may of the following when administering medications via
be useful: providing complete explanations, an NG tube?
encouraging Reworded Question: What is the appropriate way to
participation in decision making, allowing administer medications via an NG tube?
choices if feasible, using distraction, using relaxation Strategy: Consider the proper technique for
and using imagery. administering
Category: Implementation/Physiological Integrity/ medications via an NG tube. Review each
Basic Care and Comfort response and use the process of elimination to
(1) Offer choices when possible—it is appropriate to determine
offer choices when possible the correct choice.
(2) Reassure the client that the procedure will not Needed Info: Know the procedure for administering
hurt—CORRECT: client should be given medications via an NG tube. Enteric coated aspirin
ageappropriate should not be crushed because this action removes
but truthful explanations about any the protective coating and potentially clogs the tube.
procedures Medications should not be mixed with the client’s
(3) Provide complete explanations about what is feeding formula. The feeding should be stopped,
going to happen: this is appropriate the tube should be flushed, and then medications
(4) Use distraction, relaxation, and imagery—this is should be administered. A minimum of a 30-mL
appropriate syringe should be used to flush or irrigate the tube.
223. The Answer is 2 Each medication should be administered separately;
In preparation for doxorubicin administration, the medications should not be mixed and administered
nurse is assessing a client’s arm to determine where together.
to attempt venipuncture. The nurse knows which of Category: Implementation/Physiological Integrity/
the following veins is the BEST choice to start the Pharmacological and Parenteral Therapies
IV? (1) Crush the enteric coated aspirin—enteric coated
Reworded Question: Which is the best choice of medications should not be crushed
venipuncture sites to administer doxorubicin (2) Mix the medications with the client’s feeding
(Adriamycin)? formula—medications should not be mixed with
Strategy: Focus on the key word “BEST.” This the client’s feeding formula
indicates (3) Flush the tube using a 15-mL syringe—tube
there may be more than one acceptable choice. should be flushed using a minimum of a 30-mL
Consider ways to prevent complications associated oral syringe
with intravenous chemotherapy administration. (4) Administer each medication separately—
Needed Info: Doxorubicin is an antitumor antibiotic CORRECT:
medications should not be combined and (2) Creatinine 1.5 mg/dL—this is a normal range
administered together value and is not the correct response
225. The Answer is 3 (3) Fasting glucose 145 mg/dL—this is a normal
A 3-year-old client with acute otitis media has been range value and is not the correct response
prescribed ofloxacin ear drops. The nurse knows (4) Total calcium 10.0 mg/dL—this is a normal
that which of the following statements by the father range value and is not the correct response
demonstrates that he understands how to properly 227. The Answer is 2
administer the ear drops? A 64-year-old man with heart failure has recently
Reworded Question: What is the proper way to been told by his physician to increase his digoxin
administer ear drops? dose to 0.25 mg. He has 125 mcg tablets on hand.
Strategy: Consider each of the statements. Using the Which of the following statements by the client to
process of elimination, select the statement that best the home health nurse indicates the client has
reflects the technique that should be used to understood
administer the teaching provided about the medication?
ear drops to a pediatric client. Reworded Question: Which statement is correct?
Needed Info: Ear drop medication technique: warm Strategy: Recall how to calculate digoxin (Lanoxin)
the drops to body temperature to prevent pain and dosages, as well as the side effects of the drug.
dizziness; lie the client down with affected ear facing Consider
up; place drops on the wall of the ear canal to allow the consequence of each action, and eliminate
air to escape and the medication to flow into the ear; the incorrect statements.
give the medication as prescribed. Needed Info: A digoxin 125 mcg tablet is equivalent
Category: Implementation/Physiological Integrity/ to 0.125 mg. GI disturbances may be associated with
Pharmacological and Parenteral Therapies digoxin toxicity. Regular follow-up and blood work
(1) “I can stop giving the ear drops as soon as my monitoring is required for clients taking digoxin.
daughter’s fever is gone.”—medication should Over-the-counter medications can interfere with
be administered as prescribed and should not be digoxin therapy and should not be taken without
discontinued based on the presence or absence physician approval.
of a fever Category: Assessment/Physiological Integrity/
(2) “I should give the drops directly on the eardrum Reduction of Risk Potential
to help get rid of the infection quickly.”—drops (1) “I should take one tablet.”—client should take 2
should not be administered directly on the eardrum tablets
(3) “I should warm the ear drops before giving (2) “I should notify my doctor if I experience
them by wrapping the bottle in my hand.”— diarrhea.”—
CORRECT: temperature should be between CORRECT: client should notify his
95–98.6° F (35–37° C) physician if he experiences GI disturbances
(4) “My daughter should lie flat while I give the including diarrhea
drops.”—client should lie on her side with the (3) “I don’t need to follow up with my doctor unless
affected ear facing upwards I’m having a problem.”—routine follow-up and
TChoen tPernatc tRiecvei eTwes atnd Practice monitoring is required
for the NCLEX-RN ® Exam (4) “I can take over-the-counter medications without
416 the approval of my physician.”—over-thecounter
The Practice Test medications should not be taken without
226. The Answer is 1 physician approval
A 68-year-old woman recently diagnosed with 228. The Answer is 3
hypertension has started taking furosemide 40 mg The nurse is preparing to administer a tuberculin
PO twice daily. During a clinic appointment, she skin test to a pregnant 26-year-old client. The nurse
reports new onset muscle weakness and abdominal knows which of the following statements about
cramping. Lab tests are performed. The nurse knows tuberculin skin testing is TRUE?
which of the following results is the best explanation Reworded Question: How are tuberculin skin tests
for the symptoms experienced by the client? correctly administered?
Reworded Question: What is the most likely reason Strategy: Consider proper techniques associated
for the client’s symptoms? with tuberculin skin test administration. Using the
Strategy: Consider the side effects of furosemide. process of elimination, select the correct answer.
What effects does the drug have on potassium levels? Needed Info: Mantoux tuberculin skin testing is used
What symptoms might be seen with this side effect? to determine whether a client is infected with
Needed Info: Furosemide (Lasix) is a diuretic that Mycobacterium
can lead to electrolyte imbalances. The normal range tuberculosis. Tuberculin skin tests may be
for potassium is 3.5 to 5 mEq/L. Symptoms associated administered during pregnancy. They should be read
with hypokalemia include abdominal cramping 48 to 72 hours after administration. The reaction is
and muscle weakness. measured in millimeters of the induration. The test
Category: Analysis/Physiological Integrity/Reduction should be administered into the inner surface of the
of Risk Potential forearm.
(1) Potassium 3.0 mEq/L—CORRECT: a potassium Category: Implementation/Physiological Integrity/
level of 3.0 mEq/L is below the normal range Reduction of Risk Potential
and may be associated with symptoms such as Practice Test Explanations
abdominal cramping and muscle weakness 417
Practice Test Answers Taensdt (1) The test should Physiological Adaptation
not be administered during pregnancy— (1) Assess the client’s airway—not the intervention
test may be administered during pregnancy that should be performed first
(2) The test should be read between 24 and 48 (2) Stop the infusion—CORRECT: the infusion
hours after administration—test should be read should be stopped FIRST
between 48 and 72 hours after administration (3) Slow down the rate of infusion—not the
(3) The reaction is measured in millimeters of the intervention
induration—CORRECT: reaction is measured that should be performed first
in millimeters of the induration (4) Administer epinephrine—not the intervention
(4) The test should be administered into the outer that should be performed first
surface of the forearm—test should be administered 231. The Answer is 4
into the inner surface of the forearm The nurse is assigned as the team leader on a busy
229. The Answer is 1, 3, and 5 medical/surgical unit. Which of the following BEST
The nurse takes report on a client who underwent a describes the “rights” of delegation the nurse must
thyroidectomy 24 hours ago. The nurse understands consider when assigning tasks to other members of
that the client is at risk for hypocalcemia. Which of the health care team?
the following assessment findings indicate the client Reworded Question: What are the five principles of
may be hypocalcemic? Select all that apply. delegation when assigning tasks to the health care
Reworded Question: Which findings are associated peers on your shift?
with hypocalcemia? Strategy: Apply your knowledge of the topic—
Strategy: Note the instruction to “select all that delegation
apply.” More than one response may be correct. of tasks. Eliminate the answer choices that
Consider the signs and symptoms associated with are incorrect first.
hypocalcemia. Use the process of elimination to Needed Info: Delegation is an important skill. The
determine which response or responses are correct. nurse should identify the right person to do the
Needed Info: Hypocalcemia is a risk associated with TChoen tPernatc tRiecvei eTwes atnd Practice
thyroidectomy and is most common 24 to 48 hours for the NCLEX-RN ® Exam
after surgery. Signs and symptoms of hypocalcemia 418
in a client who has undergone thyroidectomy may The Practice Test
include positive Trousseau’s sign, positive Chvostek’s specific task, explain or give clear directions regarding
sign, periorbital numbness, mental status changes, the task, choose the right task for the appropriate
tetany, seizures, EKG changes, and cardiac arrest. person, and keep the client in mind. Is the client able
Category: Assessment/Physiological to tolerate the task being performed by the person
Integrity/Physiological chosen?
Adaptation Category: Analysis/Safe and Effective Care
(1) Positive Trousseau’s sign—CORRECT: positive Management/
Trousseau’s sign may be associated with Management of Care
hypocalcemia (1) Right task, right timing, right client, right person,
(2) Negative Chvostek’s sign—incorrect and right date—right client refers to the five
(3) Numbness around the mouth—CORRECT: may rights of dispensing medications
be associated with hypocalcemia (2) Right task, right client, right direction, right
(4) Positive Moro reflex test—incorrect supervision, and right date—right client and
(5) “Pins and needles” sensation in client’s feet— right date refer to medications
CORRECT: an early symptom of hypocalcemia (3) Right client, right direction, right day, right
230. The Answer is 2 medication,
A 58-year-old client is receiving the monoclonal and right unit—right day, right medication,
antibody and right unit do not refer to the delegation
rituximab and develops an infusion reaction of tasks
manifested by chest pain and dyspnea. The nurse (4) Right task, right circumstance, right person,
should do which of the following FIRST? right direction, and right supervision—CORRECT:
Reworded Question: What should the nurse do these are the five rights of delegation;
FIRST during an infusion reaction? when delegating, the charge nurse must make
Strategy: Focus on the key word “FIRST.” This certain of the following regarding the task: (1)
indicates it can be safely delegated; (2) the client is stable
there may be more than one correct answer. and a good outcome is anticipated; (3) the task
Select the intervention that must be performed first. is being delegated to the right person; (4) there
Needed Info: The nurse should assess frequently for is clear communication; (5) the charge nurse is
reactions during monoclonal antibody infusions. responsible for supervision
When an infusion reaction is suspected, the nurse 232. The Answer is 4
should immediately stop the infusion but maintain Which of the following pediatric clients should
vascular access with a normal saline infusion. the nurse provide assessment and intervention for
After that, it is appropriate for the nurse to assess FIRST?
the client, and prepare emergency equipment and Reworded Question: Which client requires immediate
medication. Depending on the situation, it may be intervention?
appropriate to restart the infusion at a slower rate. Strategy: Think ABCs and airway.
Category: Implementation/Physiological Integrity/ Needed Info: Understand the ABCs framework of
prioritizing airway, breathing, and circulation in Reworded Question: What is an important health
order of importance. Always assess the client before risk when caring for an elderly adult with dementia?
checking alarms. Strategy: Think client safety.
Category: Assessment/Safe and Effective Care Needed Info: You should know the health risks to
Environment/ consider for any elderly adult regardless of medical
Management of Care diagnosis. Elderly clients have slower reflexes,
(1) A 15-month-old who has developed hives—no decreasing ability to adapt to new environments
indication that the infant with hives is in immediate especially with dementia, and balance issues leading
distress; requires an assessment but is not to falls.
the priority Category: Implementation/Safe and Effective Care
(2) A 2-year-old who is ventilated but stable—no Environment/Management of Care
indication that an alarm is sounding; client (1) Nutrition—factored into the nursing care plan
requires an assessment but not immediately but is in the category of basic needs and comfort
(3) A 12-year-old recovering from surgical repair of (2) Hygiene—in the category of basic needs and
a fractured femur — no indication of immediate comfort
distress (3) Fall risk—CORRECT: in the category of safety
(4) The 2-month-old infant whose apnea monitor is or safe and effective care, and should be part of
sounding with an oxygen saturation reading of any care plan for an elderly adult with or without
82%—CORRECT: reading below normal on the dementia
oxygen saturation must be assessed immediately; (4) Cardiac care—particular to clients who have
a 2-month-old cannot tell you what is wrong and symptoms or a history of heart disease and
has little room for compensation when oxygen would not necessarily be included in this care
levels drop; it is imperative that this infant is plan
assessed and an intervention initiated; it may 235. The Answer is 2
be simple repositioning, suctioning, or readjusting The nurse is planning the care of an elderly male client
the probe, but apnea alarms should not be with very poor oral hygiene and gum disease.
ignored The nurse knows that the teeth and gums can be
233. The Answer is 2 which of the following in the chain of infection?
The nurse knows that she would NOT be required to Reworded Question: Which element in the chain of
use airborne precautions for which of the following infection would the mouth, teeth, and gums of an
clients? older adult with gum disease be considered?
Reworded Question: Which pathogen is not spread by Strategy: Think about the six elements in the chain
small airborne droplets traveling more than 3 feet? of infection.
Strategy: Think about the transmission of pathogens. Needed Info: A pathogen has the potential to cause
Needed Info: Transmission-based precautions help to infection under the right conditions including
limit the spread of germs. Small germ-infected drops environment,
that travel far require airborne precautions. Droplet portal of exit, transmission mode, a way to
precautions are for those germs found in droplets of enter, and a susceptible host.
secretions when a client coughs or sneezes. Contact Category: Analysis/Safe and Effective Care
precautions are for germs spread by hand-to-skin Management/
touching or from germs on an object to skin. Safety and Infection Control
Category: Implementation/Safe and Effective Care (1) The method of transmission for bacteria—
Environment/Safety and Infection Control direct, indirect, or airborne contact; the teeth
(1) A young adult with possible tuberculosis who most often are not the mode of transmission
is also HIV positive—tuberculosis is carried in (2) The portal of entry for bacteria—CORRECT:
small airborne droplets over 3 feet and requires unhealthy gums generally are red, swollen, and
airborne precautions bleed with brushing, offering the portal of entry;
(2) A middle-aged adult with herpes simplex— bacteria enters and can easily spread through the
CORRECT: herpes simplex is spread from hand blood stream due to poor oral hygiene and gum
to skin or object to skin; requires contact not disease
airborne precautions (3) The pathogen—the teeth alone are not the
Practice Test Explanations pathogen
419 (4) A portal of exit—the teeth and gums are not a
Practice Test Answers Taensdt (3) A teenager with portal of exit
chickenpox and a sore throat— 236. The Answer is 2
chickenpox is spread by small airborne droplets In the event of a fire, the nurse should do which of
so airborne precautions are appropriate the following FIRST?
(4) A college student with possible rubella—rubella Reworded Question: Which should the nurse do
is an airborne pathogen requiring airborne FIRST in the event of a fire?
precautions Strategy: Think about what you know about the
until a diagnosis is confirmed steps involved in fire safety in a hospital setting.
234. The Answer is 3 Needed Info: If a fire occurs, the first step is to get
The nurse is caring for an elderly female with clients
dementia. out of danger, then work to contain the fire, and
The nurse knows that which of the following then to determine the order in which clients should
should be the priority for care for this client? be evacuated.
Category: Implementation/Safe and Effective Care and whose responsibility it is to obtain informed
Management/Safety and Infection Control consent.
(1) Leave the building—the nurse should not leave Needed Info: The practitioner ordering the procedure
the building without attempting to remove clients is responsible for explaining the risks, benefits,
from danger and alternatives to the client. The nurse may witness
(2) Attempt to get clients out of immediate danger— the signing of the consent but should only sign if he is
CORRECT: this is the first step the nurse should confident that the client does indeed understand and
take in the event of fire wants the procedure. The nurse should also be
(3) Work to contain the fire—the first thing the familiar
nurse should do is attempt to get clients out of with the ethics of decision making for any client.
immediate danger Category: Implementation/Safe and Effective Care
TChoen tPernatc tRiecvei eTwes atnd Practice Environment/Management of Care
for the NCLEX-RN ® Exam (1) The nurse tells the client not to worry because
420 blood transfusions are very common—telling
The Practice Test the client not to worry leaves the client without
(4) Determine the order in which to evacuate clients— the information required to make an informed
not the first step consent
237. The Answer is 1 (2) The nurse informs the ordering physician that
The nurse knows that which of the following BEST the client does not understand the risks and
describes the role of a nursing supervisor? will need further explanation—CORRECT: the
Reworded Question: What do you expect from a nurse should advocate for the client and call the
supervisor? practitioner who ordered the procedure to come
Strategy: Think authority, guidance, and follow-up offer further information
responsibilities. (3) The nurse has someone else witness the signature
Needed Info: The role of a nursing supervisor is one on the consent—the nurse should not have
of authority. The nursing supervisor implements someone else witness the signature on the consent
different especially if he knows the client does not
ideas assisting in the safe and effective management fully understand the risks involved
of care for the clients in the facility. The (4) The nurse describes alternative treatments—the
nursing supervisor also manages the professional nurse’s role is not to provide alternative treatment
employees to ensure tasks are done efficiently and choices; physicians are the ones to discuss
correctly. alternative treatments
Category: Implementation/Safe and Effective Care 239. The Answer is 3
Environment/Management of Care The nurse is caring for a famous basketball player
(1) Chooses and implements interventions— who may have sustained a career-changing injury.
CORRECT: When asked by coworkers about the status of the
a nursing supervisor chooses strategies client, she responds that she is not able to discuss
and interventions that may be necessary to her client. Which of the following ethical principles
ensure timely and efficient tasks are completed BEST supports her statement?
(2) Attends meetings to keep staff up to date— Practice Test Explanations
although a nursing supervisor does attend meetings, 421
her role is to help to implement ideas when Practice Test Answers Taensdt Reworded
staff members require guidance Question: Which ethical principle should
(3) Does not require special skills to oversee other the nurse use when asked about a client by other staff
professionals—a nursing supervisor should not caring for the client?
have special skills like active listening, good Strategy: Think about ethical practice and the Code
communication skills, ability to solve problems, of Ethics for Nurses.
and knowledge about the clients and staff being Needed Info: Understand the ethical principles that
supervised determine what is right and wrong when caring for
(4) Is friendly and can make contributions to an clients. You should be able to define and review
employee evaluation—a nursing supervisor outcomes
should be friendly and not intimidating but and interventions to promote ethical practice.
friendliness does not make a nurse qualified to Justice, nonmaleficence, fidelity, confidentiality,
supervise others; supervisors may directly contribute and accountability are some of the ethical principles
information to an employee evaluation that a nurse should understand and practice.
238. The Answer is 2 Category: Implementation/Safe and Effective Care
The nurse is conversing with a young adult client Environment/Management of Care
regarding an ordered blood transfusion. It is clear (1) Justice—providing fair and appropriate treatment
to the nurse that the client does not understand the (2) Beneficence—doing what is in the best interest
risks involved with the procedure. Which of the of the client
following (3) Confidentiality—CORRECT: maintaining the
statements BEST describes the nurse’s role client’s privacy and supporting what the nurse’s
regarding informed consent for this procedure? role is by not talking to her coworkers about her
Reworded Question: What is the role of the nurse client
regarding informed consent? (4) Accountability—being responsible for one’s
Strategy: Think about the process of informed consent actions
240. The Answer is 3 The Practice Test
The nurse is on duty on a busy cardiac telemetry Category: Planning/Safe and Effective Care
unit. Which of the following situations requires the Environment/
nurse’s immediate attention? Management of Care
Reworded Question: Which scenario needs the (1) Team work—refers to peers working together to
immediate attention of the nurse? complete tasks; does not define multidisciplinary
Strategy: Think triage, establishing priorities, and actions for the benefit of the client’s plan of care
ABCs. (2) Team building—refers to a working relationship
Needed Info: You must understand priorities and between peers for a variety of reasons, not
symptoms of cardiac compromise. The nurse should always client related
know the basics for recognizing a sudden change in (3) Case management—coordinates the care,
the level of consciousness or a reduced heart rate and improves outcomes, and reduces cost but does
its importance. Know normal vital signs and not initially plan the care of the client
symptoms (4) Collaboration—CORRECT: defines the
of respiratory compromise such as snoring multidisciplinary
and being unable to arouse a client. A client sleeping team approach to building a plan of
soundly with a normal heart rate would not be care for the client
as worrisome. 242. The Answer is 1, 3, 4, and 5
Category: Analysis/Safe and Effective Care and The charge RN is preparing assignments on a busy
Environment/Management of Care medical unit. For this shift, there are several LPNs,
(1) The wife of a cardiac client states that his IV several RNs, and one NAP. Which of the following
pump is alarming and he is not receiving the pain assignments by the charge RN is appropriate? Select
medication dose due to the pump malfunctioning— all that apply.
can wait, not life-threatening Reworded Question: Which tasks are within the
(2) The daughter of an elderly client states that her scope of practice for the RN, the LPN, and the NAP?
mother is uncomfortable and that her electrodes Strategy: Eliminate the choices that are out of the
have come off—can wait, not life-threatening scope of practice for the professional assigned to the
(3) The new NAP reports that she cannot wake her task.
elderly client to take his blood pressure because Needed Info: Think through which tasks can be
he is sleeping soundly and snoring, but she appropriately delegated to trained personnel. You
obtained his pulse and it is 30. She wants you to also should review the principles of delegation, the
come to see if you can wake him—CORRECT: Nurse Practice Act, and know what it means to
a new NAP may not have enough experience to supervise those who have been assigned tasks.
recognize the difference between sleeping and an Category: Analysis/Safe and Effective Care
unconscious state in this elderly client; this client Environment/
should be assessed immediately with a heart Management of Care
rate of 30 and nursing interventions should be (1) The NAP is assigned to give morning baths—
initiated; if the client is really sleeping soundly, CORRECT: giving a bath is within the scope of
arousing him to increase his heart rate might practice for an NAP
be appropriate; if the client cannot be aroused, (2) An LPN is assigned to perform an initial
cardio pulmonary resuscitation efforts should assessment
be initiated by calling for help and following the on a newly admitted client—LPNs do not
ABCs of resuscitation perform initial client assessments
(4) The new admission from earlier today is (3) An LPN is assigned to clients who are prescribed
complaining oral medications, and will do vital signs on those
that he has not been assessed in over an clients—CORRECT: LPNs may administer oral
hour and he would like to order dinner—the new medications and do vital signs
admission requires assessment but is not as much (4) The clients with IV medications are divided
of a priority as the client in answer choice 3 among the RNs—CORRECT: RNs administer
241. The Answer is 4 IV medications
The nurse knows that which of the following terms (5) An LPN is assigned to insert a urinary catheter—
BEST defines the multidisciplinary care planning CORRECT: inserting a urinary catheter is
for a young adult with breast cancer? within the scope of practice of an LPN
Reworded Question: What is the term for 243. The Answer is 4
multidisciplinary The nurse is educating new nursing staff members
team meetings to plan client care? about safety on the pediatric unit. Which of the
Strategy: Think about reaching common goals, following
reviewing client information, and communication. comments by one of the new staff members BEST
Needed Info: You should understand the definition demonstrates that teaching has been successful?
of terms surrounding multidisciplinary team Reworded Question: Which statement is correct
interaction about childhood safety practices in and out of the
and the purpose for collaboration versus case hospital setting?
management. Strategy: Think prevention and safety with children.
TChoen tPernatc tRiecvei eTwes atnd Practice Needed Info: Begin by reviewing pediatric growth
for the NCLEX-RN ® Exam and development including age-related safety
422 regulations
for each age group. Review prevention strategies elderly client with vertigo. Which of the following
for children including car seat safety, poison interventions demonstrates that the nurse
prevention, fall risks, and preventing injuries. understands
Category: Evaluation/Safe and Effective Care the symptoms of vertigo?
Environment/ Reworded Question: What actions can the nurse take
Safety and Infection Control to make this client safe?
(1) “A toddler may be taken to the car in a wheelchair Strategy: Think safety risks for the elderly.
when discharged and, after that, the Needed Info: You should know the definition of vertigo
hospital is not responsible for how the child is and a basic understanding of the head-spinning
transported in the family car.”—toddler should feeling that this client may be experiencing. Basic
not be discharged to parents if they do not have safety for fall prevention is helpful, and knowing
an appropriate car seat; laws state that toddler that vertigo originates in the inner ear will help guide
must be in child-appropriate car seat and the your critical thinking skills.
hospital should be held accountable for making Category: Implementation/Safe and Effective Care
certain that when a child is discharged it is with Environment/Safety and Infection Control
the proper car restraint (1) The nurse recognizes this client as at risk for
(2) “School-aged children do not require booster falls and relays this to the other team members—
seats if they are less than 80 pounds, and they do CORRECT: the nurse should alert staff and the
not require bicycle helmets when they are more multidisciplinary team about fall risk precautions
than 80 pounds.”—school-aged children less to help prevent injury during a vertigo episode
than 80 pounds should still be in an appropriate with this client
booster seat or car restraint until they reach 4 (2) The nurse allows the client to ambulate alone—
foot 8 inches and weigh over 80 pounds; children an elderly client with vertigo should not ambulate
of all ages and sizes should wear bicycle helmets alone and should be instructed to call for
(3) “Medications can be left at the bedside for assistance to avoid falling
pediatric (3) The nurse encourages the client to sit up quickly
clients, and the parent will dispense when before standing—instructing a client to sit up
Practice Test Explanations quickly can make the vertigo worse; encourage
423 the client to sit up slowly and maintain that position
Practice Test Answers Taensdt needed.”— until the vertigo subsides
medications should not be left at the (4) The nurse makes no change in routine precautions
bedside of any client because vertigo is an expected symptom for
(4) “All medications and cleaning supplies must be this age group—making no change in the care of
locked in a child-proof cabinet on the pediatric a client with vertigo is reckless and could lead to
unit at all times.”—CORRECT: all medications injury
and cleaning supplies should be locked in a childproof 246. The Answer is 2
cabinet The nurse is preparing to change a sterile surgical
244. The Answer is 2 dressing. While repositioning herself, the client
The nurse knows that which of the following is the touches a sterile sponge. Which of the following
MOST appropriate infection control method when is the BEST nursing intervention to promote and
caring for clients on a surgical unit? maintain surgical asepsis?
Reworded Question: How can staff members prevent Reworded Question: What is the best way to handle
the spread of infection to a fresh surgical client? the situation when a sterile field has been
Strategy: The word MOST indicates there may be contaminated?
more than one possibility but that only one is the Strategy: Think about the principles of surgical
best choice. asepsis.
Needed Info: Review how infection is spread, the TChoen tPernatc tRiecvei eTwes atnd Practice
elements for the NCLEX-RN ® Exam
of the chain of infection, and universal precautions. 424
Category: Implementation/Safe and Effective Care The Practice Test
Environment/Safety and Infection Control Needed Info: You must understand the basis for
(1) Hand hygiene before charting or using the surgical
keyboard— asepsis and how to set up and maintain a sterile
appropriate but not the most effective field.
directly related to the surgical client Category: Implementation/Safe and Effective Care
(2) Handwashing before and after contact with Environment/Safety and Infection Control
each client—CORRECT: washing hands before (1) Reassure the client and continue with the dressing
entering the room and after contact with the client change—if the sterile field has been contaminated,
is the most effective way to prevent infection a new sterile field must be set up
(3) Use of gloves—using gloves is helpful but does (2) Reassure the client but instruct her to keep her
not take the place of handwashing hands free from the sterile field. Clear the
(4) Use of gowns with each client—gowns may be contaminated
appropriate for certain bacteria but do not take area, rewash hands, and assemble
the place of proper handwashing another sterile field to start over—CORRECT:
245. The Answer is 1 reassurance and education for the client can be
The nurse on the adult medical unit assesses an done while setting up a new sterile field; it is for
the safety of the client and to prevent infection (2) “They do make motor wheelchairs. Maybe we
(3) Have the client sterilize her hands so the episode can look into that.”—although a motor chair
is not repeated—skin cannot be sterilized, but may increase mobility, it may not be enough
the client should cleanse her hands for her own to allow the client to live alone; this would be
protection determined by a complete physical assessment
(4) Continue with the dressing change, avoiding and home visit
the items that came in contact with the client’s (3) “How often do you have episodes of diarrhea?”—
hands—if the sterile field has been contaminated, clients with MS are prone to constipation due to
a new sterile field must be set up sluggish peristalsis, not diarrhea
247. The Answer is 3 Practice Test Explanations
The nurse is caring for a client who has just undergone 425
an open laparotomy with ileostomy. The nurse Practice Test Answers Taensdt (4) “What kinds of
knows that client education should include which of meals would you like prepared
the following topics? while in the hospital?”—meal preparation would
Reworded Question: What are the principles that not be the first priority in an assessment
nursing care focuses on with ileostomy care? 249. The Answer is 3
Strategy: Think about stoma and wound care. The nurse is educating a client who is scheduled for
Needed Info: An ileostomy is an opening at the distal surgery in the near future about autologous blood
end of the small intestine. Stool will be loose and donation. Which of the following statements by the
acidic and can be irritating to the skin. Skin care, client indicates the teaching has been successful?
hygiene, diet, and fluid intake will be educational Reworded Question: What are the criteria for
topics. autologous
Category: Planning/Physiological Integrity: Basic blood donations prior to surgery?
Care and Comfort Strategy: Think client’s own blood.
(1) Constipation management—will not be an Needed Info: Know the criteria for blood donation
important topic to discuss because it is rare with and the different blood types. Understand transfusion
an ileostomy reactions and the life cycle of the red blood cell.
(2) Limited activity—activity should not be limited Know the proper procedures for both blood
after healing from the surgery; education should transfusion
center on returning to normal activity with the and donation.
ileostomy Category: Assessment/Physiological
(3) Stoma care and skin care—CORRECT: education Integrity/Pharmacological
should include stoma care, skin care, diet, and Parenteral Therapies
fluid intake, and how to safely remove a food (1) “I cannot donate blood for myself because of my
blockage from the stoma age.”—not a true statement
(4) Urinary incontinence—not a symptom associated (2) “I will not need a transfusion after major
with an ileostomy surgery.”—
248. The Answer is 1 you cannot assume that blood won’t be
The nurse is assessing a client who has multiple needed during or after a major surgical procedure
sclerosis (3) “I can be an autologous blood donor 6 weeks
and can no longer live alone due to immobility. before my surgery in the event that I may need a
Which of the following statements by the nurse transfusion.”—CORRECT: to be an autologous
demonstrates donor (donating your own blood), donation
her understanding of this client’s impaired should be done 4–6 weeks before your surgery
physical mobility? and when blood counts are within normal limits
Reworded Question: What are some of the (4) “I cannot get a transfusion reaction with my own
complications blood.”—blood transfusion reactions are rare
of immobility? with one’s own blood but they are possible and
Strategy: Think about how multiple sclerosis (MS) the risks should be discussed with the client
affects each body system and prioritize. 250. The Answer is 4
Needed Info: Understand the effects of immobility The RN is providing education to the LPN about
caused by neuromuscular disorders. Apply that administrating oral medications. Which of the
knowledge to the systems of the body. Look first at following
the cause of the immobility and the length of time it statements demonstrates to the RN that the
has occurred, and then look for complications and LPN understands the teaching?
symptoms. Reworded Question: Which statement provides
Category: Assessment/Physiological Integrity/Basic correct information about administering oral
Care and Comfort medication?
(1) “Do you have any areas of pain, pressure, or Strategy: Think about what extended-release tablets
open ulcers on your legs, ankles, or hips?”— are designed to do and the effects of crushing them.
CORRECT: an assessment of a client with MS Know the responsibility of administering oral
should include looking for pressure areas and medication
skin breakdown; look for areas of injury due in the proper dose.
to falls and potential areas of open wounds for Needed Info: Know your medications and the
sources of infection; look for bruising, atrophy, actions, length of effect, and the purpose for each
and signs of thrombosis type of medication.
Category: Analysis/Physiological Reworded Question: What are the causes of newborn
Integrity/Pharmacological jaundice after discharge from the hospital?
and Parenteral Therapies Strategy: Think breastfeeding and hydration. The
(1) “Giving oral medications is simple and requires phrase “MOST appropriate” indicates that more
little training.”—statement indicates a lack of than one answer may be correct.
knowledge regarding the training and responsibility Needed Info: You should have a basic understanding
of administering medication of newborn feeding and care, breastfeeding issues
(2) “If the client can’t swallow a time-released tablet, for the new mom, and the significance of jaundice in
I will crush it.”—time-released medications the newborn. Breastfeeding moms need to maintain
should not be crushed; if a client cannot take a good hydration and pay attention to how much milk
medication prescribed, the nurse should call the they are producing for their newborn. You should
practitioner and discuss the possibility of a different understand the risks of dehydration for the infant
medication and jaundice if it remains a long-term issue.
(3) “It is okay to crush the client’s extended-release Category: Assessment/Physiological Integrity/
tablet to put it in applesauce.”—extended-release Reduction of Risk Potential
tablets should not be crushed (1) “How often are you nursing your baby?”—this
(4) “I can break this scored tablet for the partial dose should be the second question after finding out
ordered for the client.”—CORRECT: scored tablets how the baby is being fed
can be broken for a partial dose (2) “Are you breastfeeding or bottle feeding?”—
251. The Answer is 2 CORRECT: it is important to know how the baby
The nurse is reviewing the lab work of a pediatric is being fed; breastfed babies may have jaundice
client for a few days due to lack of milk production or
admitted for chemotherapy treatment. For which not nursing well; bottle-fed infants who are taking
of the following laboratory values should the nurse the appropriate amount of formula tend to
call the physician? not be as jaundiced after discharge; determining
Reworded Question: Which laboratory values are how the infant is fed is important to assess the
most affected by chemotherapy? risk for dehydration
Strategy: Think anemia. (3) “Do you know what your baby’s bilirubin level
Needed Info: Be familiar with the normal ranges of was before discharge?”—the mother may know
routine laboratory values. WBC ranges from 5–10 the bilirubin level at discharge but this is not the
although chemotherapy will reduce the level to most important question; it may be part of the
almost zero between doses. Hemoglobin levels are conversation, but the most important information
lower in anemia, which decreases the ability for is about the way the baby is being fed
blood to carry oxygen to tissue. If the doses are too (4) “Has your baby been seen by the pediatrician?”—
low, transfusions may be ordered before the next pediatrician appointment is
chemotherapy. important but not before finding out how the
Chemotherapy destroys both good blood cells baby eats and the hydration state of the infant
and cancerous cells, so blood counts are important 253. The Answer is 1
when evaluating the risks of chemo and the ability The nurse is assessing a young adult client who
for the client to tolerate the next round. begins to have a grand mal seizure for the first time.
TChoen tPernatc tRiecvei eTwes atnd Practice Which of the following actions should the nurse do
for the NCLEX-RN ® Exam FIRST?
426 Reworded Question: What is the nurse’s first response
The Practice Test to a client having a seizure?
Category: Analysis/Physiological Integrity/Reduction Strategy: Think ABCs and priority—this suggests
of Risk Potential that all the answers may be correct; however, one
(1) BUN 5, creatinine 0.7—these BUN and creatinine takes precedence.
values are normal; would not postpone chemotherapy Needed Info: Seizures can be life threatening if the
(2) WBC 0, hemoglobin 2—CORRECT: a zero airway becomes obstructed. Know the types of
white blood cell count and a hemoglobin level of seizures,
2 would most likely postpone chemotherapy due treatments, and the emergency care of a client
to the severity of the anemia who seizes. Understanding the causes of seizures
(3) Hemoglobin 9.5, WBC 14—a normal hemoglobin can assist you in planning for the follow-up care of
level and an elevated WBC would be expected a client who has a seizure. First-time seizures often
when taking chemotherapy and would not be a require a neurological workup including a CT scan
reason to postpone the dose unless the client was or MRI of the brain. A seizing client must be protected
experiencing other symptoms of sepsis from aspiration and choking by placing the
(4) Magnesium 2—this is a normal magnesium client on her side or turning her head.
value Category: Implementation/Physiological Integrity/
252. The Answer is 2 Physiological Adaptation
The nurse is doing a follow-up telephone call with (1) Protect the client’s airway—CORRECT: the priority
a new mother regarding her newborn. The mother is to protect the client’s airway and prevent
states the baby’s eyes look yellow. Which of the aspiration; you can turn her head to the side or
following turn the client on her side
is the MOST appropriate response by the (2) Restrain the client—attempts to retrain can
nurse? cause injury; it is better to move objects away,
Practice Test Explanations (2) The headache becomes worse and the client
427 shows a decrease in the level of consciousness—
Practice Test Answers Taensdt lower the client to CORRECT: an increase in headache and
the floor, or protect the client a decrease in level of consciousness requires medical
from hitting the side rails if in bed intervention; choices are repeating CT scans,
(3) Record the length of the seizure—you must giving steroids to help with brain tissue swelling,
record the length of the seizure but timing can be inserting an ICP monitor, among other choices
done while you protect the airway; airway always (3) The client vomits one time and continues to have
comes first a slight headache—vomiting once or twice is a
(4) Report this to the physician—you must report normal response to a minor head injury
the seizure, but you would not leave a seizing (4) The client has no headache but has little memory
client to report it; you may call others for help; of the incident—amnesia affects are normal with
someone else may call the physician, but the first a minor head injury
responder stays with the client to protect the airway 256. The Answer is 3
254. The Answer is 3 The nurse is admitting a client to the neurology
The nurse is educating a client with a history of unit at the medical center. The nurse has arrived at
hyponatremia on diet choices. Which of the following the advance directive section of the initial nursing
statements by the client BEST indicates the assessment flowsheet. The nurse assisting with the
teaching was successful? admission would intervene if the primary admitting
Reworded Question: What foods promote adequate nurse made which of the following statements to the
sodium retention? client?
Strategy: Think about nutrition and the food pyramid. TChoen tPernatc tRiecvei eTwes atnd Practice
Needed Info: Know the symptoms of hyponatremia. for the NCLEX-RN ® Exam
Know the food sources best offering those nutrients. 428
Helping a client to understand the symptoms and The Practice Test
risks of low sodium may encourage the client to be Reworded Question: What are the nurse’s
more compliant with diet choices. responsibilities
Category: Planning and Implementation/Physiological when asking clients about advance directives?
Integrity/Physiological Adaptation Strategy: Think about each statement. Does it
(1) “I should maintain a low-sodium diet.”—a describe an inappropriate statement made by the
lowsodium nurse regarding advance directives?
diet is not indicated for a client with a Needed Info: The Centers for Medicare and Medicaid
history of hyponatremia (CMS) require that clients are asked about their
(2) “I can drink as much beer as I want to.”—beer is wishes regarding advance directives upon admission
a diuretic beverage whose consumption should to an inpatient hospital setting.
be limited by a client with a history of hyponatremia Category: Implementation/Safe and Effective Care
(3) “I should avoid caffeine.”—CORRECT: caffeine Environment/Management of Care
is a diuretic that a client with a history of (1) “Do you have someone who would be a surrogate
hyponatremia decision maker for you if you were unable to
should avoid make decisions for yourself?”—this is an appropriate
(4) “I should drink a lot of water.”—excess water statement by the nurse
consumption increases risk of hyponatremia (2) “Are you familiar with what an advance directive
255. The Answer is 2 is?”—this is an appropriate statement by the
The nurse is caring for an alert and oriented teen nurse
with a head injury who complains of a slight headache. (3) “I should find out if you want an advance directive,
Which of the following symptoms exhibited by but you seem tired and confused so I will
the client would require immediate intervention by ask you later.”—CORRECT: this is an inappropriate
the nurse? statement by the nurse; unless the client is
Reworded Question: What are the symptoms of unable to comprehend due to a lack of capacity,
increased intracranial pressure? the client should be asked about advance directives
Strategy: Think of the risks of a brain hemorrhage, upon admission
Glasgow coma scale, and intracranial pressure signs (4) “Let me tell you a little bit about what an advance
and symptoms. directive is so that you can decide if you want one
Needed Info: Understand the symptoms of a mild set up.”—this is an appropriate statement by the
concussion and more severe head injuries. Amnesia nurse
may be part of a mild head injury, but even mild 257. The Answer is 4
injuries The nurse is caring for a client on the medical/surgical
can be a risk for intracranial bleeding. Know the unit who is receiving an intravenous insulin drip
types of brain bleeding that can occur and what to due to severe uncontrollable episodic hyperglycemia.
look for. After several hours of administering the insulin
Category: Assessment/Physiological and monitoring blood glucose levels regularly, the
Integrity/Physiological glucose levels are normalizing. The physician orders
Adaptation the nurse to maintain the IV insulin drip despite the
(1) The client complains of a continued headache nurse’s concerns. Which of the following actions by
and becomes drowsy—headache and drowsiness the nurse is the MOST appropriate?
are common with a minor head injury Reworded Question: What is a correct behavior?
Strategy: Think about the consequences of each employed by the home care agency—CORRECT:
action. the nurse should utilize the resources
Needed Info: Nurses have a duty and ethical available within the home care agency
obligation (2) The nurse should call 911 for an emergency
to advocate for their clients even if they are in response due to concerns for safety—this is not
disagreement the best action because the nurse is not concerned
with the physician about a client’s clinical status. about an immediate risk of safety
The ultimate obligation is to the client and to providing (3) The nurse should call the client’s community
appropriate and safe care. A physician’s order center for advice—this is not the best action;
alone will not protect a nurse’s license or integrity. case managers specialize in this
Category: Implementation/Safe and Effective Care (4) The nurse should call the client’s neighbors to
Environment/Management of Care ask them to look in on the client—this is not the
(1) The nurse should explain the procedure of best action by the nurse due to confidentiality
administering 259. The Answer is 1
the insulin intravenously to the client— The new staff nurse working on the intensive care
this is good practice by the nurse but not the best unit is concerned about her client’s status. The client
answer given the circumstances of the scenario; has continued to decline throughout the shift.
this should be done before administering the drug The client’s blood pressure, heart rate, and oxygen
(2) The nurse should maintain the intravenous saturation have progressively dropped in a relatively
medication short period of time. The nurse inquires with the
according to the physician’s orders—the charge nurse assigned to that shift. The charge nurse
nurse still has an obligation to advocate for her says “Don’t worry, the client will be fine, he always
client if her clinical judgment differs from that does that.” Which of the following actions should
of the physician’s; a physician’s order alone is not the nurse take?
enough reason to maintain a medication Reworded Question: What is the most appropriate
(3) The nurse should wait until the next blood glucose action for the nurse to take?
level check is due and make a decision then Strategy: Determine the outcome of each answer
about next steps—this is an inappropriate action choice.
because the client’s blood glucose could dramatically Needed Info: The nurse caring for the client has the
drop in this time period ultimate responsibility and ethical obligation to
(4) The nurse should contact the nursing supervisor act on behalf of the client. If the new nurse is
and possibly the supervisor of the physician concerned
who ordered the medication—CORRECT: this for the client’s status, and the charge nurse
is an appropriate action by the nurse to prevent is not inducing actions that support the appropriate
the client’s blood glucose levels from dropping provision of care that the client needs, the new nurse
dramatically must escalate up the chain of command to enlist the
258. The Answer is 1 support of supervisors and leadership to help the
The home care nurse is caring for an elderly client client.
who lives alone. The nurse notices that the client is Category: Implementation/Safe and Effective Care
beginning to show signs of failure to thrive at home Environment/Management of Care
and has no family to assist him. The nurse is unsure (1) The nurse should call the nursing supervisor on
how long this client will be able to remain in his duty to assist—CORRECT: the nurse should
home alone. Which of the following is the next step escalate up the chain of command to advocate
the nurse should take based on this assessment? for the client
Reworded Question: What is the appropriate action (2) The nurse should wait and see how the client
by the nurse? does—this is incorrect because the client is
Strategy: Think about each response. exhibiting serious symptoms that could represent
Needed Info: The home care nurse has other a grave diagnosis
resources available to assist with the care of clients. (3) The nurse should agree with the charge nurse
Practice Test Explanations because that nurse has more experience—this is
429 incorrect because the nurse must trust her own
Practice Test Answers Taensdt Home care clinical judgment even if it is in conflict with a
agencies typically employ case managers, more senior nurse
social workers, physical therapists, and other (4) The nurse should discuss this with other nurses
health care providers to assist their clients with their on the unit—this might be an appropriate strategy,
activities of daily living and care needs. Case but a supervisor can provide more immediate
managers assistance
can assist with proper placement of clients who 260. The Answer is 3
require alternative levels of care and support. Case The nurse on a busy surgical unit has just received
managers are aware of services available in report from the previous shift on the clients assigned
communities to that shift. Which of the following clients should
and long-term care or rehabilitation facilities. the nurse see FIRST?
Category: Planning/Safe and Effective Care Reworded Question: Which client is least stable?
Environment/ Strategy: Think ABCs.
Management of Care Needed Info: The most unstable client should be
(1) The nurse should consult with the case manager seen first. The most urgent client needs should be
attended to first. appropriate?
Category: Analysis/Safe and Effective Care Reworded Question: What behavior is acceptable?
Environment/ Strategy: Consider each answer.
Management of Care Needed Info: Although electronic medical records
(1) A young adult client who fractured his arm while offer certain amounts of protection of protected
playing football, had surgical repair of the fracture, health information (PHI), the Health Insurance
and is awaiting discharge—client is stable Portability
(2) A middle-aged client recovering from a knee and Accountability Act (HIPAA) mandates
replacement who is currently on the continuous that other necessary safeguards be put in place.
passive motion machine with the physical therapist— Category: Analysis/Safe and Effective Care
client is stable and not the first priority Environment/
TChoen tPernatc tRiecvei eTwes atnd Practice Management of Care
for the NCLEX-RN ® Exam (1) After each use of the computer and upon leaving
430 the client room, log off from the computer—
The Practice Test CORRECT: this is the most appropriate answer
(3) A middle-aged client recovering from abdominal because the rooms are not completely private
surgery who is complaining of wheezing and has and leave the information vulnerable to be seen
a new oxygen requirement—CORRECT: a new by others
oxygen requirement is concerning postoperatively (2) After each use of the computer and upon leaving
and could indicate a complication of the the client room, face the computer away from
surgery where visitors would be able to see the screen—
(4) An elderly client 1-day post-op for a hip this is not the most appropriate action because it
replacement is still possible for visitors to see PHI
whose blood pressure is elevated—you (3) The nurse should not be concerned about the
would want to attend to the blood pressure, but security
based on the facts given, it is not as urgent as the of the information because there is a single
client recovering from abdominal surgery computer for each client and therefore no risk of
261. The Answer is 2 the information being seen—not enough security
On a medical/surgical unit, each nurse is paired with (4) The nurse should pull the curtain to cover the
nursing assistive personnel (NAP) for the night shift. computer screen so that visitors cannot view it—
The nurse should assign which of the following clients not enough security
to the NAP? 263. The Answer is 2
Reworded Question: Which client is an appropriate The nurse is working on a unit that is equipped with
assignment for the NAP? electronic medication administration processes.
Strategy: Think about the skill level involved in each This includes a computer at the bedside that allows
client’s care. for scanning a bar code on the medication order,
Needed Info: NAPs are considered unlicensed the medication label, and the client’s identification
assistive band. Which of the following is the BEST method
personnel. They assist clients with activities of for the nurse to practice regularly?
daily living (ADL). They cannot perform activities Practice Test Explanations
that require nursing judgment or assessment. 431
Category: Planning/Safe and Effective Care Practice Test Answers Taensdt Reworded
Environment/ Question: What is a best practice for nurses
Management of Care who work with medication administration bar-coding?
(1) A middle-aged client receiving chemotherapy Strategy: Consider each answer.
and complaining of nausea and vomiting— Needed Info: Electronic medication administration
requires nursing judgment/assessment programs such as scanners that use bar-coding
(2) A middle-aged client who is an unstable diabetic encourage safer practices and promote the reduction
requiring a blood glucose level check—CORRECT: of errors in the medication administration process.
although the client is considered unstable, However, a machine does not take the place of
the check can be done by the NAP; the nurse can nursing
then subsequently make a decision about how to clinical judgment. Appropriate nursing judgment
react to the level appropriately and practices are paramount.
(3) An elderly client complaining of pain from restless Category: Analysis/Safe and Effective Care
leg syndrome—this requires a nurse to assess Environment/
the pain Management of Care
(4) A young adult client recovering from a drug (1) The nurse should rely solely on the bar-coding
overdose requesting to leave the unit against scanner because it promotes safer medication
medical advice—not an appropriate assignment administration practices—although bar-coding
for the NAP scanners promote safer medication administration
262. The Answer is 1 practices, they do not replace the judgment
The nurse is working on a state-of-the-art nursing of the nurse
unit with completely electronic medical records. The (2) The nurse should rely on a combination of nursing
rooms are semi-private, with two clients to a room, judgment and decision-making along with
and equipped with a computer for each client. Which the computerized system—CORRECT: computers
of the following actions by the nurse is the MOST do not take the place of nursing judgment
(3) The nurse should never give a medication that choice.
a bar-coding system scans as “incorrect medication”— Needed Info: As a measure of tracking and of
this is not true; computerized systems preventing
can have glitches and improper codes inadvertently future events, the nurse is required to report
in the system; the nurse should confirm events or injuries that are not expected with the
the order, the medication, and the client, and TChoen tPernatc tRiecvei eTwes atnd Practice
then make the decision to administer or not for the NCLEX-RN ® Exam
based on nursing judgment 432
(4) The nurse should override any medication that The Practice Test
the machine scans as “incorrect medication” typical provision of care. When reporting, the nurse
and administer it—this may be true in certain should state events factually and objectively.
situations, but there is not enough information Category: Implementation/Safe and Effective Care
provided to determine if that is the case Environment/Safety and Infection Control
264. The Answer is 3 (1) The nurse should file an incident report stating
The nurse is administering medications to a client “Client fell, no injury noted.”—not an appropriate
on an inpatient psychiatric unit. The client states “I answer because the nurse did not witness the
don’t usually take a pink pill” when the nurse gives client fall
a cup holding 4 different pills to the client. Which of (2) The nurse should file an incident report stating
the following is the MOST appropriate response by “Client fell on floor.”—not an appropriate answer
the nurse? because the nurse did not witness the client fall
Reworded Question: How should a nurse handle a (3) The nurse should document the event only in
client’s the client’s medical record and not in an incident
concerns about medications? report—although the event should be documented
Strategy: Consider each answer. in the client’s medical record, the event should be
Needed Info: The Joint Commission (TJC) promotes documented in an incident report as well
the practice of medication reconciliation, safety of (4) The nurse should file an incident report stating
using medications, and involving the client as a “Client found on floor. Client stated ‘I fell.’
partner Assessment completed, no injury noted, physician
in the client’s care. The Institute of Medicine notified.”—CORRECT: this is the best answer
(IOM) also promotes partnering with clients. that states the facts objectively
Category: Implementation/Safe and Effective Care 14: Essentials for International Nurses
Environment/Safety and Infection Control Sample Questions
(1) The nurse checks the medication administration Directions: Carefully read the question and all answer
record, determines it is correct, and tells the client choices. Examine each answer choice
to take the medication—not an appropriate and determine whether it is an appropriate response.
response; the nurse is not including the client as Indicate your decision in the column
a partner in the client’s care labeled Correct/Incorrect and give the reason for your
(2) The nurse discounts the client’s concern because choice.
he is a psychiatric client and doesn’t know any Question
better—being a psychiatric client does not Correct/
automatically Incorrect Reason
make the client unable to participate in 1. A client has been hospitalized for 2 days for
his or her own care treatment of hepatitis A. When the nurse enters
(3) The nurse asks the client for a list of medications the client’s room, he asks the nurse to leave him
he routinely takes, and tells the client that alone and stop bothering him. Which of the following
she will review and confirm the order with the responses by the nurse would be MOST
physician—CORRECT: best response, validates appropriate?
client’s concern (1) “I understand and will leave you alone for
(4) The nurse tells the client that sometimes drugs now.”
come in different colors depending on what (2) “Why are you angry with me?”
pharmacy they come from—not an appropriate (3) “Are you upset because you do not feel
response even though it may be true; still need to better?”
confirm it is the appropriate drug and alleviate (4) “You seem upset this morning.”
the client’s concerns 2. A client states she is afraid to have her cast
265. The Answer is 4 removed from her fractured arm. Which of
The nurse is working at a skilled nursing facility. the following would be the MOST appropriate
The nurse enters the client’s room and sees the client response by the nurse?
attempting to pull himself up from a sitting position (1) “I know it is unpleasant. Try not to be
on the floor. The nurse inquires with the client as to afraid. I will help you.”
what happened. The client responds “I fell.” Which of (2) “You seem very anxious. I will stay with
the following should the nurse document in the you while the cast is removed.”
incident (3) “I don’t blame you. I’d be afraid also.”
report? (4) “My aunt just had a cast removed and she’s
Reworded Question: How does a nurse report an just fine.”
event? 460
Strategy: Determine the outcome of each answer The Licensure Process
Question 7. A client is admitted to the hospital with an
Correct/ abdominal mass and is scheduled for an exploratory
Incorrect Reason laparotomy. She asks the nurse admitting
3. A client comes to the clinic because she thinks her, “Do you think I have cancer?” Which of
she is pregnant. She tells the nurse that she wants the following responses by the nurse would be
the pregnancy terminated because she and her MOST appropriate?
husband do not want to have children, and then (1) “Would you like me to call your doctor
begins to cry. Which of the following statements so that you can discuss your specific
by the nurse would be the MOST appropriate? concerns?”
(1) “Are you upset because you forgot to use (2) “Your tests show a mass. It must be hard
birth control?” not knowing what is wrong.”
(2) “Why are you so upset? You’re married. (3) “It sounds like you are afraid that you are
There’s no reason not to have the baby.” going to die from cancer.”
(3) “If you’re so upset, why don’t you have the (4) “Don’t worry about it now; I’m sure you
baby and put it up for adoption?” have many healthy years ahead of you.”
(4) “You seem upset. Let’s talk about how 8. A client is admitted to the postpartum unit following
you’re feeling.” a miscarriage. The next day the nurse
4. A client is in the terminal stages of carcinoma of finds the woman crying while looking at the
the lung. A family member asks the nurse, “How babies in the newborn nursery. Which of the following
much longer will it be?” Which of the following approaches by the nurse would be MOST
responses by the nurse would be MOST appropriate? appropriate?
(1) “I cannot say exactly. What are your (1) Assure the woman that the loss was “for
concerns at this time?” the best.”
(2) “I don’t know. I’ll call the doctor.” (2) Explain to her that she is young enough to
(3) “This must be a terrible situation for you.” have more children.
(4) “Don’t worry, it will be very soon.” (3) Ask her why she is looking at the babies.
461 (4) Acknowledge the loss and be supportive.
14: Essentials for International Nurses 463
Question 14: Essentials for International Nurses
Correct/ Question
Incorrect Reason Correct/
5. A client is admitted to the hospital with a diagnosis Incorrect Reason
of manic-depressive disorder. The man 9. An elderly client is hospitalized with Alzheimer’s
approaches the nurse and says, “Hi, baby,” and disease. His daughter tells the nurse that caring
opens his robe, under which he is naked. Which for him is too hard, but that she feels guilty
of the following comments by the nurse would placing him in a nursing home. Which of the following
be MOST appropriate? statements by the nurse would be MOST
(1) “This is inappropriate behavior. Please appropriate?
close your robe and return to your room.” (1) “It is hard to be caught between taking
(2) “Please dress in your clothes and then join care of your needs and your father’s
us for lunch in the dining room.” needs.”
(3) “I am offended by your behavior and will (2) “Would you like me to help you find a
have to report you.” nursing home?”
(4) “Do you need some assistance dressing (3) “Don’t feel guilty. The only solution is to
today?” place your father in a nursing home.”
6. A client is placed in Buck’s traction. The nurse (4) “I think I would feel guilty too if I had to
assigned to her prepares to assist her with a place my father in a nursing home.”
bath. The woman says, “You’re too young to 464
know how to do this. Get me somebody who The Licensure Process
knows what they’re doing.” Which of the following Read the explanations to these questions and make
responses by the nurse would be MOST sure that the American approach to
appropriate? these communications questions is understandable to
(1) “I am young, but I graduated from nursing you. It will help you to choose the right
school.” answer on the NCLEX-RN® exam.
(2) “If I don’t bathe you now, you’ll have Answers to Sample Questions
to wait until I’m finished with my other 1. (4) “You seem upset this morning,” is the correct
clients.” answer choice because the nurse seeks
(3) “Can you be more specific about your to verbally validate the client’s behavior rather than
concerns?” simply respond to the behavior.
(4) “Your concerns are unnecessary. I know This response promotes the nurse-client relationship
what I’m doing.” by encouraging the client
462 to share his feelings with the nurse.
The Licensure Process (1) “I understand and will leave you alone for now,” is
Question not the best approach because it
Correct/ does not promote further communication between the
Incorrect Reason nurse and the client about
how the client is feeling. In order to interpret this inappropriate in terms of American therapeutic
client’s behavior, the nurse must communication. This response
first validate it with the client. is harsh, presumptive, and assumes that the purpose
(2) “Why are you angry with me?” is incorrect. The of every marriage is to have
nurse is drawing a conclusion about children. This is not always the case in American
the client’s behavior. This type of action is too culture. With this response, the
confrontational. “Why” questions nurse does not attempt to verify the reason for the
are considered nontherapeutic. client’s tears, thereby discouraging
(3) “Are you upset because you do not feel better?” is further conversation about what the client is actually
not the best choice. The nurse is experiencing.
drawing a conclusion about the client’s behavior (3) “If you’re so upset, why don’t you have the baby
without validating it first. This and put it up for adoption?” is also
type of response may also belittle the client’s actual inappropriate. This is a value-laden assumption
concerns. placing positive value on adoption.
2. (2) “You seem very anxious. I will stay with you Again, the nurse fails to explore with the client the
while the cast is removed,” is the best reason for the client’s tears,
response because the nurse responds to the client’s thereby discouraging further communication. The
feelings of fear. This is consistent nurse is also offering advice.
with therapeutic communication used in American 4. (1) “I cannot say exactly. What are your concerns
nursing. This response at this time?” is the most appropriate
also provides an additional opportunity for the nurse response because it is unclear why the family member
to remain with the client in a has approached the nurse at
supportive capacity, enhancing the nurse-client this point. Perhaps the client is in pain and the family
relationship. member wants to discuss it
(1) “I know it is unpleasant. Try not to be afraid. I will with the nurse. This response allows for that
help you,” is not the best response. possibility. This response is also direct
It is not clear what concerns the client has about this and factually correct.
procedure. The nurse should (2) “I don’t know. I’ll call the doctor,” is not the most
establish this before responding. The nurse falsely appropriate response. It shifts
reassures the client by saying, the focus of responsibility from the nurse to the
“I will help you.” Because you do not know the nature physician, which prevents a nursefamily
of the client’s concerns, you member relationship from developing.
cannot honestly offer help. (3) “This must be a terrible situation for you,” is not
(3) “I don’t blame you. I’d be afraid also,” is not the the most appropriate response.
correct response because the nurse It is a value-laden statement that fails to explore the
shifts the focus of the conversation from the client to family member’s reason for
the nurse. This sets up a barrier approaching the nurse.
to further communication. The nurse concedes the (4) “Don’t worry, it will be very soon,” is inappropriate
issue too quickly, leaving because the nurse offers the family
the source of the client’s fear unknown. member false reassurance. It also offers advice by
(4) “My aunt just had a cast removed and she’s just telling the family member not
fine,” is not the best choice. The to worry. This statement is demeaning and may sound
focus of the conversation is shifted from the client to as if the nurse is too busy to
the nurse’s aunt, who is of no discuss the family member’s concerns.
concern to the client. This response fails to explore the 5. (1) “This is inappropriate behavior. Please close
source of the client’s anxiety your robe and return to your room,” is the
and sets up a block to further communication. correct answer choice. This statement by the nurse
3. (4) “You seem upset. Let’s talk about how you’re responds to the client’s behavior,
feeling,” is the best answer to this sets limits on the behavior, and directs the client
question. This promotes the nurse-client relationship toward more appropriate
and illustrates therapeutic social behavior in the milieu. This statement rejects
465 the client’s behavior, not the
14: Essentials for International Nurses client as a person.
communication used in American nursing. The nurse (2) “Please dress in your clothes and then join us for
responds to the client’s feelings lunch in the dining room,” is incorrect.
in a nonjudgmental empathetic way. It ignores the behavior of the client exposing himself.
(1) “Are you upset because you forgot to use birth Instead it directs the
control?” is inappropriate because it client to dress and report to the dining room for lunch
places blame on the client. The nurse should not as though nothing has happened.
assume that the client “forgot” to This is inappropriate and nontherapeutic.
do something. This response also fails to respond to 466
the client’s feelings and does The Licensure Process
not encourage the client to discuss her concerns. (3) “I am offended by your behavior and will have to
(2) “Why are you so upset? You’re married. There’s report you,” is incorrect. It shifts
no reason not to have the baby” is the focus from the client to the nurse and the nurse’s
feelings. The nurse’s personal
feelings are irrelevant. Also, the nurse goes on to is inappropriate. The nurse is telling the client how she
threaten the client by reporting should feel, and then goes
him. This is nontherapeutic. 467
(4) “Do you need some assistance dressing today?” is 14: Essentials for International Nurses
incorrect. This question fails to on to offer false reassurance. This response fails to
respond to the client’s behavior. It is also a yes/no address or explore the actual
question, which is nontherapeutic. concerns of the client.
6. (3) “Can you be more specific about your 8. (4) “Acknowledge the loss and be supportive,” is
concerns?” is the correct answer. This is the the best answer choice. It promotes
best answer choice because it seeks to validate the the nurse-client relationship, and allows for the
client’s message. It is direct, not identification of feelings and the
defensive, and allows the client to express her point expression of sadness. The client is in an acute stage
of view. of grief. This type of response
(1) “I am young, but I graduated from nursing addresses this issue.
school,” is incorrect. It responds to only (1) “Assure the woman that the loss was ‘f or the
part of the message that the client sent to the nurse. best,’ ” is incorrect. This statement is
It assumes that the nurse knows insensitive to the client, offers false reassurance, and
what the client’s concerns are and agrees that there belittles the client’s most
is some problem associated with immediate concerns.
being too young. Further clarification is necessary in (2) “Explain to her that she is young enough to have
this situation. more children,” is inappropriate
(2) “If I don’t bathe you now, you’ll have to wait until because it is insensitive to the grief that the client is
I’m finished with my other clients,” experiencing. The nurse offers
is a nontherapeutic response. It fails to explore the false reassurance by telling the woman that she can
client’s concerns about the have other children.
nurse. It is an uncaring and punitive statement by the (3) “Ask her why she is looking at the babies,” is also
nurse that is inappropriate in incorrect. This is inappropriate
a nurse-client relationship. because it is a “why” question and because the woman
(4) “Your concerns are unnecessary. I know what I’m may become defensive when
doing,” is incorrect. The nurse answering such a question. This response also fails to
dismisses the client’s concerns by telling her that she respond to the client’s immediate
shouldn’t be concerned. The grief.
nurse should not tell a client how the client should be 9. (1) “It is hard to be caught between taking care of
feeling. It may sound as if the your needs and your father’s needs,” is the
nurse is trying to reassure the client by telling her that correct response. This is the most therapeutic
the nurse knows what he or response as it allows for continued
she is doing; however, the nurse has yet to validate development of a relationship with the family member
the concerns that underlie the of the client. This response
client’s statement. allows the nurse to explore and validate the
7. (2) “Your tests show a mass. It must be hard not daughter’s feelings about the nursing
knowing what is wrong,” is the correct home placement.
answer choice. This is the best answer choice because (2) “Would you like me to help you find a nursing
it responds to the client’s feelings. home?” is not the best answer choice.
It allows the client to continue to identify and express It is a yes/no question and doesn’t encourage
her concerns regarding discussion of the daughter’s feelings.
surgery, hospitalization, and the possibility of having (3) “Don’t feel guilty. The only solution is to place your
a potentially life-threatening father in a nursing home,” is not
illness. The nurse validates that the client has the best therapeutic response. The daughter’s
appropriate concerns and invites her concerns are minimized when the
to elaborate on them. nurse tells the daughter not to worry. Although it may
(1) “Would you like me to call your doctor so that you be true that the daughter has
can discuss your specific concerns?” done all that she can, this response cuts off an
This response is incorrect because it shifts the focus opportunity for further conversation
of responsibility from with the nurse.
the nurse to the doctor, thereby reducing the (4) “I think I would feel guilty too if I had to place my
possibility of developing an ongoing father in a nursing home,” is also
nurse-client relationship. incorrect. This statement is value-laden and
(3) “It sounds like you are afraid that you are going judgmental, and blocks any further
to die from cancer,” is inappropriate. communication between the nurse and the client’s
It fails to validate with the client that “dying from daughter. It is not important
cancer” is in fact the issue. The what the nurse thinks about the daughter’s decision,
nurse inappropriately concludes this on the basis of a nor is it the nurse’s role to
brief statement made by the make the daughter feel more guilty about her
client without giving the client a chance to elaborate. decision.
(4) “Don’t worry about it now; I’m sure that you have Language
many healthy years ahead of you,”
English is the predominant language spoken and course of study to help you earn your CGFNS®
written in the United States, and the certificate. To obtain information, please call
NCLEX-RN® exam is administered only in English. 1-800-527-8378. Outside the United States, please
With the exception of the medical terminology, call 1-212-997-5883 or log on to the website
the reading level of the NCLEX-RN® exam is that of a at kaplannursing.com.
junior in an American high Preparation for the NCLEX-RN® (National Council
468 Licensure Examination)
The Licensure Process Examination for International Nurses
school (11th grade). In order to be successful on the An internationally educated nurse must pass the
NCLEX-RN® exam, you must understand NCLEX-RN® exam in order to obtain a
English—and the terminology—as it is used in the license to practice as a registered nurse in the United
United States. States. Kaplan has a comprehensive
Vocabulary course and review products to help international
Vocabulary can be a challenge for international nurses nurses pass this exam. To obtain information,
on the NCLEX-RN® exam. Not only please call 1-800-527-8378. Outside the United
must you know what each word means, but States, please call 1-212-997-5883 or log
sometimes a word may have more than one meaning. on to the website at kaplannursing.com.
You need to be able to correctly identify words as they Kaplan English Programs
are used in context. Refer to the In addition to Kaplan Nursing programs, Kaplan also
NCLEX-RN® Exam Resources section in the back of offers English programs to help you
this book for some of the commonly improve your English skills and score on the TOEFL®.
found words on the NCLEX-RN® exam. Some other Kaplan’s English Programs were
ways to increase your vocabulary and designed to help students and professionals from
learn how the words are used in everyday English outside the United States meet their educational
include: and career goals. At locations throughout the United
• Talking with Americans States, international students
• Watching American movies and television take advantage of Kaplan’s programs to help them
• Reading American newspapers and magazines improve their academic and conversational
Abbreviations English skills, raise their scores on the TOEFL® and
Many of you are unfamiliar with the abbreviations other standardized exams, and
used in the United States. When studying, gain admission to the schools of their choice. Our staff
always look up unknown words in a medical and instructors give international
dictionary. Consult the NCLEX-RN® Exam students the individualized instruction they need to
Resources section in the back of this book for a list of succeed. The following sections provide
abbreviations used by nurses in American brief descriptions of some of Kaplan’s programs for
health care settings. non-native English speakers.
As an internationally educated nurse, you face special English Language Programs
challenges in preparing for the NCLEXRN Kaplan offers a wide range of English language
® exam. Following the tips and guidelines outlined in programs to help you improve your English
this book will increase your chances quickly and effectively, regardless of your current
of passing the NCLEX-RN® exam and will allow you to level. Each of our programs has a special
reach your career goals. focus, allowing you to direct your study in a way that
Kaplan Programs for International Nurses suits your particular language needs.
Knowing something about U.S. culture and how U.S. All of the essential language skills are covered, and
nurses fit into the overall health care your fluency and confidence will increase
industry is important for nurses trained outside the rapidly thanks to Kaplan’s communicative teaching
United States. If you are not from the method.
United States, but are interested in learning more TOEFL® and Academic English
about U.S nursing, wish to practice in the Kaplan’s world-famous TOEFL® course prepares
United States, or are exploring the possibilities of students for the TOEFL® iBT. Designed for
attending a U.S. nursing school for graduate high-intermediate to advanced-level English
study, Kaplan is able to help you. speakers, our course focuses on the academic
CGFNS® (Commission on Graduates of Foreign English skills you will need to succeed on the test. The
Nursing Schools) course includes TOEFL®-focused
Preparation for International Nurses reading, writing, listening, and speaking instruction
Many U.S. state boards of nursing require and hundreds of practice items similar
internationally educated nurses to obtain a to those on the exam. Kaplan’s expert instructors help
CGFNS® certificate before applying for initial licensure you prepare for the four sections of the
as a registered nurse. The certification TOEFL® iBT, including the Speaking section. Our
process requires that a candidate pass a two-part test simulated online TOEFL® tests help you
of nursing knowledge and demonstrate monitor your progress and provide you with feedback
English language proficiency on the TOEFL® exam. on areas where you require improvement.
Kaplan offers a comprehensive We will teach you how to get a higher score!
469 The Licensure Process
14: Essentials for International Nurses 470
Other Kaplan Programs
Since 1938, more than 3 million students have come expected
to Kaplan to advance their studies, prepare outcome?
for entry to American universities, and further their What is the
careers. In addition to the above best action for
programs, Kaplan offers courses to prepare for the the expected
SAT®, ACT®, GMAT®, GRE®, LSAT®, outcome?
MCAT®, DAT®, USMLE®, and other standardized Question?
exams both online and at locations Correct
throughout the United States. 3 Answer
Applying to Kaplan English Programs* 2
To get more information, or to apply for admission to 1
any of Kaplan’s programs for nonnative Read the
English speakers, contact us at: stem one
Kaplan English Programs time.
Phone: 1-800-818-9128 (within the United States) Read answer
Phone: +44 (0)20 7045 5000 (elsewhere) choices for
Website: kaplaninternational.com clues to topic.
*Kaplan is authorized under federal law to enroll Reword question
nonimmigrant alien students. using clues from
Test names are registered trademarks of their answer choices.
respective owners. Real World
FREE Services for International Students Correct
Kaplan now offers international students many Answer
services online—free of 1
charge! Students may assess their TOEFL® skills and Don’t use real-world
gain valuable feedback experience to
on their English language proficiency in just a few answer NCLEX-RN®
hours with Kaplan’s questions.
TOEFL® Skills Assessment. Log on to 2
kaplaninternational.com today. You have the
NCLEX-RN® time, the staff,
Exam Resources and the
Part 5 equipment.
NCLEX-RN® Exam Resources 3
The 10 charts below illustrate different paths you must Take care
choose from in order to correctly of the
answer NCLEX-RN® exam questions. The stepping client first.
stones stand for steps that you must 4
follow in order to find the correct answer for that The NCLEX-RN®
question type. Use the chart to refresh exam tests the
your memory with respect to the various steps for nurse’s judgment.
each type of question. Tear out this page Summary of cr itical thinking paths
and refer to it to practice using this book’s strategies Appen dix A
when answering practice NCLEX-RN® NCLEX-RN® Exam Resources
exam-style questions. 474
Correct Correct
Answer Answer
Read the Eliminate
stem. “Don’t
Identify worry.”
the topic. Read the Eliminate
answer “explore”
choices. answers.
Identify the Don’t ask
nursing concept “Why?”
contained in Eliminate
answer choices. “authoritarian”
4 answers.
3 Eliminate
2 “focus on
Answers? 1 the nurse”
Correct answers.
Expected Answer 5
Outcomes 34
12 2
What is the Eliminate
closedended and psychosocial.
questions. Correct
6 Answer
1 Are you trying
Therapeutic to prevent or
Communication promote?
Correct What are you
3 Answer trying to prevent
2 or promote?
Nursing 1 Think A&P.
Process 3
Recognize both 2
assess and Positioning 1
implement 123
answers. Correct
Read stem to Safety Answer
decide whether All answers
to assess or must be
implement. implementations.
Select best Try to answer
assessment or based on
implementation. knowledge;
Correct if you can’t...
Answer What will cause
Do not the client the least
delegate amount of harm?
assessment, 475
teaching, abduction – movement away from the midline
evaluation, abraded – scraped
or nursing acetonuria – acetone in the urine
judgment. adduction – movement toward the midline
Delegate tasks afebrile – without fever
that involve albuminuria – albumin in the urine
standard, ambulatory – walking
unchanging amenorrhea – absence of menstruation
procedures. amnesia – loss of or defective memory
Remember ankylosis – stiff joint
priorities: anorexia – loss of appetite
Maslow, ABCs, anuria – total suppression of urination
and stable vs. apnea – short periods when breathing has ceased
unstable arthritis – inflammation of joint
Delegate care asphyxia – suffocation
for stable atrophy – wasting
patients with auscultation, auscultate – to listen for sounds
expected bradycardia – heart rate lower than 60 beats per
outcomes. minute
4 Cheyne-Stokes respirations – increasing dyspnea
3 with periods of apnea
2 choluria – bile in the urine
Delegation 1 clonic tremor – shaking with intervals of rest
4 conjunctivitis – inflammation of conjunctiva
3 coryza – watery drainage from nose
2 cyanotic – bluish in color due to poor oxygenation
Maslow 1 defecation – bowel movement
Correct dental caries – decay of the teeth
Answer dentures – false teeth
Eliminate diarrhea – excessive or frequent defecation
psychosocial diplopia – double vision
answers. distended – appears swollen
“Does this diuresis – large amount of urine voided
make sense?” dorsal recumbent – lying on back, knees flexed and
Apply apart
ABCs. dysmenorrhea – painful menstruation
Recognize that dyspnea – difficulty breathing
answers are dysrhythmia, arrhythmia – abnormal heartbeat
both physical dysuria – painful urination
edematous – puffy, swollen lordosis – sway-back, convexity of spine
emaciated – thin, underweight manipulation, manipulate – to handle
emetic – agent given to produce vomiting menopause – cessation of menstruation
enuresis – bed-wetting menorrhagia – profuse menstruation
epistaxis – nosebleed metrorrhagia – variable amount of uterine bleeding
eructation – belching occurring frequently but at irregular intervals
erythema – redness moist – wet
eupnea – normal breathing monoplegia – paralysis of one limb
excoriation – raw surface mucopurulent – drainage containing mucus and pus
exophthalmos – abnormal protrusion of eyeball mydriasis – dilation of pupil
extension, extend – to straighten myopia – nearsightedness
fatigued – tired myosis – contraction of pupil
feigned – pretended nausea – desire to vomit
Nursing Terminology necrosis – death of tissue
Appendix B nocturia – frequent voiding at night
476 obese – overweight
NCLEX-RN® Exam Resources objective – able to be documented by other than
fetid – foul observation
fixed – motionless oliguria – scant urination, less than 400 mL per 24
flaccid – soft, flabby hours
flatus, flatulence – expulsion of gas from the orthopnea – inability to breathe or difficulty
digestive tract breathing while lying down
flexion – bending palliative – offering temporary relief
flushed – pink or hot pallor – white
Fowler’s position – semierect, knees flexed, head of palpation, palpate – to feel with hands or fingers
bed elevated 45–60 degrees 477
gavage – forced feeding through a tube passed into Nursing Terminology
the stomach paraplegia – paralysis of legs
glossy – shiny paroxysm – spasm or convulsive seizure
glycosuria – glucose in the urine paroxysmal – coming in seizures
gustatory – dealing with taste pediculi – lice
heliotherapy – using sunlight as a therapeutic agent pediculosis – lice infestation
hematemesis – blood in vomitus percussion, percuss – to strike
hematuria – blood in the urine persistent – lasting over a long time
hemiplegia – paralysis of one side of the body petechia – small rupture of blood vessels
hemoglobinuria – hemoglobin in the urine photophobia – sensitive to light
hemoptysis – spitting of blood photosensitivity – skin reaction caused by exposure
horizontal – flat to sunlight
hydrotherapy – using water as a therapeutic agent pigmented – containing color
hyperpnea – rapid breathing polyuria – excessive voiding of urine
hypertonic – concentration greater than body fluids profuse, copious – large amount
hypotonic – concentration less than body fluids projectile – ejected or projected some distance
infrequent – not often pronation – turning downward
insomnia – inability to sleep prone – on abdomen, face turned to one side
instillation – pouring into a body cavity prophylactic – preventative
intermittent – starting and stopping, not continuous protruding – extending outward
intradermal – within or through the skin pruritus – itching
intramuscular – within or through the muscle ptosis – drooping eyelid
intraspinal – within or through the spinal canal purulent drainage – drainage containing pus
intravenous – within or through the vein pyrexia – elevated temperature
involuntary, incontinent – unable to control bladder pyuria – pus in the urine
or bowels radiating – spreading to distant areas
isotonic – having the same tonicity or concentration radiotherapy – using x-ray or radium as a therapeutic
as body fluids agent
jackknife position – prone with hips over break in rales, crackles – abnormal breath sounds
table and feet below level of head rapid – quick
jaundice – yellow color rotation – to move in circular pattern
knee-chest position – in face-down position resting sanguineous drainage – bloody drainage
on knees and chest scanty – small in amount
kyphosis – humpback, concavity of spine semi-Fowler’s position – semi-erect, head of bed
labored – difficult, requires an effort elevated 30–45 degrees
lacerated – torn, ragged edged serous drainage – drainage of lymphatic fluid
lateral position – on the side, knees flexed Sims’ position – on left side, left arm behind back,
lithotomy position – on the back, buttocks near edge left leg slightly flexed, right leg slightly flexed
of table, knees well flexed and separated sprain – wrenching of a joint
lochia – drainage from the vagina after delivery stertorous – characterized by snoring
stethoscope – instrument used for auscultation bid – two times a day
strabismus – squinting; misalignment of the eyes BKA – below-the-knee amputation
stuporous – partially unconscious BLS – basic life support
subcutaneous – under the skin BMR – basal metabolic rate
subjective – observed BP – blood pressure
sudden onset – started all at once BPH – benign prostatic hypertrophy
superficial – on the surface only bpm – beats per minute
supination – turning upward BPR – bathroom privileges
suppurating – discharging pus BSA – body surface area
syncope – fainting BUN – blood urea nitrogen
syndrome – group of symptoms C – centigrade, Celsius
tachycardia – fast heartbeat, greater than 100 beats c – with
per minute Ca – calcium
tenacious – tough and sticky CA – cancer
thready – barely perceptible CABG – coronary artery bypass graft
tonic tremor – continuous shaking CAD – coronary artery disease
Trendelenburg position – flat on back with pelvis CAL – chronic airflow limitations
higher than head, foot of bed elevated 6 inches 480
tympanic – filled with gas NCLEX-RN® Exam Resources
urticaria – hives or wheals; eruptions on skin or CAPD – continuous ambulatory peritoneal dialysis
mucous membranes caps – capsules
vertigo – dizziness CBC – complete blood count
vesicle – fluid-filled blister CC – chief complaint
visual acuity – sharpness of vision cc – cubic centimeter
void, micturate – to urinate or pass urine CCU – coronary care unit, critical care unit
CDC – Centers for Disease Control and Prevention
479 CHF – congestive heart failure
Title CK – creatine kinase
Xxxx Cl – chloride
Common Medical Abbreviations CLL – chronic lymphocytic leukemia
Appendix C cm – centimeter
ABC – airway, breathing, circulation CMV – cytomegalovirus
abd. – abdomen CNS – central nervous system
ABG – arterial blood gas CO – carbon monoxide, cardiac output
ABO – system of classifying blood groups CO2 – carbon dioxide
ac – before meals comp – compound
ACE – angiotensin-converting enzyme cont – continuous
ACS – acute compartment syndrome COPD – chronic obstructive pulmonary disease
ACTH – adrenocorticotrophic hormone CP – cerebral palsy
ad lib – freely, as desired CPAP – continuous positive airway pressure
ADH – antidiuretic hormone CPK – creatine phosphokinase
ADL – activities of daily living CPR – cardiopulmonary resuscitation
AFP – alpha-fetoprotein CRP – C-reactive protein
AIDS – acquired immunodeficiency syndrome C&S – culture and sensitivity
AKA – above-the-knee amputation CSF – cerebrospinal fluid
ALL – acute lymphocytic leukemia CT – computerized tomography
ALP – alkaline phosphatase CTD – connective tissue disease
ALS – amyotrophic lateral sclerosis CTS – carpal tunnel syndrome
ALT – alkaline phosphatase (formerly SGPT) cu – cubic
AMI – antibody-mediated immunity CVA – cerebrovascular accident or costovertebral
AML – acute myelogenous leukemia angle
amt. – amount CVC – central venous catheter
ANA – antinuclear antibody CVP – central venous pressure
ANS – autonomic nervous system D&C – dilation and curettage
AP – anteroposterior DC – discontinue
A&P – anterior and posterior DCBE – double-contrast barium enema
APC – atrial premature contraction DIC – disseminated intravascular coagulation
aq. – water DIFF – differential blood count
ARDS – adult respiratory distress syndrome dil. – dilute
ASD – atrial septal defect DJD – degenerative joint disease
ASHD – atherosclerotic heart disease DKA – diabetic ketoacidosis
AST – aspartate aminotransferase (formerly dL, dl – deciliter (100 mL)
SGOT) DM – diabetes mellitus
ATP – adenosine triphosphate DNA – deoxyribonucleic acid
AV – atrioventricular DNR – do not resuscitate
BCG – Bacille Calmette-Guerin DO – doctor of osteopathy
DOE – dyspnea on exertion IAPB – intra-aortic balloon pump
DPT – vaccine for diphtheria, pertussis, tetanus IBBP – intermittent positive pressure breathing
Dr. – doctor IBS – irritable bowel syndrome
DRE – digital rectal exam ICF – intracellular fluid
DVT – deep vein thrombosis ICP – intracranial pressure
D/W – dextrose in water ICS – intercostal space
Dx – diagnosis ICU – intensive care unit
ECF – extracellular fluid I&D – incision and drainage
ECG, EKG – electrocardiogram IDDM – insulin-dependent diabetes mellitus
ECT – electroconvulsive therapy IgA – immunoglobulin A
ED – emergency department IM – intramuscular
EEG – electroencephalogram I&O – intake and output
EMD – electromechanical dissociation IOP – increased intraocular pressure
EMG – electromyography IPG – impedance plethysmogram
ENT – ear, nose, and throat IPPB – intermittent positive-pressure breathing
ERCP – endoscopic retrograde IUD – intrauterine device
cholangiopancreatography IV – intravenous
ESR – erythrocyte sedimentation rate IVC – intraventricular catheter
ESRD – end-stage renal disease IVP – intravenous pyelogram or intravenous
ET – endotracheal tube pyelography
F – Fahrenheit JRA – juvenile rheumatoid arthritis
FBD – fibrocystic breast disease K+ – potassium
FBS – fasting blood sugar kcal – kilocalorie (food calorie)
FDA – U.S. Food and Drug Administration kg – kilogram
FFP – fresh frozen plasma KO, KVO – keep vein open
FHR – fetal heart rate KS – Kaposi’s sarcoma
FHT – fetal heart tone KUB – kidneys, ureters, bladder
481 L, l – liter
Common Medical Abbreviations lab – laboratory
fl – fluid lb. – pound
FOBT – fecal occult blood test LBBB – left bundle branch block
4 × 4 – piece of gauze 4 inches by 4 inches; used for LDH – lactate dehydrogenase
dressings LDL – low-density lipoprotein
FSH – follicle-stimulating hormone LE – lupus erythematosus
ft. – foot, feet (unit of measure) LH – luteinizing hormone
FUO – fever of undetermined origin liq – liquid
g – gram LLQ – left lower quadrant
GB – gallbladder 482
GCS – Glasgow coma scale NCLEX-RN® Exam Resources
GFR – glomerular filtration rate LOC – level of consciousness
GH – growth hormone LP – lumbar puncture
GI – gastrointestinal LPN – licensed practical nurse
gr – grain Lt, lt – left
gtt – drops LTC – long-term care
GU – genitourinary LUQ – left upper quadrant
GYN – gynecological LV – left ventricle
h, hrs – hour, hours LVN – licensed vocational nurse
(H) – hypodermically m – minum, meter, micron
Hb, Hgb – hemoglobin MAO – monoamine oxidase inhibitor
HCG – human chorionic gonadotropin MAST – military antishock trousers
HCO3 – bicarbonate mcg – microgram
Hct – hematocrit MCH – mean corpuscular hemoglobin
HD – hemodialysis MCV – mean corpuscular volume
HDL – high-density lipoprotein MD – muscular dystrophy, medical doctor
Hg – mercury MDI – metered dose inhaler
HGH – human growth hormone mEq – milliequivalent
HHNK – hyperglycemia hyperosmolar nonketotic mg – milligram
coma Mg – magnesium
HIV – human immunodeficiency virus MG – myasthenia gravis
HLA – human leukocyte antigen MI – myocardial infarction
H2O – water mL, ml – milliliter
HR – heart rate mm – millimeter
HSV – herpes simplex virus MMR – vaccine for measles, mumps, rubella
HTN – hypertension MRI – magnetic resonance imaging
Hx – history MS – multiple sclerosis
Hz – hertz (cycles/second) N – nitrogen, normal (strength of solution)
NIDDM – non–insulin dependent diabetes mellitus psi – pounds per square inch
(type 2) PSP – phenolsulfonphthalein
Na+ – sodium PT – physical therapy, prothrombin time
NaCl – sodium chloride PTCA – percutaneous transluminal coronary
NANDA – North American Nursing Diagnosis angioplasty
Association PTH – parathyroid hormone
NG – nasogastric PTSD – post-traumatic stress disorder
NGT – nasogastric tube PTT – partial thromboplastin time
NLN – National League for Nursing PUD – peptic ulcer disease
noc – at night PVC – premature ventricular contraction
NPO – nothing by mouth (nil per os) q – every
NS – normal saline QA – quality assurance
NSAID – nonsteroidal anti-inflammatory drug qd – once a day
NSNA – National Student Nurses’ Association qh – every hour
NST – non-stress test q 2 h – every two hours
O2 – oxygen q 4 h – every four hours
OB-GYN – obstetrics and gynecology qid – four times a day
OCT – oxytocin challenge test qs – quantity sufficient
OOB – out of bed R – rectal temperature, respirations, roentgen
OPC – outpatient clinic RA – rheumatoid arthritis
OR – operating room RAI – radioactive iodine
os – by mouth RAIU – radioactive iodine uptake
OSHA – Occupational Safety and Health RAS – reticular activating system
Administration RBBB – right bundle branch block
OTC – over-the-counter (drug that can be obtained RBC – red blood cell or red blood count
without a prescription) RCA – right coronary artery
oz. – ounce RDA – recommended dietary allowance
p – with resp – respirations
P – pulse, pressure, phosphorus RF – rheumatic fever, rheumatoid factor
PA Chest – posterior-anterior chest x-ray Rh – antigen on blood cell indicated by + or –
PAC – premature atrial complexes RIND – reversible ischemic neurologic deficit
PaCO2 – partial pressure of carbon dioxide in arterial RLQ – right lower quadrant
blood RN – registered nurse
PaO2 – partial pressure of oxygen in arterial blood RNA – ribonucleic acid
PAD – peripheral artery disease R/O, r/o – rule out, to exclude
Pap – Papanicolaou smear ROM – range of motion (of joint)
pc – after meals Rt, rt – right
PCA – patient-controlled analgesia RUQ – right upper quadrant
pCO2 – partial pressure of carbon dioxide Rx – prescription
PCP – Pneumocystis jiroveci (formely Pneumocystitis s – without
carinii pneumonia) pneumonia S., Sig. – (Signa) to write on label
PD – peritoneal dialysis SA – sinoatrial node
PE – pulmonary embolism SaO2 – systemic arterial oxygen saturation (%)
PEEP – positive end-expiratory pressure sat sol – saturated solution
PERRLA – pupils equal, round, react to light and SBE – subacute bacterial endocarditis
accommodation SDA – same-day admission
PET – postural emission tomography SDS – same-day surgery
PFT – pulmonary function test S/E – side effects
483 sed rate – sedimentation rate
Common Medical Abbreviations SGOT – serum glutamic-oxaloacetic transaminase
pH – hydrogen ion concentration (see AST)
PICC – peripherally inserted central catheter SGPT – serum glutamic-pyruvic transaminase (see
PID – pelvic inflammatory disease ALT)
PKD – polycystic disease SI – International System of Units
PKU – phenylketonuria SIADH – syndrome of inappropriate antidiuretic
PMS – premenstrual syndrome hormone
PND – paroxysmal nocturnal dyspnea SIDS – sudden infant death syndrome
PO, po – by mouth SL – sublingual
pO2 – partial pressure of oxygen SLE – systemic lupus erythematosus
PPD – positive purified protein derivative (of SMBG – self-monitoring blood glucose
tuberculin) SMR – submucous resection
PPE – personal protective equipment SOB – shortness of breath
PPN – partial parenteral nutrition sol – solution
PRN, prn – as needed, whenever necessary sp gr – specific gravity
pro time – prothrombin time spec. – specimen
PSA – prostate-specific antigen ss – one half
SS – soapsuds
S/S, s/s – signs and symptoms
SSKI – saturated solution of potassium iodide
stat – immediately
STD – sexually transmitted disease
subcut, SubQ – subcutaneous
sx – symptoms
Syr. – syrup
T – temperature, thoracic (followed by the number
designating specific thoracic vertebra)
T&A – tonsillectomy and adenoidectomy
tabs – tablets
TB – tuberculosis
T&C – type and crossmatch
TED – antiembolitic stockings
temp – temperature
TENS – transcutaneous electrical nerve stimulation
TIA – transient ischemic attack
TIBC – total iron binding capacity
tid – three times a day
tinct, tr. – tincture
TLC – total lymphocyte count
TMJ – temporomandibular joint
TPA, t-pa – tissue plasminogen activator
TPN – total parenteral nutrition
TPR – temperature, pulse, respiration
TQM – total quality management
TSE – testicular self-examination
TSH – thyroid-stimulating hormone
tsp. – teaspoon
TSS – toxic shock syndrome
TURP – transurethral prostatectomy
UA – urinalysis
um – unit of measurement
ung – ointment
URI – upper respiratory tract infection
UTI – urinary tract infection
VAD – venous access device
VDRL – Venereal Disease Research Laboratory (test
for syphilis)
VF, Vfib – ventricular fibrillation
VPC – ventricular premature complexes
VS, vs – vital signs
VSD – ventricular septal defect
VT – ventricular tachycardia
WBC – white blood cell or white blood count
WHO – World Health Organization
WNL – within normal limits
wt – weight
X PO – 10 grains per orem

Vous aimerez peut-être aussi